You are on page 1of 261

,:carihant

ARIHANT PUBLICATIONS (INDIA) LIMITED


All Rights Reserved

©Publisher
No part of this publication may be re-produced, stored in a retrieval system or by any means,
electronic, mechanical, photocopying, recording, scanning, web or otherwise without the written
permission of the publisher. Arihant has obtained all the information in this book from the sources
believed to be reliable and true. However, Arihant or its editors or authors or illustrators don't take
any responsibility for the absolute accuracy of any information published and the damage or loss
suffered thereupon.
All disputes subject to Meerut (UP) jurisdiction only.

!Ii Administrative & Production Offices


Regd. Office
'Ramchhaya' 4577/15, Agarwal Road, Darya Ganj, New Delhi -110002
Tele:011- 47630600, 43518550

!Ii Head Office


Kalindi, TP Nagar, Meerut (UP) - 250002
Tel:0121-7156203, 7156204

!Ii Sales & Support Offices


Agra, Ahmedabad, Bengaluru, Bareilly, Chennai, Delhi, Guwahati,
Hyderabad, Jaipur, Jhansi, Kolkata, Lucknow, Nagpur & Pune.

!Ii ISBN 978-93-25292-35-2

PO No: TXT-XX-XXXXXXX-X-XX
Published by Arihant Publications (India) Ltd.

For further information about the books published by Arihant, log on to


www.arihantbooks.com or e-mail at info@arihantbooks.com
Follow us on O� a@)
AFCAT-II ~ Solved Paper 2020 03

INDIAN AIR FORCE


AFCAT-II
*
Solved Paper 2020
Time : 2 Hrs MM : 300

DIRECTIONS
■ The set contains a total of 100 questions, comprising Verbal Ability in English, General Awareness, Numerical Ability and Reasoning and

Military Aptitude Test.


■ Each correct question carry 3 Marks and there will be negative marking of 1 Mark for each incorrect attempt.

■ Total time duration will be 2 hrs (120 minutes).

■ No marks will be deducted for unattempted questions.

Directions (Q. Nos. 1-3) In these each question out of the given four 7. Which one of the following is not
questions, choose the word similar in alternatives. correct?
meaning to the word given. Some people believe that they are (a) Internals have more confidence
1. Affluent masters of their own fate. Other people in their ability
(a) Prosperous (b) Self-obsessed see themselves as pawns of fats, (b) Internals are more
(c) Genuine (d) Agitation believing that what happens to them in enterprising
their lives is due to luck or chance. (c) Internals go in for routine jobs
2. Pilfer (d) Internals are more inquisitive
The first type, those who believe that
(a) Fearless (b) Steal
they control their destinies, have been 8. Which one of these is correct?
(c) Unlawful (d) Subjugation
labelled internals, whereas the latter, (a) Internals seek jobs which call for
3. Debacle who see their lives as being controlled by more initiative
(a) Victory outside forces, have been called (b) Internals seek jobs which are
(b) Accomplishment externals. full of fun and enjoyment
(c) Notoriety Internals search more actively for (c) Internals believe in receiving
(d) Failure information before making a decision, guidance and direction from
Directions (Q. Nos. 4-6) In the are more motivated to achieve, and superiors only
following questions, choose the word make a greater attempt to control their (d) Internals believe in destiny
opposite in meaning to the given word. environment. Externals, however, are
9. Externals perceive themselves
more compliant and willing to follow
4. Mitigate (a) as having little control over
directions. Therefore, internals do well
(a) Restful (b) Exalted organisational outcome
on sophisticated tasks which include
(c) Aggravate (d) Enumerate (b) as critical to the success of the
most, managerial and professional jobs
organisation
5. Luscious that require complex information
(c) as those who can influence sales
(a) Unappetizing (b) Delicious processing and learning.
volumes
(c) Ardent (d) Twisted In addition, internals are more suited to (d) as those who search frantically
6. Allure jobs that require initiative and for more information
(a) Egotistic (b) Attract independence of action. Almost all
(c) Repel (d) Overcome successful sales people, for instance are Directions (Q. Nos. 10-12) In these
internals. In contrast, externals should questions, four alternatives are given
Directions (Q. Nos. 7-9) You have one do well on jobs that are well structured for the idiom/phrase. Choose the
brief passage with two questions and routine and in which success alternative, which best expresses the
following the passage. Read the passage depends heavily on complying with the meaning of the idiom/phrase, given in
carefully and choose the best answer to directions of others. italics in the sentence.
*Based on Memory
04 AFCAT-II ~ Solved Paper 2020

10. The sight of the accident made my 18. With little imagination and 28. Who is the first batsman to hit six
flesh creep. enterprise, (a)/the tournament sixes in an over in a T20
(a) worried me could have been transformed International?
(b) frightened me (b)/into a major attraction. (c)/No (a) Chris Gayle
(c) confused me error(d) (b) Yuvraj Singh
(d) drew my attention 19. Order has been issued (a)/for his (c) Ben Stokes
11. I just paid him a left handed transfer to another district (b)/but (d) Mathew Hayden
compliment. he has not received them so far. 29. Which Indian leader was popularly
(a) an honest (c)/No error (d) known as ‘Punjab Kesari’ or
(b) a well deserved 20. The master did not know (a)/who ‘Sher-e- Punjab’?
(c) an insincere (d) a flattering of the servants (b)/broke the glass. (a) Bhagat Singh (b) Ajit Singh
(c). No error (d) (c) Lala Hardyal (d) Lala Lajpat Rai
12. The party stalwarts have advised
the President to take it lying down for 21. Each of these players (a)/have been 30. Who is called the father of
a while. warned (b)/not to repeat the silly Germanunity?
(a) to show no reaction mistake. (c)/no error (d) (a) Helmut Kohl (b) Garybaldi
(b) to take rest (c) Bismark (d) Hitler
(c) to be on the defensive
Directions (Q. Nos. 22-25) Sentences
are given with blanks to be filled it with 31. Vienna is the capital of which
(d) to be cautious country?
an appropriate words. Four words are
Directions (Q. Nos. 13-17) Below a suggested for each question. Choose the (a) Germany (b) France
passage is given, with five blanks, and correct alternative out of the four (c) Austria (d) Poland
four options are given for each blank. alternatives. 32. In which city was the United
Choose the word that fits each blank Nation Organisation was created?
most appropriately in the content of the
22. The speed with which she typed the
draft demonstrated that she is a (a) New York (b) London
passage. (c) Washington (d) San Francisco
……… secretary.
The much-awaited New Year is finally (a) savvy 33. The term ‘bully’ is connected with
here. While you might have made …… (b) adapted which sports?
(13) to exercise daily, read 20 books (c) nimble (a) Rugby (b) Baseball
before the year ends, splurge less on (d) competent (c) Hockey (d) Cricket
clothes or learn to …… (14) a musical
instrument, but have you spared a 23. Our latest battle plan ……… some 34. Who is the author of Ain-i-Akbari?
thought about your …… (15) life? In projects ……… at saving the (a) Abu’l Fazl
case, you are still figuring out what all Earth’s Biological diversity. (b) Amir Khusrau
you can do to climb up the corporate (a) finishes, stirring (c) Ziauddin Barani
ladder in 2020, we have got you (b) covers, aimed (d) Shaikh Burhan
covered. (c) enclose, looked
(d) excludes, arriving 35. Who is the writer of Das Kapital?
We have compiled a list of six small (a) Spencer
changes that can make a huge …… (16) 24. No one was allowed to use any (b) Jeremy Benthem
in your career. Apart from increasing written materials ……… the exam. (c) Max Weber
your daily productivity and managing (a) on (b) by (d) Karl Marx
the stress level at workplace, these (c) over (d) during
……… (17) would also help you to 36. Where is Hawaii located in the
25. If you smuggle goods into the world?
achieve a better work-life balance.
country, they may be ……… by the (a) Indian Ocean (b) Pacific Ocean
13. (a) Plans (b) reports customs authority. (c) Atlantic Ocean (d) Arctic Ocean
(c) expenses (d) resolutions (a) possessed
(b) punished 37. Which of the following is the
14. (a) play (b) make lightest gas?
(c) confiscated
(c) teach (d) perform (a) Oxygen (b) Nitrogen
(d) fined
15. .(a) personal (b) professional (c) Hydrogen (d) Methane
26. Who is known as the “Napolean of
(c) economical (d) social India”? 38. The first oil well in India was dug at
16. (a) gap (b) difference (a) Samudragupta (a) Naharkatiya (b) Digboi
(b) Chandragupta II ‘Vikramaditya’ (c) Akleshwar (d) Mumbai High
(c) deviation (d) profit
(c) Skandagupta 39. Equator doesn’t pass through
17. (a) circumstances (b) habits (d) Ashoka which continent?
(c) paths (d) goals
27. Chris Evert’s name is associated (a) Africa (b) South America
Directions (Q. Nos. 18-21) Read each with which sports? (c) Asia (d) Australia
sentence to find out which part of the (a) Cricket 40. In which layer of the atmosphere
sentence has an error. If you find no (b) Football do Aurora Borealis appear?
error, your answer should be indicated (c) Tennis (a) Ionosphere (b) Troposphere
as option (d) No error. (d) Badminton (c) Stratosphere (d) Mesosphere
AFCAT-II ~ Solved Paper 2020 05

41. CTPS thermal power plant is per litres of petrol if he spends 60. Difference between SI and CI is
located in which city? ` 4000 each year? [Numerical may ` 604. If the sum of amount is given
(a) Jamshedpur (b) Nasik differ] at the rate of 25% for 3 yr, find SI
(c) Chandrapura (d) Ranchi (a) ` 9.17 (b) ` 11.2 and CI.
(c) ` 7.12 (d) ` 12.6 (a) 2230.15 and 2834.15
42. Separation of the Judiciary from
the Executive is enjoined by 53. In a library 5 per cent books are in (b) 2430.25 and 2622.24
English; 10 per cent of the (c) 2625.25 and 2700.30
(a) Fundamental Right
remaining are in Hindi and (d) 2010.12 and 2220.27
(b) Directive Principles of State Policy
(c) Fundamental Duty 15 per cent of the remaining are in 61. Akash leaves Mumbai at 6 am and
(d) Preamble Sanskrit. The remaining 11628 reaches Bangalore at 10 am.
books are in French. Then, find the Prakash leaves Bangalore at 8 am
43. First Afro-Asian Games held in total number of books in the and reaches Mumbai at 11:30 am.
Hyderabad in which year?
library. At what time do they cross each
(a) 2003 (b) 2005
(a) 15000 (b) 12000 other?
(c) 2008 (d) 2010
(c) 16000 (d) 10000 (a) 8 : 56 am (b) 9 am
44. The term ‘Chukker’ is used in 54. Two poles cast shadow 12 m and (c) 8 : 56 pm (d) 9 pm
which game?
14 m respectively. If height of one 62. Two trains traveling in opposite
(a) Chess (b) Kho-Kho is 48 m, what can be possible direction crosses a man in 17 sec
(c) Polo (d) Volleyball height of second? and 27 sec respectively and they
45. First SAARC Gold Championship (a) 56 m (b) 62 m cross each other in 23 sec. Find the
held in which country? (c) 50 m (d) 45 m ratio of their speeds.
(a) India (b) Bangladesh 55. (30% of 6500) ÷ 26 ÷ 25 = ? (a) 1 : 2 (b) 2 : 1
(c) Sri Lanka (d) Pakistan (c) 2 : 3 (d) 5 : 7
(a) 4 (b) 8
46. At which place will you find (c) 2 (d) 3 63. A seller has a 20% loss after selling
maximum sunlight in December? an item at ` 1200. At what price
56. CP of 30 kg of rice is same as the SP
(a) Kanyakumari of “X” kg of wheat. If shopkeeper should sell it to get 5% profit?
(b) Kolkata makes profit of 20%, find X? (a) ` 1600 (b) ` 1440
(c) Mumbai (c) ` 1550 (d) ` 1575
(a) 20 kg (b) 24 kg
(d) Hyderabad
(c) 25 kg (d) 28 kg 64. A sum invested at SI will be twice
47. AWACS warning system was 57. Two guns are fired from the same in 12 yr. Find the rate of interest.
equipped with which aircraft? place at an interval of 15 min. A (a) 10% (b) 8.33%
(a) Mig 21 (b) Sukoi 30 person approaching the place (c) 15% (d) 5%
(c) IL-76 (d) TEJAS observes that 14 min 30 sec have 65. A sum invested at SI gets thrice in
48. Which of the following forts was elapsed between the hearings of the 8 yr. Then, after 20 yr it will be?
not built by Akbar? sound of the two guns. If the (a) 6 times (b) 2 times
(a) Gwalior Fort (b) Agra Fort velocity of the sound is 330 m/sec, (c) 5 times (d) 4 times
(c) Lahore Fort (d) Allahabad Fort the man was approaching that
place at what speed (in km/h)?
66. Ratio of volumes of 3 containers is
49. Who was the author of 3 : 4 : 5. All the three containers are
(a) 50.23
Kitab-ul-Hind? full of a mixture of milk and water.
(b) 40.9655
(a) Minhaj Siraj Ratio of milk and water is 4 : 1,
(c) 45.1826
(b) Ziauddin Barani 3 : 1 and 3 : 2 in the three
(d) 32.2632
(c) Badauni containers respectively. If the liquid
(d) AL-Biruni 58. A boy starts painting a fence, 2nd of all the three containers is poured
day 2 boys join him, 3rd day 3 more in the 4th container, then find the
50. Gondwana Hills are located where?
boys joined them and so on, fence ratio of milk and water.
(a) Uttarakhand
is painted in 20 days. How many (a) 1 : 2 (b) 2 : 3
(b) Madhya Pradesh
days will be taken by 10 men to (c) 7 : 3 (d) 5 : 4
(c) Himachal Pradesh paint the whole fence if efficiency
(d) Maharashtra 67. A boy goes from his house to school
of 1 man is equal to efficiency of 2
at 3 km/h and return at 2 km/h. If
51. Calculate compound interest on a boys?
he takes total 5 h in the whole
sum of ` 64000 compounded (a) 67 (b) 74
journey, find the distance between
quarterly for 1 year at a rate of (c) 84 (d) 77
school and his house.
10%?
59. Two classes X and Y have students (a) 10 km (b) 7 km
(a) ` 7425.221 (b) ` 6644.025 32 and 30. Average of X is 45 and Y (c) 4 km (d) 6 km
(c) ` 5602.028 (d) ` 6244.155 is 50. Find average of both the
68. 60% of total donation required by
52. A car owner or a man buys petrol at class.
school was collected when 50% of
` 8, ` 9 and ` 11 per litres for three (a) 54.32 (b) 47.42
the parents donated an average of
successive years. What (c) 61.23 (d) 40.25
` 1200. Find the average amount
approximately is the average cost
06 AFCAT-II ~ Solved Paper 2020

that should be donated by rest 50% Directions (Q. Nos. 79 and 80) What figure from the answer figures which
parents so that school has the comes in place of question mark in the when placed next to the problem figures
whole amount that was required. following letter series? will continue the sequence of problem
(a) 1000 (b) 2000 79. ABC, PQR, DEF, STU, ? figures.
(c) 800 (d) 500 84. Problem Figures
(a) GKL (b) VWX
Directions (Q. Nos. 69-72) In each of (c) GHI (d) IJK
the following questions, there is a 80. XWA, VTC, SPF, OKJ, ?
certain relationship between two given
(a) JDN 1 2 3 4 5
words on one side of (::) and one word is
(b) JEO Answer Figures
given on another side (::) while another
(c) LPN
word is to be found from the given
(d) JDP
alternatives, having the same relation
with this word as the words of the given 81. Which of the following diagrams (a) (b) (c) (d)
pair bear. Choose the correct represents the relation among
alternative. Currency, Rupee and Dollar? 85. Problem Figures
69. House : Door :: Compound : ?
(a) Gate (b) Fence
(c) Foundation (d) Wall 1 2 3 4 5
70. Hongkong : China :: Vatican : ? (a) (b) Answer Figures
(a) France (b) Mexico
(c) Canada (d) Rome
71. Giant : Dwarf : : Genius : ? (a) (b) (c) (d)
(a) Wicked (b) Gentle (c) (d)
(c) Idiot (d) Tiny 86. Problem Figures
72. Aryabhatta : Mathematician : : 82. Which one of the following
Varahamihira : ? diagrams best depicts the
(a) Physician relationship among Elephants,
Wolves and Animals? 1 2 3 4 5
(b) Astronomer
Answer Figures
(c) Scientist
(d) Architect
Directions (Q. Nos. 73-77) In the
following questions, three alternatives (a) (b) (c) (d)
(a) (b)
are same in a certain way out of four Directions (Q. Nos. 87-90) In the
and so form a group. Find the odd word following questions a group of four
that does not belong to the group. figures are given. Out of which three
73. (a) Father (b) Mother figures are similar to each other in a
(c) (d) certain pattern. Find the figure which
(c) Friend (d) Brother
83. Which diagram correctly represents does not belong to the group.
74. (a) Red (b) Blue
(c) Yellow (d) Black the relationship among Politicians, 87.
Poets and Women?
75. (a) Kanpur (b) Lucknow
(c) Lahore (d) Patna
(a) (b) (c) (d)
76. (a) Ample (b) Copious
(c) Plentiful (d) Abundance 88.
(a) (b)
77. (a) Rifle (b) Cannon
(c) Sword (d) Pistol
(a) (b) (c) (d)
78. If in a certain code language
‘NAME’ is written as ‘OYPA’, then
(c) (d) 89.
how will ‘TEAM’ be coded in that
language? Directions (Q. Nos. 84-86) In each of
(a) UCDI (b) UCID the following questions, find out the (a) (b) (c) (d)
(c) UICD (d) UDCI
AFCAT-II ~ Solved Paper 2020 07

90. 94. Question Figure Answer Figures

(a) (b) (c) (d)


(a) (b) (c) (d)
Directions (Q. Nos. 91-95) In each of ?
the following questions, a part of 98. Question Figure
question figure is missing. Find out Answer Figures
from the given answer figures (a), (b),
(c) and (d), that can replace the ‘?’ to
complete the question figure.
Answer Figures
91. Question Figure (a) (b) (c) (d)

95. Question Figure

(a) (b) (c) (d)


?
? Direction (Q. No. 99) In the question
below is given a statement followed by
Answer Figures assumptions numbered I and II.
Consider the statement and decide
Answer Figures which of the given assumptions is
implicit.
(a) (b) (c) (d) 99. Statement Get your child
examined by a specialist doctor, X
92. Question Figure (a) (b) (c) (d)
tells Y.
Directions (Q. Nos. 96-98) In the Assumptions I. Y will not listen
? following questions, a question figure what X tells to him.
and a set of four answer figures (a), (b), II. Y will hear X’s advice.
(c) and (d) are given. Find out that (a) Only Assumption I is implicit
answer figure in which the question (b) Only Assumption II is implicit
figure is embedded. (c) Both I and II are implicit
Answer Figures (d) Neither I nor II is implicit
96. Question Figure
Direction (Q. No. 100) In the question
below are given two statements followed
by two conclusions. You have to take the
(a) (b) (c) (d) two given statements to be true even if
Answer Figures they seem to be at variance from
93. Question Figure commonly known facts and decide
which of the conclusion(s) logically
follow(s) from the two given statements.

(a) (b) (c) (d) 100. Statements Some dogs are cats.
All cats are pigs.
?
97. Question Figure Conclusions I. Some cats are dogs.
II. Some dogs are pigs.
Answer Figures
(a) Only Conclusion I follows
(b) Only Conclusion II follows
(c) Either Conclusion I or II follows
(d) Both Conclusions I and II follow
(a) (b) (c) (d)
08 AFCAT-II ~ Solved Paper 2020

Answers
1. (a) 2. (b) 3. (d) 4. (c) 5. (a) 6. (c) 7. (c) 8. (a) 9. (a) 10. (b)
11. (c) 12. (b) 13. (d) 14. (a) 15. (b) 16. (b) 17. (b) 18. (a) 19. (a) 20. (b)
21. (b) 22. (d) 23. (b) 24. (d) 25. (c) 26. (a) 27. (c) 28. (b) 29. (d) 30. (a)
31. (c) 32. (d) 33. (c) 34. (a) 35. (d) 36. (b) 37. (c) 38. (b) 39. (d) 40. (a)
41. (c) 42. (b) 43. (a) 44. (c) 45. (d) 46. (a) 47. (c) 48. (a) 49. (d) 50. (b)
51. (b) 52. (a) 53. (c) 54. (a) 55. (d) 56. (c) 57. (b) 58. (d) 59. (b) 60. (a)
61. (a) 62. (c) 63. (d) 64. (b) 65. (a) 66. (c) 67. (d) 68. (c) 69. (a) 70. (d)
71. (c) 72. (b) 73. (c) 74. (d) 75. (c) 76. (d) 77. (c) 78. (a) 79. (c) 80. (b)
81. (b) 82. (a) 83. (a) 84. (d) 85. (d) 86. (d) 87. (b) 88. (d) 89. (b) 90. (d)
91. (a) 92. (d) 93. (d) 94. (c) 95. (d) 96. (a) 97. (b) 98. (d) 99. (b) 100. (d)

Hints and Solutions


1. (a) ‘Affluent’ means rich/wealthy. So, 11. (c) A left-handed compliment is one that 24. (d) Preposition ‘during’ will be used to fill
‘prosperous’ would be its most similar sounds like praise but has an insulting the given blank. ‘During’ is used in the
meaning word. meaning. So, ‘insincere’ would be its expressions of time.
2. (b) ‘Pilfer’ means to steal or to rob. correct answer. 25. (c) In the context of the sentence, option
Hence, ‘steal’ would be its correct similar 12. (b) According to the given sentence, ‘to (c) confiscated is the correct choice to fill
meaning word. take rest’ is the correct answer of the the given blank. Confiscated means taken
3. (d) ‘Debacle’ means a sudden and given idiom. or seized (possession) with authority.
ignominious failure. Hence, failure 13. (d) resolutions 26. (a) Samudragupta of the Gupta dynasty is
would be its correct similar meaning 14. (a) play known as the Napolean of India. Historian
word. AV Smith called him Napolean of India
15. (b) professional
4. (c) ‘Mitigate’ means make (something because of his great military conquests
16. (b) difference known from the ‘Prayag Prashati’ written
bad) less severe, serious or painful.
So, among the given options, aggravate 17. (b) habits by his courtier and poet Harisena, who
would be its correct antonym as it 18. (a) Little means none or negligible. Thus, also describes him as the hero of hundred
means make (a problem, injury or we use article ‘a’ before little as ‘a little’ battles. He succeeded his father around
offense) worse or more serious. means ‘some’. 335 or 350 CE and ruled until c375 CE.
He was also a great player of Veena.
5. (a) ‘Luscious’ means delicious or tasty 19. (a) When a person is assigned to a new
(of food or drink). Among the given station, he receives his ‘orders’. This is a 27. (c) Chris Evert is associated with Tennis.
options ‘unappetizing’ would be its set of instructions, including where to be, She won 18 Grand Slam singles
correct antonym as it means flavourless, when to be there etc. So, the correct championships and three doubles titles.
uneatable. sentence would be ‘orders have been She was the year-ending world No.1
issued ……… . singles player in 1974, 1975, 1976,
6. (c) ‘Allure’ means powerfully attract or 1977, 1978, 1980 and 1981. Overall,
charm. So, among the given options, 20. (b) As the sentence refers to a choice
Evert won 157 singles championships
repel would be its correct antonym as it among more than two persons (servants),
and 32 doubles titles. She took
means disgust. ‘which’ will be used in place of ‘who’.
retirement in 1987.
7. (c) Internals are not go for routine jobs. 21. (b) The phrase ‘Each of ’ takes a singular
28. (b) Yuvraj Singh is the first batsman to
Externals should do well on jobs that are verb. So, ‘have’ would be replaced by
hit six sixes in an over in a T20
well-structured and routine. ‘has’.
International. He punched six sixes in an
Hence, option (c) is the correct answer. 22. (d) ‘Competent’ means having the over on 19th September, 2007 against
8. (a) Internal seek jobs which call for more necessary ability, knowledge or skill to do Stuart Broad (England) in a league stage
initiative, is the correct one. something successfully. So, option (d) match of the inaugural T20 World Cup.
9. (a) Externals perceive themselves as ‘competent’ is a suitable choice to fill the
29. (d) Lala Lajpat Rai is popularly known as
having the little control over given blank.
‘Punjab Kesari’ or ‘Sher-e-Punjab’. He
organisational outcome. 23. (b) The suitable choice to fill the given was prominent freedom fighter. In 1881,
10. (b) Make one’s flesh creep means to blanks is option (b) covers, aimed. Other he joined the Indian National Congress at
cause one to feel disgusted, frightened or choices are irrelevant in the context of the the age of 16. In 1885, he established
unnerved. sentence. the Dayanand Anglo-Vedic School in
AFCAT-II ~ Solved Paper 2020 09

Lahore. He was among the leaders of the to classical political economists such as particles in both solar wind and
movement opposing the Simon Adam Smith, Jean-Baptiste Say, David magnetospheric plasma.
Commission and was severely Ricardo and John Stuart Mill. He was 41. (c) Chandrapura Super Thermal Power
lathi-charged during a protest in Lahore German philosopher and socialist Station (CTPS) is a thermal power plant
on 30th October, 1928. revolutionary. located in Chandrapura city of
30. (a) Helmut Kohl was served as 36. (b) Hawaii is located in the Pacific Jharkhand. The coal for the power plant
Chancellor of Germany from 1982 to Ocean. It is one of the states of the is sourced from Durgapur and Padmapur
1998 (of West Germany 1982-1990 United States. And, it only state located Collieries of Western Coalfields limited. It
and of reunified Germany 1990-1998). outside North America, the only island was inaugurated in 1984.
He played key role in unification of East state, and the only state in the tropics. 42. (b) The separation of the Judiciary from
and West Germany, he is referred as It consists of 137 volcanic islands the Executive is enjoined by Article 50 of
father of Germanunity. He was also the spanning 1,500 miles (2400 km), which the Directive Principles of State Policy
architect of the Maastricht Treaty, which are physiographically and ethnologically (DPSP). The separation of the Judiciary
established the European Union (EU) part of the Polynesian subregion of from the Executive is important to ensure
and the Euro currency. Oceania. balance of power in the system.
31. (c) Vienna is the capital of Austria. It is 37. (c) Hydrogen is the lightest gas. It is also DPSP are ideals which are meant to be
Austria’s most-populous city. Austria is a is a colorless and odorless gas. It is kept in mind by the state when it
landlocked East Alpine country in the represented by symbol H and atomic formulates policies and enacts laws.
southern part of Central Europe. It is number 1. It has a standard atomic Articles 36-51 under Part-IV of
composed of nine federated states one of weight of 1.008. Henry Cavendish was Constitution deals with DPSP.
which is Vienna. Austria is a the first to recognise that hydrogen gas 43. (a) The first Afro-Asian Games were held
parliamentary representative democracy was a discrete substance, and that it in 2003 at Hyderabad (Telangana).
with a directly elected Federal President produces water when burned. It can be China topped the medal tally. They are
as head of state and a Chancellor as found anywhere throughout the human jointly supervised by the Olympic Council
head of the federal government. cell, such as in lysosome, endoplasmic of Asia (OCA), and the Association of
32. (d) The United Nation Organisation reticulum, golgi and peroxisome. National Olympic Committees of Africa
(UNO) was established after World War II 38. (b) The first oil well in India was dug at (ANOCA). These Games are supposed to
with the aim of preventing future wars, Digboi in Tinsukia district in the be held once every four years. Till now,
succeeding the ineffective League of north-eastern part of the state of Assam. only one edition of these games has been
Nations. On 25th April, 1945, 50 Oil was discovered in the year 1889. organised.
governments met in San Francisco for a Assam Oil Company was formed in 1899 44. (c) The term ‘Chukker’ is used in Polo. In
conference and started drafting the UN to look after the running of the oil Polo, each team has four mounted riders,
Charter, which was adopted on 25th business in this area. India (and Asia) and the game usually lasts one to two
June, 1945 and took effect on 24th established its first refinery in Digboi in hours, divided into periods called
October, 1945. The UN is headquartered the year 1901. It is also known as Oil chukkas (or “chukkers”).
on international territory in New York City City of Assam.
and it consists of 193 member states 45. (d) The first South Asian Association for
39. (d) Among the given options, Equator Regional Co-operation Gold Cup was
and 2 observer states.
doesn’t pass through Australia. Equator held in Lahore, Pakistan between 16th
33. (c) The term ‘bully’ is connected with passes through 13 countries, 3 July, 1993 and 23rd July, 1993. The
Hockey. It refers to the set play that is continents and 3 water bodies. countries that competed in this
used to restart the game, from where the
Continent Country tournament were India, Nepal, Pakistan
infringement took place. Two opposing
South and Sri Lanka. Title was clinched by
players stand facing each other and tap Ecuador, Colombia, Brazil
America India after defeating Sri Lanka.
each other’s sticks and the ground three
times. Gabon, Congo, Democratic 46. (a) During the month of December Sun
Republic of Congo, shifts toward Tropic of Capricorn from
34. (a) Abu’l Fazl is the author of Ain-i-Akbari Africa
Uganda, Kenya, Sao Tome
and Principe, Somalia Tropic of Cancer. As a result of which
(Administration of Akbar). He was court
Sun’s rays falls vertically over Tropic of
historian of Emperor Akbar and had Maldives, Indonesia,
Asia Capricorn.
written it in the Persian language. In this Kiribati (Oceania)
book, he described various aspects of the Among the given options Kanyakumari is
40. (a) Aurora Borealis appear in the layer of located nearest to Tropic of Capricorn,
administration of the Mughal Empire
Ionosphere. It is a beautiful light show hence, it receives maximum sunlight in
under Emperor Akbar. The Akbarnama
that happens in the sky of North Pole. It December.
(Account of Akbar) is also authored by
is also known as northern lights. Auroras
Abu’l-Fazl. 47. (c) AWACS warning system was
are the result of disturbances in the
35. (d) Das Kapital was written by Karl Marx. equipped with IL-76. It is a
magnetosphere caused by solar wind.
In this book, Marx aimed to reveal the multi-purpose four-engine turbofan
These disturbances are sometimes strong strategic airlifter designed by the Soviet
economic patterns underpinning the
enough to alter the trajectories of charged Union’s Ilyushin design bureau. It was
capitalist mode of production in contrast
10 AFCAT-II ~ Solved Paper 2020

designed to deliver heavy machinery to 12000 57. (b) Difference of time


=
remote, poorly served areas. 4000  1 1
+ +
1 = 15 min − 14 min 30 sec = 30 sec
 8 9 11
48. (a) Except Gwalior Fort all the other given Distance covered by man in 14 min 30 sec
forts were built by Akbar. Gwalior Fort is 3
= = Distance covered by sound in 30 sec
a hill fort near Gwalior, Madhya Pradesh. 99 + 88 + 72
It was built by Man Singh Tomar 8 × 9 × 11 = Speed × Time = 330 × 30 = 9900 m
(reigned 1486-1516 CE). It consisted of Distance
3 × 8 × 9 × 11 ∴Speed of man =
two main palaces, Gujari Mahal and Man = = ` 9.17 per liters Time
259
Mandir. The Gujari Mahal palace was 9900
53. (c) Let the total number of books in the = m/s
built for Queen Mrignayani. 14 × 60 + 30
library = x
49. (d) Kitab-ul-Hind was authored by 9900 18 Q1 m / s = 18 km / h
According to the question, = ×
AL-Biruni in Arabic language. It was 870 5  5 
100 − 5  100 − 10
divided into subjects such as religion and x ×   × 
philosophy, festivals, astronomy,  100   100  = 409655
. km/h
alchemy, manners and customs, social 100 − 15 58. (d) Let us first consider a general series.
×  
life, weights and measures, iconography,  100  1 + (1 + 2) + (1 + 2 + 3) + ...
laws and metrology. He was also familiar
= 11628 + (1 + 2 + 3 ... n)
with translations and adaptations of
Sanskrit, Pali and Prakrit texts into Arabic 95 90 85 The sum of above series for n terms
⇒x × × × = 11628
100 100 100 n (n + 1) (n + 2)
which ranged from fables to works on =
astronomy and medicine. 11628 × 1000000 6
⇒x =
50. (b) Gondwana Hills are located in 95 × 90 × 85 The number of boys for 20 days
Madhya Pradesh. Gondwana hills are not 20 (20 + 1) (20 + 2)
∴ x = 16000 =
just limited in the Madhya Pradesh, 6
54. (a) According to the question,
rather they spread over some regions of 20 × 21 × 22
Telangana, Andhra Pradesh and A P = = 1540
6
Maharashtra. Gondwana hills are named
But efficiency of 1 man = efficiency of
after Gond dynasty by which it was ruled 48 m
2 boys
during the era of Mughals. Gonds were
1540
the tribal community more often found in C R ∴10 men will take = = 77 days
the Gond forest in central India and 12 m B 14 m Q 2 × 10
spoke the Dravidian language. Q∆ABC ~ ∆PQR Sum of terms
59. (b) Required average =
51. (b) Given, P = ` 64000, R = 10%, AB PQ Number of terms
∴ = 32 × 45 + 30 × 50 1440 + 1500
n = 1 yr BC RQ = =
We know that, if the interest is 48 PQ 32 + 30 62
⇒ = ⇒ PQ = 4 × 14
compounded quarterly, then 12 14 2940
= = 47.42
 R 
4n  ∴PQ = 56 m 62
CI = P  1 +  − 1 60. (a) Given, D = ` 604, R = 25%, t = 3 yr
 4 × 100  Hence, the possible height of second
  pole can be 56 m. PR2 (300 + R )
 4 ×1  We know that, D =
= 64000  1 +
10 
− 1 55. (d) ? = (30% of 6500) ÷ 26 ÷ 25 (100)3

 
  =  × 6500 ÷ 26 ÷ 25
400 30
P (25)2 (300 + 25)
 100  ⇒ 604 =
 41 4 
= 64000    − 1
1000000
  = 1950 ÷ 26 ÷ 25 = 75 ÷ 25 = 3 604 × 1000000
 40  ⇒ P= = 297354
.
56. (c) Let CP of 1 kg rice = ` 1 625 × 325
− 2560000
= 64000 
2825761
CP of 30 kg rice = ` 30 P×R×T
 2560000  Then, SI =
CP of x kg wheat = ` x 100
265761
= = ` 6644.025 297354 . × 25 × 3
40 SP of x kg wheat = CP of 30 kg rice = = ` 2230.15
100
52. (a) Total cost = 3 × 4000 = 12000 = ` 30
 T 
and CI = P  1 +
SP − CP R 
4000 4000 4000 ∴Profit percentage = × 100  − 1
Total quantity = + + CP   100  
8 9 11
30 − x  
Total Cost ⇒ 20 = × 100 3
.  1 +
∴Required average = 25 
x = 297354  − 1
Total quantity   100  
12000 ⇒ x = (30 − x) × 5
= −
. × 
⇒ x + 5x = 150 ⇒ 6x = 150 125 64
4000 4000 4000 = 297354
+ +  64 
8 9 11 ∴ x = 25 kg
= ` 2834.15
AFCAT-II ~ Solved Paper 2020 11

100 + 5
61. (a) Mumbai
d km
Bangalore = 1500  67. (d) Let the distance between school and
 100  house = d km
Let the distance between Mumbai and
= 15 × 105 According to the question,
Bangalore = d km
= ` 1575 d d  distance 
Time taken by Akash = 4 h + =5 Q time = 
64. (b) Let the Principal be x. Then 3 2  speed 
Distance d
∴Speed of Akash = = km/h 2d + 3d
Time 4 Amount = 2 × x = 2x ⇒ = 5 ⇒ 5d = 5 × 6
So, SI = Amount − Principal 6
7
. h= h
Time taken by Prakash = 35 ∴ d = 6 km
2 = 2x − x = x
d 2d We know that, 68. (c) Let total number of parents = 100
∴Speed of Prakash = = km/h
7 7 Principal × Rate × Time Total amount donated by 50% of parents
SI =
2 50
100 = 100 × × 1200
QAkash leaves 2 h earlier than Prakash x × Rate × 12 100
⇒ x=
∴Distance covered by Akash in 2 h 100 = ` 60000
d d 100
= Speed × Time = 2 × = km ⇒ Rate = Total donated amounts
4 2 12 60000 × 100
d d = = ` 100000
∴Remaining distance = d − = km ∴ Rate = 833
. % 60
2 2
65. (a) Given, T1 = 8 yr, N1 = 3, T2 = 20 yr, Remaining donated amount
Distance
∴Required time = N2 = ? = 100000 − 60000
Speed
d We know that, = ` 40000
1 28 14 T1 N −1 ∴Required average amount that should
= 2 = × = h = 1
d 2d 2 15 15
+ T2 N2 − 1 be donated by rest 50% parents
Total amount
4 7 8 3−1 =
14 ⇒ = Total parents
= × 60 = 56 min 20 N2 − 1
15 40000
2 × 20 = = ` 800
Hence, they cross each other at ⇒ N2 − 1 = =5 50
8
= 8:56 am 69. (a) ‘Doors’ are entry to a ‘House’ or we
∴N2 = 5 + 1 = 6 times
62. (c) Let the speeds of two train are a and b can get into the house through door.
respectively. Length of first train = 17a 66. (c) Let the volumes of 3 containers are
In the same way, gates are entry to a
30 L, 40 L and 50 L.
[Qdistance = speed × time] Compound or we can get into a
Quantity of milk in container 1 compound through ‘Gate’.
Length of second train = 27 b 4
= 30 × = 24 L 70. (d) As, ‘Hongkong’ is in ‘China’, in the
Time taken to cross each other 5
Distance 17a + 27b same way ‘Vatican’ is situated in Rome.
= = Quantity of water in container 1
Time a+ b 1 71. (c) ‘Dwarf ’ is the antonym of ‘Giant’, in
= 30 × = 6 L the same way ‘Genius’ is the antonym of
17a + 27b 5
⇒ 23 = ‘Idiot’.
a+ b Quantity of milk in container 2
3 72. (b) ‘Aryabhatta’ was the famous
⇒ 23a + 23b = 17a + 27b = 40 × = 30 L ‘Mathematician’ of the ancient period, in
4
⇒ 6a = 4b the same way ‘Varahamihira’ was a
Quantity of water in container 2 famous ‘Astronomer’ of ancient period.
a 4 2
∴ = = 1
= 40 × = 10 L
b 6 3 4 73. (c) Except friend, all others are family
∴Required ratio = 2 : 3 members.
Quantity of milk in container 3
63. (d) Given, SP of the item = ` 1200, 3 74. (d) Except Black, all others are primary
= 50 × = 30 L colours.
Loss = 20% 5
75. (c) Except Lahore, all others are Indian
∴CP of the item Quantity of water in container 3
2 cities.
 100  = 50 × = 20 L
= SP   5 76. (d) Except ‘Abundance’ all mean
100 − loss %  sufficient amount but abundance is used
If the liquid of all three containers is
 100  poured in the 4th container, then for more than sufficient amount.
= 1200  
100 − 20 Quantity of milk in container 4 77. (c) Except sward, all others are fire arms.
1200 × 100 = 24 + 30 + 30 = 84 L 78. (a) As, 14 1 13 5 15 25 16 1
= = ` 1500 N A M E O Y P A
80 Quantity of water in container 4 +1
Now, profit = 5% = 6 + 10 + 20 = 36 L –2
100 + Profit %  ∴Required ratio = 84 : 36
∴Required SP = CP 
+3
 100  =7:3 –4
12 AFCAT-II ~ Solved Paper 2020

Similarly, 20 5 1 13 21 3 4 9 politicians are poets. This can be the alternative figure has been shown in
T E A M U C D I expressed as given in option (a). the below figure.
+1
–2 Poets Women
+3
–4 Politicians
∴ TEAM = UCDI 84. (d) It is clear from the pattern of series 97. (b) Clearly, the question figure is
79. (c) The pattern is as follows that the figure rotates 90°, 135°, 180°, embedded in answer figure (b).
+1 225° and finally 270°, respectively in the The portion which question figure
clockwise direction. occupies in the alternative figure has
P Q R S T U 85. (d) It is clear from the pattern of series been shown in the below figure.
16 17 18 19 20 21 that the outer figure rotates through an
1 23 45 6 7 89
A BC DE F GH I angle of 45° in each subsequent block
+1 +1 whereas the inner figure rotates through
an angle of 90° in anti-clockwise
∴? = GHI
direction in each subsequent block. 98. (d) Clearly, the question figure is
80. (b) The pattern is as follows embedded in figure (d) only.
86. (d) The number of sides in the outer
24 22 −3 19 −4 15 −5 10 figure is increased by one and number of
1st Letter : X −2 V S O J sides in the inner figure is decreased by
23 −3 20 −4 16 −5 11 −6 5 one side successively in each block.
2nd Letter : W T P K E
87. (b) In all the figures, except figure (b),
1 +2 3 +3 6 +4 10 +5 15 arrow is one step away from the shaded 99. (b) We advice anybody with the
3rd Letter : A C F J O
sector of the circle in anti-clockwise assumption that our advice will be
∴ ? = JEO direction. listened to. Why will we advice someone
88. (d) All the figures, except figure (d) are with a thinking that our advice will not be
81. (b) Rupee and dollar are entirely different
divided into four equal parts. heard? While saying something, we go
from each other but both come under the
with a positive frame of mind that the
category of currency. This can be 89. (b) Except figure (b), all others are
targeted people will hear and pay
expressed as given in option (b). rotated forms of the same figure.
attention towards our point of view.
90. (d) In all the figures, except figure (d), Hence, II is a valid assumption but I is
Dollar Currency arrows are facing in opposite direction. not.
91. (a) Answer figure (a) will complete the 100.(d) According to the question,
Rupee question figure.
Pigs
92. (d) Answer figure (d) will complete the Dogs

82. (a) Wolves and elephants both are question figure.


animals but are different from each 93. (d) Answer figure (d) will complete the
other.This can be expressed as given in question figure.
option (a). 94. (c) Answer figure (c) will complete the
question figure.
95. (d) Answer figure (d) will complete the Cats
Wolves Elephants
question figure. Conclusions
Animals
96. (a) Clearly, the question figure is I. Some cats are dogs. (ü)
embedded in answer figure (a). The
83. (a) Some women are poets. Some II. Some dogs are pigs. (ü)
portion which question figure occupies in
women are politicians. Some women
A FCA T - I ~ Solved Paper 2020 13

INDIAN AIR FORCE


AFCAT-I
*
Solved Paper 2020
Time 2 Hrs MM : 300
DIRECTIONS

■ The set contains a total of 100 questions, Comprising Verbal Ability in English, General Awareness, Numerical Ability and Reasoning and
Military Aptitude Test.
■ Each correct question carry 3 Marks and there will be negative marking of 1 Mark for each incorrect attempt.
■ Total time duration will be 2 hrs (120 minutes).
■ No marks will be deducted for unattempted questions.

1. The cost price of 20 articles is the 6. Salaries of Ravi and Sumit are in 11. The average weight of 8 person’s
same as the selling price of x the ratio 2 : 3. If the salary of each increases by 2.5 kg when a new
articles. If the profit is 25%, then is increased by ` 4000, the new ratio person comes in place of one of
the value of x is? becomes 40 : 57. What is Sumit’s them weighing 65 kg. What might
(a) 15 (b) 16 (c) 18 (d) 25 salary? be the weight of the new person?
(a) ` 17000 (b) ` 20000 (a) 76 kg
2. In a certain store, the profit is 320%
(c) ` 25500 (d) ` 38000 (b) 76.5 kg
of the cost. If the cost increases by
7. In a mixture 60 L, the ratio of milk (c) 85 kg
25% but the selling price remains
constant, approximately what and water is 2 : 1. If this ratio is to (d) Data inadequate
percentage of the selling price is the be 1 : 2, then the quantity of water 12. Present ages of Sameer and Anand
profit? to be further added is are in the ratio of 5 : 4 respectively.
(a) 30% (b) 70% (a) 20 L (b) 30 L Three years hence, the ratio of their
(c) 100% (d) 250% (c) 40 L (d) 60 L ages will become 11 : 9 respectively.
3. If A = x% of y and B = y% of x, then 8. There is 60% increase in an amount What is Anand’s present age in
which of the following is true? in 6 yr at simple interest. What will years?
be the compound interest of (a) 24 yr
(a) A is smaller than B
` 12000 after 3 yr at the same rate? (b) 27 yr
(b) A is greater than B
(a) ` 2160 (b) ` 3120 (c) 40 yr
(c) None of these
(c) ` 3972 (d) ` 6240 (d) Cannot be determined
(d) If x is smaller than y, then A is
greater than B 9. The sum of ages of 5 children born 13. If a − b = 3 and a 2 + b 2 = 29, find
4. If 20% of a = b, then b% of 20 is the at the intervals of 3 yr each is 50 yr. the value of ab.
same as What is the age of the youngest (a) 10
(a) 4% of a (b) 5% of a child? (b) 12
(c) 20% of a (d) None of these (a) 4 yr (b) 5 yr (c) 15
(c) 8 yr (d) 10 yr (d) 18
5. Seats for Mathematics, Physics and
Biology in a school are in the ratio 10. An accurate clock shows 8 o’clock 14. What should come in place of both
5 : 7 : 8. There is a proposal to in the morning. Through how may x 162
x in the equation = ?
increase these seats by 40%, 50% degrees will the hour hand rotate 128 x
and 75% respectively. What will be when the clock shows 2 o’clock in (a) 12
the ratio of increased seats? the afternoon? (b) 14
(a) 2 : 3 : 4 (b) 6 : 7 : 8 (a) 144° (b) 150° (c) 144
(c) 6 : 8 : 9 (d) None of these (c) 168° (d) 180° (d) 196
*Based on Memory
14 A FCA T - I ~ Solved Paper 2020

15. A alone can do a piece of work in 25. (a) Biology (b) Chemistry 34. If Arun’s birthday is on May 25
6 days and B alone in 8 days. A and (c) Theology (d) Zoology which is Monday and his sister’s
B undertook to do it for ` 3200. birthday is on July 13. Which day of
With the help of C, they completed Directions (Q.Nos. 26-29) In each of the week is his sister’s birthday?
the work in 3 days. How much is to the following questions, select the
(a) Monday
be paid to C? related word/letter/figure from the
(b) Wednesday
(a) ` 375 (b) ` 400
given alternatives.
(c) Thursday
(c) ` 600 (d) ` 750 26. Cup is to coffee as bowl is to (d) Friday
16. A man complete a journey in 10 h. (a) Dish (b) Soup
35. Fill the blank in the middle of the
He travels first half of the journey (c) Spoon (d) Food series or end of the series.
at the rate of 21 km/h and second 27. Play is to actor as concert is to DEF, DEF2 , DE2F2 , .... D 2E2F3
half at the rate of 24 km/h. Find the (a) Symphony (b) Musician (a) DEF3
total journey (in km). (c) Piano (d) Percussion (b) D3 EF3
(a) 220 (b) 224 (c) 245 (d) 278 28. Palette Easel Brush (c) D2 E 3 F
17. Two trains running in opposite Text book Lesson plan? (d) D2 E 2 F2
directions cross a man standing on (a) Artist (b) Teacher 36. Read all the conclusions and then
the platform in 27 s and 17 s (c) Report Card (d) Paint decide which of the given
respectively and they cross each conclusions logically follows from
29.
other in 23 s. The ratio of their the given statements disregarding
speeds is? commonly known facts.
(a) 1 : 3 (b) 3 : 2 Statements Some noise are steel.
(c) 2 : 3 (d) None of these ? No steel is blood.
18. The value of Some steel is free.
. × 01
01 . × 01
. + 0.02 × 0.02 × 0.02 Conclusions
is I. Some noise are not blood
0.2 × 0.2 × 0.2 + 0.04 × 0.04 × 0.04
II. All blood is noise.
(a) 0.0125 (b) 25 (a) Only Conclusion I follows
(a) (b) (c) (d)
(c) 0.25 (d) 0.125 (b) Only Conclusion II follows
19. The ratio between the perimeter 30. Select the related word pair from (c) Either Conclusion I or II follows
and the breadth of a rectangle is the given alternatives. (d) Both Conclusions I and II follow
5 : 1. If the area of the rectangle is EXPLORE : DISCOVER 37. Which of the following diagrams
216 sq. cm what is the length of the (a) Read : Skim indicates the best relation between
rectangle? (b) Research : Learn Profit, Dividend and Bonus?
(a) 24 cm (b) 18 cm (c) Write : Print
(c) 16 cm (d) None of these (d) Think : Relate (a) (b)
5x 1
20. If = , then the value of 31. Pointing to a photograph, a man
2x + 5x + 1 3
2
said, "I have no brother or sister but
 1 that man’s father is my father’s
 x +  is (c) (d)
 2x son.” Whose photograph was it?
(a) 15 (b) 10 (a) His own
(c) 20 (d) 5 (b) His son 38. The diagram given below represents
(c) His father those students who play Cricket,
Directions (Q.Nos. 21-25) Three of the (d) His grandfather Football and Kabaddi.
words will be in the same classification,
the remaining one will not be. Your 32. A group of 1200 persons consisting Kabaddi
answer will be the one word that does of captains and soldiers is travelling
not belong in the same classification as in a train. For every 15 soldiers R
the others. there is one captain. The number of
captains in the group is? S T U
21. (a) Parsley (b) Basil (a) 70 (b) 75 (c) 80 (d) 85
Football V
(c) Dill (d) Mayonnaise
33. I. Mara runs faster than Gail.
22. (a) Tulip (b) Rose Cricket
II. Lily runs faster than Mara.
(c) Bud (d) Daisy
III. Gail runs faster than Lily. Study the diagram and identify the
23. (a) Rye If the two statements are true, the students who play all the three
(b) Sourdough third statement is games.
(c) Pumpernickel (a) True (a) P + Q + R
(d) Loaf (b) False (b) V + T
(c) Uncertain (c) S + T + V
24. (a) Scythe (b) Knife
(d) None of the above (d) S
(c) Pliers (d) Saw
A FCA T - I ~ Solved Paper 2020 15

39. Which of the following diagrams 43. Question Figure 46. Question Figures
indicates the best relation between
Hockey, Football and Cricket? × + – × + –

(a) (b) – + ×

Answer Figures Answer Figures


– =
(c) (d)
(a) + × (b) × +

40. In an organisation of pollution


control board, engineers are (a) (b) (c) (d)
(c) × + (d) × +
represented by a circle, legal experts
by a square and environmentalist
Directions (Q.Nos. 44 and 45) In the –
following questions there is a problem
by a triangle. Who is most
represented in the board as shown figure, a part of which is missing. Direction (Q.No. 47) : There are two
in the following figure? Observe the answer figures (a), (b), (c), sets of figure given. There is a definite
(d) and find out the answer figure relationship between first two figures.
which without changing the direction, Establish a similar relationship
fits in the missing part of the problem between third and fourth figures by
figure in order to complete the pattern selecting a suitable figure from answer
in the problem figure. that would replace the question mark.
44. 47. Question Figurs
?
(a) Environmentalists
(b) Legal Experts
(c) Engineers with legal background
(d) Environmentalists with Answer Figures
Engineering background (a) (b)

Directions (Q.Nos. 41-42) Following


four figures (a), (b), (c) and (d) have
(a) (b) (c) (d)
been given in each question. Of these (c) (d)
four figures, three figures are similar in Direction (Q.No. 48) In question, one
some way and one figure is different. part of a geometrical figure (Triangle,
Select the figure which is different. 45. Square, Circle) is as question figure
41. and the other one is among the four
answer figures (a), (b), (c) and (d). Find
(a) (b)
the figure on the right side that
completes the geometrical figure.
?
48. Question Figure
(c) (d)

(a) (b)
42.
Answer Figures
(a) RUN (b) UNR
(a) (b)
(c) (d)

(c) NKU (d) RNU


Direction (Q. No. 46) There are three (c) (d)
question figures and the space for the
Direction (Q. No. 43) Question figure fourth figure is left blank. The question
is followed by four alternatives (a), (b), figures are in a series. Find out one Direction (Q.No. 49) There is a
(c) and (d). Select the figure from figure from among the answer figures question figure and four answer figures
Answer figure which is exactly similar given which occupies the blank space marked (a), (b), (c) and (d) are given.
to question figure. for the fourth figure and completes the Select the answer figure which is
series. exactly the mirror image of the question
figure when the mirror is held at XY.
16 A FCA T - I ~ Solved Paper 2020

49. Question Figure Directions (Q. Nos. 57-60) In each of society is a necessary condition to hold
X the question part, find out which part the State to account. It is not about one
has an error. If there is no mistake, the seemingly revolutionary protest, even if
answer is ‘No error’. the well-meaning antagonists of the
57. (a) The poor CAA …66… believe so.
Y When violence and lawlessness spill
(b) is oppressed
Answer Figures over into the streets and to constrain
(c) all around the world
our work-a-day lives, as it has across
(d) No error
India, when the police are routinely
(a) (b) 58. (a) Mohan and me are challenged by those on the wrong side
(b) going to learn of the …67…; when the police force is
(c) many tricks overworked and overwhelmed by
(d) No error political subterfuge, our society will be
(c) (d) on the brink. As a society, we are
59. (a) We shan’t (b) invite them
currently engaged in a negative-sum
(c) won’t we? (d) No error
game. The rule of law cannot, unlike
Direction (Q.No. 50) Question figure 60. (a) The USA is aiming at riots, be engineered. It can only be
is followed by four answer figures (b) double its bilateral trade practiced if we understand that liberty
namely (a), (b), (c) and (d). Find out the (c) with India by 2015 has to be balanced with equality; and
correct answer figure from the given (d) No error pluralism entails a measure of negative
alternatives in which question figure is liberty.
hidden.
Directions (Q.Nos. 61-64) In the
following questions, the sentence given 65. (a) establish (b) establishing
50. Question Figure with blank to be filled in with an (c) established (d) None of these
appropriate word. Select the correct
66. (a) genuine (b) genuinely
alternative out of the four and indicate
it by selecting the appropriate option. (c) wrong (d) right

61. A leaderless police force ……… the 67. (a) face (b) law
Answer Figures path of least resistance since there (c) police (d) None of these
was no one in authority to spur it to Directions (Q.Nos. 68-70) In the
action. following questions, four alternatives
(a) take (b) took are given for idioms/phrases. Choose
(c) tread (d) negotiate the one that best expresses the meaning
(a) (b) (c) (d)
62. This has provided a ray of hope and of the given idiom/phrase.
Directions (Q.Nos. 51-56) In the there is no dearth of data as far as 68. What is the meaning of Idiom ‘Get
following questions, choose the correct cervical cancer and prevention are into hot water’?
synonym/antonym. ……… (a) To swim in the deep water
51. What is the antonym of Naive? (a) concerned (b) forgotten (b) To think over difficult problem
(a) Unsophisticated (c) discernible (d) detectable (c) To get into difficult situation
(b) Artless 63. The program has ……… a DNA (d) To discuss a burning topic
(c) Experienced based HPV test and offered it for 69. What is the meaning of Idiom ‘rain
(d) Inexperienced free to these women. cats and dogs’?
52. What is the antonym of Cajole? (a) relate (b) deploying (a) Rain heavily
(a) Persuade (b) Wheedle (c) deployed (d) refrain (b) To fight in rain
(c) Coax (d) Bully 64. There were four instances when (c) Cats and dogs fight
Delhi Police gave short shrift to the (d) Rain water wastage
53. What is the synonym of Parochial?
(a) Conventional laws of the land and, indeed, to the 70. What is the meaning of Idiom ‘At
(b) Broad minded very Constitution they are ……… to daggers drawn’?
(c) Cosmopolitan uphold. (a) To throw daggers
(d) Liberal (a) swear (b) sworn (b) To be bitterly hostile
(c) sword (d) firm (c) To be confused
54. What is the antonym of Sporadic?
Directions (Q.Nos. 65-67) In these (d) To think deeply
(a) Uneven (b) Spasmodic
(c) Frequent (d) Intermittent questions, in the following passage Directions (Q.Nos. 71-75) : A passage
some of the words have been left out. is given with questions following it.
55. Antonyms of intrinsic? Read the passage carefully and choose Read the passage carefully and choose
(a) Innate (b) Natural the correct out of four alternatives. the best answer to each question out of
(c) Acquired (d) Built in
For the rule of law to be well …65… the four alternatives.
56. What is the synonym of inquisitive? and enforced, we need to start at the My brother, David, was always close to
(a) Curious (b) Intrigued very beginning-mobilising our local our grandmother. Both of them shared
(c) Agog (d) All of these communities. A strong, mobilized, civil
A FCA T - I ~ Solved Paper 2020 17

a love of Mother Nature and of food 78. Who among the following first 90. Dr. M.S. Subbulakshmi related
that they had grown themselves. person to take hat-trick in a World with
Whenever his schedule permitted, he Cup? (a) Hindustani Music
would drop in for a short visit and a (a) Chetan Sharma (b) Dhrupad
cup of coffee. One day, when he found (b) Hardik Pandya (c) Kathak
no one home, he left a chunk of dirt on (c) Md. Shami (d) Carnatic Classical Music
her porch. This started what was later (d) Jasprit Bumrah 91. Which case Ram Prasad Bismil
to be known as his ‘‘calling card”.
79. Capital of Barbados related?
Grandmother would come home
(a) Andorra la Vella (a) Alipore conspiracy
occasionally and instantly know that
(b) Manama (b) Kakori conspiracy
Dave had been by when she spotted the
(c) Bridge town (c) Kanpur conspiracy
chunk of dirt on her porch. Although
(d) Nassau (d) Meerut conspiracy
Grandmother had a poor upbringing in
Italy, she managed to do well in the 80. How many number of bodies in 92. Akbar Buland Darwaza victory over
United States. She was always healthy UNO? (a) Gujarat
and independent and enjoyed a (a) Two (b) Four (c) Six (d) Seven (b) Asirgarh fort
fulfilling life. Recently she had a stroke (c) Panipat battle
81. Article 343 is related to which?
and died. Everyone was saddened by (d) Chittorgarh fort
(a) Official languages
her death. David was disconsolate. His (b) Election Commission 93. What is the capital of Ghana?
life-long friend was now gone. (c) National Emergency (a) Tirana
71. Which of the following is the (d) Supreme Court (b) Tbilisi
synonym of the word ‘upbringing’? (c) Accra
82. Who among the following
(a) Devastated (b) Hilarious introduced Preamble in Constituent (d) Addis Ababa
(c) Raising (d) Surrounding Assembly? 94. Which of the following Mountain
72. The adjective form of the word (a) Dr. B R Ambedkar passes are not in India?
‘saddened’ is (b) Dr. Rajendra Prasad (a) Rohtang Pass
(a) sadly (b) sad (c) Sardar Ballabhbhai Patel (b) Khyber Pass
(c) suddenly (d) sudden (d) Jawaharlal Nehru (c) Baralachala Pass
83. Irani Cup is related to which of the (d) Lipulekh Pass
73. What was condition of
grandmother earlier? game? 95. Who wrote the book ‘‘Gone with
(a) Rich in Italy but poor in the (a) Badminton (b) Football the Wind’’?
United states (c) Hockey (d) Cricket (a) Anand Neelakantan
(b) In the United States but is now in 84. Tropic of Cancer does not pass (b) Gita Sahgal
Italy through which country? (c) Douglas Noel Adams
(c) Poor earlier but became rich later (d) Margaret Mitchell
(a) Iran (b) India
on (c) Algeria (d) Egypt 96. Nautical Mile is unit of
(d) Rich earlier but now poor (a) Mass
85. Metallic coin first used in India in
74. Grandmother enjoyed a ……… life. (a) Bihar and eastern U.P.
(b) Work
(a) healthy but sickly (c) Length
(b) Southern India
(b) good and healthy (d) Energy
(c) Western India
(c) rich but sickly (d) Central India 97. Dravidian Art associated with
(d) poor and healthy (a) Sangam period
86. Cryolite is mainly found in which
75. Grandmother’s death made country? (b) Mughal period
everyone (a) Canada (b) Iceland (c) Gupta period
(a) sad including David (c) Greenland (d) Norway (d) Maurya period
(b) disconsolate excluding David 98. Where is UNIDO headquarters?
(c) happy and disconsolate 87. Hargobind Khurana won the model
prize in (a) New York (b) Brussels
(d) sad excluding David (c) Vienna (d) Rome
(a) Dermatology (b) Physiology
76. Who among the following writer of (c) Nephrology (d) Oology 99. How many players are there in
the ‘Mother India’ book? Kabaddi?
(a) Hans Aanrud 88. Prime Meridian is also known as
(a) Arctic Meridian (a) Six (b) Eleven
(b) Katherine Mayo (c) Five (d) Seven
(c) Rachel Aaron (b) Latin Meridian
(d) Ben Aaronovitch (c) Greenwich Meridian 100. Which rivers flow into the Arabian
(d) Antarctic Meridian Sea?
77. Mallet term is related to which (a) Ganga
sports? 89. Retreating monsoon occurs during
which month? (b) Narmada
(a) Cricket (b) Tennis (c) Tapti
(c) Hockey (d) POLO (a) November (b) September
(c) October (d) December (d) Both (b) and (c)
18 AFCAT-I ~ Solved Paper 2020

Answers
1. (b) 2. (b) 3. (c) 4. (a) 5. (a) 6. (d) 7. (d) 8. (c) 9. (a) 10. (d)
11. (c) 12. (a) 13. (a) 14. (a) 15. (b) 16. (b) 17. (b) 18. (d) 19. (b) 20. (d)
21. (d) 22. (c) 23. (a) 24. (c) 25. (c) 26. (b) 27. (b) 28. (c) 29. (c) 30. (b)
31. (b) 32. (b) 33. (b) 34. (a) 35. (d) 36. (a) 37. (b) 38. (d) 39. (b) 40. (d)
41. (b) 42. (c) 43. (c) 44. (a) 45. (b) 46. (c) 47. (b) 48. (a) 49. (c) 50. (c)
51. (c) 52. (d) 53. (a) 54. (c) 55. (c) 56. (d) 57. (b) 58. (a) 59. (c) 60. (b)
61. (b) 62. (a) 63. (c) 64. (b) 65. (c) 66. (b) 67. (b) 68. (c) 69. (a) 70. (b)
71. (c) 72. (b) 73. (c) 74. (b) 75. (a) 76. (b) 77. (d) 78. (a) 79. (c) 80. (c)
81. (a) 82. (d) 83. (d) 84. (a) 85. (a) 86. (c) 87. (b) 88. (c) 89. (c) 90. (d)
91. (b) 92. (a) 93. (c) 94. (b) 95. (d) 96. (c) 97. (a) 98. (c) 99. (d) 100. (d)

Hints and Solutions


1. (b) Let the cost price of 1 article = ` 1 4. (a) Given, 7. (d) Given, quantity of mixture = 60 L
Let the cost price of 20 articles = ` 20 20% of a = b 2
20a Quantity of milk = 60 × = 40 L
Let the cost price of x article = ` x ⇒ =b (2 + 1)
Selling price of x article = Cost price of 100 1
a Quantity of water = 60 × = 20 L
20 articles ⇒ =b … (i) (2 + 1)
= ` 20 5
20b 20 a Let the quantity of water to be further
∴ Profit = Selling price − Cost price Now, b% of 20 = = × added be x litres.
= 20 − x 100 100 5
According to the question,
profit [from Eq. (i)]
Q Profit % = × 100 4
40
=
1
⇒ 20 + x = 80
cost price = a = 4% of a 20 + x 2
20 − x 100
⇒ 25 = × 100 ∴ x = 80 − 20 = 60 L
x 5. (a) Let the number of seats for
⇒ x = (20 − x) × 4 Mathematics, Physics and Biology be 8. (c) Let Principal (P) = ` 100
⇒ x = 80 − 4x 5 x, 7x and 8 x respectively. 60
Then, SI = 100 × = ` 60
⇒ 5x = 80 Now, increased seats for Mathematics 100
140 and T = 6 yr
∴ x = 16 = 5x × = 7x
100 We know that,
2. (b) Let cost price = ` 100 Increased seats for Physics P× R×T 100 × R × 6
320 150 21x SI = ⇒ 60 =
Then, profit = 100 × = ` 320 = 7x × = 100 100
100 100 2 ⇒ 6R = 60
Selling price = Cost price + Profit Increased seats for Biology ∴ R = 10%
= 100 + 320 175
= 8x × = 14 x Now, P = ` 12000, R = 10%, T = 3 yr
= ` 420 100 We know that,
125 ∴The ratio of increased seats
New cost price = 100 × = ` 125  T 
CI = P  1 +
R 
100 = 7x :
21x
: 14x  − 1
QSelling price remains constant 2   100 
∴New profit = selling price − cost price = 14 : 21 : 28 = 2 : 3 : 4  3 
= 12000  1 +
10 
= 420 − 125 = ` 295 6. (d) Let Ravi’s salary = 2x  − 1
  100  
∴Required percentage Sumit’s salary = 3 x
295   11 3 
= × 100 According to the question, = 12000    − 1
420 2x + 4000 40
=   10 
= 70. 23 3x + 4000 57  1331 − 1000
≈ 70% = 12000 = 12 × 331
⇒114 x + 228000 = 120 x + 160000  1000 
3. (c) Given, A = x% of y ⇒ 120 x − 114 x = 228000 − 160000 = ` 3972
xy ⇒ 6x = 68000
A= ...(i) 9. (a) Let the ages of children be
100 68000 34000 x,(x + 3), (x + 6), (x + 9) and (x + 12) yr.
∴ x= =
and B = y% of x 6 3 According to the question,
xy ∴Sumit’s increased salary = 3x + 4000 x + x + 3 + x + 6 + x + 9 + x + 12
B= ...(ii)
100 3 × 34000 = 50
= + 4000
From Eqs. (i) and (ii), 3 ⇒ 5x + 30 = 50 ⇒ 5x = 20 ⇒ x = 4
∴ A=B = ` 38000 ∴The age of the youngest child = 4 yr
AFCAT-I ~ Solved Paper 2020 19

10. (d) Time difference = 2 pm − 8 am 16. (b) Let the total journey = d km 20. (d) Given,
5x 1
=6h According to the question, =
QHour hand rotate in 1 h = 30° d d 2x + 5x + 1
2
3
∴Hour hand will rotate in 6 h 2 + 2 = 10 Q Time = distance  ⇒ 15x = 2x 2 + 5x + 1
 speed 
= 6 × 30° 21 24  2x 2 5x 1
d 8 + 7 ⇒ 15 =+ +
= 180° ⇒  = 10 x x x
11. (c) Let the average weight of eight 2  168  1
168 × 10 × 2 ⇒ 15 = 2x + 5 +
persons = x ∴ d= x
The weight of the new person = y 15

1
2x + = 15 − 5 = 10
According to the question, = 224 km x
8x − 65 + y = 8 (x + 2 .5) 17. (b) Let the speeds of first and second 1 1  10
⇒  2x +  =
⇒ 8x − 65 + y = 8x + 20 train be x and y respectively. 2 x 2
⇒ y = 65 + 20 = 85 Length of first train = 27x ∴ x+
1
=5
∴The Weight of the new person = 85 kg [QDistance = Speed × Time] 2x
12. (a) Let Sameer’s present age = 5x years Length of second train = 17 y 21. (d) All except Mayonnaise are different
Time taken to cross each other types of herbs.
Anand’s present age = 4x years Distance
According to the question, = 22. (c) Except bud all others are different
Speed
5x + 3 11 flowers.
= 27x + 17 y
4x + 3 9 23 = 23. (a) Except Rye all others are different
x+ y
⇒ 45x + 27 = 44x + 33 types of bread.
⇒ 23x + 23y = 27x + 17 y
⇒ 45x − 44x = 33 − 27 24. (c) Except pliers all other tools are used
⇒ 27x − 23x = 23y − 17 y
x =6 for cutting.
⇒ 4x = 6 y
∴Anand’s present age = 4 × 6 = 24 yr x 6 3 25. (c) Except Theology all others are
⇒ = =
13. (a) Given, a − b = 3 and a2 + b2 = 29 y 4 2
different branches of Science, but
theology is the study of religion.
We know that, ∴The ratio of their speed = 3 : 2
(a − b)2 = a2 + b2 − 2ab 26. (b) As, a cup contains coffee, similarly a
0.1 × 0.1 × 0.1 + 002. × 002
. × 002
.
18. (d) bowl contains soup.
⇒ (3) = 29 − 2ab
2
0. 2 × 0. 2 × 0. 2 + 004
. × 004
. × 0.04
27. (b) As, an actor acts in a play, similarly,
⇒ 2ab = 29 − 9 = 20 (0.1)3 + (002
. )3
= a musician performs in a concert.
∴ ab = 10 (0. 2)3 + (004
. )3 28. (c) Palette, Easel and Brush are things
x 162 (0.1)3 + (002
. )3
14. (a) Given, = = used by an artist. Similarly, Textbook,
128 x 23 [(0.1)3 + (002
. ) 3] lesson plan and report card are used by a
⇒ x 2 = 162 × 128 1 teacher.
=
⇒ x 2 = 162 × 128 8 29. (c) As, Scissor and knife are used for
= 81 × 2 × 64 × 2 = 0.125 cutting, similarly pitcher and watering
⇒ x 2 = 9 × 2 × 8 = 144 can are used for watering.
19. (b) Let the length and breadth of
rectangle be l and b respectively, 30. (b) One explores to discover, similarly,
⇒ x = 144
According to the question, one researches to learn.
∴ x = 12
Perimeter 5 31. (b)
1 = Father
15. (b) A’s 1 day’s work = breadth 1
6 2 (l + b) 5 Son
1 ⇒ =
B’s 1 day’s work = b 1
8
1 ⇒ 2 l + 2b = 5b Father Man
(A + B + C)’s 1 day’s work = ⇒ 2l = 3b
3 Self
2l
C’s 1 day’s work ⇒ b= … (i)
3 Photograph
1 1 1 8− 4− 3 1
= − − = = Area of rectangle = 216
3 6 8 24 24 Clearly, the man is the father of the man
∴(A’s wages) : (B’s wages) : (C’s wages) ⇒ l × b = 216
2l in the photograph. So, he was pointing
1 1 1
= : : = 4 : 3 :1 ⇒ l× = 216 [Q from Eq. (i)] towards his son’s photograph.
6 8 24 3
216 × 3 32. (b) A small group contains 15 soldiers
∴C’s share ⇒ l2 =
2 and one captain. So, a small group
1
= 3200 × = 324 contains = 15 + 1 = 16 members
(4 + 3 + 1) ∴Total number of captains in a group of
1 ⇒ l = 18
= 3200 × = ` 400 1200 members = 1200 ÷ 16 = 75
∴The length of the rectangle = 18 cm
8
20 AFCAT-I ~ Solved Paper 2020

33. (b) From I and II Statements, 45. (b) Answer figure (b) will complete the 59. (c) In the given sentence, the question
Lily > Mara > Gail pattern of the problem figure. tag is not correct. The correct question
Here, Gail runs slower than Lily. 46. (c) The main figure i.e triangle with a tag should be used here is ‘shall we’.
So, the third statement is false. slant line is same in each alternate Hence, option (c) is the correct answer.
34. (a) Total number of odd days from May figure. The symbol ×, + and − are 60. (b) Here, the word ‘double’ should be
25 to July 13 = 6 + 30 + 13 moving one side in clockwise direction in replaced with ‘doubling’, to make the
= 49 ⇒ 49 ÷ 7 = 0 each step. Clearly, answer figure (c) will sentence grammatically correct.
So, the day on July 13 complete the series. 61. (b) According to the given sentence, the
= Monday + 0 day 47. (b) Second figure is the mirror image of word ‘took’ is correct usage here.
= Monday first figure. So, the answer figure (b) will 62. (a) The word ‘concerned’ is appropriate
complete the second pair. here to fill the given blank.
35. (d) The pattern of the series is as follows
DEF, DEF2 , DE2 F2 , D2 E2 F2 , D2 E2 F 3 48. (a) Answer figure (a) will complete the 63. (c) ‘Deployed’ is the correct alternative
Here, the letters remain same and the given geometrical figure. here after the word ‘has’.
subscript numbers follow the below 49. (c) Answer figure (c) is the correct mirror 64. (b) Here, ‘sworn’ should be used in the
pattern image of the given question figure. given blank.
111, 112, 122, 222, 223 X
65. (c) The word ‘established’ is the correct
36. (a) According to the given statements, usage here.
Noise Steel Blood 66. (b) In the given sentence, the word
‘genuinely’ is the appropriate to be used
Free Y
before the verb ‘believe’.
50. (c) The given question figure is 67. (b) ‘law’ is the appropriate word here to
Conclusions I. ü embedded in answer figure (c). be used in the given blank.
II. û
68. (c) The idiom ‘get into hot water’ means
37. (b) Dividend and Bonus are different to get into difficult situation.
parts of profit.
69. (a) The given idiom ‘rain cats and dogs’
Profit means rain heavily.
51. (c) Naive means showing a lack of
Dividend Bonus 70. (b) The given idiom ‘at daggers drawn’
experience, wisdom or judgement. So,
among the given options, ‘Experienced’ means to be bitterly hostile.
would be its correct antonym. 71. (c) Upbringing means early training;
38. (d) Clearly, S represents the students,
52. (d) Cajole means to persuade by flattery especially a particular way of bringing up
who play all the three games.
or promises. So, ‘Bully’ would be its a child. So, ‘raising’ would be its correct
39. (b) Hockey, football and cricket are synonym. Raising means to cause
correct antonym as bully means seek to
different sports. something to increase or become better,
harm, intimidate or coerce.
Hockey higher.
53. (a) Parochial means having a limited or
Cricket narrow outlook or scope. So, 72. (b) The adjective form of the word
Football conventional would be its correct ‘saddened’ is ‘sad’.
synonym as its also means the same. 73. (c) The earlier condition of grandmother
40. (d) was poor but become rich later on.
54. (c) Sporadic means occurring at irregular
Legal intervals or only in a few places; 74. (b) Grandmother enjoyed a good and
Experts scattered or isolated. So, ‘frequent’ would healthy life.
Engineers be its correct antonym. 75. (a) Grandmother’s death made everyone
55. (c) Intrinsic means belonging naturally or sad including David.
essential. So, ‘acquired’ would be its 76. (b) Katherine Mayo is the writer of book
Environmentalist correct antonym. ‘Mother India’. In this book, she attacked
Environmentalists with engineering 56. (d) Inquisitive means having or showing Hindu society and religion and the
background is most represented in the an interest in learning things. The given culture of India. She was an American
board. words curious, intrigued, agog are all the historian. She was opposed to Indian
synonyms of the given word. Independence from British rule.
41. (b) Except figure (b), in all other figures
the number of circles is equal to the Hence, option (d) is its correct answer. 77. (d) The Mallet term is associated with the
number of sides in the innermost shape. 57. (b) Here, the word ‘Poor’ is plural here. POLO. It is an implement with a long
So, the helping verb should also be in handle and a head like a hammer used
42. (c) Except (c), all other words have same
plural form. Hence, ‘are oppressed’ is hit a ball. Polo is a horseback mounted
letters i.e. R, U and N.
used in the given sentence. team sport. The game is played by two
43. (c) Answer figure (c) is exactly similar to opposing teams with the objective of
58. (a) Here the word ‘me’ is in objective
question figure. scoring goals by using a long-handled
case, it should be replaced with
44. (a) Answer figure (a) will complete the nominative case ‘I’, to make the given wooden mallet to hit a small hard ball
pattern of the problem figure. sentence error free. through the opposing team’s goal.
AFCAT-I ~ Solved Paper 2020 21

78. (a) Chetan Sharma was the first man to Constitution by the 42nd Constitutional foot of Pikes Peak in Colorado, Falcon
take a hat-trick in the World Cup. He Amendment Act of 1976. Quarry near Montreal in Quebec, Canada
achieved this feat in the World Cup of 83. (d) Irani Trophy (now known as Z. R. and also in Miask, Russia.
1987 (Held in Nagpur, Maharashtra). He Irani Cup) is related to cricket. The 87. (b) Har Gobind Singh Khurana won the
dismissed three New Zealand batsmen in tournament was conceived during the Nobel Prize in Physiology or Medicine in
succession (all bowled). 1959-60 season to mark the completion 1968 ‘for the interpretation of the genetic
He was as a fast bowler. In his career he of 25 years of the Ranji Trophy code and its function in protein
played 23 Test and 65 ODI matches. championship. synthesis’.
79. (c) The capital of Barbados is Bridgetown. The fixture is played annually between He was renowned biochemist famous for
Barbados is an island country in the the incumbent Ranji Trophy winners his work in the field of genetics and
Lesser Antilles of the West Indies, in the and the Rest of India Team. The current DNA. He was the first person to
Caribbean region of North America. champion (2018-19) of this Trophy is demonstrate the role of nucleotides in
It is inhabited by Kalinago people since Vidarbha. protein synthesis.
the 13th century, and prior to that by 84. (a) The Tropic of Cancer does not pass He was successful in constructing the
other Amerindians. In November 1966, through Iran. It is the most northerly first ever artificial gene in 1972.
Barbados became an independent state circle of latitude (23°26'11.7'') on Earth 88. (c) Prime Meridian is also known as
(was colony of Britain). at which the Sun can be directly
Greenwich Meridian because it passes
overhead. There are 16 countries, 3
80. (c) The United Nations Organisation through Greenwich, where the British
continents and 6 water bodies’ lies on
(UNO) consist of six members is an Royal Observatory is located. Its value is
Tropic of Cancer.
intergovernmental organisation that aims 0° longitude and from it, we count 180°
to maintain international peace and The countries are as follows- Mexico, eastward as well as 180° westward.
Bahamas (North America), Nigeria,
security, develop friendly relations among The Prime Meridian and 180° meridian
Algeria, Mauritania, Egypt, Libya, Mali,
nations, achieve international divide the Earth into two halves, the
Western Sahara (Africa), Myanmar,
cooperation, and be a centre for Eastern hemisphere and the Western
Omen, Bangladesh, India, Saudi Arabia,
harmonising the actions of nations. China, United Arab Emirates and hemisphere. When the Prime Meridian of
It was established on 24th October, 1945 Taiwan (Asia). Greenwich has the Sun at the highest
(as per the UN Charter). Six bodies 85. (a) The first metallic coin was used in point in the sky, all the places along this
includes General Assembly, Secretariat, the Bihar and Eastern Uttar Pradesh meridian will have mid-day or noon.
International Court of Justice, Security region by the Indo-Greeks. The coins 89. (c) Retreating monsoons occur during the
Council, Economic and Social Council had royal portraits on the obverse and month of October. Retreating means
and UN Trusteeship Council. Greek deities (Zeus, Apollo and Athena) withdrawal. So, withdrawal of
81. (a) Article 343 of the Indian Constitution on the reverse. South-West monsoon winds from skies of
deals with the official language. There were gold, silver, copper and North India during the months of October
It states that the official language of the nickel coins. The Indo-Greek kingdom and November is known as retreating
Union shall be Hindi in Devanagari script. was ruled by over 30 Hellenistic (Greek) monsoon.
Part XVII of the Indian Constitution deals kings in the North-West and North India The withdrawal is gradual and takes
with the official languages in Articles 343 from the 2nd century BC to the about three months. With the onset of
to 351. beginning of the first century AD. retreating monsoon, skies become clearer
The Eighth Schedule to the Constitution The kingdom started when and clouds disappear. The
consists of the 22 languages. Graeco-Bactrian king Demetrius (son of disappearance of clouds makes the
Other important Articles Euthydemus I) invaded India around climate of various places hotter gradually.
National Emergency- 352 180 BC. He conquered southern 90. (d) M. S. Subbulakshmi was related with
Election Commission- 324 Afghanistan and parts of Punjab. the Carnatic Classical Music. She was
Supreme Court- 124- 147 86. (c) Cryolite is mainly found in from Madurai, Tamil Nadu. She was the
82. (d) The Preamble of Indian Constitution is Greenland. Its chemical formula is first musician ever to be awarded the
based on the Objective Resolution moved Na 3AlF 6 (Sodium hexafluoroaluminate). Bharat Ratna, India’s highest civilian
by Jawaharlal Nehru in the Constituent Cryolite was first described in 1798 by honour. She was the first Indian who
Assembly on 13th, December, 1946. Danish physician Peder Christian performed in United Nations General
Abildgaard. It was historically used as an Assembly in 1966.
The Resolution was adopted on 22nd
January, 1947. It is a brief introductory ore of aluminium and later in the 91. (b) Ram Prasad Bismil is related with
statement that sets out the guiding electrolytic processing of the Kakori Conspiracy. It was an armed
purpose and principles of the aluminium-rich oxide ore bauxite. robbery which took place on 9th August,
Constitution. The term ‘secular’ was Small deposits of cryolite have also been 1925, on a train in central Uttar
added to the Preamble of the Indian reported in some areas of Spain, at the Pradesh. It was executed by Bismil,
Ashfaqullah Khan, Chandrashekhar
22 AFCAT-I ~ Solved Paper 2020

Azad, Rajendra Lahiri and many others. important trade route between Central Sangam Literature reflects the earliest
All of them were members of newly Asia and the Indian subcontinent and a literature of South India. It is a group of
formed Hindustan Republican vital strategic military choke point for texts in old Tamil.
Association, a revolutionary organisation, various states that came to control it. 98. (c) The United Nations Industrial
later renamed as Hindustan Socialist Other Passes Development Organisation (UNIDO) is
Republican Association (HSRA). Lipulekh Pass- Uttarakhand the specialised agency of the United
92. (a) Mughal Emperor Akbar built Buland Rohtang Pass- Himachal Pradesh Nations that promotes industrial
Darwaza (Door of victory) to Baralachala Pass- Himachal Pradesh development for poverty reduction,
commemorate his victory over Gujarat. It 95. (d) ‘Gone with the Wind’ is a novel inclusive globalisation and environmental
is the main entrance to Jama Masjid at written by American writer Margaret sustainability.
Fatehpur Sikri (Agra). Mitchell. It was published in 1936. It is also a member of UNDP. It is
It is the highest gateway in the world and It depicts the struggles of young Scarlett headquartered at the UN Office in
is an example of Mughal architecture. It O'Hara, the spoiled daughter of a Vienna, Austria. It was formed on
is made of red and buff sandstone, well-to-do plantation owner, who must 17th November, 1966.
decorated by white and black marble and use every means at her disposal to claw 99. (d) There are seven player in Kabaddi. It
is higher than the courtyard of the her way out of poverty following is a contact team sport played between
mosque. Sherman’s destructive “March to two teams.
93. (c) The capital of Ghana is Accra. It is a the Sea”. It is popular in the Indian Subcontinent
country along the Gulf of Guinea and the 96. (c) A nautical mile is a unit of length. and other surrounding Asian countries. It
Atlantic Ocean, in the subregion of West One nautical mile is equal to 1852 is the national sport of Bangladesh.
Africa. It has a unitary constitutional metres (about 1.15 miles). It is used for 100.(d) Correct answer is option (d).
democracy led by a President who is measurement in air, marine and space
both head of state and head of the Rivers flowing in Rivers flowing in
navigation, and for the definition of
government. Arabian Sea Bay of Bengal
territorial waters.
94. (b) The Khyber Pass is a mountain pass 97. (a) Dravidian Art is associated with the Narmada, Tapi, Ganga, Godavari,
in the Khyber Pakhtunkhwa province of Mahi, Sabarmati, Krishna, Pennar,
Sangam period. This period was from the Mandovi river, Kaveri
Pakistan. It connects the town of Landi first century BCE to the end of the 2nd Periyar river,
Kotal to the Valley of Peshawar at century CE in South India. Bharathapuzha River
Jamrud by traversing part of the Spin and Pamba river
Three dynasties ruled during the Sangam
Ghar Mountains.
period are the Cheras, Cholas and
Throughout history, it has been an
Pandyas.
AFCAT-II ~ Solved Paper 2019 03

INDIAN AIR FORCE


AFCAT-II

Solved Paper 2019


Time : 2 Hrs MM : 300
INSTRUCTIONS
■ The set contains a total of 100 questions, Comprising Verbal Ability in English, General Awareness, Numerical Ability and Reasoning and
Military Aptitude Test.
■ Each correct question carry 3 Marks and there will be negative marking of 1 Mark for each incorrect attempt.
■ Total time duration will be 2 hr (120 minutes).
■ No marks will be deducted for unattempted questions.

Directions (Q. Nos. 1-3) In these 7. He ...... a map of the caribbean next 11. I did not want to listen to him,
questions, choose the word similar in to the letter, to trace the ship’s ...... . (a)/but he was adamant and
meaning to the word given. (a) unfolded, plan (b)/discussed about the matter.
(b) unfurled, course (c)/No error (d)
1. Allegiance
(a) Treachery (b) Appreciation (c) unrolled, course 12. Please note (a)/that the interview
(c) Loyalty (d) Careless (d) used, course for the post (b)/shall be held on
2. Abash 8. She noisily ........... the water, ........... 15th June, 2019 between 10.00 a.m.
half of it onto her chin. to 2.00 p.m. (c)/No error (d)
(a) Ashamed (b) Credulous
(a) drank, dropping 13. The legendary hero (a)/laid down
(c) Unbashed (d) Undaunted
(b) inhaled, spilling his precious life (b)/ for our
3. Altruism (c) gurgled, slopping country. (c)/ No error (d)
(a) Selfish (b) Weak (d) gulped, trailing
(c) Attractive (d) Selflessness 14. A year has been gone by (a)/
9. It is ............ for every listed since he left us (b)/and whenever
Directions (Q. Nos. 4-6) In the company to .......... its annual hear any news of him. (c)/
following questions, choose the word financial statements with the No error (d)
opposite in meaning to the given word. registrar of companies.
(a) necessary, lodge Directions (Q. Nos. 15-18) In these
4. Adroit questions, four alternatives are given
(a) Skilful (b) Competent (b) binding, pay
(c) obligatory, file for the idiom/phrase. Choose the
(c) Canny (d) Foolish
(d) possible, remit alternative, which best expresses the
5. Vague meaning of the idiom/phrase.
(a) Clear (b) Precise 10. ............ dark clouds had been piling
up all afternoon in .......... layers. 15. A little gush of gratitude
(c) Firm (d) Yielded
(a) Flourishing, threatening (a) gradual recovery
6. Exodus (b) Healing, menacing (b) friendly feeling
(a) Influx (b) Home-coming (c) Festering, ominous (c) excessive labour
(c) Return (d) Restoration (d) Decaying, promising (d) excessive enthusiasm
Directions (Q. Nos. 7-10) Sentences Directions (Q. Nos 11-14) Read each 16. To lose ground
are given with blanks to be filled it
sentence to find out which part of the (a) to become less powerful
with an appropriate word(s). Four
sentence has an error. If you find no error (b) to become less popular
alternatives are suggested for each
in the sentence, your response should be (c) to lose foundation
question. Choose the correct alternative
indicated as option (d) No error. (d) to be without a leader
out of the four alternatives.
04 AFCAT-II ~ Solved Paper 2019

17. To fall back on How to express your ideas so that the 31. The First Indian player to get
(a) to oppose something important reader will understand? It’s good to Padma Vibhushan is
(b) to suffer an injury on the back in write as if you were having a ...(21)... (a) Virat Kohli
an accident with the reader. Use complete (b) Sunil Gavaskar
(c) to fail to do something important sentences and liven up your text with (c) Viswanathan Anand
in time ...(22)... . A good practice is to raise (d) Kapil Dev
(d) to seek support out of necessity questions and ...(23)... upon the topic 32. ISRO launched the highest number
from different perspectives. The of satellites from which launching
18. To make one’s blood boil benefits of good preparation will
(a) to make somebody furious vehicle?
become apparent at this stage at the (a) GSLV MK III (b) PSLV-C46
(b) to develop fever
latest. Remember that if you try to (c) PSLV-C37 (d) PSLV-C45
(c) to get excited
collect information and think about the
(d) to make someone nervous
style and content at the same time, it is 33. The first Battle of Panipat was held
Directions (Q. Nos. 19 and 20) You very ...(24)... that you will not succeed in
have one brief passage with two with either the style or the content. It (a) 1539 (b) 1527 (c) 1526 (d) 1556
questions following the passage. Read will be difficult to get a proper flow, 34. Who discovered penicillin?
the passage carefully and choosed the with the text proceeding smoothly from (a) Benjamin Franklin
best answer to each question out of the start to finish. Sometimes it’s best to (b) Louis Pasteur
four alternatives. start from the easiest or most ...(25)... (c) Edward Jenner
To eat and not be eaten—that’s the part and then work through the more (d) Alexander Fleming
imperative of a caterpillar’s existence. difficult parts at a later stage.
35. The book ‘Argumentative Indian’ is
The leaf roller reduces its risks of being 21. (a) argument (b) chat written by
picked off by predators by silking (c) talk (d) discussion (a) Amartya Sen
together a temporary shelter in which (b) Abhijeet Banerjee
to feed and rest. Adopting a different 22. (a) quotes (b) phrases
(c) Jagdish Bhagwati
line of defense, the jelly slug extrudes a (c) notations (d) examples
(d) Manmohan Singh
sticky tranlucent coating that may foul 23. (a) examine (b) explanation
the mouth-parts of marauding ants. 36. The Spektr-RG space telescope is a
(c) illuminate (d) highlight
For its part, the aquatic larva, by its joint venture of which of the
watery element, fashions a portable 24. (a) unlikely (b) probable following countries?
hideout from fragments of aquatic (c) pertinent (d) necessary (a) India and France
leaves. Cutting a serpentine trail as it (b) France and USA
25. (a) crucial (b) difficult
feeds on tender young levels, the (c) Russia and Germany
(c) important (d) defining
minute citrus leaf miner spends its (d) India and China
entire larval life inside its host plant, 26. Ozone layer depletion is due to
37. Who is considered the ‘Father of
thus keeping its appetising body safely which gas?
Indian Revolutionary Ideas’?
under wraps. (a) Carbon Dioxide
(a) Raja Rammohan Roy
(b) Methane (b) Aurobindo Ghosh
19. Which varieties of caterpillars (c) Sulphur dioxide
‘build’ shelters to protect (c) Bipin Chandra Pal
(d) Chlorofluorocarbons (d) MG Ranade
themselves?
(a) Leaf roller and aquatic larva 27. In a T20, each bowler can bowl 38. The Khilafat Movement was
(b) leaf roller and jelly slug a maximum of how many overs? merged with which movement?
(c) Jelly slug and aquatic larva (a) 6 (b) 5 (a) Quit India Movement
(d) Jelly slug and citrus leaf miner (c) 10 (d) 4 (b) Non-Cooperation Movement
20. Which one of the following 28. What is the India’s rank in the 2019 (c) Civil Disobedience Movement
caterpillars produces a sticky edition of the IMD World (d) Swadeshi Movement
covering? Competitiveness Rankings? 39. Who is known as the ‘Female Don
(a) Leaf roller (a) 43rd (b) 41st Bradman’ in the Cricketing World?
(b) Jelly slug (c) 44th (d) 42nd (a) Meghann Moira Lanning
(c) Aquatic larva 29. The Constitution of India was (b) Katherine Helen Brunt
(d) Citrus leaf miner adopted on (c) Jhulan Goswami
Directions (Q. Nos. 21-25) Below a (a) 26th Jan, 1949 (d) Elizabeth Rebecca Wilson
passage is given with five blanks. (b) 26th Jan, 1950 40. The capital of Columbia is
Below the passage, four options are (c) 26th Nov, 1950 (a) Lisbon (b) Seville
given for each blank. Choose the (d) 26th Nov, 1949 (c) Bogota (d) Lima
word that fits each blank most 30. Which of the following is a
appropriately in the context of the 41. NASA will launch its Mars Rover in
non-ferrous element? 2020 from which launching vehicle?
passage and mark the corresponding (a) Copper (b) Iron
answer. (a) Atlas V-541 (b) Atlas V-550
(c) Steel (d) Magnetite (c) GSLV MIC-III (d) PSLV-C47
AFCAT-II ~ Solved Paper 2019 05

42. Who is the winner of UEFA 53. 16 children are to be divided into 61. Two friends started for a place one
champions league 2019? two groups A and B of 10 and 6 by motorcycle and the other by
(a) Spain (b) Brazil children, respectively. The average train. The speed of motorcycle is
(c) Portugal (d) England marks obtained by the children of 30 km/h and that of train is
group A is 75 and that of all the 24 km/h. The first one takes 6 h
43. Which of the following metal is
children is 76. Then, the average 12 min to reach the destination.
used for making aeroplane parts?
marks of the children of group B is Find the time of reaching of second
(a) Silver (b) Aluminium 1 2 one.
(c) Iron (d) Zinc (a) 77 (b) 77
3 3 (a) 8.00 h (b) 7.25 h
44. The Kailasa Temple in Ellora caves (c) 78
1
(d) 78
2 (c) 7.50 h (d) 7.75 h
was built by which dynasty? 3 3 62. The speed of the current is 5 km/h.
(a) Satvahana (b) Rashtrakuta
54. B was born when A was 4 yr A motorboat goes 10 km upstream
(c) Pallavas (d) Chalukyas and back again to the starting point
7 months old and C was born, when
45. Which of the following desert is not B was 3 yr 4 months old. When C in 50 min. The speed (in km/h) of
located in Asian Continent? was 5 yr 2 months old, then their the motorboat in still water is
(a) Sahara Desert average age was (a) 20 (b) 26
(b) Gobi Desert (a) 8 yr 9 months (b) 7 yr 3 months (c) 25 (d) 28
(c) Thar Desert (c) 8 yr 7 months (d) 8 yr 11 months 63. If a 2 + b 2 + c 2 = ab + bc + ac, then
(d) Taklamakan Desert 55. When the price of an article was a+ c
the value of is
46. Which radioactive material is used reduced by 20%, its sale increased b
to treat cancer? by 80%. What was the net effect on (a) 0 (b) 2 (c) 1 (d) − 1
(a) Cobalt-60 (b) Thorium the sale?
(c) Uranium (d) Caesium (a) 44% increase (b) 44% decrease 64. A and B can together finish a work
(c) 66% increase (d) 75% increase in 30 days. They worked together
47. The 2024 Summer Olympics will be for 20 days and then B left. After
held in? 56. A sample of 50 L of glycerine is another 20 days, A finished the
(a) Tokyo (b) Qatar found to be adulterated to the remaining work. The number of
(c) Paris (d) London extent of 20%. How much pure
days, in which B alone can finish
glycerine should be added to it so as
48. Who was the founder of INA? to bring down the percentage of
the work is
(a) Mohan Singh (a) 54 (b) 60
impurity to 5%?
(b) Subhash Chandra Bose (c) 50 (d) 48
(a) 155 L (b) 150 L
(c) Rash Behari Bose 65. A dealer offers a discount of 10% on
(c) 150.4 L (d) 149 L
(d) C R Das the marked price of an article and
57. A reduction of 20% in the price of still makes a profit of 20%. If its
49. Kamaljeet Sandhu won gold medal apples enables a man to buy 16 kg
in which sports event? marked price is ` 800, then the cost
more apples for ` 4000. What is the will be
(a) Commonwealth Games reduced price per kg of apples?
(b) Bangkok Asian Games (a) ` 1000 (b) ` 500
(a) ` 64 (b) ` 60 (c) ` 800 (d) ` 600
(c) Summer Olympic Games (c) ` 50 (d) ` 40
(d) World Boxing Championships 1 1 2 1
58. If x : y = 5 : 6, then + −
50. What is the name of India’s surface (3 x − 2 y) : (y − x ) is
2 2 2 2 3 4  5 2 
to surface ballistic missile with
66. Simplify .
2 3 3 4
(a) 7 : 6 (b) 11 : 3 1 of − of
5000 km range which is developed 3 4 4 5
(c) 3 : 11 (d) 6 : 7
by DRDO? 37 37
(a) Brahmos (b) Prithvi III 59. Ramesh bought 10 cycles for ` 500 (a) (b)
each. He spent ` 2000 on the repair 78 13
(c) Agni IV (d) Agni V 74 74
of all cycles. He sold five of them (c) (d)
51. If 18225 = 135, then the value of for ` 750 each and the remaining 78 13
18225 + 182.25 + 1.8225 for ` 550 each. Then, the total gain 67. Pure milk costs ` 16 per litre. After
+ 0.018225 is or loss% is adding water the milkman sells the
1 1 mixture at ` 15 per litre and thereby
(a) 14.9985 (b) 149.985 (a) Gain of 8 % (b) Loss of 8 %
3 3 makes a profit of 25%. In what
(c) 1499.85 (d) 1.49985 2 1
(c) Gain of 7 % (d) Loss of 7 % respective ratio does he mix milk
52. The sum of a 2-digit number and 3 7 with water?
the number obtained by reversing (a) 3 : 1 (b) 4 : 3
60. A , B and C can complete a piece of
its digits is a square number. (c) 3 : 2 (d) 5 : 3
work in 12, 24 and 36 days,
How many such numbers are respectively. In how many days will 68. 3.25 × 3.20 − 3.20 × 3.05 is equal to
there? they together complete the same work? 0.064
(a) 5 (b) 6 6 6 1 1
(c) 7 (d) 8 (a) 5 (b) 4 (c) 6 (d) 6 (a) 1 (b) (c) (d) 10
11 11 2 10
06 AFCAT-II ~ Solved Paper 2019

69. Which of the following diagrams 79. USA : Congress : : Iran : ? Directions (Q. Nos. 86-90) In each
represents the relationship among (a) Althing (b) Storting of the following questions, choose
Sun, Moon and Star? (c) Majlis (d) Cortes the correct answer figure which will
complete the problem figure pattern.
80. Eye : Wink : : Heart : ?
(a) Throb (b) Move 86. Problem Figure
(c) Pump (d) Respirate
(a) (b) (c) (d) 81. Two statements are given
followed by two conclusions.
70. Which diagram correctly represents Read the conclusions and then
the relationship between Human decide which of the conclusions, ?
beings, Teachers, Graduates?
if any, logically follows from the
two given statements, disregarding Answer Figures
(a) (b) the known facts.
Statements
All rivers are mountains.
(c) (d) All forests are mountains.
(a) (b) (c) (d)
Conclusions
71. Which one of the following Venn I. Some rivers are forests. 87. Problem Figure
diagrams represents the best II. No river is a forest.
relationship between Snake, Lizard, (a) Only Conclusion I follows
Reptiles? (b) Only Conclusion II follows
(c) Neither I nor II follows
(d) Either Conclusion I or II follows
(a) (b) ?
82. In each question below is given a
statement followed by two
Answer Figures
assumptions numbered I and II.
(c) (d) You have to consider the statement
and the following assumptions and
decide which of the assumptions is
Directions (Q. Nos. 72-76) In each of implicit in the statement. (a) (b) (c) (d)
the following questions, three out of Statement Because of the large 88. Problem Figure
four words are similar in a certain way number of potholes in road X,
and one is different. Choose the word reaching airport in time has become
which is different from others. difficult.
72. (a) AC (b) Calculator Assumptions
(c) Cooler (d) Computer I. Reaching airport in time may ?
not be always necessary.
73. (a) Tongue (b) Lips II. There is no other convenient
(c) Brain (d) Nose road to the airport. Answer Figures
74. (a) Slip (b) Diamond (a) Only Assumption I is implicit
(c) Googli (d) Doosra (b) Only Assumption II is implicit
(c) Neither I nor II is implicit
75. (a) Orange (b) Apple (d) Both I and II are implicit (a) (b) (c) (d)
(c) Guava (d) Grapes
83. If ‘STYLE’ is written as PQVIB, how
76. (a) Albatross (b) Ostrich can ‘SMELL’ be written in that code? 89. Problem Figure
(c) Pelican (d) Penguin (a) PJBII (b) PVBII
(c) PVHII (d) PJHII
Directions (Q. Nos. 77-80) In each of
the following questions, choose the word Directions (Q. Nos. 84 and 85) In the
that will complete the second pair in the following series choose the term that
?
same way as first pair. will complete the series.
77. Pituitary : Brain : : Thymus : ? 84. TSMD, TSDM, TMDS, ? Answer Figures
(a) Larynx (b) Spinal Cord (a) TSDM (b) SDTM
(c) Throat (d) Chest (c) TMDS (d) SDMT
78. Scientist : Laboratory : : Actor : ? 85. AFI, JOR, MRU, ?
(a) Casino (b) Gallery (a) GJN (b) HMP
(c) PMO (d) RJL (a) (b) (c) (d)
(c) Stage (d) Site
AFCAT-II ~ Solved Paper 2019 07

90. Problem Figure Answer Figures Directions (Q. Nos. 98-100) In each
of the following question, find
? the answer figure in which
problem figure is embedded.
(a) (b) (c) (d)
98. Problem Figure
93. Problem Figures

Answer Figures

(1) (2) (3) (4) (5) Answer Figures


Answer Figures
(a) (b) (c) (d)
Directions (Q. Nos. 91-93) In each of
the following questions, a group of five (a) (b) (c) (d)
figures following a certain sequence is (a) (b) (c) (d)
given as problem figures. Problem 99. Problem Figure
Directions (Q. Nos. 94-97) In each
figures are followed by another group of of the following question, three out of
four answer figures marked as (a), (b), four figures are similar in a certain
(c) and (d). Find out the figure from the way and one is different, choose the
answer figures which when placed next different figure.
to the problem figures will continue the Answer Figures
sequence of problem figures. 94.
91. Problem Figures
(a) (b) (c) (d)
(a) (b) (c) (d)
95.
(1) (2) (3) (4) (5) 100. Problem Figure
Answer Figures (a) (b) (c) (d)
96.

(a) (b) (c) (d) Answer Figures


(a) (b) (c) (d)
92. Problem Figures
97.
(a) (b) (c) (d)
(1) (2) (3) (4) (5) (a) (b) (c) (d)

Answers
1. (c) 2. (a) 3. (d) 4. (d) 5. (a) 6. (a) 7. (b) 8. (a) 9. (c) 10. (c)
11. (c) 12. (c) 13. (c) 14. (c) 15. (b) 16. (a) 17. (d) 18. (a) 19. (a) 20. (b)
21. (a) 22. (d) 23. (c) 24. (b) 25. (a) 26. (d) 27. (d) 28. (a) 29. (d) 30. (a)
31. (c) 32. (c) 33. (c) 34. (d) 35. (a) 36. (c) 37. (c) 38. (b) 39. (d) 40. (c)
41. (a) 42. (d) 43. (b) 44. (b) 45. (a) 46. (a) 47. (c) 48. (a) 49. (b) 50. (d)
51. (b) 52. (d) 53. (b) 54. (d) 55. (a) 56. (b) 57. (c) 58. (c) 59. (d) 60. (c)
61. (d) 62. (c) 63. (b) 64. (b) 65. (d) 66. (a) 67. (a) 68. (d) 69. (d) 70. (a)
71. (d) 72. (b) 73. (c) 74. (b) 75. (d) 76. (b) 77. (d) 78. (c) 79. (c) 80. (a)
81. (d) 82. (b) 83. (a) 84. (d) 85. (b) 86. (d) 87. (b) 88. (c) 89. (c) 90. (a)
91. (c) 92. (d) 93. (b) 94. (b) 95. (c) 96. (b) 97. (c) 98. (b) 99. (d) 100. (b)
08 AFCAT-II ~ Solved Paper 2019

Hints and Solutions


1. (c) ‘Allegiance’ means ‘loyalty’ or 17. (d) To fall back on means to go to the most satellites launched
commitment to a superior or to a group somebody for support. simultaneously on one rocket.
or cause. 18. (a) To make one’s blood boil means to 33. (c) The First Battle of Panipat was fought
2. (a) ‘Abash’ means make someone feel make somebody extremely angry. on 21st April, 1526 between the
embarrassed. So, ‘ashamed’ is its similar 19. (a) ‘Leaf roller and aquatic larva’ build invading forces of Babur and Lodhi
meaning word. shelters to protect themselves. Empire. The battle took place in North
3. (d) ‘Altruism’ means disinterested and India and marked the beginning of the
20. (b) Jelly slug produces a sticky covering.
selfless concern for the well-being of Mughal Empire and end of the Delhi
21. (a) argument Sultanate.
others. Hence, ‘Sselflessness’ is its
correct answer. 22. (d) examples 34. (d) Alexander Fleming was a Scottish
23. (c) illuminate Physician who was recognised for
4. (d) ‘Adroit’ means clever or skilful. So,
24. (b) probable discovering Penicillin.
‘foolish’ is its correct antonym.
25. (a) crucial Penicillin is the first naturally occurring
5. (a) ‘Vague’ means uncertain, indefinite or
26. (d) Ozone layer depletion is the gradual antibiotic drug discovered and used
unclear. Hence, ‘clear’ is its correct
thinning of Earth’s ozone layer in the therapeutically.
antonym.
upper atmosphere caused by the release 35. (a) The book ‘Argumentative Indian’ is
6. (a) ‘Exodus’ means a sudden departure of chemical compounds containing written by Nobel Laureate Indian
of number of people. So, ‘influx’ is its gaseous chlorine or bromine from industry Economist Amartya Sen. It is a collection
correct antonym which means ‘a sudden and other human activities. of essays that discuss India’s history and
arrival of number of people’.
The major gases responsible for ozone identity, focusing on the traditions of
7. (b) ‘Unfurled’ means make or become layer depletion include public debate and intellectual pluralism.
spread out from a rolled or folded state, chlorofluorocarbons (CFCs), 36. (c) The Spektr-RG telescope is a joint
especially in order to be open to the methylchloride, bromide, hydrochloro- venture of Russia and Germany that was
wind. So, it is best fitted in first blank. fluorocarbons (HCFCs) and halons. launched on 13th July, 2019.
Hence, option (b) is the correct answer.
27. (d) T-20 or Twenty-20 is a shortened The telescope aims to survey the sky in
8. (a) According to the given question, format of cricket. T-20 cricket match is X-ray, mapping all galaxy clusters visible
‘drank and dropping’ are the appropriate restricted to a maximum of 20 overs in across the universe.
words to fill the blanks. which each bowler can bowl a maximum 37. (c) Bipin Chandra Pal is considered as
9. (c) ‘Obligatory and file’ are the correct of 4 overs. the ‘Father of Indian Revolutionary Ideas’.
alternatives to fill the blank according to 28. (a) As per 2019 edition of the IMD World He was an Indian nationalist, writer,
the given sentence. Competitiveness Rankings, India ranks orator and a great social reformer.
10. (c) In the first blank, ‘festering’ fits in the 43rd most competitive economy in the 38. (b) Khilafat Movement was an agitation
blank as here it means unpleasant and world. Singapore has topped the chart by Indian Muslims, to pressure the
full of anger. For the second blank following Hongkong and United States British Government to preserve the
‘ominous’ is appropriate as it means respectively. authority of the Ottoman Sultan as Caliph
something that suggests bad things will 29. (d) The Constitution of India was adopted of Islam after World War I.
happen soon or threatingly. on 26th November, 1949 and came into Gandhiji in order to get the support of
11. (c) Use of preposition ‘about’ after force on 26th January, 1950. It is the Muslim community merged his
discussed is not correct, remove it. The supreme law of India that frames Non-Cooperation Movement with
word ‘discussed’ itself means to talk fundamental political principles, Khilafat Movement.
about something. procedures, practices, rights, powers and
39. (d) Elizabeth Rebecca Wilson was a
12. (c) Use of ‘shall’ is incorrect here. As duties of the government.
former Australian cricket player who was
interview is singular in form, it should be 30. (a) Non-ferrous elements are those that do popularly known as ‘Female Don
replaced by ‘will be held’. not contain iron in appreciable amounts. Bradman’ in the Cricketing World.
13. (c) Use of pronoun ‘our’ is incorrect here. Copper, lead, nickel, tin, lead, zinc etc are
In 1985, she became the first woman
Use ‘his’ in its place. some of the examples of non-ferrous
cricketer to be inducted into the
metals.
14. (c) Replace ‘hear’ by ‘heard’ as the Australian Sporting Hall of Fame.
sentence is in past tense. As the time is 31. (c) In 2007, Indian chess grandmaster
40. (c) Bogota is the capital of Colombia, a
given, so second form of verb should be Viswanathan Anand was awarded India’s
country at the northern tip of South
used. second highest civilian award, the
America. The official currency of
Padma Vibhusan, making him the first
15. (b) A little gush of gratitude means Colombia is ‘Peso’.
sports person to receive the award.
‘friendly feeling’. 41. (a) The launch vehicle to be used in
32. (c) ISRO’s Polar Satellite Launch
16. (a) To lose ground means to allow NASA’s MARS 2020 rover mission is
Vehicle, PSLV-C37 has successfully
somebody to have an advantage or to Atlas V-541.
launched 104 satellites into orbit on
lose an advantage for yourself.
14th Feb, 2017 setting a new record for
AFCAT-II ~ Solved Paper 2019 09

135 135 135 40 + x 95


The MARS 2020 rover mission is part of = 135 + + + =
NASA’s Mars Exploration program with a 10 100 1000 50 + x 100
planned launch in July 2020. = 135 + 135
. + 135
. + 0135
. 40 + x 19
⇒ =
42. (d) Liverpool Football Club of England = 149985
. 50 + x 20
has won the UEFA Champions League 52. (d) Let unit’s digit be x and tens digit be y. ⇒ 800 + 20x = 950 + 19x
2019. The UEFA Champions League is ∴ Number = 10y + x ∴ x = 950 − 800 = 150 L
an annual club football competition Number obtained by reversing digits 57. (c) Let the original price of apples be ` x
organised by the Union of European = 10x + y
Football Associations (UEFA). per kg.
Sum of both numbers 80 4x
43. (b) The metals used for making = (10x + y) + (10y + x) New price = x × =`
aeroplane parts include steel, aluminium 100 5
= 11(x + y)
and titanium. According to the question,
For sum to be perfect square (x + y) 4000 × 5 4000
44. (b) The Kailasa Temple is one of the largest should be 11. − = 16
4x x
Indian rock-cut ancient Hindu temples ∴Possible pairs of x and y are 1000 1000
located in the Ellora Caves, Maharashtra. (2, 9), (3, 8), (4, 7) and (5, 6). ⇒ = 16 ⇒ x = = 62.5
x 16
The temple was made by the Rashtrakuta ∴Total possible pairs = 4 × 2 = 8
∴ New price = x = `  × 62.5
4 4
dynasty as a temple for Lord Shiva. 53. (b) Total marks obtained by all the 5 5 
45. (a) The continent of Asia features several children = 16 × 76 = 1216 = ` 50 per kg
deserts including Arabian desert, Gobi Total marks obtained by group A x 5
desert, Thar desert, Taklamakan desert etc. = 10 × 75 = 750 58. (c) Given, =
y 6
The Sahara desert is located on the Marks obtained by group B
Squaring on both sides, we get
African Continent and is the third largest = 1216 − 750 = 466 x2 25
desert in the world after Antarctica and Average marks obtained by group B =
466 233 2 y2 36
the Arctic desert.
= = = 77 x2
46. (a) The radioactive form of cobalt, 6 3 3 3 2 −2
3x − 2 y
2 2
Cobalt-60 has been used for radiotherapy 54. (d) Age of B, when C is 5 yr 2 months ∴ 2 = y 2
cancer treatment. = 3 yr 4 months + 5 yr 2 months y − x2 x
1− 2
It is also used for food irradiation and = 8 yr 6 months y
industrial applications. Age of A, when C is 5 yr 2 months (dividing numerator and denominator
47. (c) The 2024 Summer Olympics is a = 8 yr 6 months + 4 yr 7 months by y2 )
25
forthcoming international multi-sport = 13 yr 1 month 3× −2
36 75 − 72 3
event that is scheduled to take place ∴Average age = = = = 3 : 11
1−
25 36 − 25 11
from 26th July to 11th August, 2024 in [5 yr 2 months + 8 yr
Paris, France. 36
6 months + 13 yr 1 month]
= 59. (d) Given, cost price of 1 cycle = ` 500
48. (a) The Indian National Army (INA) was 3
an armed force formed by Captain 26 yr 9 months ∴Cost price of 10 cycles = 500 × 10
= = 8 yr 11 months
Mohan Singh in Singapore in September, 3 = ` 5000
1942. Its aim was to secure Indian But Ramesh spent ` 2000 on the repair.
55. (a) When the value of an object is
Independence from British rule. ∴Total cost price
changed by x% and then changed by y%.
49. (b) Kamaljeet Sandhu is the first Indian = 5000 + 2000 = ` 7000
The net effect is given as
Now, (for 5 cycles)
woman athlete to win gold in Asian  ± x ± y + ± x ± y %.
Selling price of 1 cycle = ` 750
games. She won gold medal at 1970  100  ∴Selling price of 5 cycles = 750 × 5
Bangkok Asian Games in 400m race. x × y
So, required effect =  x + y + % = 3750
50. (d) Agni V is an intercontinental surface  100  and (for remaining 5 cycles)
to surface ballistic missile developed by
Here, x = 80, y = − 20 Selling price of 1 cycle = ` 550
the Defence Research and Development
80 × 20 ∴Selling price of 5 cycles = 550 × 5
Organisation (DRDO) of India. =  80 − 20 − % = ` 2750
 100 
The missile with a strike range of So, total selling price = 3750 + 2750
5000 km can carry 1500 kg of nuclear = (60 − 16)% = 44% = 6500
warhead. Positive sign shows increase. Loss = Cost price − Selling price
51. (b) Given, 18225 = 135 56. (b) Glycerine in mixture = 80% of 50 L = 7000 − 6500 = ` 500
Loss
Expression = 18225 + 18225 . 80 × 50 ∴ Loss per cent = × 100
= = 40 L CP
+ 18225
. + 0018225
. 100 500 50 1
= × 100 = =7 %
On converting the above expression into Water in mixture = 20% of 50 = 10 L 7000 7 7
decimal fraction Let x L of pure glycerine is mixed with
18225 18225 18225 60. (c) Given, A, B and C can complete the
= 135+ + + the mixture.
100 10000 1000000 According to the question, work in 12, 24 and 36 days, respectively.
∴Work done by A in one day = 1 / 12
10 AFCAT-II ~ Solved Paper 2019

Work done by B in one day = 1 / 24 64. (b) Given, (A + B) s work time = 30 days 69. (d) Star
Work done by C in one day = 1 / 36 1
One day’s work (A + B)' s =
Then, (A + B + C)’s one day work 30 Sun
1 1 1 Then, work done by A and B in 20 days
= + + Moon
12 24 36 20 2
= =
6 + 3 + 2 11 30 3
= = Sun is a star. Moon is a satellite.
Remaining 1 −  , i.e., work is
72 72 2 1
70. (a)
∴ (A + B + C) together will complete  3 3 Human Beings
completed by A in 20 days.

Graduates
72 6

Teachers
the work = =6 days
11 11 ∴Time taken by A to complete the whole
work = 20 × 3 = 60 days.
61. (d) Let the total distance of journey be
∴Time taken by B to complete the work
D km. 1 1 1
Now, the speed of motorcycle and time = − = = 60 days
taken by him is 30 km/h and 6 h 12 30 20 60
65. (d) Marked price = ` 800 Some teachers may be graduates and
min, respectively.
vice-versa.
We know that, 100 − discount%
Distance = Speed × Time SP = × MP All teachers and all graduates are human
100 beings.
12 90 × 800
= 30 × 6
60 = `   = ` 720 71. (d)
 100 
372 Reptiles
= 30 × Now, CP = SP ×
100
60 100 + Profit% Snake Lizard
= 186 km 720 × 100
Now, in second case, distance travelled = = ` 600
120
will be same and given that speed of
train is 24 km/h. 66. (a) Using VBODMAS rule,
Snake is different from Lizard, but both
∴ Time taken =
Distance 1 1  4 − 5 1  1 1 
+ − ×  are reptiles.
Speed 3 4  10  = 3  4 10
186 9 72. (b) All except ‘Calculator’ need electricity
= =7 = 775 5 3 3 4 5 3
. h × − × − to function.
24 12 3 4 4 5 4 5
Hence, the time taken by second man to 1 1 40 − 3 73. (c) All except ‘Brain’ are sense organs of
− human.
reach is 7.75 h. 3 40
= = 120
62. (c) Let the speed of motorboat in still 25 − 12 13 74. (b) All except ‘Diamond’ are terms related
water be x km/h. 20 20 to cricket.
Speed upstream = (x − 5) km /h and 37 / 120 37 20
= = × 75. (d) Only Grapes grow in bunches and so
Speed downstream = (x + 5) km /h 13 / 20 120 13 it is odd one among others.
37
According to the question, = 76. (b) All except Ostrich are water birds.
10 10 50 78
+ = 77. (d) ‘Pituitary’ is the gland present in the
x − 5 x + 5 60 67. (a) ∵ SP of the mixture = ` 15 ‘Brain’ and ‘Thymus’ is the gland present
 x + 5 + x − 5 5 SP × 100 in the ‘Chest’.
⇒ 10   = ∴CP of the mixture =
 (x + 5) (x − 5)  6 100 + Gain 78. (c) Second is the working place of the first.
100
⇒ 20x × 6 = (x 2 − 25) × 5 = 15 × 79. (c) Latter is the Parliament of the country
125 represented by former.
⇒ x 2 − 24x − 25 = 0
= ` 12
x 2 − 25x + x − 25 = 0 80. (a) Latter represents the movements of
By Alligation rule
former.
⇒ x(x − 25) + 1(x − 25) = 0 Milk Water 81. (d)
⇒ (x − 25) (x + 1) = 0 16 0
⇒ x = 25 km / h because x ≠ − 1 12 Rivers
Speed of motorboat = 25 km / h Rivers Forests
12 4 Mountains or
63. (b) Given, a + b + c = ab + bc + ca
2 2 2
Mountains
∴ Ratio of milk and water in mixture Forests
⇒ a2 + b2 + c2 − ab − bc − ca = 0 = 12 : 4 = 3 : 1
⇒ a(a − b) + b(b − c) + c(c − a) = 0
3.25 × 3.20 − 3.20 × 3.05 Conclusions
⇒ a(a − b) = 0 or a = 0 and a = b 68. (d)
0.064 I. May be true II. May be true
Similarly, b = c and c = a
a+ c 3.20(3.25 − 3.05) 3.20 × 0.20 ∴ Either Conclusion I or II follows.
Hence, the value of = =
b 0.064 0.064 82. (b) The statement presents the issue of
a + c 2b 320 × 20 ‘not reaching airport in time’ as a
= = =2 = = 10
b b problem. This means that reaching
640
airport in time is necessary.
AFCAT-II ~ Solved Paper 2019 11

So, I is not implicit. Besides, it is H M P 93. (b) In every successive figure, there is an
mentioned that reaching airport in time addition of 1, 2, 3, 4, 5, … lines,
has become difficult due to large number +5 +3 respectively.
of potholes in road X. ∴ ? = HMP 94. (b) In figure (b), position of S is different
This implies that road X is the only 86. (d) Answer figure (d) will complete the from other figures.
possible way. So, II is implicit. pattern of problem figure. 95. (c) In all other figures except (c), both the
83. (a) As, 87. (b) Answer figure (b) will complete the line segments are perpendicular to each
S T Y L E P Q V I B pattern of problem figure. other.
–3 88. (c) Answer figure (c) will complete the 96. (b) Only in figure (b), the cross inside the
–3
–3 pattern of problem figure. circle is attached in line with the main
–3 line segment.
–3 89. (c) Answer figure (c) will complete the
pattern of problem figure. 97. (c) In all other figures except (c), both the
Similarly, 90. (a) Answer figure (a) will complete the arrows are perpendicular to each other.
S M E L L P J B I I pattern of problem figure. 98. (b) The question figure is embedded in
–3 91. (c) Problem figure (1) is identical to answer figure (b).
–3
–3 figure (5) but with different symbols.
–3 Problem figure (2) is obtained by moving
–3
to the top, T to the opposite side and ∆
84. (d) From element first to second last two to the bottom (replacing by =). Similarly,
answer figure will be obtained from
letters are written in reverse order, from 99. (d) The question figure is embedded in
problem figure (5) by moving symbol S to
element second to third, last three letters answer figure (d).
the top, P to the opposite side and 0 to
are written in reverse order and from
the bottom (replacing by*). Figure (b)
element third to fourth, all the four letters cannot be the correct answer as symbol
are written in reverse order. ∆ is not a new symbol.
∴Required answer = SDMT
92. (d) It is clear from the above figure that
85. (b) The pattern followed in each term is the arrow rotates clockwise making an 100.(b) The question figure is embedded in
First letter + 5 = second letter angle of 45°, 90°, 135° and 180° in answer figure (b).
and second letter + 3 = Third letter each subsequent block and the dot
As, rotates in the same way but in
A F I J O R M R U anti-clockwise direction. Thus, the arrow
, and dot will rotate by an angle of 225°
+5 +3 +5 +3 +5 +3 clockwise and anti-clockwise,
respectively in the answer figure.
12 AFCAT-I ~ Solved Paper 2019

INDIAN AIR FORCE


AFCAT-I

Solved Paper 2019


Time : 2 Hrs MM : 300
INSTRUCTIONS
■ The set contains a total of 100 questions, Comprising Verbal Ability in English, General Awareness, Numerical Ability and Reasoning and
Military Aptitude Test.
■ Each correct question carry 3 Marks and there will be negative marking of 1 Mark for each incorrect attempt.
■ Total time duration will be 2 hrs (120 minutes).
■ No marks will be deducted for unattempted questions.

Directions (Q. Nos. 1 and 2) In the The woman turned it over and 7. The woman returned the letter to
following questions, choose the word examined it and then returned it, the postman because
similar in meaning to the word given. saying she could not pay the postage, (a) she could not pay the postage
1. Narcissist which was a shilling. Hearing that the (b) the letter was not addressed to her
(a) Companion (b) Follow letter was from her brother, Mr. Hill (c) she already knew the contents of
(c) Self-Obsessed (d) Self-effacing paid the postage, in spite of the the letter
manifest unwillingness of the woman. (d) she hated the person who wrote
2. Elucidate As soon as the postman was out of the letter
(a) Clarify (b) Calculate sight, she showed Mr. Hill how his
(c) Summarise (d) Update 8. Mr. Hill paid the postage because
money had been, wasted as far as she (a) the letter was from her brother
Directions (Q. Nos. 3-5) In the was concerned. (b) the woman was his relative
following questions, choose word The sheet was blank. There was an (c) the letter was addressed to him
opposite in meaning to the word given. agreement between her brother and (d) he wanted to be kind to her
3. Placidity herself that as long as all went well
with him, he should send a blank sheet 9. The envelope contained
(a) Calmness (b) Agitation (a) a currency note
in this way once a quarter and she
(c) Presence (d) Placidness (b) two written sheets
thus had tidings of him without
4. Incandescent expense of postage. (c) no sheet at all
(a) Blazing (b) Genuine (d) a blank sheet
6. The story uses irony as a technique
(c) Luminous (d) Dark Directions (Q. Nos. 10-15) In the
because
5. Dwindled (a) the woman returned her following passage, some of the words
(a) Wane (b) Unnecessary own brother’s letter without have been left out. Choose the correct
(c) Sink (d) Increase opening it answer to each question out of the four
Directions (Q. Nos. 6-9) Read the (b) the woman broke the agreement alternative and fill the blanks.
following passage carefully and choose of receiving blank the letters Whatever prosperity India enjoyed
to convey well being of her in the seventeenth centuries
the best answer to each question out of
brother
the four alternatives. disappeared when the Mughal Empire
(c) Mr. Hill accepted the letter
Mr. Rowland Hill, when a young man ...(10)... apart. The most immediate
addressed to the woman
was walking through the Lake district, cause of this breakdown was the
(d) In the modern times a brother
when he one day saw the postman deliver has no time to write a letter to his religious intolerance, which led to open
a letter to a woman at a cottage door. own sister rebellion.
AFCAT-I ~ Solved Paper 2019 13

It was to ...(11)... these revolts that the 23. Commencement of adjacent words 34. In which of the following positions
bigot ruler spent ...(12)... years in the with the same letter is the kinetic energy maximum in
field with immense armies consuming (a) Pun SHM?
the revenues of the country. There (b) Alliteration (a) Extreme position
were, however, more deep-seated (c) Transferred epithet (b) Mean position
...(13)... . The corruption of officials and (d) Oxymoron (c) Can be at extreme or mean
the oppression of the masses steadily 24. A specialist who tests eyesight position
...(14)... away the empire’s life blood. (a) Optician (d) None of the above
For some time there had been a
noticeable deterioration in the
(b) Ophthalmologist 35. India receives maximum rainfall
(c) Ichthyologist due to which monsoon?
character of the ruling class. Wars of (d) Neurologist (a) South-West Monsoon
secession ...(15)... wiped out the leading
25. A wall built to prevent the sea or a (b) Western Disturbance
families, and new blood from central
river from flooding an area (c) North-East Monsoon
Asia was no longer recruited for the (d) Retreat Monsoon
higher governmental posts. (a) Dam (b) Mound
(c) Dyke (d) Embankment 36. Who is the first woman speaker of
10. (a) joined (b) broke Loksabha in India?
(c) dashed (d) banged 26. The famous Battle of Haldighati
(a) Meira Kumar
was fought between Maharana
11. (a) run out (b) crash
Pratap and Akbar’s forces led by
(b) Sushma Swaraj
(c) crush (d) cajole (c) Sumitra Mahajan
(a) Man Singh I
(d) Indira Gandhi
12. (a) no (b) hardly (b) Amar Singh I
(c) many (d) inexpensive (c) Udai Singh 37. Derby Cup is associated with which
(d) Jai Singh I of the following sports games?
13. (a) variables (b) attributes (a) Chess (b) Horse Racing
(c) characteristics (d) causes 27. Kakrapar Dam is situated on which
(c) Hockey (d) Table Tennis
river?
14. (a) drained (b) gone (a) Narmada (b)Tapi 38. Who was the chief guest of
(c) sucked (d) released (c) Sabarmati (d) Mahi Republic Day of India in 2019?
15. (a) has (b) have (a) Cyril Ramaphosa
28. The natural rubber is a polymer
(b) Joko Widodo
(c) was (d) had of
(c) Benjamin Netanyahu
Directions (Q. Nos. 16-20) Four words (a) isoprene (b) teflon (d) Francois Hollande
are given in each questions, out of (c) bakelite (d) elastomer
39. Who is the first woman fighter pilot
which one word is wrongly spelt. Find 29. The United Nations was established in India?
the wrongly spelt word. in (a) Anjana Bhaduria
16. (a) Deliquency (b) Friquency (a) 1950 (b) 1945 (b) Priya Jhingan
(c) Discrepency (d) Hesitancy (c) 1942 (d) 1946 (c) Bhawana Kanth
17. (a) Harassment 30. Where is Satish Dhawan Space (d) Divya Ajith Kumar
(b) Commitment
centre located? 40. Which of the following country is the
(c) Breevement (d) Temparament
(a) Andhra Pradesh largest producer of rice in the world?
18. (a) Handicaped (b) Frolicked (b) Odisha (a) India (b) China
(c) Kidnaped (d) Developed (c) Tamil Nadu (c) Indonesia (d) Brazil
19. (a) Mischeivous (b) Miscariage (d) Andaman and Nicobar
41. Which Indian Personality has been
(c) Misdemeanour (d) Misnomar 31. Who was the founder of Swatantra conferred with the Global
Party? Education Leaders Award 2018?
20. (a) Capracious (b) Auspicious
(a) Motilal Nehru (a) Trilok Chaudhary
(c) Fallicious (d) Dalicious (b) C Rajagopalachari (b) Saroj Suman Gulati
Directions (Q. Nos. 21-25) Out of the (c) MG Ranade (c) Sujoy Ghosh
four alternatives, choose the one which (d) Dada Bhai Naoroji (d) Gopal Das Banerjee
can be substituted for the given word 32. Who was the Chairman of
sentence. 42. Saksham 2019 is an annual high
drafting committee of Indian intensity people-centric mega
21. A government by officials Constitution? campaign under the aegis of which
(a) Oligarchy (a) K M Munshi ministry?
(b) Aristocracy (b) BR Ambedkar (a) Ministry of Drinking Water and
(c) Plutocracy (c) Dr Rajendra Prasad Sanitation
(d) Bureaucracy (d) Mohammad Saadullah (b) Ministry of Petroleum and
22. One who walks in sleep 33. Which of the following is the Natural gas
(a) Somniloquist hardest substance? (c) Ministry of Finance
(b) Egoist (a) Diamond (b) Lead (d) Ministry of Information and
(c) Somnambulist (c) Graphite (d) Carbon Broadcasting
(d) Altruist
14 AFCAT-I ~ Solved Paper 2019

43. Which of the following state 53. The strength of a school increases 61. The average of marks in
celebrates the Hornbill festival? and decreases in every alternate Mathematics for 5 students was
(a) Arunachal Pradesh year by 10%. It started with found to be 50. Later, it was
(b) Manipur increase in 2000. Then, the strength discovered that in the case of one
(c) Mizoram of the school in 2003 as compared student, the marks 48 were misread
(d) Nagaland to that is 2000 was as 84. The correct average is
44. Who is the Golden Boot winner (a) increased by 8.9% (a) 40.2 (b) 40.8
FIFA 2018? (b) decreased by 8.9% (c) 42.8 (d) 48.2
(a) Harry Kane (c) increased by 9.8% 62. 1 1
(d) decreased by 9.8% 1 ÷1
(b) Thomas Muller 4 2 is equal to
(c) James Rodriguez 54. A retailer buys a radio for ` 225. His 1 9
 +1− 
(d) Ronaldo overhead expenses are ` 15. He sells 15 10
45. Who is the author of ‘A Better the radio for ` 300. The profit per
cent of the retailer is (a) 3 (b) 6
India : Better World’? 2
2 (c) (d) 5
(a) Raghuram Rajan (a) 25% (b) 26 % 5
(b) NR Narayana Murthy 3
(c) Aziz Premji (c) 20%
1
(d) 33 % Direction (Q.No. 63) In the question
(d) Nandan Nilekani 3 below is given a statement followed by
55. The cost price of 25 articles is equal two assumptions numbered I and II.
46. Silao Khaja, which recently got GI You have to consider the statement and
tag, is the traditional delicacy of to the selling price of 20 of them.
The gain or loss per cent is given by the following assumptions and decide
which state? which of the assumptions is implicit in
(a) Nagaland (b) Bihar (a) 20% loss (b) 25% gain
(c) 60% loss (d) 75% gain the statement.
(c) Manipur (d) Jharkhand
56. The difference between compound 63. Statement Of all the radio sets
47. Who is the current president of manufactured in India, the ‘X’
Maldives? and simple interests on a sum of
money at 4% per annum for 2 yr in brand has the largest sale.
(a) Abdulla Yameen Assumptions
` 8. The sum is
(b) Ibrahim Mohamed Solih
(a) ` 400 (b) ` 800 1. The sale of the radio sets
(c) Joko Widodo
(c) ` 4000 (d) ` 5000 manufactured in India is
(d) Mohamed Nasheed
57. A, B and C can complete a work in known.
48. Out of the given responses, one of 2. The manufacturing of no other
2 h. If A does the work alone in 6 h
the factors of x − 3 x + 3 x + 7 is
3 2
and B in 5 h, how long will it take radio set in India is as large as
(a) x 2 − 4 x + 7 (b) x 2 + 4 x + 7 for C for finish the work alone? ‘X’ brand radio.
(c) x + 4 x − 7
2
(d) x 2 − 4 x − 7 1 1 1 (a) Only Assumption I is implicit
(a) 5 h (b) 7 h (c) 9 h (d) 4 h
2 2 2 (b) Only Assumption II is implicit
49. ‘a’ divides 228 leaving a remainder 3 1 (c) Neither Assumption I nor II is
18. The biggest two digit value of ‘a’ 58. If 3 x + = 1, then x 3 + 3 + 1 is implicit
x x
is (d) Both Assumptions I and II are
1
(a) 21 (b) 35 (a) 0 (b) implicit
(c) 30 (d) 70 27
(c)
5
(d)
28 Directions (Q. Nos. 64 and 65) In each
50. The digit in unit’s place of the 27 27 of the following question, a group of four
number words is given. Choose the word which
59. 1 man and 1 woman together can
(1570)2 + (1571)2 + (1572)2 + (1573)2 complete a piece of work in 8 days.
is odd.
is A man alone can complete the work 64. (a) Sub-lieutenant (b) Lieutenant
(a) 4 (b) 1 (c) 2 (d) 3 in 10 days. In how many days, can (c) Major (d) Colonel
51. The average of 9 numbers is 30. The one woman alone complete the
work? 65. (a) April (b) June
average of first 5 numbers is 25 and (c) July (d) September
that of the last 3 numbers is 35. 140
(a) days (b) 30 days
What is the 6th number? 9 Directions (Q. Nos. 66-75) In each of
(a) 20 (b) 30 (c) 40 (d) 50 (c) 40 days (d) 42 days the following question, choose the term to
complete the second pair in the same way
52. The average weight of a group of 60. Equal sum of money are lent to
as first pair.
20 boys was calculated to be 89.4 kg X and Y at 7.5% per annum for the
and it was later discovered that one period of 4 yr and 5 yr respectively. 66. Oxygen : Burn : : Carbon dioxide : ?
weight was misread as 78 kg If the difference in interest paid by (a) Isolate (b) Foam
instead of 87 kg. The correct them was ` 150, then sum lent to (c) Extinguishes (d) Explode
average weight is each was 67. Sitar : Guitar : : Tanpura : ?
(a) 88.95 kg (b) 89.25 kg (a) ` 500 (b) ` 1000
(a) Trumpet (b) Violin
(c) 89.55 kg (d) 89.85 kg (c) ` 2000 (d) ` 3000
(c) Harmonium (d) Mridanga
AFCAT-I ~ Solved Paper 2019 15

68. Money : Misappropriation : : 78. Statements 84.


Writing : ? All skaters are good swimmers.
(a) Deception (b) Mistake All good swimmers are runners. RA QB QC PD RB ?
(c) Plagiarism (d) Theft (a) QB (b) PA
Conclusions
69. Diamond : Baseball : : Court : ? (c) CP (d) QA
I. Some runners are skaters.
(a) Squash (b) Joker II. Some skaters are good Directions (Q. Nos. 85-89) The second
(c) Poker (d) Grass swimmers. figure of the first part of the problem
70. Veer : Path : : ? : Subject figures bears a certain relationship to
79. A statement is given followed by the first figure. Similarly, one of the
(a) Object (b) Prove two inferences. You have to take
(c) Math (d) Degree figure in answer figures bears the same
the given statement to be true even,
relationship to the first figure of the
71. 583 : 488 :: 293 : ? if they seem to be at variance from
second part. You have to select that
(a) 581 (b) 291 (c) 378 (d) 487 commonly known facts. Read the
figure from the set of answer figures
inferences and then decide which
72. NEUROTIC : TICRONEU : : which would replace the question
of the Inference, if any, logically
PSYCHOTIC : ? mark “?”.
follows from the two given
(a) TICCOHPSY (b) TICOCHPSY statements, disregarding the 85. Problem Figures
(c) TICCHOPSY (d) TICHCOPSY known facts.
73. ACEG : DFHJ : : QSUW : ? Statement Use “Kraft” colours. :: ?
(a) TVNZ (b) TVZX They add colour to our life. An
(c) TVXZ (d) XVTZ advertisement.
Inferences Answer Figures
74. 64 : 8 : : 289 : ?
I. Catchy slogans do not attract
(a) 17 (b) 27
(c) 26 (d) 19 people.
II. People like dark colours.
75. 23 : 72 : : 38 : ? (a) Only Inference I follows (a) (b) (c) (d)
(a) 110 (b) 117 (b) Only Inference II follows
(c) 123 (d) 112
(c) Neither Inference I nor II 86. Problem Figures
Directions (Q. Nos. 76-78) Two follows
statements are given in each of the (d) Both Inferences I and II follow
?
following question, followed by two Directions (Q. Nos. 80-84) Choose the
Conclusions I and II. You have to take correct code for the uncoded term.
the two statements to be true even, if Answer Figures
they seems to be at variance from 80.
commonly known facts. Read the
conclusions and then decide which of AP CR BS DQ CS ?
the given conclusion logically follows (a) (b) (c) (d)
(a) DR (b) CP
the given two statements, disregarding (c) BR (d) DS
the known facts. 87. Problem Figures
81.
Give Answer
(a) If only Conclusion I follows :: ?
(b) If only Conclusion II follows
(c) If both Conclusions I and II follow BY DZ BX CY DX ? Answer Figures
(d) If none of the conclusion follows
(a) DZ (b) DY
76. Statements (c) CZ (d) BZ
Some men are great. 82.
Some men are wise.
(a) (b) (c) (d)
Conclusions
I. Men are either greater or wise. 88. Problem Figures
II. Some men are neither great nor PA BP QA RQ CB ?
wise.
(a) PQ (b) RB :: ?
77. Statements (c) PR (d) AB
All players are doctors.
83. Answer Figures
Some doctors are actors.
Conclusions PQ RA BC RX PX ?
I. Some doctors are players as well
as actors. (a) PX (b) RP
(c) BQ (d) AC (a) (b) (c) (d)
II. All actors are doctors.
16 AFCAT-I ~ Solved Paper 2019

89. Problem Figures Answer Figures Answer Figures

?
(a) (b) (c) (d)
Answer Figures (a) (b) (c) (d)
95. Problem Figure
98. Problem Figure

(a) (b) (c) (d) ?

Directions (Q. Nos. 90-93) In each of Answer Figures


the following question, Choose
the figure which is different from Answer Figures
others.
90. (a) (b) (c) (d)

Direction In the following question, a (a) (b) (c) (d)


question figure is given with four
(a) (b) (c) (d) answer figure (a), (b), (c) and (d). Find 99. Problem Figure
out that answer figure are which is
91.
embedded in the problem figure. ?

96. Problem Figure


(a) (b) (c) (d)
Answer Figures
92.
3 3 3 3
3 3 3
3 3 3
Answer Figures
(a) (b) (c) (d)
(a) (b) (c) (d)
93. 2 1 7 5
100. Problem Figure
2 4 1 0 3 4 2 3 (a) (b) (c) (d)
(a) (b) (c) (d)
Directions (Q. Nos. 97-100) In each of ?

Directions (Q. Nos. 94-95) In each of the following questions, choose the
the following question, find the answer figure which will complete the problem
figure pattern. Answer Figures
figure in which problem figure is
embedded. 97. Problem Figure
94. Problem Figure
(a) (b) (c) (d)

Answers
1. (c) 2. (a) 3. (b) 4. (d) 5. (d) 6. (a) 7. (c) 8. (d) 9. (d) 10. (b)
11. (c) 12. (c) 13. (d) 14. (c) 15. (d) 16. (b) 17. (c) 18. (a) 19. (b) 20. (d)
21. (d) 22. (c) 23. (b) 24. (b) 25. (c) 26. (a) 27. (b) 28. (a) 29. (b) 30. (a)
31. (b) 32. (b) 33. (a) 34. (b) 35. (a) 36. (a) 37. (b) 38. (a) 39. (c) 40. (b)
41. (b) 42. (b) 43. (d) 44. (a) 45. (b) 46. (b) 47. (b) 48. (a) 49. (d) 50. (a)
51. (c) 52. (d) 53. (a) 54. (a) 55. (b) 56. (d) 57. (b) 58. (b) 59. (c) 60. (c)
61. (c) 62. (d) 63. (a) 64. (a) 65. (c) 66. (c) 67. (b) 68. (c) 69. (a) 70. (d)
71. (c) 72. (c) 73. (c) 74. (a) 75. (b) 76. (d) 77. (d) 78. (c) 79. (c) 80. (a)
81. (c) 82. (c) 83. (c) 84. (b) 85. (d) 86. (b) 87. (b) 88. (d) 89. (d) 90. (d)
91. (d) 92. (d) 93. (a) 94. (b) 95. (d) 96. (a) 97. (c) 98. (b) 99. (d) 100. (b)
AFCAT-I ~ Solved Paper 2019 17

Hints and Solutions


1. (c) ‘Narcissist’ means having or showing 25. (c) Dyke is a thick wall to prevent the 34. (b) The Kinetic energy is maximum in
an excessive interest in or admiration of water from a sea or river from flooding an mean position of Simple Harmonic
oneself and one’s physical appearance. area. Motion (S.H.M). Mean position in SHM
So, ‘self-obsessed’ is its correct 26. (a) The famous Battle of Haldighati was is the position when the freely suspended
synonym. fought in 1576, between Maharana pendulum is at rest.
2. (a) ‘Elucidate’ and ‘clarify’ have the same Pratap and the Mughal emperor Akbar’s As the pendulum is at its lowest position,
meaning which means make clear or forces, led by Man Singh I of Amber. its potential energy is minimum while its
throw light upon. The site of the Battle was a narrow kinetic energy will be maximum.
3. (b) ‘Placidity’ means a feeling of mountain pass at Haldighati near 35. (a) India receives maximum rainfall
calmness. So, its correct antonym is Gogunda in Rajasthan. The battle of due to South-West monsoon.
‘agitation’. Agitation means a state of Haldighati was a pyrrhic victory for the
South-West monsoon occurs from
anxiety or nervous excitement. Mughals, as they were unable to oust
July-September and majority of Indian
4. (d) Incandescent means white, glowing Maharana Pratap.
states receives the rainfall from this
or luminous, with intense heat. So, 27. (b) Kakrapar Dam is situated on Tapi river monsoon. South-West monsoon is the
‘Dark’ is its correct antonym. in Gujarat. Ukai Dam is other important rain bearing seasonal winds that flow
5. (d) ‘Dwindled’ means diminish gradually dam situated on river ‘Tapi’ in Gujarat.
from Arabian sea to the main land of
in size, amount or strength. So, 28. (a) The natural rubber is a polymer of India from the South-West direction.
‘increase’ is its correct antonym. ‘Isoprene’. Natural rubber or polyisoprene
36. (a) The first woman speaker of the Lok
6. (a) Woman returned her own brother’s is composed of the monomer isoprene :
Sabha is Meira Kumar. She is an Indian
letter without reading it. (2 methyl-1, 3-butadiene). It is an
Politician and the 15th speaker of Lok
additional polymer that is obtained as a
7. (c) The woman returned the letter to the Sabha from 2009 to 2014. The speaker
milky white fluid known as ‘latex’ from a
postman because she already knew the of the Lok Sabha is the presiding officer
tropical rubber tree.
contents of the letter. of the Lok Sabha, the lower house of the
29. (b) The United Nations (UN) is an Parliament of India.
8. (d) Mr Hill paid the postage because he
intergovernmental organisation
wanted to be kind to the woman as letter 37. (b) Derby Cup is associated with Horse
responsible for maintaining international
was from her brother. racing. Other cups/trophies related to this
peace and security, developing friendly
9. (d) As mentioned in the passage, the game includes Grand National wellington
relations among nations and achieving
envelope contained a blank sheet only. cup etc.
international cooperation. The UN is
10. (b) broke 11. (c) crush established in 1945 with its headquarters 38. (a) The South African President Cyril
located in New York city, USA. It has 193 Ramaphosa was the chief guest of
12. (c) many 13. (d) causes
member states. Republic Day of India in 2019. This was
14. (c) sucked 15. (d) had his first visit to India as a Head of State.
30. (a) Satish Dhawan space centre or
16. (b) The wrongly spelt word is ‘friquency’. He was the second President of
Sriharikota range is a rocket launch centre
The correctly spelt word is frequency South-Africa after Nelson Mandela to be
operated by the Indian Space Research
means number of repeat, occurence. the chief guest at the Republic Day
Organisation (ISRO). It is located in
17. (c) The wrongly spelt word is celebrations.
Sriharikota in Andhra Pradesh.
‘breevement’. The correctly spelt word is 39. (c) Flight Lieutenant Bhawana Kanth has
‘bereavement’ means loss, death. 31. (b) The Swatantra Party was an Indian
become first female fighter pilots in India
classical liberal political party that existed
18. (a) The wrongly spelt word is along with Flight Lieutenant Avni
handicaped. The correctly spelt word is from 1959 to 1974. The party was
Chaturvedi and Mohana Singh Jitarwal.
‘handicapped’ means disabled. founded by C Rajagopalachari. It had a
The trio was inducted into the Indian Air
number of distinguished leaders such as
19. (b) The wrongly spelt word is Force fighter squadron in June 2016.
Tanguturi Prakasam Pantulu, NG Ranga
miscariage. ‘miscarriage’ means failure. 40. (b) China is the largest producer of rice in
and KM Munshi.
The correctly spelt word is the world. China accounts for 30% of all
20. (d) The wrongly spelt word is ‘dalicious’. 32. (b) A drafting committee on 29th August,
world rice production. India is the second
The correctly spelt word is ‘delicious’ 1947 was appointed with BR Ambedkar
largest producer of rice in the world.
mens pleasing, tasteful. as the chairman along with six other
members to prepare a Draft Constitution 41. (b) Dr Saroj Suman Gulati, Director of
21. (d) A government run by officials is Blue Bells Group of Schools, has been
for India.
called ‘Bureaucracy’. conferred with the Global Education
33. (a) Diamond is the hardest naturally
22. (c) A person who walks is sleep is called Leaders Award 2018.
occurring substance in the world.
‘Somnambulist’. She was awarded for her exemplary
Diamond is the solid form of the
23. (b) Alliteration is the commencement of element carbon with its atoms arranged contribution to K-12 education and
adjacent words with the same letter. in a crystal structure called diamond promotion of Art and Culture during the
24. (b) A eye specialist who tests eyesight is cubic. 2nd India-UAE Partnership
called Ophthalmologist. Summit-2018.
18 AFCAT-I ~ Solved Paper 2019

x + 1)x 3 − 3x 2 + 3x + 7(x 2 − 4x + 7 90 × 110


42. (b) Saksham 2019 is an annual high = = 99
intensity people-centric mega campaign, x 3 + x2 100
which is organised by PCRA (Petroleum − − Again in year 2003, it increased by 10%.
Conservation Research Association) − 4x 2 + 3x + 7 So, new strength = 110% of 99
under the aegis of Ministry of Petroleum − 4x 2 − 4x 110 × 99
= = 1089
.
and Natural gas. + + 100
The purpose of the Saksham 7x + 7 ∴The strength in 2003 as compared to
(Sanrakshan Kshamta Mahotsav) is to 7x + 7 2000 is increased by 8.9%.
sensitise the masses about conservation − −
54. (a) Actual CP
and efficient use of petroleum products ×
= Cost of purchase + Overhead charges
which will lead towards better health and ∴ (x 2 − 4x + 7) is the factor of = 225 + 15 = ` 240
environment.
expression x 3 − 3x 2 + 3x + 7. Profit = SP − CP
43. (d) The Hornbill festival is celebrated = 300 − 240 = ` 60
every year from 1st to 10th December in 49. (d) Given, divisor = a, remainder = 18 Porfit × 100
and dividend = 228 ∴Profit per cent =
Nagaland state. It is also called the CP
‘Festival of Festivals’. We know that, 60
= × 100 = 25%
All the tribes of Nagaland take part in this Dividend = (Divisor × Quotient) + Remainder 240
festival. The aim of the festival is to ⇒ 228 = a × Quotient + 18 55. (b) Let the CP of 1 article = ` x
revive and protect the rich culture of ⇒ 210 = a × Quotient ∴ CP of 25 articles = 25 x
Nagaland and displays its culture Hence, biggest two digit value = 70 and SP of 20 articles = 25x
richness and traditions. 50. (a) (1570)2 + (1571)2 + (1572)2 + (1573)2 25x 5x
∴SP of 1 article = =
44. (a) The 2018 FIFA World Cup was held Then, unit’s digit of (1570)2 = (0)2 = 0 20 4
from 14th June to 15th July 2018 in 5x x
Unit’s digit of (1571)2 = (1)2 = 1 ∴ Gain = −x=
Russia. England captain Harry Kane has 4 4
won the Golden Boot after finishing as Unit’s digit of (1572)2 = (2)2 = 4 Gain × 100
Gain per cent =
the world Cup’s top scorer. After Units digit of (1573)2 = (3)2 = 9 CP
defeating Croatia, France became the ∴Required unit’s digit x
× 100
winner of the 2018 FIFA World Cup.
= Unit’s digit in (0 + 1 + 4 + 9) = 14 = 4 = 25%
45. (b) The author of the book ‘A Better i.e., unit’s digit in 14 = 4 x
India: A Better World’ is NR Narayana
51. (c) Sum of 9 numbers = 9 × 30 = 270 56. (d) Given, Cl − SI = 8 and r = 4%
Murthy. He is an Indian IT industrialist
Sum of first 5 numbers = 25 × 5 = 125 By the Formula,
and the co-founder of Infosys, a P× r ×t
multinational corporation providing Sum of last three numbers SI =
= 3 × 35 = 105 100
business consulting, technology,
 r 
t 
engineering and outsourcing services. ∴6th number = (Sum of 9 numbers) and Cl = P  1 +  − 1
− (Sum of first 5 number)   100  
46. (b) Silao Khaja, which recently got GI tag
is the traditional delicacy of Nalanda − (Sum of last 3 numbers) According to the question,
district of Bihar. Silao Khaja is known for = 270 − 125 − 105 = 40  t  P× r ×t
P  1 +
r 
its taste, crispness and multi-layered  − 1 − =8
52. (d) Given, average weight of 20 boys
  100   100
appearance. The sweets consists of = 894
.
 2  P × 4×2
⇒ P  1 +
twelve to sixteen very thin dough-sheets ∴ Sum of weight of 20 boys 4 
 − 1 − =8
  100
placed over one another. = 894. × 20 = 1788  100
47. (b) The incumbent President of Maldives According to the question,
 26 2  8P
is Ibrahim Mohamed Solih. He is the
New average =
1788 − 78 + 87 ⇒ P    − 1 − =8
head of the state and head of the 20 
 25   100
government of the Republic of Maldives. 1788 + 9 1797
= =  676   8P
He is elected in 2018 by a vast majority 20 20 ⇒ P    − 1 − =8
  625   100
of 58.4%, defeating former President = 8985
. kg
676 − 625 8P
Abdulla Yameen. ⇒P  − =8
53. (a) Let the strength of school in 2000 be  625  100
48. (a) Given expression, 100.
f(x) = x 3 − 3x 2 + 3x + 7 51P 8P
Then, in year 2001 strength increased ⇒ − =8
625 100
Put x = − 1 by 10%
f(− 1) = (− 1)3 − 3(− 1)2 + 3(− 1) + 7 So, new strength = 110% of 100 ⇒ 5100P − 5000P = 500000
110 × 100 ⇒ 100P = 500000
= −1− 3− 3+ 7 = 0 = = 110 500000
∴ (x + 1) is the factor of the expression. 100 ⇒ P=
In year 2002, strength decreased by 10% 100
Now, dividing expression by (x + 1)
So, new strength = 90% of 110 ∴ P = ` 5000
AFCAT-I ~ Solved Paper 2019 19

Alternate Method According to the question, Here, difference of both are


Difference between the Cl and Sl on a difference in SI = 150 (20 − 16) = (18 − 14) = 4
certain sum of money for 2 yr at r% rate is x × 7.5 × 5 x × 7.5 × 4 ∴ ? = 378
∴ − = 150
Pr2
= 100 100 72. (c) As, N E U R O T I C
(100)2 x × 7.5 × 1
⇒ = 150
P × 16 100
⇒ 8=
(100)2 150 × 100
⇒ x= = ` 2000 T I C R O N E U
16P 7.5
⇒ 8=
10000 61. (c) Given, average marks of 5 students in Similarly,
⇒ 16P = 80000 Mathematics = 50
P S Y C H O T I C
80000 ∴Sum of marks = 250
⇒ P=
16 Hence, correct average
∴ P = ` 5000 250 + 48 − 84 214
= = = 42.8 T I C C H O P S Y
57. (b) Let C alone can finish the work in x h. 5 5
1 1 5 3
According to the question, 1 ÷1 ÷
4 2 4 2 73. (c) As, A C E G
Work done by A, B and C in 1 h =
1 62. (d) =
2  1 + 1 − 9   2 + 30 − 27 
    +3 +3 +3 +3
1 1 1 1  15 10  30 
⇒ + + = 5 2 D F H J
6 5 x 2 ×
5 30
1 1 1 1 = 4 3 = × =5 Similarly,
⇒ = − − 5 6 5 Q S U W
x 2 6 5
30 +3 +3 +3
15 − 5 − 6 4 2
= = = 63. (a) Clearly, the comparison could not be
30 30 15 T V X Z
1 made without knowing the sale of all the
⇒ x =7 h radio sets. So, I is implicit. The statement 74. (a) As, 64 ⇒ 64 = 8
2
mentions only that the sale is largest and Same as, 289 ⇒ 289 = 17
Hence, C alone can finish the work
1 nothing is mentioned about the
in 7 h. manufacture. 75. (b) As, 23 72
2
3 So, II is not implicit.
58. (b) 3x + = 1 ×3 +3
x 64. (a) Except Sub-Lieutenant, all others are
different ranks in army while Same as,
On dividing of equation by 3, we get 38 117
1 1 Sub-Lieutenant is a rank in Navy.
x+ = …(i)
x 3 65. (c) Except July, all others months have ×3 +3

On cubing the Eq. (i), we get 30 days.


76. (d) According to the statements, Venn
x 3 + 3 + 3 × x ×  x +  =
1 1 1 1 66. (c) ‘Oxygen’ enhances the fire and diagram is as follows
x x  x  27 ‘Carbon dioxide’ extinguishes the fire.
1 
∵x + = 
1 1 1 1 67. (b) ‘Sitar’, ‘Guitar’, ‘Tanpura’ and ‘Violin’
⇒ x3 + 3 + 3 × =
x 3 27  x 3 are all string instruments. Wise Men Great
1 1 68. (c) As, ‘Money’ can be
∴ x3 + 3 + 1 =
x 27 ‘Misappropriation’, in the same way
‘Writing’ can be ‘Plagiarism’. Conclusions
59. (c) Given, work done in 1 day I. 7 II. 7
1 69. (a) As playground of baseball is known
by (1 man + 1 woman) = From above figure, it is clear, that none
8 as Diamond, similarly playground of
of the conclusion follows.
1 squash is known as court.
Work done by 1 man in 1 day = 77. (d) According to the statements, Venn
10 70. (d) One veers from a Path and one
diagram is as follows
Work done by 1 woman in 1 day Degress from a subject.
1 1 5− 4 1 71. (c) Taking option (c),
= − = =
8 10 40 40 A sum of digits of
∴ 1 woman will complete the work in 40 583 = 5 + 8 + 3 = 16 Players Actors
days. and sum of digits of
60. (c) Given, R = 7.5%, T1 = 4, T2 = 5 488 = 4 + 8 + 8 = 20
Doctors
Let the sum lent be ` x. Similarly, sum of digits of
Then, difference in SI 293 = 2 + 9 + 3 = 14 Conclusions I. 7 II. 7
P × R × T2 P × R × T1 and sum of digits of 378
= − From above, it is clear, that none of the
100 100 = 3 + 7 + 8 = 18 conclusion follows.
20 AFCAT-I ~ Solved Paper 2019

78. (c) According to the statements, Venn remaining portion of the figure rotates 90°
P, B, R, C and
diagram is as follows clockwise and is inverted.
A, Q
90. (d) Except figure (d), all other figures
Good Swimmers have even number of sides.
From the above codes, we see that code 91. (d) Two lines on the extreme ends of all
for the last figure is PR. the figures, except figure (d) have same
Skaters direction of arrows. In figure (d), the
83. (c) Here, the coded series can be
decoded as, direction of arrows are different.
92. (d) Except (d), in all other figures, the
Runners P, Q, R, A,
symbols in opposite sectors are same.
B, C 93. (a) Except (a), in all other figures, the
Conclusions I. 3 II. 3
sum of two lower numbers is given on
Hence, both Conclusions I and II follow. From the above codes, we see that code top of each figure.
79. (c) The given slogan is Catchy, which for the last figure is BQ.
94. (b) The figure is embedded in answer
indicates that catchy slogans do attract 84. (b) Here, the coded series can be
figure (b).
people. decoded as
So, Inference I does not follows. Nothing R, Q, P
about people’s preference for colours can
be deduced from the statements. Thus, A, B, C, D
Inference II also does not follows. From the above codes, we see that code
80. (a) Here, the coded series can be for the last figure is PA. 95. (d) The problem figure is embedded in
decoded as answer figure (d).
85. (d) Considering the first two figures, both
P, R,
the figures interchange their positions
S, Q and
and get reversed at the new position. The
A, C, B, D arrow head is shifted to the arm of
U-shaped figure and becomes reversed.
From the above codes, we see that code 86. (b) The shaded figure moves inside the 96. (a) Answer figure (a) is embedded in
for the last figure is DR. pentagon and the lower half of it problem figure.
81. (c) Here, the coded series can be becomes unshaded.
decoded as 87. (b) The lower element gets enlarged and
becomes the outer most element. The
upper element also gets enlarged and
B, D, C and for the circles
becomes the middle element while a new
element with one more side than the 97. (c) Answer figure (c) will complete the
lower element appears at the innermost problem figure pattern.
Y, Z and X position. 98. (b) Answer figure (b) will complete the
88. (d) The number of sides in the main problem figure pattern.
From the above codes, we see that code
figure increases by one and the number 99. (d) Answer figure (d) will complete the
for the last figure is CZ.
of line segments attached to the main problem figure pattern.
82. (c) Here, the coded series can be figure decreases by one. 100.(b) Answer figure (b) will complete the
decoded as,
89. (d) The line attached to the small black problem figure pattern.
dot rotates 90° anti- clockwise. The
AFCAT-II ~ Solved Paper 2018 21

INDIAN AIR FORCE


AFCAT-II

Solved Paper 2018


Time : 2 Hrs MM : 300
INSTRUCTIONS
■ The set contains a total of 100 questions, Comprising Verbal Ability in English, General Awareness, Numerical Ability and Reasoning and
Military Aptitude Test.
■ Each correct question carry 3 Marks and there will be negative marking of 1 Mark for each incorrect attempt.
■ Total time duration will be 2 hrs (120 minutes).
■ No marks will be deducted for unattempted questions.

Directions (Q. Nos. 1-4) You have one (c) without dementia and with 6. Sandhya .......... me from the top of
brief passage with four questions Alzheimer’s disease the house.
following the passage. Read the passage (d) With dementia and with (a) shouted to (b) shouted at
carefully and choosed the best answer Alzheimer’s disease (c) shouted on (d) shouted
to each question out of the four 2. Which word in the passage means 7. Ravi has the habit of … a headache.
alternatives. ‘earlier’? (a) complaining (b) complain
Daily consumption of a certain form of (a) Performance (b) Absorbed (c) complaining to (d) complaining of
curcumin improved memory and mood (c) Properties (d) Previously 8. I always want to go alone for a ride, but
in people with mild, age-related my mother .... going with my brother.
3. Eating turmeric
memory loss. The research examined (a) insists (b) insists on
the effects of an easily absorbed (a) will reduce the chance of getting
Alzheimer’s disease (c) insists in (d) insisted
curcumin supplement on memory
performance in people without (b) will increase curcumin 9. The new student found it difficult
dementia, as well as curcumin’s (c) will enhance dementia to …… with his classmates.
potential impact on the microscopic (d) will reduce chance of getting (a) get along (b) get among
cancer (c) get well (d) get up
plaques and tangled in the brains of
people with Alzheimer’s disease. 4. …… of a disease in a region Directions (Q. Nos. 10-12) In the
Found in turmeric, curcumin has depends on the food habits following questions, a sentence with a
previously been shown to have too. bold word/words followed by four
anti-inflammatory and antioxidant (a) Dominance (b) Prevalence words. Select the option that is nearer
properties in laboratory studies. It has (c) Affection (d) Death to meaning to the bold word/words.
also been suggested as a possible Directions (Q. Nos. 5-9) Each of the
reason that senior citizens in India,
10. Rahul is always thrifty.
following sentence in this section has a (a) reckless
where curcumin is a dietary staple, blank space and four words or group of
have a lower prevalence of Alzheimer’s (b) economical
words given after the sentences. Select (c) naive
disease and better cognitive the word or group of words you consider
performance. (d) extravagant
most appropriate for the blank space
1. Curcumin has positive effect of and indicate your response on the 11. His salubrious words calmed the
people answer sheet accordingly. students.
5. My teacher was ...... us for being late. (a) provoking (b) pleasant
(a) without dementia
(a) annoyed at (b) annoyed with (c) ridiculous (d) thanking
(b) with Aizheimer’s disease
(c) annoyed about (d) annoys
22 AFCAT-II ~ Solved Paper 2018

12. He felt desolated after he lost-his 17. (a) originating (b) originates 29. Who has won the 2018 Formula
business. (c) originated (d) organising One French Grand Prix
(a) deserted (b)joyful Tournament 2018?
18. (a) had been (b) has been
(c) strong (d) annoyed (a) Valtteri Bottas
(c) was (d) is
(b) Lewis Hamilton
Directions (Q. Nos. 13-16) Each item 19. (a) instance (b) incident (c) Max Verstappen
in this section has a sentence with three (c) accident (d) events (d) Kimi Raikkonen
underlined parts labelled (a), (b) and
(c). Read each sentence to find out Directions (Q. Nos. 20-22) Given 30. Where is the headquarters of
whether there is any error in any below are some idiom/phrases followed NATO?
by four alternative meanings to each. (a) Brussels, Belgium
underlined part. If you find no error,
Choose the response (a), (b), (c) or (d) (b) Geneva, Switzerland
your response should be indicated as
which is the most appropriate (c) Vienna, Austria
(d).
expression. (d) Paris, France
13. The letter has been written I insist on 20. Follow suit 31. The instrument Taseometer is used
(a) (b) (a) Following someone’s suit to measure
it being sent at once. No error, (b) Suiting to someone (a) pressure
(c) (d) (c) Doing the same as someone else (b) strain
14. “I’m tired of my boys”, said the mother, has just done (c) force
(a) (d) Doing the same kind of mistake (d) density
“Both of them keep quarrelling all 21. Close shave 32. Which Bollywood personality to
the time (a) Shaving very closely promote NITI Aayog’s Women
(b) (b) Miraculous escape Entrepreneurship Platform (WEP)?
right now also they are quarrelling (c) Saving someone from danger (a) Deepika Padukone
With one another.” No error (d) Easy escape (b) Aamir Khan
(c) Priyanka Chopra
(c) (d) 22. At the cross roads,
(d) Sushant Singh Rajput
15. Sherly wants to know (a) At important point of a decision
(a) (b) At an Important point of journey 33. What was the term used for
(c) At the important road of a journey measurement of land in the Delhi
whether you are going
(d) At an important stage or decision Sultanate period?
(b) (a) Yakut (b) Masahat
to Delhi today night. Directions (Q. Nos. 23-25) Select the
(c) Paibos (d) Visayas
(c) option that is opposite in meaning to
the underlined word/words. 34. Who was the founder of Brahmo
No error Samaj?
(d) 23. It appears that the whole group is
(a) Raja Rammohan Roy
mutinous.
16. The visitor’s to the zoo are (b) Swami Vivekananda
(a) Arrogant (b) Lucky
requested, (c) Dayananda Saraswati
(c) Obedient (d) Sincere (d) Atmaram Pandurang
(a)
in the interest of all concerned, 24. They consider themselves as foes 35. What is biome?
from birth. (a) a large naturally occurring
(b)
(a) Protagonists (b) Opponents community of flora and fauna.
not to carry sticks, stones or food
(c) Friends (d) Soul mates (b) a large naturally occurring
inside and not to tease animals.
(c) 25. This painting has a distinctive community of flora.
element which can be noticed well. (c) a large artificially occurring
No error
(a) Salient (b) Common community of flora and fauna.
(d) (c) Great (d) Unique (d) an artificially occurring
Directions (Q. Nos. 17-19) Each of the 26. Which of the following is the oldest
community of fauna.
following sentences in this section has a Veda? 36. Who was the captain of Indian
blank space with four words or group of hockey team in 1928?
(a) Sama Veda (b) Atharva Veda
words given. Select whichever word or (a) Jaipal Singh Munda
(c) Rig Veda (d) Yajur Veda
group of words you consider most (b) Dhyanchand
appropriate for the blank. 27. Hook pass is related to which game? (c) KD Singh
(a) Volleyball (b) Basketball
This cultural from ...(17)... and indicate (d) Udham Singh
(c) Football (d) Base ball
your response on the answer sheet 37. Who is the author of the book
accordingly. Japan has a name which 28. LPG is a hydrocarbon consisting of ‘Underground Railroad’?
means whimsical or impromptu a mixture of
(a) Nissim Mishal
pictures. It ...(18)... in existence since (a) Methane and Butane
(b) Michael Bar-Zohar
the 12th century when the first ...(19)... (b) Propane and Butane
(c) Colson Whitehead
for this art form was seen. (c) Ethane and Propane
(d) Ethane and Butane (d) Harold Robbins
AFCAT-II ~ Solved Paper 2018 23

38. The light year is a unit of 49. Who was the first Woman Prime 58. The price of an article has been
(a) gravitational force Minister in the world? reduced by 25%. In order to restore
(b) distance (a) Goldan Meir the original price, the new price
(c) velocity (b) Milka Planinc must be increased by
(d) atmospheric pressure (c) Indira Gandhi 1 1 1 2
(a) (b)11 (c) 33 (d) 36
39. Which country has been designated (d) Sirimavo Bandaranaike 11 9 3 3
as a Nodal centre for preparing 50. What is the theme of the 10th 59. Difference of two numbers is 1660.
flash-flood forecasts to Asian edition of the Delhi Dialogue 2018? 1 1
If 6 % of one number is 8 % of the
nations by the World (a) ASEAN–India Relations : Peaceful 2 2
Meteorological Organisation? Village other number, the smaller number is
(a) Thailand (b) Sri Lanka (b) ASEAN–India Relations : Way for (a) 7055 (b) 5395 (c) 3735 (d) 2075
(c) Vietnam (d) India next 25 years 60. 75 g of sugar solution has 30%
(c) Strengthening India–ASEAN sugar in it. Then, the quantity of
40. Who was the first posthumous
Maritime Co-operation sugar that should be added to the
recipient of Bharat Ratna Award? (d) Strengthening India–ASEAN
(a) Lal Bahadur Shastri solution to make the quantity of the
Eco-social Co-operation.
(b) Atal Bihari Vajpayee sugar 70% in the solution, is
(c) Nanaji Deshmukh 51. The sum of a natural number and (a) 125 g (b) 100 g (c) 120 g (d) 130 g
(d) APJ Abdul Kalam its square equals the product of the
61. A reduction of 20% in the price of
first three prime numbers. The
41. Which of the following Vitamin is an apple enables a man to buy
number is
responsible for blood clotting? 10 apples more for ` 54. The
(a) 2 (b) 3 (c) 5 (d) 6 reduced price of apples per dozen is
(a) Vitamin E (b) Vitamin C
(c) Vitamin K (d) Vitamin B 52. If the sum of the digits of any (a) ` 4.32 (b) ` 12.96
integer lying between 100 and 1000 (c) ` 10.80 (d) ` 14.40
42. The Ministry of HRD has launched
is subtracted from the number, the 62. A sum of ` 9000 is to be distributed
which scheme for holistic
result always is among A , B and C in the ratio
development of school education?
(a) divisible by 2 (b) divisible by 9 4 : 5 : 6. What will be the difference
(a) Sarva Shiksha
(c) divisible by 5 (d) divisible by 6 between A’s and C’s shares?
(b) Samagra Shiksha
(c) Rashtriya Shiksha 53. The simplified value of (a) ` 600 (b) ` 1000
(d) Moral Shiksha (c) ` 900 (d) ` 1200
5 + 11 + 19 + 29 + 49 is
43. Which act is known as the Black Act? 63. A bag contains three types of coins,
(a) Rowlatt Act (a) 3 (b) 2 (c) 4 (d) 6 rupee coins, 50 paise coins and
(b) Regulating Act of 1773 25 paise coins totalling 175 coins. If
54. The LCM of two numbers, multiples
(c) Morley-Minto Act 1909 the total value of the coins of the
of 12 is 1056. If one of the numbers
(d)Government of India Act, 1935 each kind be the same, the total
is 132, the other number is
amount in the bag is
44. What were the number of (a) 12 (b) 72 (c) 96 (d) 132
constituencies in India during the (a) ` 75 (b) ` 175 (c) ` 300 (d) ` 126
55. If the sum of two numbers be
first parliamentary election? 64. I purchased 120 exercise books at
multiplied by each number 1
(a) 401 (b) 402 (c) 400 (d) 410 separately, the products so obtained the rate of ` 3 each and sold of
45. Which of the following is India’s are 247 and 114. The sum of the 3
1
first indigenously developed light numbers is them at the rate of ` 4 each, of
combat fighter aircraft? (a) 19 (b) 20 (c) 21 (d) 23 2
(a) Tejas them at the rate of ` 5 each and the
56. The average salary of all the rest at the cost price. My profit per
(b) Su-30 MKI
workers in a workshop is ` 8000. cent was
(c) Jaguar The average salary of 7 technicians 4
(d) MIG-21 BISON is ` 12000 and the average salary of (a) 44% (b) 44 %
9
46. Riga is the capital of which the rest is ` 6000. The total number 2
(c) 44 % (d) 45%
country? of workers in the workshop is 3
(a) Latvia (b) Finland (a) 20 (b) 21 (c) 22 (d) 23
(c) Sweden (d) Czech Republic 65. A man sells two chairs at ` 120
57. The average age of 11 players of a each and by doing so gains 25% on
47. Where is Indira point located? cricket team is increased by 2 one chair and loses 25% on the
(a) North Andaman Island months, when two of them aged other. His loss on the whole (in `) is
(b) Little Nicobar Island 18 yr and 20 yr are replaced by two (a) 20 (b) 16 (c) 25 (d) 30
(c) Great Nicobar Island new players. The average age of the
(d) Car Nicobar Island new players is Directions (Q. Nos. 66-68) In each of
(a) 19 yr 1 month the following questions, choose the word
48. JAXA is an aerospace agency of which is different from others.
which country? (b) 19 yr 6 months
(a) Russia (b) China (c) 19 yr 11 months 66. (a) Admiral (b) Colonel
(c) Japan (d) UK (d) 19 yr 5 months (c) Commodore (d) Commander
24 AFCAT-II ~ Solved Paper 2018

67. (a) Henry Bequeeral Directions (Q. Nos. 77-79) What 82. If ‘MOTHER’ is coded as ‘TOMREH’,
(b) Otto Hohn comes in place of question mark (?) in what should be the code for the
(c) Madam Curie the series given below? word ‘NEPHEW’?
(d) Einstein 77. BDF, HJL, NPR, ? (a) ENHPWE (b) PENWEH
(c) WEHPEN (d) HPENWE
68. (a) Prism (b) Cube (a) OQS (b) TUV
(c) Cone (d) Square (c) TVX (d) UVW 83. In a code language, the following
alphabets are coded in a particular
69. Which of the following diagrams 78. ABCD, BCDA, CDAB, ?, ABCD
way
best depicts the relationship among (a) BADC (b) DABC
Thief, Criminal, Police? (c) DBAC (d) DACB P N C Y AD J R B Q
2 7 5 1 6 8 4 3 9 0
79. YVP, WTN, URL, ?
(a) (b) Which group of numbers can be
(a) VSP
(b) SRJ decoded from the following?
(c) SPJ QAJYNR
(c) (d) (a) 064713
(d) TQL
(b) 064173
70. Which of the following diagrams 80. Two statements are given followed (c) 064513
best depicts the relationship among by two conclusions. Read the (d) 061473
Animals, Amphibian, Tortoise? conclusions and then decide which
of the conclusions, if any, logically Directions (Q. Nos. 84-87) In each of the
(a) (b) follows from the two given following questions, find the answer figure
statements, disregarding the in which problem figure is embedded.
known facts. 84. Problem Figure
(c) (d) Statements
Some cows are deer.
71. Which of the following diagrams Some deer are fish.
best depicts the relationship among Conclusions Answer figures
Haryana, Gurgaon, Punjab and I. Some cows are fish.
Ludhiana? II. Some fish are cows.
(a) Only Conclusion I follows
(a) (b) (a) (b) (c) (d)
(b) Only Conclusion II follows
(c) Neither I nor II follows 85. Problem Figure
(d) Both Conclusions I and II follow
(c) (d)
81. One statement is given followed by
two assumptions numbered I and
II. You have to consider the Answer Figures
Directions (Q. Nos. 72-76) In each of statement and the following
the following questions, find the correct
assumptions and decide which of
alternative which has the same relation the assumptions is implicit in the
with the third word as the words of the statement. (a) (b) (c) (d)
given pair bear.
Statement
72. Sheep: Mutton:: Deer? 86. Problem Figure
The government has decided to pay
(a) Veal (b) Meat compensation to the tune of ` 1 lakh
(c) Flesh (d) Venison to the family members of
73. Tea: Leaves : : Coffee: ? those who are killed in railway
accidents. Answer Figures
(a) Plant (b) Leaves
(c) Seeds (d) Stimulant Assumptions
74. Deep: Shallow: : Sharp: ? I. The government has enough
(a) Knife (b) Blade funds to meet the expenses due (a) (b) (c) (d)
(c) Blunt (d) Ocean to compensation.
II. There may be reduction in 87. Problem Figure
75. Horse : Stable : : Man : ? incidents of railway accidents in
(a) Woman (b) Den near future.
(c) Clothes (d) House
(a) Only Assumption I is implicit
76. Mason : Wall : : Carpenter :? (b) Only Assumption II is implicit Answer Figures
(a) Glass (c) Neither Assumption I nor II is
(b) Chair implicit
(c) Pen (d) Both Assumptions I and II are
(a) (b) (c) (d)
(d) Book implicit
AFCAT-II ~ Solved Paper 2018 25

Directions (Q. Nos. 88-91) In each of Answer Figures Answer Figures


the following questions, choose the
figure from the given answer figures,
that can replace the ‘?’ to complete the
problem figure. (a) (b) (c) (d) (a) (b) (c) (d)
88. Problem Figure Directions (Q. Nos. 92 and 93) In the 95. Problem Figures
following questions, select that figure
from the set of answer figures
which would come in place of question
(1) (2) (3) (4) (5)
mark ‘?’
? 92. Problem Figures
Answer Figures

Answer Figures
?
(a) (b) (c) (d)
(1) (2) (3) (4)
96. Problem Figures
(a) (b) (c) (d) Answer Figures

89. Problem Figure


S

(1) (2) (3) (4) (5)


? (a) (b) (c) (d) Answer Figures
93. Problem Figures

? (a) (b) (c) (d)


Answer Figures
(1) (2) (3) (4) Directions (Q. Nos. 97-100) In each of
Answer Figures the following questions, three out of
four figures are similar in a certain
(a) (b) (c) (d) way and so form a group. Choose
the figure that does not belong to the
90. Problem Figure
(a) (b) (c) (d) group.
97.
Directions (Q. Nos. 94-96) In each of
the following questions, a group of five
figures following a certain sequence is (a) (b) (c) (d)
? given as problem figures. Problem
figures are followed by another group of 98.
Answer Figures four figures known as answer figures.
Find out the figure from the answer
figures which when placed next to the (a) (b) (c) (d)
problem figures will continue the 99.
sequence of problem figures.
(a) (b) (c) (d)
94. Problem Figures
91. Problem Figure (a) (b) (c) (d)
100.
(1) (2) (3) (4) (5)

? (a) (b) (c) (d)


26 AFCAT-II ~ Solved Paper 2018

Answers
1. (c) 2. (d) 3. (a) 4. (b) 5. (b) 6. (b) 7. (d) 8. (a) 9. (a) 10. (b)
11. (b) 12. (a) 13. (b) 14. (c) 15. (c) 16. (a) 17. (a) 18. (b) 19. (a) 20. (c)
21. (b) 22. (a) 23. (c) 24. (c) 25. (b) 26. (c) 27. (b) 28. (b) 29. (b) 30. (a)
31. (b) 32. (d) 33. (b) 34. (a) 35. (a) 36. (a) 37. (c) 38. (b) 39. (d) 40. (a)
41. (c) 42. (b) 43. (a) 44. (a) 45. (a) 46. (a) 47. (c) 48. (c) 49. (d) 50. (c)
51. (c) 52. (b) 53. (a) 54. (c) 55. (a) 56. (b) 57. (c) 58. (c) 59. (b) 60. (b)
61. (b) 62. (d) 63. (a) 64. (b) 65. (b) 66. (b) 67. (d) 68. (d) 69. (b) 70. (a)
71. (a) 72. (d) 73. (c) 74. (c) 75. (d) 76. (b) 77. (c) 78. (b) 79. (c) 80. (c)
81. (a) 82. (b) 83. (b) 84. (c) 85. (c) 86. (a) 87. (b) 88. (d) 89. (b) 90. (c)
91. (d) 92. (d) 93. (c) 94. (d) 95. (b) 96. (d) 97. (d) 98. (a) 99. (c) 100. (d)

Hints and Solutions


1. (c) As stated in the passage curcumin 11. (b) ‘Pleasant’ is nearer to the meaning of 23. (c) The word ‘mutinuous’ means a
has positive effect on people without underlined word ‘salubrious’. Other person or soldier refusing to obey the
dementia and also on people suffering options are not relevant here. authority. Its antonym is ‘obedient’. The
from Alzheimer’s disease. 12. (a) ‘Deserted’ is closest in meaning to word means a person who is willing to
2. (d) Previously means earlier or in the word desolated. The word means to comply with an order or request.
time gone by. leave someone in a situation when they 24. (c) ‘Foe’ means enemy or opponent. Its
3. (a) As inferred from the passage, have no one to support. The word antonym is friends which means a
consumption of turmeric can reduce the desolated also means to feel miserable or person with whom one has bond of
chance of getting Alzheimer’s diseases as gloomy. affection.
it contains as antioxidant called curcumin. 13. (b) Delete ‘on’ after insist. The word does 25. (b) The word ‘distinctive‘ means unique
4. (b) Prevalence is the best option for this not require any preposition. It means to or remarkable or uncommon. So, from
sentence, The word means widespread say,firmly or demand forcefully. the options given ‘common’ is
presence or a condition which is quite 14. (c) Use ‘each other’ in place of ‘one appropriate antonym which means
common. another’ as reference is for two as something which is found or done very
suggested by the use of word ‘both’. often or which is prevalent.
5. (b) ‘annoyed with’ is the most
appropriate option as when we are angry 15. (c) Today night is not correct. We must 26. (c) The Rig veda is the oldest known Vedic
with people we use ‘annoyed with’. write ‘tonight’. That means approaching Sanskrit text most likely between 1900 to
evening or night. 1100 BCE. It is one of the four sacred
6. (b) With verb ‘shouted’, preposition to be canonical texts (Sruti) of hinduism.
used is ‘at’. So option ‘b’ ‘shouted at’ is 16. (a) Use of apostrophe is incorrect here.
Just write the ‘visitors’. The text is layered consisting of the
correct here as we shout at some or the
Samhita, Brahmanas, Aranyakas and
other person. When we ‘shout to’ some 17. (a) originating Upanishads. The Rig veda Samhita is a
person, we want to be heard. Here, 18. (b) has been collection of 10 mandalas with 1028
option (b) should be used. 19. (a) instance hymns.
7. (d) ‘complaining of’ is the most suitable 20. (c) ‘Follow suit’ means ‘doing the same 27. (b) The term hook pass is associated
option. When we speak about illnesses as someone else has just done.’ e.g. with basketball. Other terminology
we use complaining of. When one airline reduces ticket prices, associated with sport include dribbling,
8. (a) ‘insists’ is appropriate option. ‘Insist’ the others usually follow suit. free throw, jump ball etc.
means to say firmly or demand forcefully. 21. (b) The idiom ‘close shave’ means 28. (b) Liquid Petroleum Gas (LPG) is a
So, here no need to use any preposition. miraculous escape. A close shave means hydrocarbon consisting of a mixture of
9. (a) ‘get along’ is the phrasal verb which a situation where one avoids something propane and butane. It is used as fuel in
should be used here in the blank. ‘Get dangerous. e.g. The car almost hit the heating appliances, cooking equipments
along’ means to have a friendly child who ran out in front of it. It was a and vehicles.
relationship with others. ‘close shave.
It is increasingly used as an aerosol
10. (b) ‘Economical’ is synonym of the 22. (a) The idiom ‘At the cross roads’ means prepellant and a refrigerant replacing
underlined word ‘thrifty’ which means at important point of a decision. When a chlorofluorocarbons in an effort to reduce
using money and other resources choice must be made, it is called at the damage to the ozone layer.
carefully. cross roads.
29. (b) Lewis Hamilton, a British racing
Other options are antonyms and do not e.g. He is at the cross roads in his driver who races in Formula One for
give similar meaning. An economic career-either he stays in his current job Mercedes-AMG Petronas Motorsport has
person is one who is careful with and waits for promotion or accept new
spending money. won the 2018 Formula One French
post abroad.
Grand Prix Tournament.
AFCAT-II ~ Solved Paper 2018 27

30. (a) The North Atlantic Treaty 40. (a) Lal Bahadur Shastri was the first 50. (c) The 10th edition of the Delhi
Organisation (NATO) is an person to have received the Bharat Ratna Dialogue 2018 was held at New Delhi
intergovernmental military alliance Award posthumously in 1966. hosted by former minister of External
between 29 North American and Nanaji Deshmukh and Bhupen Hazarika Affairs of India Sushma Swaraj. The
European Countries. The headquarters of are the Posthumous recipient of Bharat theme of this edition was ‘Strengthening
NATO is located in Brussels, Belgium. Ratna Award 2019. India-ASEAN Maritime Co-operation’.
31. (b) Taseometer is an instrument, used for 41. (c) Vitamin K is responsible for blood Delhi Dialogue is the major annual
measuring physical strain in a structure. clotting. The human body requires program for discussing poltical-security,
vitamin K for complete synthesis of economic and socio-cultural partnership
32. (d) Actor Sushant Singh Rajput has between India and South-east Asian
certain proteins that are needed for blood
collaborated with NITI Aayog for the Nations (ASEAN).
clotting.
promotion of Women Entrepreneurship
42. (b) The Union Minister for Human 51. (c) Let the required number be x.
Platform (WEP) launched by the Central
Resource Development (HRD), Shri According to the question,
Government.
Prakash Javedkar has launched the x2 + x = 2 × 3 × 5
The aim of the WEP is to build an
ecosystem for women across India to ‘Samagra Shiksha’ Scheme on 24th May, ⇒ x2 + x − 30 = 0
realize their entrepreneurial aspirations, 2018 for holistic development of school ⇒ x2 + 6x − 5x − 30 = 0
scale up innovative initiatives and education. It is an integrated scheme for
school education extending support to ⇒ x(x + 6) − 5 (x + 6) = 0
chalk-out sustainable, long-term
strategies for their business. states from pre-school to senior ⇒ (x − 5) (x + 6) = 0
secondary level. ∴ x=5
33. (b) The term ‘Masahat’ was used for
43. (a) The Anarchical and Revolutionary Hence, the number is 5.
measurement of land in the Delhi
Sultanate Period. Crimes Act of 1919 or Rowlatt Act is 52. (b) Such number is always divisible by 9.
34. (a) The Brahmo Samaj was founded by populary known as ‘Black Act’. Rowlatt To make it clear, you can take some
Act was a legislative act passed by example.
Raja Rammohan Roy in 1828 in
Calcutta. Imperial Legislative Council in 1919 that e.g., 496 − (4 + 9 + 6) = 477,
It was the first important organisation of allowed the British to arrest and jail which is divisible by 9.
religious reforms. The Brahmo Samaj anyone without trial if suspected to be 971 − (9 + 7 + 1) = 954,
forbade idol worship and discarded plotting against them. which is divisible by 9.
meaningless rites and rituals. 44. (a) The first parliamentary election were 53. (a) Expression
35. (a) Biome is a large naturally occurring held at 401 constituencies. Voters from
these constituencies voted for 489 seats = 5 + 11 + 19 + 29 + 49
community of flora and fauna occupying
a major habitat. e.g. tundra, forests, out of which INC won complete majority
with 364 seats. = 5 + 11 + 19 + 29 + 7
grasslands, deserts etc.
36. (a) Jaipal Singh Munda was the captain 45. (a) Tejas is India’s first indigenously = 5 + 11 + 19 + 6
of India Hockey team in 1928 Olympics developed Light Combat fighter aircraft
held at Amsterdam. Under his captaincy, designed by the Aeronautical = 5 + 11 + 5 = 5+ 4
the India team won its first Olympic Gold Development Agency (ADA) and = 9=3
medal. Manpreet Singh is the current Hindustan Aeronautics Limited (HAL) for
Indian Air force and the Indian Navy. 54. (c) Given, first number = 132
captain of India Hockey team.
46. (a) Riga is the capital of Latvia. Latvia is Since, each of the two numbers is a
37. (c) ‘The Underground Railroad’, multiple of 12 (given),
published in 2016, is the sixth novel by a country in the Baltic region of Northern
Europe having ‘Euro’ as its official currency. ∴ HCF = 12 and LCM = 1056 (given)
American author Colson Whitehead.
We know that,
The novel is about American Slavery and 47. (c) Indira point is the name of the
LCM × HCF =First number × Second
has won the 2017 Pulitzer Prize for Southernmost point of India’s territory. It
number
Fiction. is located on Great Nicobar Island in the 1056 × 12
38. (b) Light year is a unit of distance. It is the Nicobar Islands. ∴Second number = = 96
132
distance that light can travel in one year. 48. (c) Japan Aerospace Exploration Agency
55. (a) Let the numbers be x and y.
1 light year = 946. × 1015 m (JAXA) is the Japanese national
aerospace and space agency. Through According to the question,
39. (d) The World Meteorological the merger of three previously x(x + y) = 247
Organisation (WMO) has designated India independent organisations, JAXA was ⇒ x2 + xy = 247 … (i)
as a nodal centre for preparing flash- formed on 1st October, 2003.
flood forecasts to Asian Nations including and y (x + y) = 114
Vietnam, Sri Lanka, Myanmar and Thailand. 49. (d) Sirimavo Bandaranaike was elected y2 + xy = 114 … (ii)
It implies that Indian Meteorological as the world’s first female Prime Minister
On adding Eqs. (i) and (ii), we get
in 1960. She served as the Sri Lankan
Department (IMD) under Ministry of x2 + xy + xy + y2 = 247 + 114
Earth Sciences (MoES) will have to Prime Minister for three times and was
the leader of the Sri Lankan Freedom ⇒ x2 + 2xy + y2 = 361
develop a customised weather model that
can issue advance warning of floods. Party. Indira Gandhi was the first female ⇒ (x + y)2 = 192
Prime Minister of India. ⇒ x + y = 19
28 AFCAT-II ~ Solved Paper 2018

56. (b) Let total number of workers in 61. (b) Let the original price of apples be ` x 64. (b) CP of 120 exercise books at the rate
workshop = x per dozen. ` 3 each
Then, total salary of all workers 80 4x = ` (120 × 3) = ` 360
∴ New price = x × =` per dozen
= 8000 × x 100 5 SP of 40 at the rate ` 4 each
According to the question, According to the question, = ` (40 × 4) = ` 160
8000x = 7 × 12000 + (x − 7) × 6000 54 54 10
− = SP of 60 at the rate ` 5 each
⇒ 8000x = 84000 + 6000x − 42000 4x x 12 = ` (60 × 5) = ` 300
⇒ 8000x − 6000x = 84000 − 42000 5 SP of remaining 20 books
⇒ 2000x = 42000 [∵ 1 dozen = 12 pieces] = ` (20 × 3) = ` 60
∴ x = 21 ⇒ 
54
5 1 5
−  = Total SP = ` (160 + 300 + 60)
57. (c) Total increase in age  4x x  6 = ` 520
5 − 4 5
= 11 × 2 = 22 months ⇒ 54  =
Profit = ` (520 − 360)
∴Sum of the ages of both new players  4x  6 = ` 160
= (18 + 20) yr 22 months 54 5 Profit × 100
⇒ = ∴Profit% =
= 39 yr 10 months 4x 6 CP
54 × 6 =
160
× 100 =
400
∴Average age of the new players ⇒ 4x =
39 yr 10 months 5 360 9
= 324 4
2 ∴ x= = 162
. = 44 %
= 19 yr 11 months 20 9
4 × 162
58. (c) Percentage increase Hence, reduced price = 65. (b) Given,
a 5 Total SP = 120 + 120 = ` 240
= × 100% 648. SP × 100
100 − a = = ` 12.96 Then, CP of first chair =
25
5 100 + Profit%
= × 100 [∵ here, 62. (d) Total amount = ` 9000 120 × 100
100 − 25 =
A’ s share = 4x 125
100 1 120 × 4
a = 25] = = 33 % B’ s share = 5x =
3 3 5
and C’ s share = 6x
59. (b) Let the numbers be x and y and
Then, 4x + 5x + 6x = 9000 = 24 × 4 = ` 96
x > y. SP × 100
⇒ 15x = 9000 CP of second chair =
According to the question,
∴ x = 600 100 − Loss%
1 1 120 × 100
6 % of x = 8 % of y Now, =
2 2 75
13 17 A’ s share = 4 × 600 = ` 2400
⇒ % of x = % of y and C’ s share = 6 × 600 = ` 3600 = 40 × 4 = ` 160
2 2
17 Difference between A’ s and C’ s share Total CP = 160 + 96 = ` 256
⇒ 13x = 17 y ⇒ x = y Loss = 256 − 240 = ` 16
13 = ` (3600 − 2400) = ` 1200
According to the question, Alternate Method 66. (b) Except Colonel, all others are different
4 ranks in Navy while colonel is a rank in
x − y = 1660 A’ s share = 9000 ×
17 15 army.
⇒ y − y = 1660
13 = 600 × 4 = ` 2400 67. (d) Except Einstein, all others belong to
17 y − 13y 6
⇒ = 1660 ⇒ 4y = 1660 × 13 C’ s share = 9000 × the research in Radio activity.
13 15 68. (d) Except square, all others are 3-D objects.
1660 × 13 = 600 × 6 = ` 3600
⇒ y= = 5395 69. (b)
4 ∴Difference between A’ s and C’ s shares
Criminals
60. (b) Sugar in original solution = 3600 − 2400 = ` 1200
Thieves Police
75 × 30 63. (a) Let the number of coins of ` 1 be x.
= = 22.5 g
100 Total value of the coins of each kind is All the thieves are criminals but police is
Let x g of sugar be mixed. same, then the number of 50 paisa coins different.
According to the question, is 2x and the number of 25 paisa coins is
22.5 + x 70. (a)
4x.
× 100 = 70
75 + x According to the question, Animals Amphibians
⇒ 2250 + 100x = 75 × 70 + 70x x + 2x + 4x = 175
⇒ 2250 + 100x = 5250 + 70x ⇒ 7x = 175
175 Tortoise
⇒ 30x = 5250 − 2250 ∴ x= = 25
= 3000 7 Tortoise are animal and amphibians
3000 ∴Total amount in bag both. Some animals are amphibians and
⇒ x= = 100 g
30 = 25 + 25 + 25 = ` 75 vice-versa.
AFCAT-II ~ Solved Paper 2018 29

71. (a) Assumption II is not implicit because by 90. (c) Answer figure (c) will complete the
Haryana
giving compensations we cannot reduce pattern of problem figure.
Punjab
the number of accidents. 91. (d) Answer figure (d) will complete the
The accidents can be avoided by taking pattern of problem figure.
Gurgaon Ludhiana appropriate precaution any measures.
92. (d) From figure (1) to (2), the whole
Gurgaon is in Haryana and Ludhiana is 82. (b) As, M O T H E R figure rotates by 45° in clockwise
in Punjab. direction and gets inverted on its base
72. (d) As ‘Mutton’ is the meat of ‘Sheep’, line. One of the end elements is
T O M R E Hs
similarly ‘Venison’ is the meat of ‘Deer’. replaced.
Therefore, N E P H E W 93. (c) The figure is inverted and bent lines
73. (c) As ‘Tea’ is obtained from ‘Leaves’
similarly ‘Coffee’ is obtained from on the base get reversed. The arrow
‘Seeds’. inside the figure rotates through 135°
P E N W E H
anti-clockwise.
74. (c) ‘Deep’ is opposite to ‘Shallow,’ in the
same way ‘Sharp’ is opposite to ‘Blunt’. 83. (b) Q A J Y N R 94. (d) It is clear from the pattern of series
that the figure rotates 90°, 135°, 180°,
75. (d) A ‘Horse’ lives in ‘Stable’, similarly a
0 6 4 1 7 3 225° and finally 270°, respectively in the
‘Man’ lives in a ‘House’.
clockwise direction.
76. (b) ‘Mason’ builds a ‘Wall’, in the same 84. (c) The problem figure is embedded in 95. (b) In each subsequent figure, 0, 1, 2, 3,
way a ‘Carpenter’ makes a ‘Chair’. answer figure (c). … lines are added alternatively in
77. (c) The pattern is as follows anti-clockwise and clockwise
2 +6 8 +6 14 +6 20 directions.
1 st Letter: B H N T
4 10 +6 16 +6 22
96. (d) Symbols of first block move in such a
2nd Letter: +6
D J P V way that figure at the top right position is
6 +6 12 +6 18 +6 24
85. (c) The problem figure is embedded in moved diagonally opposite, symbol at
3rd Letter: F L R X
answer figure (c). bottom right is moved at top right
position, symbol at left bottom position is
78. (b) In the following series, the next term
shifted to bottom right and figure at the
is obtained by shifting the first letter of
left top position is reversed at the same
the previous term to the last place.
place. The method is same from problem
∴? = DABC
figure (3) to (4) and hence, it will be
79. (c) The pattern is as follows, 86. (a) The problem figure is embedded in same from problem figure (5) to answer
–2 –2 –2 answer figure (a). figure.

Y V P W T N U R L S P J
97. (d) In consecutive figure the U-shaped
element gets inverted in order as 1 in first
–2 –2 –2 step, 2 in second step, 3 in third and so
–2 –2 –2 on this rule gets rotated in figure (d), so it
is the odd figure among the options.
∴ ? = SPJ 87. (b) The problem figure is embedded in
answer figure (b). 98. (a) Figure (a) does not belong to the
80. (c) group as all the other figures contain
Cows Deer Fish same elements.
99. (c) In all other figures, if both the figures
Conclusions on right hand side are first inverted and
I. (û) II. (û) then joined together, the figure on left
88. (d) Answer figure (d) will complete the
81. (a) Assumption I is implicit because a hand side is obtained.
pattern of problem figure.
decision is made keeping in view the 100.(d) In figure (d), both the shaded designs
89. (b) Answer figure (b) will complete the
availability of resources to execute it. have the common base i.e., the middle
pattern of problem figure.
line.
30 AFCAT-I ~ Solved Paper 2018

INDIAN AIR FORCE


AFCAT-I

Solved Paper 2018


Time : 2 Hrs MM : 300
INSTRUCTIONS
■ The set contains a total of 100 questions, Comprising Verbal Ability in English, General Awareness, Numerical Ability and Reasoning and
Military Aptitude Test.
■ Each correct question carry 3 Marks and there will be negative marking of 1 Mark for each incorrect attempt.
■ Total time duration will be 2 hrs (120 minutes).
■ No marks will be deducted for unattempted questions.

Directions (Q. Nos. 1-4) Each sentence (a) extraordinary, complaisance 10. A damp squib
in the below given questions has two (b) specific, obedience
(a) Rainy weather
blanks, each blank indicates that (c) tantalising, adherence
something has been omitted. Beneath the (b) A disappointing result
(d) extenuating, compliance
sentence are sets of words. Choose the set (c) A skirt in a laundry
of words for each blank that best fits into
Directions (Q.Nos. 5-9) Some parts of (d) None of the above
the sentences have errors and some are
the meaning of the sentence as a whole.
correct. Find out which part of a sentence
11. To smell a rat
1. It is a well-known ….. that the lover has an error and blacken the oval [.] (a) To smell foul (b) To see a rat
of the sea craves for dry land, the corresponding to the appropriate letter (c) To chase a rat (d) To be suspicious
age old …… to be where we are not. (a), (b), (c). If a sentence is free from 12. Yeoman’s services
(a) belief, antipathy error, blacken the oval corresponding to (a) Medical help (b) Excellent work
(b) anomaly, demiurge (d) in the Answer Sheet. (c) Social work (d) Hard work
(c) credo, inspiration 5. I was (a)/ laying down (b)/ when 13. Flippant
(d) paradox, yearning the door bell rang. (c)/ No error (d) (a) Highly critical
2. Such stalling tactics are ……… to 6. I told the teacher (a)/ that the (b) Not showing deserved respect
all fans and cannot be ........... . homework set for the day (b)/ was (c) Casual
(a) repugnant, condoned much too heavy for us to complete. (d) Indifferent
(b) anathema, ascertained (c)/ No error (d)
(c) injurious, explained Directions (Q. Nos. 14-16) In these
(d) unfair, superseded
7. Someone, they don’t know (a)/ who questions, you have a brief passage with
knocked at (b)/ their door in three questions following passage. Read
3. The ……… of democratic freedom midnight. (c)/ No error (d) the passage carefully and choose the
is dialogue and the ………
interchange of diverse ideas.
8. Seldom if ever (a)/ nature does best answer to each question out of the
operate (b)/ in closed and separate four alternatives.
(a) deterioration, untrammeled compartments. (c)/ No error (d)
(b) height, restrained From the world of magic, hypnosis is
(c) essence, unhampered 9. Mohan leapt (a)/ on the moving into the world of medicine.
(d) alienation, compulsory opportunity (b)/ that came his way. From hocus-pocus performed by men in
(c)/ No error (d) black capes, to hypnotherapy practised
4. Although, there were ………
circumstances in this particular Directions (Q. Nos. 10-13) In these by doctors in white coats. The purpose
violation of the law, the judge ruled questions, four alternatives are given is to help people stop smoking, lose
that there had to be strict ……… or for the idiom/phrase. Choose the weight, overcome phobias and control
there would be no law at all. alternative which best expresses the pain in a variety of medical situations
meaning of the idiom/phrase. from childbirth to cancer.
AFCAT-I ~ Solved Paper 2018 31

Research laboratories are currently At the time, the White House was as 30. With which country India has
checking out the success rate of serene as a resort hotel out of season. recently signed a MoU for water
therapy under hypnosis, while medical The corridors were …(23)… in the conservation in India?
journals stand by to publish the results. various offices. …(24)… gray men on (a) France (b) Germany
And the important thing is, nobody is waistcoats talked to one another in (c) Israel (d) Bangladesh
laughing. low-pitched voices. The only colour or
31. Who built the Vijay Stambha Tower
In the 1840’s, a British doctor in choler, curiously enough, was provided of victory in Chittorgarh?
Calcutta created a controversy by by President Eisenhower himself.
(a) Maharana Pratap
performing over 1000 operations with Apparently, his …(25)… was easily set
(b) Rana Kumbha
hypnosis as the only anaesthesia. off; he scowled when he paced the
(c) Rana Sanga
During the World Wars, German and corridors.
(d) Kunwar Durjan Singh
British doctors used hypnosis to treat 23. (a) striking (b) hollow
war neuroses. 32. Who raised the slogan ‘Swaraj is
(c) empty (d) white my birthright and I shall have it’?
14. Hypnosis means
24. (a) Quiet (b) Faded (a) Mahatma Gandhi
(a) auto-suggestion
(c) Loud (d) Stentorian (b) Subhash Chandra Bose
(b) suggestion made in trance
(c) Bal Gangadhar Tilak
(c) anaesthesia 25. (a) laughter (b) curiosity (d) Lala Lajpat Rai
(d) hocus-pocus (c) humour (d) temper
33. Axillary bud develops into which of
15. ‘Nobody is laughing’ at hypnothe 26. ‘Vikalp’ is a scheme launched by the following part of the plant?
rapy now, because they are Indian Railways to help wait-listed (a) Fruit (b) Leaf
(a) sad (b) angry passengers. Which of the following (c) Branch (d) Roots
(c) taking it seriously is not true about this scheme?
(d) annoyed 34. Xylem helps in transportation of
(a) Confirmed berths in alternate
which of the following ?
16. The purpose of hypnotherapy is to trains
(a) Food (b) Water
(a) cure patients (b) No-extra charges will be taken (c) Nutrients
(b) make life easier from passengers (d) Both food and water
(c) carry out research (c) Wait-listed passengers can avail
(d) check out the success rate 35. Match the following.
opportunity of travelling in
Directions (Q. Nos. 17-19) In these List I List II
Rajdhani/Shatabdi/Special trains (Revolution) (Leader)
questions, choose the word similar in even when booking made is in
meaning to the word given. A. Green
other mail/express trains 1. Durgesh Patel
Revolution
17. Barren (d) Vikalp scheme will be initially B. White MS
(a) Good (b) Wholesome 2.
available for e-tickets only Revolution Swaminathan
(c) Unproductive (d) Profitable C. Pink Verghese
27. Who discovered the Cholera 3.
18. Infamy Revolution Kurien
causing germ?
(a) Notoriety (b) Glory (a) Filippo Pacini Codes
(c) Integrity (d) Familiarity (b) Robert Koch A B C A B C
19. Intrepid (c) M Laveran (a) 3 2 1 (b) 2 3 1
(a) Hesitant (b) Fearless (d) Felix Hoffman (c) 1 2 3 (d) 1 3 2
(c) Extrovert (d) Rash 36. Who shot dead John Saunders on
Directions (Q. Nos. 20-22) Choose the 28. Match the following 17th December, 1927?
word opposite in meaning to the word List I List II (a) Bhagat Singh
given. A. Mithali Raj 1. Hockey (b) Mangal Pandey
20. Compassionate 3000 m (c) Sukhdev
B. Poonam Rani 2.
Steeplechases (d) Bipin Chandra Pal Singh
(a) Unlawful (b) Heartless
C. Lalita Babar 3. Cricket 37. Sardar Vallabhbhai Patel was the
(c) Untrustworthy (d) Indecisive
Codes leader of ………
21. Tasty
A B C A B C (a) Bhoodan Movement
(a) Delicious (b) Insipid
(b) Rowlatt Satyagraha
(c) Appetising (d) Palatable (a) 3 2 1 (b) 1 2 3
(c) Bardoli Satyagraha
(c) 1 3 2 (d) 3 1 2
22. Triumph (d) Swadeshi Movement
(a) Defeat (b) Victory 29. Who is the author of the book titled
Citizen and Society? 38. Who amidst the following is a
(c) Success (d) Subjugation
distinguished painter?
(a) Pranab Mukherjee
Directions (Q. Nos. 23-25) Fill up the (b) Hamid Ansari
(a) Uday Shankar
blanks in the passage given below with (b) Sonal Mansingh
(c) Nandan Nilekani
the most appropriate word from the (c) Amrita Shergill
(d) Satyajit Ray
options given for each blank. (d) Yamini Krishnamurthy
32 AFCAT-I ~ Solved Paper 2018

39. Who among the following was 49. ……… scheme by the Central 59. When coded OPTRRE reads
awarded with Padma Shri 2017 in Government will strength the bond as PORTER. In the same way,
the field of Sports Hockey? between all the state, regions and what does the following
(a) Sardar Singh products of India. read as?
(b) PR Shreejesh (a) Uday Desh Ka Aam Nagrik EROPTR
(c) Ramandeep Singh (b) Urja Ganga (a) ROPE (b) PROPER
(d) Yuvraj Walmiki (c) Ek Bharat Shreshtha Bharat (c) PORT (d) REPORT
(d) Namami Ganga Yojana
40. Which of the following is the Direction (Q. No. 60) Two statements
largest gland in human body? 50. The Union Government on 22nd are given below followed by two
(a) Thyroid (b) Liver May, 2016 appointed whom as the Conclusions I and II. You have to
(c) Kidney (d) Pancreas new Lieutenant Governor of the consider statements to be true even,
Union Territory (UT) of if they seem to be at variance from
41. On which principle does the Puducherry?
hydraulic lift works? commonly known facts. You have
(a) Kiran Bedi to decide which of the given conclusions,
(a) Newton’s law
(b) Kalyan Singh if any, follow from the given statements.
(b) Pascal’s law
(c) Ram Naik Indicate your answer.
(c) Archimedes’ law
(d) Mukul Sangma
(d) Joule’s law 60. Statements No man is a donkey.
51. Choose the word which is different Rahul is a man.
42. At what temperature (in degree from the other.
celsius), the numerical values on Conclusions
(a) Leo (b) Equator I. Rahul is not a donkey.
Celsius and Fahrenheit scales
(c) Libra (d) Cancer
become equal? II. All men are not Rahul.
(a) −40 (b) 40 Directions (Q. Nos. 52-55) In these (a) Only I follows
(c) 273 (d) −273 questions, select the related (b) Only II follows
letter/word/ number from the given (c) Both follow
43. In MICR, what does ‘I’ stands for?
alternatives. (d) Neither I nor II follows
(a) Interactive (b) Information
(c) Ink (d) Instruction 52. Cataract : Eye : : Pneumonia : ? Direction (Q. No. 61) Which one set of
(a) Brain letters when sequentially placed at the
44. What is the process of conversion of
solid state directly to gaseous state (b) Ear gaps in the given letter series shall
called? (c) Lungs complete it?
(d) Nerves and Limbs
(a) Evaporation (b) Condensation 61. ab__d__aaba__na__badna__b
(c) Sublimation (d) Distillation 53. TTT : 777 : : RRR : ? (a) andaa (b) babda
45. Which of the following is an Indian (a) 555 (b) 666 (c) 888 (d) 999 (c) badna (d) dbanb
military decoration awarded for valour, 54. YAD : NUS : : ? : NOOM 62. Below two positions of a dice
courageous action or self-sacrifice (a) NTHIG (b) HIGIN are shown. What will be on
away from the battlefield? (c) GHTIN (d) THGIN opposite of the face at which 1 is
(a) Ashok Chakra written?
(b) Dada Saheb Phalke Awards 55. 7 : 24 : : ?
(c) Arjuna Award (a) 30 : 100 (b) 23 : 72 4 5
(d) Padma Shri (c) 19 : 58 (d) 11 : 43 1 5 4 2

46. Which nation will host the FIFA Men’s 56. Akbar is standing facing East. After
walking 15 m he turned left and (a) 2 (b) 3 (c) 4 (d) 1
World Cup to be held in the year 2018?
(a) Japan
walked 25 m. Then, he turned right 63. A statement with two assumptions
(b) South Korea
and walked 10 m. Again, he turned are given followed by four
right and walked 25 m. How far is alternatives. Select the one which is
(c) China
he from his original position? most appropriate.
(d) Russia
(a) 15 m (b) 20 m Statement Nobody can predict
47. In Microsoft Word, ……… allows (c) 25 m (d) 30 m as to how long our country
us to move selected paragraphs to
the right. 57. If in a certain code, TWENTY would take to contain the
is written as 863985 and unfortunate and disastrous
(a) decrease indent
ELEVEN is written as 323039, terrorist activities.
(b) increase indent
how is TWELVE written in that Assumptions
(c) double indent
code? I. It is impossible to put on end to
(d) single indent
(a) 863203 (b) 863584 terrorist activities.
48. Blue Vitriol is another name for (c) 863903 (d) 863063 II. Efforts to control the terrorist
which of the following?
58. If the 25th of August in a year is activities are on.
(a) Copper sulphate
Thursday, the number of Mondays (a) Only I follows
(b) Oxygen
in that month is (b) Only II follows
(c) Copper
(a) 3 (b) 4 (c) Either I or II follows
(d) Magnesium oxide
(c) 5 (d) 6 (d) Neither I nor II follows
AFCAT-I ~ Solved Paper 2018 33

64. A statement with two conclusions Answer Figures Answer Figures


are given followed by four
alternatives. Select the one
alternative which is the most
appropriate. (a) (b) (c) (d) (a) (b) (c) (d)
Statement No country is 71. Problem Figure
absolutely self-dependent these 76. Problem Figure
days.
Conclusions
I. It is impossible to grow and
produce all that a country needs.
Answer Figures
II. Countrymen in general have Answer Figures
become lazy.
(a) Only I follows
(b) Only II follows (a) (b) (c) (d)
(c) Either I or II follows 72. Problem Figure
(d) Neither I nor II follows (a) (b) (c) (d)
Directions (Q. Nos. 65-68) Select 77. Problem Figure
the odd figure from the given
responses.
65.
Answer Figures

(a) (b) (c) (d) Answer Figures


66. = = (a) (b) (c) (d)

= = Directions (Q. Nos. 73-77) From the given


(a) (b) (c) (d) answer figures, select the one in which (a) (b) (c) (d)
the problem figure is hidden/ embedded.
67. 73. Problem Figure Directions (Q.Nos. 78-82) In each of
the following question, which answer
figure will come next in the series of
(a) (b) (c) (d) problem figures.
68. 78. Problem Figures
k k k
Answer Figures
?
(a) (b) (c) (d)
Directions (Q. Nos. 69-72) In each of (1) (2) (3) (4)
the following questions, choose the Answer Figures
(a) (b) (c) (d)
correct answer figure that will complete k k
the pattern of problem figure. 74. Problem Figure
69. Problem Figure k k
(a) (b) (c) (d)

79. Problem Figures

Answer Figures
Answer Figures ?
1 2 3 4
(a) (b) (c) (d) Answer Figures
(a) (b) (c) (d)

70. Problem Figure 75. Problem Figure

(a) (b) (c) (d)


34 AFCAT-I ~ Solved Paper 2018

80. Problem Figures 86. Two trains of equal length are running 93. The sum of two numbers is 24 and
on parallel lines in the same direction their product is 143. The sum of
at 46 km/h and 36 km/h. The faster their squares is
? train passes, the slower train in 36 s. (a) 296 (b) 295
The length of each train is (c) 290 (d) 228
(1) (2) (3) (4) (a) 82 m (b) 50 m 94. 3
0.000064 is equal to
Answer Figures (c) 80 m (d) 72 m
(a) 0.0002 (b) 0.002
87. Two trains start from a certain (c) 0.02 (d) 0.2
place on two parallel tracks in the
same direction. The speed of the 95. A box filled with paper bundles
(a) (b) (c) (d) trains are 45 km/h and 40 km/h, weighs 36 kg. If the weight of the
respectively. The distance between box and paper bundles
81. Problem Figures the two trains after 45 min will be respectively are in the ratio of
(a) 2 km 500 m (b) 2 km 750 m 3 : 22, then the weight of the
(c) 3 km 750 m (d) 3 km 250 m papers (in g) is
? (a) 30680 (b) 30710
88. A, B and C can complete a work in (c) 31500 (d) 31680
(1) (2) (3) (4) 2 h. If A does the job alone in 6 h
and B in 5 h, how long will it take 96. The LCM of two numbers is 520
Answer Figures and their HCF is 4. If one of the
for C to finish the job alone?
1 1 numbers is 52, then the other
(a) 5 h (b) 7 h
2 2 number is
1 (a) 40 (b) 42
(c) 9 h (d) 4 h
(a) (b) (c) (d) 2 (c) 50 (d) 52

82. Problem Figures 89. ` 6000 becomes ` 7200 in 4 yr at a 97. Weight of a bucket when filled fully
certain rate of simple interest. If the with water is 17 kg. If the weight of
rate becomes 1.5 times of itself, the the bucket when half filled with
? amount of the same principal in water is 13.5 kg, what is the weight
5 yr will be of empty bucket?
(1) (2) (3) (4) (a) ` 8000 (b) ` 8250 (a) 12 kg
Answer Figures (c) ` 9250 (d) ` 9000 (b) 8 kg
(c) 10 kg
90. A can do a work in 20 days and B (d) 7 kg
in 40 days. If they work on it
together for 5 days. Then, the 98. In a fixed time, a boy swims double
(a) (b) (c) (d) fraction of the work that left is the distance along the current that he
swims against the current. If the speed
83. Out of the given responses, one of 5 8 7 1
(a) (b) (c) (d) of the current is 3 km/h, then find
the factors of x 3 − 3 x 2 + 3 x + 7 is 8 15 15 10 the rate of swimming in still water.
(a) x 2 − 4 x + 7 (b) x 2 + 4 x + 7 91. A car driver leaves Bengaluru at (a) 6 km/h
(c) x 2 + 4 x − 7 (d) x 2 − 4 x − 7 8 : 30 am and expects to reach a (b) 9 km/h
place 300 km from Bengaluru at (c) 10 km/h
84. If n is even, (6n − 1) is divisible by 12 : 30 pm. At 10 : 30, he finds that (d) 12 km/h
(a) 37 (b) 35 (c) 30 (d) 6 he has covered only 40% of the
distance. By how much he has to
99. A boat goes 40 km upstream in
85. Ramesh bought 10 cycles for ` 500 increase the speed of the car in 8 h and 36 km downstream in 6 h.
each. He spent ` 2000 on the repair order to keep up his schedule? The speed of the boat in still
of all cycles. He sold five of them (a) 45 km/h (b) 40 km/h water is
for ` 750 each and the remaining (c) 35 km/h (d) 30 km/h (a) 6.5 km/h
for ` 550 each. Then, the total gain (b) 5.5 km/h
or loss% is 92. The average monthly expenditure
of a family for the first four months (c) 6 km/h
1
(a) Gain of 8 % (d) 5 km/h
3 is ` 2570, for the next three months
(b)
1
Loss of 8 %
is ` 2490 and for the last five 100. If a 2 + b 2 + 2b + 4 a + 5 = 0, then
3 months is ` 3030. If the family a−b
saves ` 5320 during the whole year, the value of is
(c)
2
Gain of 7 % then the average monthly income a+ b
3 (a) 3 (b) − 3
of the family during the year is
1 1 1
(d) Loss of 7 % (a) ` 3000 (b) ` 3185 (c) (d) −
7 (c) ` 3200 (d) ` 3580 3 3
AFCAT-I ~ Solved Paper 2018 35

Answers
1. (d) 2. (a) 3. (c) 4. (d) 5. (b) 6. (c) 7. (c) 8. (b) 9. (b) 10. (b)
11. (d) 12. (b) 13. (b) 14. (b) 15. (c) 16. (b) 17. (c) 18. (a) 19. (b) 20. (b)
21. (b) 22. (a) 23. (c) 24. (a) 25. (d) 26. (a) 27. (a) 28. (d) 29. (b) 30. (c)
31. (b) 32. (c) 33. (c) 34. (b) 35. (b) 36. (a) 37. (c) 38. (c) 39. (b) 40. (b)
41. (b) 42. (a) 43. (c) 44. (c) 45. (a) 46. (d) 47. (b) 48. (a) 49. (c) 50. (a)
51. (b) 52. (c) 53. (d) 54. (d) 55. (b) 56. (c) 57. (a) 58. (c) 59. (d) 60. (a)
61. (a) 62. (a) 63. (b) 64. (a) 65. (b) 66. (d) 67. (c) 68. (c) 69. (a) 70. (b)
71. (c) 72. (b) 73. (c) 74. (b) 75. (a) 76. (a) 77. (d) 78. (c) 79. (b) 80. (b)
81. (d) 82. (b) 83. (a) 84. (b) 85. (d) 86. (b) 87. (c) 88. (b) 89. (b) 90. (a)
91. (d) 92. (b) 93. (c) 94. (d) 95. (d) 96. (a) 97. (c) 98. (b) 99. (b) 100. (c)

Hints and Solutions


1. (d) ‘Paradox’ and ‘Yearning’ are the 9. (b) ‘Leapt on’ is wrong usage as ‘leapt’ is 20. (b) ‘Compassionate’ means showing
correct words to fill the blank as for first followed by ‘at’. sympathy towards weak. So, heartless is
blank the sentence is shows 10. (b) The given phrase ‘a damp squib’ its correct antonym as it means cruel or
contradiction and second statement means something which is less unkind and not worrying about other
showing the desire. impressive or exciting than it was people.
‘Paradox’ means a seemingly absurd or expected to be. 21. (b) ‘Tasty’ means having pleasant flavour
contradictory statement to expectations, Hence, among the given options, a so insipid is its correct antonym as it
existing belief or perceived opinion. disappointing result is its correct answer. means flavourless or tasteless.
‘Yearning’ means a feeling of intense 11. (d) The idiom ‘to smell a rat’ means 22. (a) ‘Triumph’ means a great
longing for something or want, wish etc. causing a feeling that something is wrong achievement. So, ‘defeat’ is its correct
2. (a) According to the given sentence, or that someone is behaving wrongly. antonym which means failure or
‘repugnant, condoned’ are the correct Hence, ‘to be suspicious’ is its correct unsuccessful.
words to fill the blanks. ‘Repugnant’ answer. 23. (c) empty
means unacceptable or objectionable. 12. (b) Yeoman’s services means very good, 24. (a) quiet
‘Condoned’ means accept behaviour that hard and valuable work that someone 25. (d) temper
is considered morally wrong or offensive. does especially to support a cause, to
help a team etc. 26. (a) The Vikalp scheme, also known as
3. (c) According to the given sentence,
Alternate Train Accommodation Scheme
‘essence’ and ‘unhampered’ are the 13. (b) ‘Flippant’ means not showing a (ATAS) was launched in 2015 to make
appropriate words as Essence means the serious or respectful attitude. sure available berths are optimally used.
basic or most important idea or quality of 14. (b) Hypnosis means suggestion made in In this scheme, wait-listed passengers of
something and unhampered means not trance. a train can opt for accommodation in
held in check or subject to control.
15. (c) ‘Nobody is laughing’ at hypnotherapy alternate trains.
4. (d) Here, ‘Extenuating’ and ‘Compliance’ now, because they are taking it seriously. The scheme does not guarantee a
are the correct words to fill the given blanks. confirmed berth in alternative train and
‘Extenuating’ means causing a wrong act to 16. (b) The purpose of hypnotherapy is to
make life easier. is subjected to train and berth
be judged less seriously by giving reasons availability.
for it. ‘Compliance’ means the action or 17. (c) ‘Barren’ means unable to
produce plants or fruits or Under the scheme, no-extra charges are
fact of complying with a command.
taken from passengers or any refund of
5. (b) Here, ‘lying’ should be used in place unproductive land. Hence,
fare is provided for the difference in fare
of ‘laying’. Because ‘laying’ means put ‘unproductive’ is similar in meaning to
when a reservation is provided in another
something down gently or carefully while the word given.
train.
‘lying’ means to be in or move into a 18. (a) ‘Infamy’ is the state of being Initially available for e-tickets only, the
horizontal position on a surface. well-known for some bad quality or deed. scheme is now made available to
6. (c) Remove ‘much’ which is superfluous Hence, ‘Notoriety’ is the word similar in passenger booking tickets from PRS
usage here as much is not used with too meaning to the word given, as notoriety counters.
because both have same meaning. also means the same. 27. (a) The cholera causing germ, Vibrio
7. (c) Replace ‘in’ by ‘at’ to make the syntax 19. (b) ‘Intrepid’ means extremely brave cholerae was discovered by Italian
correct. and showing no fear of dangerous anatomist Filippo Pacini in 1854.
situations or adventurous. Cholera is a water borne bacterial disease
8. (b) The correct grammatical structure Hence, fearless is its correct similar that causes diarrhea and severe
should be ‘does nature operate’ in place word. dehydration.
of nature does operate.
36 AFCAT-I ~ Solved Paper 2018

28. (d) 36. (a) The Indian socialist revolutionist 46. (d) The 21st FIFA Men’s World Cup
List I List II Bhagat Singh and an associate, 2018 was held in Russia. It was the first
Shivaram Rajguru shot dead British world cup to be held in Eastern Europe.
A. Mithali Raj 3. Cricket
police officer, John Saunders on 17th The FIFA World Cup 2018 was won by
B. Poonam Rani 1. Hockey December, 1927. Bhagat Singh wanted France, who beat Croatia by 4-2. The
C. Lalita Babar 2. 3000 m to take revenge for the death of popular
Steeplechase 2022 FIFA World Cup is scheduled to be
Indian nationalist leader Lala Lajpat Rai. held in Qatar.
29. (b) The author of the book titled ‘Citizen 37. (c) The Bardoli Satyagraha in 1928 was 47. (b) In Microsoft Word, increase indent
and Society’ is former Vice-President a movement in the independence allows us to move selected paragraphs to
Hamid Ansari. This book is a collection struggle led by Sardar Vallabhbhai Patel the right.
of Ansari’s lectures on diverse themes for the farmers of Bardoli against the
such as polity, security and 48. (a) Blue Vitriol is another name of Copper
unjust raising of taxes.
empowerment. sulphate. Blue Vitriol is used as a
38. (c) Amrita Shergil was an eminent Indian fungicide, insecticide, mordant and in
30. (c) The Indian Government has signed a Painter. She has been called one of the
engraving.
MoU (Memorandum of Understanding) greatest avant-garde women artists of the
with Israel on National campaign for early 20th century and a ‘pioneer’ of 49. (c) With an aim to strength the bond
water conservation in India. This will modern Indian art. The Government of between all the state, regions and
benefit the country in conserving water India has declared her works as National products of India, Central government
for future generation. As a part of the Art Treasures and most of them are has launched ‘EK Bharat Shreshtha
MoU, the governments intended to housed in the National Gallery of Modern Bharat’ Scheme. The scheme was
achieve several objectives such as Art in New Delhi. announced by Hon’ble Prime Minister
putting water conservation on the 39. (b) The Indian hockey player PR Narendra Modi on 31st October, 2015
national agenda in India, encouraging Shreejesh was awarded with Padma Shri on the occasion of 140th birth
each citizen to save water in everyday 2017 in the field of sports. He is the anniversary of Sardar Vallabhbhai Patel.
life, and promoting its re-use, recharge former captain of Indian hockey team. Through this scheme, the government
and recycling. He has also been awarded with Arjuna will promote the spirit of national
31. (b) The Vijay Stambha is victory Award in 2015. integration through a deep and structured
monument located within Chittor fort in engagement between All Indian states
40. (b) Liver is the largest gland found in the
Chittorgarh, Rajasthan. The victory tower and Union Territories.
human body. It is reddish brown in
was built by the Mewar King, Rana colour, situated in the upper part of the 50. (a) The Union Government on 22nd
Kumbha in 1448 to commemorate his abdominal cavity. The liver has many May, 2016 appointed Kiran Bedi as the
victory over the combined armies of functions and plays a major role in new Lieutenant Governor of the Union
Malwa and Gujarat led by Mahmud Khilji. human metabolism and digestion. Territory of Puducherry. She is a retired
32. (c) The slogan ‘Swaraj is my birthright 41. (b) Hydraulic lift works on the principle Indian Police Service Officer and social
and I shall have it’ was first raised by Bal of Pascal’s law. The law states that if any activist. She is the first woman to have
Gangadhar Tilak in Belgaum in 1916. force is applied to any point of a confined joined the officer ranks of Indian Police
Tilak was one of the first and strongest fluid, then the pressure is equally and Service. She had been conferred with
advocates of ‘Swaraj’ or ‘Self-rule’, which undiminishedly transmitted through out Ramon Magsaysay Award in 1994 for
became the guiding principle of the the liquid. Government Service.
Indian freedom movement.
42. (a) At − 40° C, the numerical values on 51. (b) Except Equator all are zodiac signs
33. (c) The axillary bud is an embryonic or whereas Equator is an imaginary line on
Celsius and Fahrenheit scales become
organogenic shoot located in the axil of a Earth.
equal.
leaf of a plant. Axillary buds develop from
43. (c) The abbreviation MICR stands for 52. (c) Cataract is a disease, which effects on
the nodes which then becomes a new
‘Magnetic Ink Character Recognition.’ eye. Similarly, Pneumonia is a disease
stem branch.
MICR code is a character recognition which effects on Lungs.
34. (b) Xylem is one of the two types of
technology used mainly by the banking 53. (d) In the given pair, 7 is the backward
transport tissue in vascular plants. The
industry to streamline the processing and position of T. Similarly, 9 is the
basic function of Xylem is to transport
clearance of cheques and other backward position of R. Thus, the
water from roots to stems and leaves.
documents. missing term will be
The other transport tissue is Phloem.
44. (c) Sublimation is the process of R R R
35. (b)
transition of solid state directly to
List I List II gaseous state, without passing through 9 9 9
(Revolution) (Leader) the intermediate liquid phase. Some of Reverse
A. Green 2. MS the example of sublimation is dry ice or 54. (d) As, NUS → SUN rises in
Revolution Swaminathan Reverse
solid carbon dioxide sublimes. YAD → DAY.
B. White 3. Verghese
Revolution Kurien 45. (a) Ashok Chakra is India’s highest Reverse
Similarly, NOOM → MOON
C. Pink 1. Durgesh
peacetime military decoration awarded Reverse
for valour, courageous, action or shines in THGIN → NIGHT.
Revolution Patel
self-sacrifice away from the battlefield.
AFCAT-I ~ Solved Paper 2018 37

55. (b) As, 7 × 3 + 3 = 21 + 3 ⇒ 24 61. (a) The series is 75. (a) The answer figure (a) is hidden in
Same as, 23 × 3 + 3 = 69 + 3 ⇒ 72 abadna/abadna/abadna/ab. Thus, the problem figure.
Hence, 23 : 72 will be came in place of pattern ‘abadna’ is repeated.
question mark. Hence, required set of letters = andaa
56. (c) According to the question, the 62. (a) In the given two positions of the die 4
direction diagram is as given below. and 5 are common. Hence, 1 is opposite 76. (a) The answer figure (a) is hidden in
10 m to 2. problem figure.
North 63. (b) The statement expresses concern over
25 m 25 m the issue as to when our country would
10 m West East be able to curb terrorism completely. This
A B
15 m means that efforts are on and it is quite
South possible to put an end to terrorist 77. (d) The problem figure is hidden in the
∴ Required distance, AB = 15 + 10 activities although it could take longer. answer figure (d).
= 25 m So, only II is implicit.
57. (a) As, 64. (a) Clearly, only Conclusion I provides a
T W E N T Y suitable explanation to the given
statement. So, only it follows the
statement. 78. (c) In each subsequent figure, all the
8 6 3 9 8 5 65. (b) Except figure (b), in all other figures, designs move one step in anti-clockwise
and there are three similar designs. direction. So, answer figure (c) will come
66. (d) Except figure (d), in all other figures, next.
E L E V E N
all the five designs are similar. In figure 79. (b) In each subsequent figure, the
(d), symbol‘+’ is replaced by ‘~’. number of lines in the design decreases
3 2 3 0 3 9 gradually. So, option figure (b) will come
67. (c) Except figure (c), in all other figures,
line segments divide the shape into equal next.
Similarly,
T W E L V E parts. 80. (b) From figure (1) to (2), one line is
68. (c) Except figure (c), in all other figures increasing in both the given shapes.
8 6 3 2 0 3 two crosses and two circle are given Also, both the different sized shapes are
together. In figure (c), cross and circle places at the same positions in both
58. (c) 25th August is a Thursday. figures. So, from figure (3) to (4), one
are placed alternately.
So, 22nd August is a Monday. line will be increased and both the
69. (a) Answer figure (a) will complete the
So, Mondays fall on 1st, 8th, 15th, 22th shapes will be placed at the same
and 29th of August. given pattern.
position of figure (3).
Thus, there are five Mondays. 70. (b) Answer figure (b) will complete the
81. (d) In each successive figure, the entire
given pattern.
59. (d) As, design is rotated through 90° in
O P T R R E 71. (c) Option (c) figure will complete the clockwise direction.
pattern of the figure. So, option (d) will come next.
72. (b) Option (b) figure will complete the 82. (b) In each successive figure, one
P O R T E R pattern of the figure. curve is deleted and the shaded parts of
Similarly, square move two sectors in clockwise
E R O P T R direction. So, option (b) will replace
the ‘?’.
83. (a) Given expression,
73. (c) The answer figure (c), is hidden in
R E P O R T f(x) = x 3 − 3x2 + 3x + 7
problem figure.
So, the coded word will be read as Put x = −1
REPORT. f(− 1) = (− 1)3 − 3(− 1)2 + 3(− 1) + 7
60. (a) From the given statements, = −1− 3− 3+ 7 = 0
Man ∴ (x + 1) is a factor of the expression.
It is embedded in the problem figure. Now, dividing expression by (x + 1).
× Donkey 74. (b) The answer figure (b) is embedded in
Rahul
problem figure as shown below. x + 1) x 3 − 3x2 + 3x + 7(x2 − 4x + 7
x 3 + x2
− −

I. (3) II. (7) − 4x + 3x + 7
2
38 AFCAT-I ~ Solved Paper 2018

= 5 
− 4x2 − 4x 87. (c) According to the question, 1 1
+ 
+ + Relative speed for same direction  20 40
= 45 − 40 = 5 km/h 2 + 1
= 5 

7x + 7 
∴Distance between two trains after  40 
7x + 7 45 min 15 3
– – = =
= Speed × Time 40 8

3 5
=  5 ×
× 45 ∴ Remaining work = 1 − = part
 km
 60  8 8
∴(x2 − 4x + 7) is also a factor of 91 (d) According to the question,
15
expression x 3 − 3x2 + 3x + 7. = km distance covered by car in 2 h i.e. from
4
84. (b) We have, (6n − 1). 8:30 am to 10:30 am = 40% of 300
= 3 km 750 m
40 × 300
If n is even, then taking n = 2, 88. (b) Let C alone can finish the job in x h. = = 120 km
100
6n − 1 = 62 − 1 = 36 − 1 = 35 1
Work done by A in 1 h = Average speed of driver for first two hours
Here, number 35 is divisible by 35. 6 Distance
Hence, for any even value of n, (6n − 1) =
1
is divisible by 35. Work done by B in 1 h = Time
5 120
85. (d) Given, cost price of 1 cycle = 500 = = 60 km/h
1
Work done by C in 1h = 2
∴Cost price of 10 cycles = 500 × 10 x Remaining distance
= 5000 According to the question, = 300 − 120 = 180 km
But Ramesh spent ` 2000 on the repair. 1 and time = 2 h
Work done by A, B and C in 1 h =
∴ Total cost price = 5000 + 2000 2 ∴ Average speed for next two hours
= 7000 1 1 1 1 180
Now, (for 5 cycles) ⇒ + + = = = 90 km/h
6 5 x 2 2
Selling price of 1 cycle = 750 ∴ Increase the speed
1 1 1 1
∴ Selling price of 5 cycles = 750 × 5 ⇒ = − −
x 2 6 5 = (90 − 60) = 30 km/h
= 3750
15 − 5 − 6 92. (b) Total earning of the family
and (for remaining 5 cycles) =
30 = 4 × 2570 + 3 × 2490
Selling price of 1 cycle = 550
4
∴ Selling price of 5 cycles = 550 × 5 = + 5 × 3030 + 5320
= 2750 30
2 [∵ total earning = average earning
So, Total selling price = 3750 + 2750 =
= 6500 15 × number of months]
Loss = Cost price − Selling price 1
⇒ x = 7 h.
= 7000 − 6500 = 500 2 Total income

Loss per cent =


Loss
× 100 89. (b) SI = Amount − Principal = 10280 + 7470 + 15150 + 5320
CP = ` (7200 − 6000) [∵ income = earning + savings]
500 50 1
= × 100 = =7 % = ` 1200 = 38220
7000 7 7 ∴Average monthly income
P× R×T
86. (b) Let the length of each train be x m. ∴ SI = Total income
100 =
Total length = x + x = 2x Number of Months
6000 × R × 4
Relative speed for same direction ⇒1200 = 38220
100 = = ` 3185
= 46 − 36 = 10 km/h 12
1200 × 100
10 × 5 ⇒ R= = 5%
m/s ∵ 1 km /h = m /s 
5 93. (c) Let the numbers be x and y.
= 6000 × 4
18  18 
∴ x + y = 24 and xy = 143
25 New rate of R = 5 × 1.5
= m/s ∴ x2 + y2 = (x + y)2 − 2xy
9 = 7.5%
Time taken passes slower train by faster 6000 × 7.5 × 5 = (24)2 − 2 × 143
Sum of length of both trains Then, SI =
train = 100 = 576 − 286 = 290
Relative speed of faster train
= ` 2250
2x 94. (d) Expression = 3 0.000064
∴ 36 = ∴ Amount = ` (6000 + 2250)
25
9 = ` 8250 = 3
64
= 3
8
36 × 25 1 106 103
⇒ 2x = 90. (a) A’s one day work =
9 20 2×2×2
36 × 25 = 3
⇒ x= 1 10 × 10 × 10
B’s one day work =
18 40 2
∴ x = 50 m Then, (A + B)’s 5 days work = = 0.2
10
AFCAT-I ~ Solved Paper 2018 39

95. (d) Let weight of the box = 3 x g ∴Weight of water when bucket is fully ∴Speed of boat in still water
and weight of the bundle = 22x g filled = 2 × 35
. kg = 7 kg 1
=
According to the question, 2
Now, weight of empty bucket
(speed of upstream + speed of
⇒ 3x + 22x = 36 ×1000 g = (17 − 7) kg
downstream)
⇒ 25x = 36000 g = 10 kg 1
= (5 + 6) = 5.5 km / h
36000 98. (b) Let the rate of swimming in still water 2
⇒ x= = 1440 g
25 be x km/h and time taken to go upstream
100.(c) Given, a2 + b2 + 2b + 4a + 5 = 0
∴ Weight of the paper = 22x or downstream is t.
∴Speed of downstream = (x + 3) km/h ⇒ a2 + 4a + b2 + 2b + 5 = 0
= 22 × 1440
= 31680 g ∴Speed of upstream = (x − 3) km/h ⇒ a + 4a + 4 + b2 + 2b + 1 = 0
2

96. (a) HCF × LCM = Product of two According to the question, ⇒ (a + 2)2 + (b + 1)2 = 0
(x + 3) t = 2 (x − 3) × t
numbers [∵ (x + y)2 = x2 + y2 + 2xy]
[∵ distance = speed × time]
∴ 4 × 520 = 52 × Second number It is possible only, when
⇒ x + 3 = 2x − 6
4 × 520 a+2= 0
∴Second number = = 40 ∴ x = 9 km/h
52 ⇒ a = − 2 and b + 1 = 0
40
99. (b) Speed of upstream =
97. (c) Total weight of bucket, fully filled by 8 ⇒ b = −1
water = 17 kg a− b −2+1
∵ speed = distance  ∴ =
a + b − 2 −1
Now, weight of bucket, half filled by  time 
water = 13.5 kg −1 1
= 5 km/h = =
∴ Weight of water when bucket is half 36 −3 3
Speed of downstream = = 6 km/h
filled = (17 − 135
. ) kg = 35
. kg 6
40 AFCAT-II ~ Solved Paper 2017

INDIAN AIR FORCE


AFCAT-II

Solved Paper 2017


Time : 2 Hrs MM : 300
INSTRUCTIONS
■ The set contains a total of 100 questions, Comprising Verbal Ability in English, General Awareness, Numerical Ability and Reasoning and

Military Aptitude Test.


■ Each correct question carry 3 Marks and there will be negative marking of 1 Mark for each incorrect attempt.

■ Total time duration will be 2 hrs (120 minutes).

■ No marks will be deducted for unattempted questions.

Directions (Q.Nos. 1-5) You have one 3. It appears that the author of the All of these changes will be for the
brief passage with five questions following sentence criticises economic better if they help ....10.... the quality of
the passage. Read the passage carefully liberalisation for the people’s police service.
and choosed the best answer to each (a) lack of formality
6. (a) predicted (b) has predicted
question out of the for alternatives. (b) poor mannerism (c) is predicted (d) was predicting
As a result of economic liberalisation (c) lack of naturalness
(d) lack of grace 7. (a) any (b) some
and entry of a large number of
(c) such (d) much
multinational companies into India, the 4. Which of the following is not true
life of most middle class and upper - about most of the middle and 8. (a) particularly (b) placidly
middle class people in the last two (c) roughly (d) widely
upper-middle class people of the
decades of the present century has been period mentioned in the passage? 9. (a) had to make (b) would be made
characterised by artificiality in dress, (a) They are wicked (c) has made (d) should make
manners and conversation and (b) They are formal
elaborate surface of formality and grace 10. (a) improving (b) be improved
(c) They are witty (c) to improve (d) have improved
covering a certain amount of vice and a (d) They are silly
great deal of plain silliness. Directions (Q. Nos. 11-15) In the
5. It can be inferred from the sentence questions given below, there are four
1. The period discussed in the that the middle and upper-middle different words out of which one is
sentence is class people of the period lived
(a) from AD 1981 to 1990 correctly spelt. Find the correctly spelt
(a) wicked lives word.
(b) from AD 1981 to 2000 (b) conventional lives
(c) from AD 1990 to 2000 (c) affluent lives 11. (a) Efflorescence (b) Eflorescence
(d) from AD 1980 to 2000 (d) very simple lives (c) Efflorrescence (d) Efloreescence
2. The sentence lays emphasis on the Directions (Q.Nos. 6-10) Select the 12. (a) Liaission (b) Liaison
fact that the economic liberalisation most appropriate word from the options (c) Lieaison (d) Liaaison
has against each number.
(a) raised the standard of living of the 13. (a) Entreepreneur (b) Entrepreeneur
middle and upper middle class It ....6.... that policing in the future will (c) Entrepreneur (d) Entreppreneur
people be ....7.... more different than it is
(b) made people more artificial out today. Advances in technology ....8.... in 14. (a) Equanemity (b) Equaninity
wardly, without change in attitude computers, television and (c) Equamimity (d) Equanimity
(c) helped the people dispense with communication will assist the police in 15. (a) Embarrassment
their vicious behaviour solving and preventing crimes. (b) Embarassment
(d) removed their poverty and raised Advances in forensic science ....9.... (c) Embarrasment
their quality of life evidence more reliable and meaningful. (d) Embarasment
AFCAT-II ~ Solved Paper 2017 41

Directions (Q. Nos. 16-20) In these 27. Michael Ferreira is related to which 37. Who is rightly called the ‘Father
questions out of the four alternatives of the following sports? of Local Self Government’ in
choose the one which can be (a) Cricket India?
substituted for the given (b) Golf (a) Lord Mayo (b) Lord Ripon
words/sentences. (c) Hockey (c) Lord Curzon (d) Lord Clive
(d) English Billiards
16. A group of three books, films etc, 38. Razia Sultan belonged to which of
that have the same subject or 28. Which of the following is India’s the following dynasty?
characters first indigenously developed (a) Slave Dynasty
(a) Trinity (b) Trilogy nuclear powered submarine? (b) Maurya Dynasty
(c) Trio (d) Tripod (a) INS Vikrant (c) Khilji Dynasty
(b) INS Trikhand (d) Koravi Dynasty
17. A study of the human race.
(c) INS Arihant
(a) Anthropology (b) Archaeology 39. Which is the second nearest star to
(d) INS Sindhurakshak
(c) Ethnology (d) Etymology the Earth after the Sun?
29. Thomas Cup is associated with (a) Vega
18. A remedy for all diseases is
(a) Hockey (b) Sirius
(a) Stoic (b) Marvel
(b) Badminton (c) Proxima Centauri
(c) Panacea (d) Recompense
(c) Kabaddi (d) Alpha Centauri
19. A place where treasures of (d) Table Tennis (Women)
art, curios are preserved or 40. Which is an extra-constitutional
30. Sattriya dance originated in body?
exhibited.
(a) Assam (a) Language Commission
(a) Museum (b) Stable
(b) Andhra Pradesh (b) Planning Commission
(c) Library (d) Studio
(c) Rajasthan (c) Election Commission
20. Associated with God or religion (d) Gujarat (d) Finance Commission
(a) Humbly (b) Holy
(c) Virtuous (d) Godly 31. Which of the following is home 41. Which of the following states won
base of the aircraft carrier INS Vijay Hazare Trophy 2016-2017?
Directions (Q.Nos. 21-23) In these Vikramaditya? (a) Tamil Nadu
questions choose the opposite word in (a) Visakhapatnam (b) Cochin (b) Bengal
meaning to the given word. (c) Karwar (d) Jamnagar (c) Madhya Pradesh
(d) Mumbai
21. Hasten 32. Who is the Defence Secretary
(a) Dash (b) Dawdle during Indo-China war in 1962? 42. Which is not an All India Service?
(c) Hurry (d) Scurry (a) Sanjay Mitra (a) Indian Administrative Service
(b) VK Krishna Menon (b) Indian Police Service
22. Spirited (c) VK Singh (c) Indian Foreign Service
(a) Animated (b) Excited (d) Govind Narain (d) Indian Forest Service
(c) Lively (d) Dull
33. The accounting year of Reserve 43. Hiuen Tsang visited India during
23. Forthright Bank of India runs from the reign of
(a) Blunt (b) Tricky (a) April to March (a) Chandra Gupta I
(c) Candid (d) Plainspoken (b) July to June (b) Chandra Gupta II
(c) Harshavardhana
Directions (Q.Nos. 24 and 25) In (c) January to December
(d) Rudradaman
these questions out of the four (d) August to July
alternatives, choose the one which best 34. Agricultural commodities, are 44. The call of ‘Back to the Vedas’ was
expresses the meaning of the given graded with given by
word. (a) Swami Vivekananda
(a) ISI (b) Eco-products
(b) Swami Dayanand Saraswati
24. Right (c) AGMARK (d) Green product (c) Aurobindo Ghosh
(a) Correct (b) Marked
35. The book titled ‘The Indian War of (d) Raja Ram Mohan Roy
(c) Straight (d) Finished
Independence’ was written by 45. The commando unit of the Indian
25. Apprehended (a) Krishna Verma Air Force is named
(a) Understood (b) Madame Cama (a) Baaz (b) Garud
(b) Arrested (c) MARCOS (d) Ghatak
(c) BG Tilak
(c) Feared
(d) Questioned (d) VD Savarkar 46. The monetary policy in India is
36. Who won the Nobel Peace Prize for formulated by
26. Who got Nobel Prize for ‘The Old (a) Central Government
Man and the Sea’? 2016?
(a) Juan Manuel Santos (b) Industrial Financial Corporation
(a) Ernest Hemingway
(b) Tawakkol Karman of India
(b) Satyajeet Ray
(c) Al Gore (c) Reserve Bank of India
(c) Rabindranath Tagore
(d) Ruskin Bond (d) Industrial Development Bank of
(d) Kailash Satyarthi
India
42 AFCAT-II ~ Solved Paper 2017

47. What is the capital of Uzbekistan? delivering the tickets at the (c) No man is a monkey. John is a
(a) Dushanbe (b) Damascus doorstep through courier service at man. Therefore, John is not a
(c) Tashkent (d) Bishkek a little extra cost. monkey.
Assumptions (d) All birds are parrots. Manish is a
Directions (Q.Nos. 48-50) In the parrot. Therefore, Manish is a bird.
following questions, complete the series. I. Many customers may now book
their tickets through internet, Directions (Q. Nos. 59-62) Choose the
48. Z, ?, T, Q, N resulting into less crowed at word which is least like the other words
(a) X (b) W (c) V (d) T ticket booking offices. in the group.
49. A, I, P, V, A, ? II. Most of the customers may still 59. (a) Chemistry (b) Physics
(a) F (b) D (c) E (d) G buy their railway tickets at the (c) Botany (d) Geography
booking counters.
50. YEB, WFD, UHG, SKI, ? 60. (a) Tomato (b) Potato
(a) QOL (b) POL (c) QLO (d) LOQ 55. Statement Many people fell ill (c) Cucumber (d) Peas
after consuming meal at a wedding
51. In a certain code FORGET is written 61. (a) Seal (b) Scorpion
reception and were rushed to the (c) Fish (d) Cat
as DPPHCU. How would DOCTOR nearby Government and private
be written in that code? hospitals. 62. (a) Zinc (b) Iron
(a) BPAUPS (b) BPAUMS Assumptions (c) Aluminium (d) Mercury
(c) EMDRPP (d) EMDRPD
I. The relatives of the affected Directions (Q.Nos. 63-67) The
52. If rat is called dog, dog is called following questions consist of two words
people may refuse to take
mongoose, mongoose is called lion, that have a certain relationship to each
them to the Government
lion is called snake and snake is called other, followed by four alternatives.
hospitals.
elephant, which is reared as a pet? Select the best alternative that has
(a) Rat (b) Dog
II. The nearby hospitals may be
same relationship as the original pair
(c) Mongoose (d) Lion able to attend to all the affected
of words.
people.
Direction (Q.No. 53) Two statements 63. Behead : Guillotine :: ?
are given followed by two Conclusions Directions (Q.Nos. 56-58) Each of the (a) Polish : Nail
I and II. You have to consider the following question has four statements. (b) Perfect : Picture
statements to be true even if they seem to Three are incorrect. Choose the (c) Thief : Prison
be at variance from commonly known statement which is logically correct. (d) Hang : Gallows
facts. You are to decide which of the
given conclusions, if any, follow from the 56. (a) All chances are fakes. No chance 64. Graphite : Lubricant :: ?
given statements. Indicate your answer. are genuine. Therefore, all fakes (a) Movement : Friction
are genuine. (b) Iron : Steel
53. Statements All animals are dogs. (b) Some papers are white. All white (c) Wool : Cloth
All dogs are birds. are brown. Therefore, all brown (d) Diamond : Abrasive
Conclusions are white.
65. Money : Transaction :: ?
I. All animals are birds. (c) Some goats are lambs. All kids are
(a) Life : Death
II.All birds are animals. lambs. Therefore, some goats are
(b) Water : Drink
(a) Only Conclusion I follows kids.
(c) Ideas : Exchange
(b) Only Conclusion II follows (d) Some cuts are dabs. No dabs is (d) Language : Conversation
(c) Both Conclusions I and II follow pricks. Therefore, some dabs are
(d) Neither Conclusion I nor II follows cuts. 66. Abrupt : Gradual :: ?
(a) Barren : Fertile
Directions (Q.Nos. 54 and 55) In each 57. (a) No trees are decorations. Some (b) Reverse : Backward
question below is given a statement trees are decorated. Therefore, no (c) Motion : Forward
followed by two Assumptions I and II. decorated are decorations. (d) Agile : Calm
You have to consider the statement and (b) All men lie. Only liars are
the following assumptions and decide welcome. Therefore, all men are 67. Jute : Sack :: ?
which of the assumption is implicit in welcome. (a) Shoe : Sock
the statement. (c) Some robots fly. All birds (b) Wool : Sweater
fly. Therefore, some birds are (c) Cotton : Fibre
Give Answer (d) Mill : Cloth
robots.
(a) Only Assumption I is implicit
(d) Some tablets are round. Some Directions (Q. Nos. 68-70) There is
(b) Only Assumption II is implicit
tablets are oval. Therefore, all certain relationship between two given
(c) Neither Assumption I nor II is tablets are oval.
implicit words on one side of (: :) and one word
(d) Both Assumptions I and II are 58. (a) All books are friends. All friends is given on another side of (::) while
implicit are dangerous. Therefore, all another word is to be found from the
dangerous are books. given alternatives, having the same
54. Statement Railway authority has relation with this word as the given
(b) All tables are ants. Some chairs
started internet booking facility of pair has. Select the best alternative/
are ants. Therefore, all ants are
long-distance trains and also relationship.
tables.
AFCAT-II ~ Solved Paper 2017 43

68. Iran : Majlis :: Finland : ? 78. A person distributes his pens Directions (Q. Nos. 86-89) In these
(a) Eduskunta among four friends A , B , C and D in tests you will find an problem figure and
(b) Helsinki 1 1 1 1 four answer figures. You have to select
the ratio : : : . What is the one diagram from the answer figures
(c) Congress 3 4 5 6
(d) Vantaa minimum number of pens the which fits into the blank column in
person should have? problem figure in order to complete it.
69. Oil : Seed :: Butter : ?
(a) Cow (b) Curd (a) 60 (b) 120 86. Problem Figure
(c) Milk (d) Amul (c) 180 (d) 240

70. Horse : Foal :: Dog : ? 79. Four horses are tethered at 4


corners of a square field of side ?
(a) Calf (b) Bitch
(c) Jenny (d) Whelp 70 m, so that they just cannot reach
one another. The area left ungrazed Answer Figures
71. Velocity of a man in steady water is by the horses is
4.5 km/h. Velocity of the stream is (a) 1050 m2 (b) 3850 m2
1.5 km/h. If he goes upstream and (c) 950 m2 (d) 1075 m2
come back. What is the average (a) (b) (c) (d)
velocity? 80. The ratio between boys and girls in
(a) 4 km/h (b) 5 km/h
a school is 4 : 6 respectively. If the 87. Problem Figure
(c) 6 km/h (d) 7 km/h number of boys is increased by 200
the ratio becomes 5 : 6, respectively. ?
72. Ratio of boys and girls in a school is How many girls are there in the
3: 2. 20% boys and 25% girls have school?
scholarships. What is the total (a) 1200
percentage of students who have (b) 800 Answer Figures
scholarship? (c) 1000
(a) 21% (b) 22% (d) Cannot be determined
(c) 23% (d) 24%
81. A man whose bowling average is (a) (b) (c) (d)
73. A is thrice efficient than B. A takes 12.4 takes 5 wickets for 26 runs and
40 days less than B to complete a thereby decreases his average by 88. Problem Figure
work. If they both work together, 0.4. The number of wickets taken
how many days it will take to finish by him before his last match, is
the work? (a) 85 (b) 78
(a) 10 days (b) 25 days (c) 72 (d) 64 ?
(c) 15 days (d) 20 days
82. The length and breadth of a square
74. 40% of a number, exceeds 25% of it are increased by 30% and 20%, Answer Figures
by 45. What is the number? respectively. The area of the
(a) 150 (b) 320 rectangle so formed exceeds the
(c) 250 (d) 300 area of the square by
(a) (b) (c) (d)
75. There are 15 students whose (a) 20% (b) 36%
average increases by 1.5 kg, when a (c) 50% (d) 56%
89. Problem Figure
student of 40 kg was replaced by 83. The ratio of father’s age to his son’s
another one. What is his weight? age is 7 : 3. The product of their age
(a) 62.5 kg (b) 60 kg is 756. The ratio of their ages after
(c) 61 kg (d) 58.5 kg ?
6 yr will be
76. A train takes 18 s to cross a man (a) 2 : 1 (b) 5 : 2 (c) 11 : 7 (d) 13:9
running at a speed of 6 km/s in the
84. The simplified value of Answer Figures
same direction of the train. How
112 576 256
much time it takes to cross a person × × is
runing at 9 km/s in the same 196 12 8
direction of the train? (a) 12 (b) 8 (c) 16 (d) 32
(a) 15 s 85. A, B and C can do a piece of work in (a) (b) (c) (d)
(b) 20 s 20, 24 and 30 days, respectively.
(c) 18 s They undertook to do the piece of Directions (Q. Nos. 90-93) In these
(d) Cannot be determined work for ` 5400. They begin the tests find which code matches the shape
1 or pattern given at the end of each
77. th tank is full and if 22 L fuel work together but B left 2 days
5 3 before the completion of work and question.
poured in then th tank is full.
4 C left 5 days before the completion 90.
Find the capacity of tank? of work. The share of A from the HC ND ZU NC ZK ?
(a) 30 L (b) 60 L assured money is
(c) 40 L (d) 20 L (a) ` 2700 (b) ` 540 (a) HU (b) ND
(c) ` 1800 (d) ` 600 (c) ZD (d) NK
44 AFCAT-II ~ Solved Paper 2017

91. 96. 99. Problem Figure

EM FZ CH ED CZ ?
(a) FD (b) EH (c) FH (d) CZ (a) (b) (c) (d)
(X)
92. 97.
Answer Figures
DK CN FG CK DN ?
(a) CN (b) NK (c) CG (d) DG
(a) (b) (c) (d)
93. (a) (b) (c) (d)
Directions (Q. Nos. 98-100) In these
test figure (X) is hidden in the option 100. Problem Figure
LG JF OK PK JK ?
figures. Find the correct option.
(a) PF (b) JG (c) OG (d) OF 98. Problem Figure
Directions (Q. Nos. 94-97) In these
tests, find the odd figure out. (X)

Answer Figures
94.
(X)
Answer Figures
(a) (b) (c) (d)
(a) (b) (c) (d)
95.

(a) (b) (c) (d)


(a) (b) (c) (d)

Answers
1. (b) 2. (b) 3. (c) 4. (c) 5. (a) 6. (c) 7. (d) 8. (a) 9. (d) 10. (c)
11. (a) 12. (b) 13. (c) 14. (d) 15. (a) 16. (b) 17. (a) 18. (c) 19. (a) 20. (b)
21. (b) 22. (d) 23. (b) 24. (a) 25. (a) 26. (a) 27. (d) 28. (c) 29. (b) 30. (a)
31. (c) 32. (b) 33. (a) 34. (c) 35. (d) 36. (a) 37. (b) 38. (a) 39. (c) 40. (b)
41. (a) 42. (c) 43. (c) 44. (b) 45. (b) 46. (c) 47. (c) 48. (b) 49. (c) 50. (a)
51. (b) 52. (c) 53. (a) 54. (a) 55. (b) 56. (d) 57. (b) 58. (c) 59. (d) 60. (b)
61. (a) 62. (d) 63. (d) 64. (d) 65. (d) 66. (a) 67. (b) 68. (a) 69. (c) 70. (d)
71. (a) 72. (b) 73. (c) 74. (d) 75. (a) 76. (d) 77. (c) 78. (a) 79. (a) 80. (a)
81. (a) 82. (d) 83. (a) 84. (d) 85. (a) 86. (b) 87. (d) 88. (b) 89. (d) 90. (a)
91. (c) 92. (a) 93. (d) 94. (c) 95. (d) 96. (d) 97. (c) 98. (c) 99. (d) 100. (c)

Hints and Solutions


1. (b) The period discussed in the sentence 6. (c) is predicted 7. (d) much 14. (d) The correctly spelt word is ‘Equanimity’.
is last two decades i.e., 1981 to 2000. 8. (a) particularly 9. (d) should make Equanimity means calmness and com
Hence, option (b) is its correct answer. posure, especially in a difficult situation.
10. (c) to improve
2. (b) The sentence lays emphasis on the 15. (a) The correctly spelt word is ‘Embar
11. (a) The correctly spelt word is
fact that the economic liberalisation has rassment’ which means a feeling of self-
‘Efflorescence’ and it means the period
made people more artificial outwardly, consciousness, shame or awkwardness.
when flowers start to appear on a plant.
without change in attitude. 16. (b) A ‘trilogy’ is a set of three works of art
12. (b) The correctly spelt world is ‘Liaison’ that are connected and that can be seen
3. (c) It appears that the author of the
which means communication between either as a single work or as three
sentence criticises economic liberalisation
two or more groups, or co-operating or individual works. Hence, option (c) is the
for the people’s lack of naturalness.
working together. correct answer.
4. (c) In context of the passage, the middle
13. (c) The correctly spelt word is 17. (a) ‘Anthropology’ is the scientific study
and upper-middle class people of the
‘Entrepreneur’. Entrepreneur means a of humans, human behaviour.
period are witty, is not true.
person who sets up a business or
5. (a) It can be inferred from the sentence 18. (c) A remedy for all diseases is ‘Panacea’.
businesses, taking on financial risks in
that the middle and upper-middle class the hope of profit. 19. (a) A ‘Museum’ is a place where treasures
people of the period lived wicked lives. of art, curious are preserved or exhibited.
AFCAT-II ~ Solved Paper 2017 45

20. (b) Associated with God or religion is 32. (b) VK Krishna Menon was the Defence 40. (b) An extra constitutional body is an
‘Holy’. Secretary during Indo-China War in institution of government which is not
21. (b) ‘Hasten’ means be quick to do 1962. He was an Indian nationalist defined in constitution of India but are
something. So, ‘Dawdle’ is its correct diplomat and politician. The Sino-Indian established by a resolution (decision) of
antonym which means to do something war, also known as Indo-China war was union cabinet. Planning commission,
very slowly. a war between India and China that National Development Council etc are
occurred in 1962. some examples of extra constitutional
22. (d) ‘Spirited’ means full of energy or
33. (a) The accounting year of Reserve Bank body.
enthusiasm. Hence, ‘Dull’ is its correct
antonym which means less energetic. of India (RBI) runs from April to March. 41. (a) Vijay Hazare Trophy 2016-17 was
The RBI is India’s Central Bank, which won by Tamil Nadu state. The Vijay
23. (b) ‘Forthright’ means a person, statement
controls the issuance and supply of the Hazare Trophy, also known as the Ranji
or action that is direct and straight forward.
Indian rupee. RBI is the regulator of entire One-day Trophy was started in
So, ‘Tricky’ is its correct antonym which
banking in India. It was established in 2002-03. It is named after the famous
means difficult or complicated, not direct.
1935 under the RBI Act, 1934. Indian cricketer Vijay Hazare.
24. (a) ‘Right’ means morally good, justified It’s headquarters is situated in Mumbai, The 2019-20 Vijay Hazare Trophy
or true. Hence, ‘Correct’ is its correct Maharashtra. winner is Karnataka.
meaning.
34. (c) Agricultural commodities are graded 42. (c) The All India Service (AIS) comprises
25. (a) ‘Apprehended’ means understand or with AGMARK. It is a certification mark Indian Administrative Service (IAS), the
perceive. Hence, ‘understood’ express the employed on agricultural products in Indian Forest Service (IFS) and the
meaning of the given word. India, assuring that they conform to a set Indian Police Service (IPS). Hence,
26. (a) Ernest Hemingway was on American of standards approved by the Directorate Indian Foreign Service is not the part of
author and journalist who was awarded of Marketing and Inspection. The All India Services.
the 1954 Nobel Prize in literature for his AGMARK is legally enforced in India by
43. (c) The Chinese traveller, Hiuen Tsang
novel ‘The old man and the sea.’ Other the Agricultural produce (Grading and
visited India during the reign of
famous works of the author include ‘The Marketing) Act of 1937.
Harshavardhana. He visited India with
Torrents of Spring’, ‘The Sun Also Rises’ 35. (d) The book titled ‘The Indian War of an aim of securing authentic Buddhist
etc. Independence’ was written by VD scripts.
27. (d) Michael Ferreira is notable amateur Savarkar. The book was first published in
He has given a vivid description of the
player of English Billiards from India. 1909. The book describes the 1857
social, economic and religious conditions
A three time Amateur World Champion revolt as a unified and national uprising
under the rule of Harshvardhana.
and was awarded with Dronacharya of India as a nation against British
Award in 2001 for his coaching authority. VD Savarkar was an Indian 44. (b) The call of ‘Back to Vedas’ was given
achievements in Billiards and Snooker. independence activist, politician, lawyer by Swami Dayanand Saraswati. He was
and writer. an Indian philosopher, social leader and
28. (c) India’s first indigenously developed
founder of the Arya Samaj, reform
nuclear powered submarine is INS 36. (a) The 2016 Nobel Peace Prize was
movement of the Vedic Dharma.
Arihant. It is the lead ship of India’s awarded to the President of Columbia
He worked towards reviving Vedic
Arihant class of nuclear powered ballistic Juan Manuel Santos ‘for his resolute
ideologies.
missile submarines. It was commissioned efforts to bring the country’s more than 50
in August, 2016 into the Indian Navy. year long civil war to an end’. The Nobel 45. (b) The commando unit of the Indian Air
Peace Prize of 2019 was awarded to Force is named Garud. The unit derives
29. (b) The Thomas Cup is an international
Ethiopian Prime Minister Abiy Ahmed Ali. its name from Garuda, a divine bird-like
badminton competition among teams
creature in Hinduism. The Garud is the
representing member nations of the 37. (b) Lord Ripon was the viceroy of India
special forces unit of Indian Air force.
Badminton World Federation. It was who is popularly known as the ‘Father of
It was formed in September 2004 and
founded in 1949. Local Self Government’ in India. Lord
has a current strength of over 1500
30. (a) Sattriya dance is originated in the Ripon introduced the resolution on local
personnel.
state of Assam. It is a dance-drama Self-Government in 1882. He also
repealed the Vernacular Press Act in 46. (c) The monetary policy in India is
performance art with origins in the
1882. formulated by the Reserve Bank of India.
Krishna-centered Vaishnavism
The policy involves measures taken to
monasteries of Assam and attributed to 38. (a) Razia Sultan belonged to Mamluk
regulate the supply of money, availability
the 15th century Bhakti movement dynasty (also called Slave dynasty). She
and cost of credit in the economy.
scholar Srimanta Sankaradeva. was the first and the last Muslim women
ruler of Medieval India. She succeeded her 47. (c) The capital of Uzbekistan is
Sattriya dance is recognised as a
father Shams-ud-din Iltutmish and ruled Tashkent. Tashkent is the largest city in
classical dance of India by Sangeet Natak
the court of Delhi from 1236 to 1240. Central Asia and is located in the
Akademi of India in 2000.
North-eastern part of the country.
31. (c) The home base of the aircraft carrier 39. (c) The second nearest star to the Earth
after the Sun is Proxima Centauri. 48. (b) The pattern of series is as follows.
INS Vikramaditya is Karwar, Karnataka.
It is a modified Kiev-class aircraft carrier Proxima Centauri is small, low-mass star Z W T Q N
and the flagship of the Indian Navy, located 4.244 lights years away from the
–3 –3 –3 –3
which entered into service in 2013. Sun. This object was discovered in 1915
by Robert Innes.
46 AFCAT-II ~ Solved Paper 2017

49. (c) The pattern of series is as follows 59. (d) Except Geography, all others are
Kids
A I P V A E (c) Goats subjects of Science stream.
Lambs
+8 +7 +6 +5 +4 60. (b) Except potato, all others can be eater
50. (a) The pattern of series is as follows Here, we cannot say that some goats are raw.
kids. 61. (a) All except Seal are creatures related to
–2 –2 –2 –2
Y W U S Q signs of zodiac.
+1 +2 +3 +4 (d) Cuts Dabs Pricks
E F H K O 62. (d) Mercury is the only liquid metal at
+2 +3 +2 +3 room temperature in the group.
B D G I L Here, we can say that some dabs are cuts.
63. (d) Behead is a punishment, to cut the
57. (b)
51. (b) As, Head, which was done by a machine
(a) Decorated Trees Decorations
called Guillotine same as with the help of
F O R G E T
Gallows people were Hang to death.
–2 +1 –2 +1 –2 +1 Here, we cannot say that no
64. (d) Graphite is used as Lubricant.
decorated are decorations.
D P P H C U Similarly, Diamond is used to scratch as
a Abrasive tool.
Similarly, D O C T O R 65. (d) Money is used for transaction same
Men
–2 +1 –2 +1 –2 +1 (b) as language is used for Conversation.
Lie 66. (a) Gradual is antonym of Abrupt, and
B P A U M S Welcome Barren is antonym of Fertile.
52. (c) We know that, dog is reared as a pet 67. (b) Sack is made up of Jute. Similarly,
Here, it is clear, that all men are welcome.
but in question dog is called as Sweater is made by Wool.
mongoose. Hence, mongoose is reared Fly 68. (a) Majlis is known as parliament of Iran.
as a pet. (c) Robots
Similarly, Eduskunta is the parliament of
Birds
53. (a) According to the statements, Venn Finland.
diagram is 69. (c) Oil is extract from Seeds. Similarly,
Here, we cannot say that some birds are
robots. Butter is extract from Milk.
Animals 70. (d) Young one of Horse is called Foal and
(d) Oval Tablets Round young one of Dog is called Whelp.
Dogs
71. (a) Let distance covered one side be
Birds Here, we cannot say that all tablets are ‘D’ km.
Conclusions oval. Speed upstream = Speed of man in
I. 3 II. 7 58. (c) steady water − Speed of stream
Hence, only Conclusion I follows from the = 45 − 15
. = 3 km/h
statements. Books
(a) Speed downstream = Speed of man in
54. (a) Assumption I is implicit because it Friends steady water + Speed of stream
serves the purpose of starting internet = 45
. + 15
. = 6 km/h
Dangerous
booking facility. Total distance
∴Average velocity =
Assumption II is not implicit because it is Here, we cannot say that all dangerous Total time
irrelevant. are books. D+ D
=
55. (b) Assumption I is not implicit because D D
+
it is vague. 6 3
(b) Tables Chairs
Assumption II is implicit because it  distance 
∵ time = speed 
follows from the given statement and Ants
 
positively assumed (positive thinking). Here, we cannot say that all ants are
2D × 6 12
56. (d) tables. = = = 4 km/h
D + 2D 3
(a) Chances Genuine Man
(c) Monkey 72. (b) Let number of boys and girls are 3x
Fakes John and 2x respectively.
Total numbers of students
Here, we cannot say that all fakes are Clearly, John is not a monkey.
genuine. = 3x + 2x = 5x
Number of boys who have scholarship
Birds 20 3x
(b) Papers White = 3x × =
(d) h 100 5
Brown a ni s
Parrots M Number of girls who have scholarship
25 x
Here, we cannot say that all brown are = 2x × =
Here, we cannot say that Manish is a 100 2
white.
bird.
AFCAT-II ~ Solved Paper 2017 47

New area = 
Total number of students who have 79. (a) Area of square field = (side)2 = (70)2 13x 6 y 
× 
scholarship =
3x x
+ = 4900 m2  10 5
= 
5 2 39x y  2
3x x 70 m m
+  25 
∴Required percentage = 5 2 × 100
Increase in area =  − x y
39x y
5x
6x + 5x  25 
= × 100 = 22% 14x y 2
50x = m
25
73. (c) Let A can do the work in x days, then
Percentage increase
B can do the same work in 3x days. increase in area
According to the question, =
first area
3x − x = 40 ⇒ 2x = 40 Area grazed by horses =
Area of circle
 14x y 1 
⇒ x = 20 4 = × × 100 % = 56%
 25 xy 
∴One day work of A and B together 4 × πr2 22
= = × (35)2 83. (a) Let the father’s age = 7x,
1 1 1 1 4 7
= + = +
x 3x 20 60 22 son’s age = 3x
= × 35 × 35 = 3850 m2
3+1 4 1 7 Now, from given conditions,
= = =
60 60 15 Hence, ungrazed area by the horses ⇒ 7x × 3x = 756 ⇒ 21x2 = 756
Hence, they both will complete the work = (4900 − 3850) m2 ⇒ x2 =
756
= 36
in 15 days. 21
= 1050 m2
74. (d) Let the number is ‘x’. ⇒ x = 36 = 6
80. (a) Let the number of boys and girls be ∴ Father’s age = 7 × 6 = 42 yr
According to the question,
40 25 4x and 6x, respectively. Son’s age = 3 × 6 = 18 yr
x× =x× + 45 According to the question,
100 100 After six years,
40 x × 25 4x + 200 5 Father’s age = 42 + 6 = 48 yr
⇒ x× − = 45 ⇒
15x
= 45 =
100 100 100 6x 6 Son’s age = 18 + 6 = 24 yr
∴ x = 3 × 100 = 300 ⇒ 5x = 4x + 200 48 2
∴ Required ratio = = = 2 :1
75. (a) Let the average of 15 students be ‘x’ ⇒ x = 200 24 1
and weight of new student be ‘y’ kg. Therefore, number of girls 112 576 256
84. (d) × ×
According to the question, = 6x = 6 × 200 196 12 8
15x − 40 + y = 15 (x + 15 = 1200 112 24 16
.) = × × = 8 × 2 × 2 = 32
[∵ total weight of students = average × 81. (a) Let he took ‘x’ wickets before last 14 12 8
number of students] match. Then, 1
85. (a) One day work of A, B and C are ,
⇒15x − 40 + y = 15x + 22.5 According to the question, 20
Total runs given before last Match 1 1
⇒ y = 40 + 22.5 12.4 = and , respectively.
x 24 30
∴ y = 62.5 kg
∵Average = Sum of terms  Let the number of days to complete the
76. (d) Let the speed and length of the train  Number of terms  work be x, then according to the
be ‘v’ km/s and ‘d’ km/s respectively t be
question,
required time. According to the question, ⇒12.4 × x = Total runs given before last x x −2 x − 5
d  Distance  match … (i) + + =1
v −6= ∵ Speed = … (i) 20 24 30
18  Time  After last match 6x + 5(x − 2) + 4(x − 5)
d ⇒ =1
v − 9= … (ii) Total runs given before 120
t Last Match + 26 6x + 5x + 4x = 120 +10 + 20
Here, we have 3 variables but only two . =
12.4 − 04
x+ 5 ⇒ 15x = 150
equations.
⇒12(x + 5) = 12.4x + 26 ∴ x = 10
Hence, time cannot be determined. 10 1
[∵ from Eq. (i)] ∴ Work done by A = =
77. (c) Let the capacity of tank be ‘x’. 20 2
⇒12x + 60 = 12.4x + 26 ⇒ 34 = 04
. x
According to the question, ∴Share of A from the assured money
x 3x 3x x 34 1
+ 22 = ⇒ − = 22 ⇒ x= × 10 = 85 = × 5400 = ` 2700
5 4 4 5 4 2
15x − 4x 82. (d) Let length = x and breadth = y m
⇒ = 22 86. (b) Answer figure (b) will complete the
20
Area = Length × breadth = (x y) m2 problem figure.
11x = 22 × 20
New length = 
130  13 87. (d) Answer figure (d) will complete the
x = 40 L x = xm
 100  10 problem figure.
78. (a) LCM of 3, 4, 5 and 6 = 60
New breadth = 
120  6 88. (b) Problem figure will be completed by
y = ym
∴Required minimum number of pens  100  5 answer figure (b).
= 60
48 AFCAT-II ~ Solved Paper 2017

89. (d) Answer figure (d) will complete the 93. (d) Here, O represents and K
problem figure. represents .
90. (a) Here, H represents and C also J represents and F represents ∆.
represents—
thus . is represented by OF. 99. (d) The problem figure is hidden in answer
Also Z represents and U
represents . 94. (c) Except figure (c), in all other figure (d).
figures the triangle is blank.
Similarly, will be represented by HU.
95. (d) Except figure (d) in all other figures
91. (c) Here, F represents and Z represents
seven straight lines are used.
.
96. (d) Except figure (d), in all other
Also C represents and H represents .
figures, the inner two shapes are
100.(c) The problem figure is hidden in answer
Then, will be represented by FH. similar.
figure (c).
92. (a) Here, C represents ∆ and N 97. (c) Except figure (c), in all other
represents . figures are divided into four parts.
Also D represents . 98. (c) The problem figure is hidden in
answer figure (c).
Then, will be represented by CN.
AFCAT-I ~ Solved Paper 2017 49

INDIAN AIR FORCE


AFCAT-I

Solved Paper 2017


Time : 2 Hrs MM : 300
INSTRUCTIONS
■ The set contains a total of 100 questions, Comprising Verbal Ability in English, General Awareness, Numerical Ability and Reasoning and
Military Aptitude Test.
■ Each correct question carry 3 Marks and there will be negative marking of 1 Mark for each incorrect attempt.
■ Total time duration will be 2 hrs (120 minutes).
■ No marks will be deducted for unattempted questions.

Directions (Q.Nos. 1-3) Read the 1. How many days was the SS Titanic 4. (a) to (b) in
passages carefully and choose the best at sea before sinking? (c) with (d) for
answer to each question. (a) 2 (b) 4
(c) 6 (d) 12
5. (a) plenty (b) scarce
In an effort to produce the largest, (c) minute (d) enough
fastest and most luxurious ship afloat, 2. All of the following contributed to
British built the SS Titanic. It was so the large death toll except 6. (a) really (b) coldly
superior to anything else on the seas (a) panic (b) fire (c) badly (d) happily
that it was dubbed ‘unsinkable’. So (c) speed (d) the Carpathia 7. (a) have (b) had
sure of this were the owners that they
3. ‘Maiden voyage’ is closest in (c) being (d) has
provided only twenty life boats and
meaning to 8. (a) ecological (b) biological
rafts, less than one-half the number
(a) inaugural (b) most elegant
needed for the 2227 passengers on board. (c) logical (d) chronological
(c) longest (d) final
Many passengers were aboard the Directions (Q.Nos. 9-12) Choose the
night it rammed an iceberg only two Directions (Q.Nos. 4-8) Read the one which can be substituted for the
days at sea and more than halfway passage carefully and fill in the blanks given words/ sentences.
between England and its New York with the help of the alternatives given.
destination. Because the luxury liner 9. Belonging to all parts of the world.
Man has been tampering …(4)… the (a) Versatile (b) Universal
was travelling so fast, it was impossible
Ecosphere for a very long time and is (c) Cosmopolitan (d) Secular
to avoid the ghostly looking iceberg. An
forced to see that the environmental
unextinguished fire also contributed to 10. The study of the origin and
resources are …(5)… environmental
the ship's submersion. Panic increased history of words.
problems are …(6)… social problems.
number of casualties as people jumped (a) Linguistics (b) Etymology
They begin with people as cause and
into the icy water or fought to be among (c) Verbose (d) Anthology
end with people as victims. Unplanned
the few to board the life boats. Four
hours after the mishap, another ship, use of resource …(7)… resulted in the 11. Tough tissues in joints
the Carpathia, rescued 705 survivors. depletion of fossil fuels, pollution of air (a) Ligaments (b) Endoderm
The infamous SS Titanic had enjoyed and water, deforestation which has (c) Muscles (d) Fibre
only two days of sailing glory on its resulted in …(8)… imbalance and 12. One who goes to settle in another
maiden voyage in 1912 before plunging draining away of national wealth country
into 12000 feet of water near the coast through heavy expenditure on oil and (a) Immigrant (b) Alien
of Newfoundland where it lies today. power generation. (c) Citizen (d) Emigrant
50 AFCAT-I ~ Solved Paper 2017

Directions (Q.Nos. 13-15) Find Directions (Q.Nos. 23-26) Choose the 33. In India, which of the following
out which part of a sentence has an alternative which best expresses the have the highest share in the
error. If there is no error, your meaning of the Idiom/Phrase of bold disbursement of credit to
answer is (d). part. agriculture and allied activities?
13. We were looking forward 23. It is evident from the minister (a) Commercial Banks
(a)/to hear news (b)/ about statement that heads will roll in (b) Co-operative Banks
the missing fishermen. (c)/ the Secretariat. (c) Regional Rural Banks
No error (d) (a) transfers will take place (d) Micro-finance Institutions

14. It is better (a) / to keep one’s head (b) heads will be cut off 34. The first Law Minister of Independent
in the face of danger than (c) people will die India was
(b)/losing one’s courage. (c)/ No (d) dismissals will occur (a) MC Setalvad
error (d) 24. Ramesh takes after his father. (b) BR Ambedkar
(a) follows (c) Kailashnath Katju
15. Although the police officer (d) Rafi Ahmed Kidwai
sympathised with the poor (a)/ he (b) imitates
refused to (b)/ take an action (c) obeys 35. Comptroller and Auditor-General of
against the rich man. (c)/ No (d) resembles India is a friend and guide of
error(d) 25. If the phone again, I am going (a) Select Committee
to give him a piece of my (b) Estimate Committee
Directions (Q.Nos. 16-18) Choose the (c) Prime Minister
correct alternative to make the sentence mind.
(a) to be nice to him (d) Public Accounts Committee
meaningful.
(b) to take a revenge on him 36. What can be the maximum interval
16. The more electricity you use, .... (c) to reprimand him between two sessions of Parliament?
(a) your bill will be higher (d) to support him (a) 3 months (b) 4 months
(b) will your bill be higher (c) 6 months (d) 9 months
(c) will be higher your bill 26. The poet drew on his fancy not
(d) the higher your bill will be his knowledge of nature, when he 37. Which of the following Constitutional
wrote his poem on birds. Amendments are related to raising
17. Madhu has not been able to recall (a) used his understanding the number of members of Lok
where ...... (b) used his knowledge Sabha to be elected from the state?
(a) does she live (b) she lived (c) used his imagination (a) 6th and 22nd (b) 13th and 38th
(c) did she live (d) lived the girl (d) used his skill (c) 7th and 31st (d) 11th and 42nd
18. The teacher gives many examples to Directions (Q.Nos. 27-30) Find out the 38. Which one of the following was the
........... the idea contained in the misspelt word. official language of Gupta Period?
poem. 27. (a) Impetuous (b) Impertinant (a) Pali (b) Magadhi
(a) bring about (b) bring in (c) Prakrit (d) Sanskrit
(c) Imperial (d) Implication
(c) bring forth (d) bring out
28. (a) Mercenary (b) Machinery
39. Who abolished Iqta system?
Directions (Q.Nos. 19 and 20) Choose (a) Babur
(c) Missionery (d) Visionary
the one which best expresses the same (b) Muhammad-bin-Tughlaq
meaning of the given word. 29. (a) Seize (b) Decieve (c) Iltutmish
(c) Believe (d) Reign (d) Alauddin Khalji
19. Consignee
(a) Delegate (b) Representative 30. (a) Prefer (b) Defer 40. During the colonial rule in India,
(c) Nominee (d) Messenger (c) Difer (d) Refer the Permanent Settlement was
introduced by
20. Momentous 31. What are ‘Open Market
(a) Lord Bentick (b) Lord Cornwallis
(a) Important (b) Temporary Operations’?
(c) Lord Curzon (d) Lord Wellesley
(c) Fleeting (d) Monumental (a) Activities of SEBI- registered
brokers 41. Where was the first cotton mill in
Directions (Q.Nos. 21 and 22) Choose India established?
the word opposite in meaning to the (b) Selling of currency by the RBI
(c) Selling of gilt-edged securities by (a) Surat (b) Bombay
given word.
the RBI (c) Ahmedabad (d) Coimbatore
21. Fallible (d) Sale of shares by FIIS 42. Who initiated regeneration of Indian
(a) Unerring (b) Reliable
(c) Falsehood (d) Trustful 32. Which of the following is a Muslims in the 19th century?
project to develop watersheds (a) Syed Ahmed Khan
22. Rough in India? (b) Nawab Salimullah
(a) Refined (b) Charming (a) DRDO (b) CARE (c) Badshah Khan
(c) Smooth (d) Polite (c) AVARD (d) NWDPRA (d) Abul Kalam Azad
AFCAT-I ~ Solved Paper 2017 51

43. Gas released during Bhopal tragedy 53. Four prime numbers are written in 62. A tradesman gives 4% discount on
was ascending order of their the marked price and gives 1 article
(a) sodium isothiocyanate magnitudes. The product of the first free for buying every 15 articles and
(b) ethyl isothiocyanate three is 385 and that of the last thus gains 35%. The marked price is
(c) potassium isothiocyanate three is 1001. The largest given above the cost price by
(d) methyl isothiocyanate prime number is (a) 20% (b) 39%
(a) 11 (b) 13 (c) 17 (d) 19 (c) 40% (d) 50%
44. Biodegradable wastes can usually
be converted into useful substances 54. A paper is in the form of rectangle 63. A fruit seller buys mangoes at the
with the help of having length 28 cm and breadth 21 rate of 15 for ` 12 and sells them at
(a) bacteria cm. A semicircular portion with the rate of ` 15 per dozen. Find the
(b) nuclear proteins breadth as a diameter is cut off. The gain percentage.
(c) radioactive substances area of remaining portion is (a) 25.65% (b) 32.25%
(d) viruses (a) 400 cm 2 (b) 404 cm 2 (c) 51.35% (d) 56.25%
45. Cadmium pollution is associated (c) 410.75 cm 2 (d) 414.75 cm 2
64. A man had ` 8400. He lent a part
with 55. If the radius of circle is increased by of it at 8% at SI and the remaining
(a) minamata disease 75%, then the area is increased by 2
(b) black foot disease (a) 150% (b) 160% at 6 % SI. His total annual income
3
(c) dyslexia (c) 210% (d) None of these
was ` 588. Find the sum lent at
(d) itai-itai 56. A sphere, a cylinder and a cone 2
6 % rate.
46. The first Indian to cross Seven having same radius and height of 3
important seas by swimming cylinder and cone are equal to (a) ` 5200 (b) ` 5300 ]
(a) Amrendra Singh diameter of sphere. Ratio of their (c) ` 6300 (d) ` 6400
(b) Bula Chaudhary volume are
(a) 2 : 3 : 1 (b) 1 : 3 : 2 65. A man can row 6 km/h in still
(c) Junko Tabei
(c) 1 : 2 : 3 (d) 7 : 4 : 1 water. It takes him twice as long to
(d) Yuri Gagarin
row up as to row down the river.
47. 15th August is the Independence 57. A contractor undertook to do a work Find the rate of the stream.
Day of India and in 60 days. He employed 50 workers (a) 2 km/h (b) 3 km/h
(a) South Korea (b) Indonesia to carry out the job, but after
(c) 4 km/h (d) 5 km/h
(c) China (d) Pakistan 40 days he found that only half
work had been done. Now, how Directions (Q. Nos. 66-70) Each of
48. Which one of the following pairs is many more worker should he the question given below contains three
not correctly matched? employee to finish the work in elements. These elements may or may not
(a) Kalinga Award—Popularisation of time? have some inter-linkage. Each group of
Science (a) 50 (b) 100 (c) 150 (d) 200 element may fit into one of these diagram
(b) David Cohen Award—Literature at (a), (b), (c) and (d). You have to
58. Which of the following numbers is
(c) Borlaug Award— Agriculture indicate the group of elements which
(d) Pulitzer Prize—Progress in the least? correctly fits the diagram.
Religion (0.5)2 , 0.49, 3 0.008, 0.23 66. Which of the following diagram
49. Where is the Punjab Lalit Kala (a) (0.5)2 (b) 0.49 correctly represents the relation
Academy located? 3
(c) 0.008 (d) 0.23 between Animal, Dog and Pet?
(a) Muktasar (b) Ludhiana
(c) Patiala (d) Chandigarh 59. By selling toffees at 20 for a rupee, a
man loses 4%. In order to gain 20%,
50. Who is known as ‘Iron Man of how many for a rupee must be sell?
India’? (a) 15 (b) 16 (c) 17 (d) 18
(a) (b) (c) (d)
(a) Jawaharlal Nehru
(b) Bal Gangadhar Tilak 60. A and B are two alloys of gold and 67. Which one of the following diagram
(c) Sardar Vallabhbhai Patel copper prepared by mixing metals best depicts the relationship among
(d) Mahatma Gandhi in the proportion 7 : 2 and 7:11 Nose, Hand and Body?
respectively. If equal quantities of
51. For what value(s) of a is the alloy are melted to form a third
1
x± x + a 2 a perfect square? alloy C, the proportion of gold and
4 copper in C will be (a) (b) (c) (d)
1 1 1 1
(a) ± (b) ± (c) − (d) (a) 5 : 9 (b) 5 : 7 (c) 7 : 5 (d) 9 : 5
18 8 5 4 68. Which of the following figure best
61. A man whose bowling average is describes the relationship among
52. The difference of squares of two 12.4 takes 5 wickets for 26 runs and Ocean, Ship and Sailor?
consecutive numbers is 21. What thereby decreases his average by 0.4.
are the numbers? The number of wickets taken by
(a) 10 and 11 (b) 11 and 10 him before his last match, is
(a) 85 (b) 78 (c) 72 (d) 64 (a) (b) (c) (d)
(c) 12 and 9 (d) 8 and 13
52 AFCAT-I ~ Solved Paper 2017

69. Which of the following figure 77. (a) Prod (b) Sap 84. Problem Figure
represents, Degree students, BA (c) Jab (d) Thrust
students and B.Sc students? 78. (a) Trousers (b) Coats
(c) Shirts (d) Jackets
79. (a) Grapes (b) Pineapple ?
(c) Cashew (d) Apple
(a) (b) (c) (d) 80. (a) Uncle (X)
(b) Nephew Answer Figures
70. Which of the following Venn- (c) Brother
diagram correctly represents Planet, (d) Cousin
Earth and Sun?
Directions (Q. Nos. 81-85) In each of
the following question, select a figure
(a) (b) (c) (d)
from amongst the four alternatives,
which when placed in the blank space 85. Problem Figure
(a) (b) (c) (d) of figure (X) would complete the
pattern.
Directions (Q. Nos. 71-75) In each of
the following question, there is a certain 81. Problem Figure
relationship between two given words (X)
on one side of (::) and one word is given ?
Answer Figures
on another side of (::) while another
word is to be selected from the given
alternatives, having the same
relationship with this word as the (X)
words of the given pair bear. Choose the (a) (b) (c) (d)
correct alternative. Answer Figures
Directions (Q. Nos. 86-90) Each of the
71. Country : President :: State : ? following question, consist of five
(a) Chief Minister figures marked (1), (2), (3), (4) and (5)
(b) Prime Minister (a) (b) (c) (d) called the problem figures followed by
(c) Speaker four other figures marked (a), (b), (c)
(d) Governor 82. Problem Figure and (d) called answer figures. Select a
72. Mirage : Desert :: ? figure from amongst the answer figures
(a) Sky : Illusion which will continue the same series as
(b) Rainbow : Sky established by the five problem figures.
(c) Rain : Rainbow 86. Problem Figures
(d) Image : Mirror ?

73. Radio : Marconi :: Television : ? (X)


(a) Picture tube
Answer Figures (1) (2) (3) (4) (5)
(b) Fardey
(c) Receiver Answer Figures
(d) JL Baird
74. Perpetual : Irregular :: Prevent : ? (a) (b) (c) (d)
(a) Check (b) Appeal (a) (b) (c) (d)
(c) Stop (d) Allow 83. Problem Figure
87. Problem Figures
75. South : North-West :: North : ?
(a) North-West (b) South-East
(c) West (d) East
Directions (Q. Nos. 76-80) In each of ? (1) (2) (3) (4) (5)
the following question, four words have (X) Answer Figures
been given out of which three are alike in
Answer Figures
same manner, while the fourth one is
different. Choose the word which is
different from the rest. (a) (b) (c) (d)

76. (a) Flute (b) Violin


(a) (b) (c) (d)
(c) Guitar (d) Sitar
AFCAT-I ~ Solved Paper 2017 53

88. Problem Figures 91. Problem Figures 95. Problem Figures

? ?
(1) (2) (3) (4) (5) (1) (2) (3) (4) (1) (2) (3) (4)
Answer Figures Answer Figures Answer Figures

(a) (b) (c) (d)


(a) (b) (c) (d) (a) (b) (c) (d)
89. Problem Figures
92. Problem Figures
Directions (Q. Nos. 96-100) In each
problem, out of the four figures marked
?
(a), (b), (c) and (d), three are similar in
(1) (2) (3) (4) (5)
(1) (2) (3) (4) a certain manner. However, one figure
Answer Figures is not like the other three. Choose
Answer Figures the figure which is different from the
rest.
96.
(a) (b) (c) (d)
(a) (b) (c) (d)
90. Problem Figures
(a) (b) (c) (d)
93. Problem Figures
97.
(1) (2) (3) (4) (5) ?

Answer Figures (1) (2) (3) (4) (a) (b) (c) (d)
Answer Figures 98.

(a) (b) (c) (d)


(a) (b) (c) (d)
Directions (Q. Nos. 91-95) Each of the (a) (b) (c) (d)
following question, consist of two sets of 99.
94. Problem Figures
figures. Figures (1), (2), (3) and (4)
constitute the problem set while figures
? (a) (b) (c) (d)
(a), (b), (c) and (d) constitute the answer
set. There is a definite relationship (1) (2) (3) (4) 100.
between figures (1) and (2) establish a Answer Figures
similar relationship between figures
(3) and (4) by selecting a suitable figure (a) (b) (c) (d)
from the answer set that would replace
the question mark (?) in figure (4). (a) (b) (c) (d)

Answers
1. (a) 2. (d) 3. (a) 4. (c) 5. (b) 6. (a) 7. (d) 8. (a) 9. (c) 10. (b)
11. (a) 12. (d) 13. (b) 14. (c) 15. (b) 16. (d) 17. (b) 18. (d) 19. (c) 20. (a)
21. (a) 22. (c) 23. (d) 24. (d) 25. (c) 26. (c) 27. (b) 28. (c) 29. (b) 30. (c)
31. (c) 32. (d) 33. (a) 34. (b) 35. (d) 36. (c) 37. (c) 38. (d) 39. (d) 40. (b)
41. (b) 42. (a) 43. (d) 44. (a) 45. (d) 46. (b) 47. (a) 48. (d) 49. (d) 50. (c)
51. (b) 52. (a) 53. (b) 54. (d) 55. (d) 56. (a) 57. (a) 58. (c) 59. (b) 60. (c)
61. (a) 62. (d) 63. (d) 64. (c) 65. (a) 66. (c) 67. (b) 68. (c) 69. (b) 70. (c)
71. (d) 72. (b) 73. (d) 74. (d) 75. (b) 76. (a) 77. (b) 78. (a) 79. (c) 80. (c)
81. (a) 82. (b) 83. (b) 84. (b) 85. (c) 86. (c) 87. (b) 88. (d) 89. (b) 90. (a)
91. (a) 92. (a) 93. (c) 94. (b) 95. (b) 96. (d) 97. (c) 98. (b) 99. (c) 100. (d)
54 AFCAT-I ~ Solved Paper 2017

Hints and Solutions


1. (a) The SS Titanic was two days at sea 21. (a) ‘Fallible’ means capable of making 33. (a) In India, Commercial Banks have the
before sinking. mistakes or being wrong. So, ‘Unerring’ highest share in the disbursement of
2. (d) All of the contributed to the large is its correct answer, which means credit to agriculture and allied activities.
death toll except the Carpathia. always right or accurate. 34. (b) The first Law Minister of Independent
3. (a) ‘Maiden Voyage’ means the first 22. (c) The opposite of ‘Rough’ will be ‘Smooth’ India was BR Ambedkar who served in
journey made by a ship or spacecraft. So, as rough means not smooth or level. Prime Minister Jawaharlal Nehru’s
‘inaugural’ is closest in meaning as 23. (d) The idiomatic meaning of ‘heads will Cabinet from 1947-51. He was a
inaugural means opening, Maiden, roll’ means someone will be severely scholar, a social reformer and a leader
launching etc. punished or will lose their jobs because who dedicated his life to eradicating
of something. Hence, ‘dismissals will social inequality in India.
4. (c) with 5. (b) scarce
occur’ is its correct answer. 35. (d) The Constitution of India provides
6. (a) really 7. (d) has
24. (d) The idiomatic meaning of ‘takes after’ for an independent office of the
8. (a) ecological Comptroller and Auditor General (CAG)
is to resemble someone, especially a
9. (c) Containing or having experience of parent or grandparent. So, ‘resembles’ is of India. He is the head of the Indian
people and things from many its correct meaning. Audit and Accounts Department and
different parts of the world is called enjoys the same status as a judge of
25. (c) The given idiom ‘to give him a piece
cosmopolitan. Hence, option (c) is its Supreme Court of India in Indian order of
of my mind’ means to voice one’s
correct answer. precedence. The CAG of India also acts
disagreement or dissatisfaction, especially
10. (b) The study of the origin and history of as a friend and guide of Public Accounts
with another person. So, ‘to reprimand
words is called Etymology. Committee.
him’ is its correct answer as reprimand
11. (a) ‘Ligaments’ connect the ends of means a formal expression of disapproval. 36. (c) The maximum interval between two
bones together in order to form a joint. sessions of Parliament can be 6 months.
26. (c) The idiomatic meaning of ‘drew on
Article 81 of the Constitution empowers
12. (d) An ‘emigrant’ is a person who leaves his fancy is ‘used his imagination’.
the President to summon each house at
their own country in order to settle Hence, option (c) is its correct answer.
such intervals that there should not be
permanently in another. 27. (b) The misspelt word is ‘Impertinant. Its more than six months gap between the
13. (b) Use ‘hearing’ in place of ‘hear’. correct spelling is ‘Impertinent’, which two session.
Always use verb + ing form after the means rude or ill-mannered.
37. (c) The Constitutional Amendments 7th
expression look forward to. Here, ‘to’ is 28. (c) The misspelt word is ‘Missionery’. Its and 31st are related to raising the
used as a preposition and it takes particle correct spelling is ‘Missionary’ and it means number of members of the Lok Sabha to
form of the verb. a person sent on a religionus mission. be elected from the state. The 31st
14. (c) Here, use of ‘to loose’ is more 29. (b) The misspelt word is ‘Decieve’. Its Constitution amendment or the
appropriate in place of losing. correct spelling is ‘Deceive’ which means Constitution Act, 1973 increased the
15. (b) Use ‘yet’ before ‘he’ because although to persuade someone that something elective strength of the Lok Sabha from
is used with yet. false is the truth. 525 to 545. Under the act, the upper
16. (d) To make the given sentence complete 30. (c) The misspelt word is Difer. Its correct limit of representatives of the states goes
and meaningful, ‘the higher your bill will spelling is ‘Differ’ that means be unlike or up from 500 to 525 and that of Union
be’ is the correct alternative. dissimilar. Territory decreases from 25 to 20.
17. (b) ‘She lived’ is the correct alternative to 31. (c) The Open Market Operations refers to 38. (d) The official language of Gupta Period
fill the given blank. the sale and purchase of government was Sanskrit. The Gupta Empire founded
securities (gilt-edged securities) and by king Sri Gupta was an ancient Indian
18. (d) ‘Bring out’ is the correct phrasal verb
treasury bills by the Central bank of the empire existing from the mid to late
to fill the blank. ‘Bring out’ means to
country RBI. Open market operations in 3rd Century CE to 543 CE. This period is
make something appear, which is
considered as the ‘Golden Age of India’
suitable according to the given India is a tool that the RBI uses to
by some historians.
alternative. smoothen liquidity conditions through the
year and regulate money supply in the 39. (d) Alauddin Khalji abolished the Iqta
19. (c) ‘Consignee’ is the person or company
economy. System. Iqta System was first introduced
to whom goods or documents are
by Iltutmish in Delhi Saltanate. Under
officially sent or delivered. Among the 32. (d) The scheme of NWDPRA (National
this system the land of empire was
given options, ‘Nominee’ express the Watershed Development Project for
divided into several large and small
same meaning. Nominee is a person Rainfed Areas) is a project to develop
tracts called ‘Iqta’ and assigned these
whose name a stock or registered but watersheds in India. The project was
‘Iqtas’ to his soldiers, officers and nobles.
who is not the actual owner. launched in 1990-91 in 25 States and
2 Union Territories based on twin 40. (b) During the Colonial rule, the
20. (a) ‘Momentous’ means of great
concept of integrated watershed Permanent Settlement was introduced by
importance or significance. So,
management and sustainable farming Lord Cornwallis in 1793. The system
‘Important’ expresses the same meaning was basically an agreement between the
of the word ‘momentous’. systems.
AFCAT-I ~ Solved Paper 2017 55

British East India Company and the 1 2


49. (d) The Punjab Lalit Kala Akademi is Area of semi-circle = πr
zamindars of Bengal to fix the land located in Chandigarh. The academy has 2
revenue. This system was also called the 2
the responsibility to establish, promote, 1 22  21
‘Zamindari System’. = × × 
preserve and disseminate visual and 2 7 2
41. (b) The first Indian cotton mill ‘The performing arts in and outside the state.
[∵ diameter = 2 radius]
Bombay Spinning Mill’ was set up in 1854 50. (c) Sardar Vallabhbhai Patel is popularly Area of remaining portion
in Bombay by Cowasji Nanabhai Davar. known as the ‘Iron Man of India’. He was = Area of rectangle − Area of semi-circle
42. (a) Syed Ahmed Khan initiated an Indian politician who played a leading 2
1 22  21
regeneration of Indian Muslims in the role in the country’s struggle for = ( 28 × 21) − × × 
2 7 2
19th century. He was a Muslim independence.
Progmatist, Islamic reformist, philosopher A monument named ‘Statue of Unity’
of 19th century British India. dedicated to Sardar Vallabhbhai Patel is
In 1875, he founded the Muhammadan built in the state of Gujarat, India.
Anglo-Oriental College, the first Muslim 51. (b) Given, 11 21 21
University in South Asia. He strongly 1 1 = 588 − × × = 41475
. cm2
x± x + a2 = ( x )2 ± 2 ⋅ x ⋅ + (a)2 7 2 2
influenced other Muslim leaders 4 8
including Allama Iqbal and Jinnah. On comparing with 55. (d) Let radius of circle be r.
43. (d) The Bhopal disaster, also referred to (a ± b)2 = (a2 + b2 ± 2ab) Area of circle = πr2
75
as the Bhopal Gas tragedy was a gas 1 Radius after increase = r + r
leakage incident on the night of 2nd-3rd Clearly, a=± 100
8
December 1984. The gas leaked in the 3 7
2
=r+ r= r

Then, expression =  x ± 
disaster was Methyl Isothiocynate (MIC) 1
4 4
which resulted in killing thousands of  8 2
Area after increase = π  r  =
7 49 2
people. The tragedy is considered to be 52. (a) Let two consecutive numbers are πr
the world’s worst industrial disaster. 4  16
x, (x + 1).
44. (a) Biodegradable wastes are such According to the question, 49πr2
Increase in area = − πr 2
wastes which can be decomposed and (x + 1)2 − (x)2 = 21 16
converted into useful substances by 33 2
microbes like bacteria. These wastes can ⇒ x2 + 2x + 1 − x2 = 21 = πr
16
be easily broken down into ⇒ 2x + 1 = 21
33 2
carbondioxide, water, methane or simple ⇒ 2x = 21 − 1 = 20 πr
organic molecules by micro-organisms. ⇒ x=
20
= 10 Per cent increase = 16 2 × 100
Plant products food waste, wood, human 2 πr
and animal waste etc. are examples of ∴ Numbers are 10 and 11. 3300
= = 206.25%
biodegradable wastes. 16
53. (b) Let the prime numbers are w, x, y, z.
45. (d) Cadmium pollution is associated with 56. (a) Let radius of sphere, cylinder and
Where, w > x > y > z.
itai-itai disease. The term itai-itai disease cone be r.
was coined by locals for the severe pains, According to the question,
x × y × z = 385 ∴Height of cylinder and cone = 2r
people with the condition felt in the spine
and joints. Cadmium poisoning can also and w × x × y = 1001 … (i) Volume of sphere : Volume of cylinder :
Now, HCF of (x × y × z) Volume of cone
cause softening of the bones and kidney
and (w × x × y) = x × y 4 π
failures. Itai-itai disease is known as one = πr 3 : πr2 (2r) : r2 (2r)
of the four big pollution diseases of Japan. 385)1001(2 3 3
∵ volume of cylinder = πr2 h
46. (b) The first Indian to Cross Seven 770  
important seas by swimming is Bula 231)385(1 ∵ volume of cone = 1 πr2 h 
Chaudhary. She is a former Indian 231  3 
National Women’s Swimming Champion. 154)231(1 = 2 : 3 :1
Who have won several awards including 154 57. (a) Let be employed ‘x’ extra worker,
Arjuna and Padma Shri Award.
77)154(2 then
47. (a) 15th August is the Independence day 154 1
of India and South Korea. The Republic M1 = 50, D1 = 40, W1 =
× 2
of India gained its independence from
⇒ 77 = x × y 1
the rule of the British on 15th August, and M2 = (50 + x), D2 = 20, W2 =
1947, and South Korea gained its From Eq. (i), w × 77 = 1001 2
independence on 15th August, 1945. ⇒ w=
1001
= 13 M1 × D1 M2 × D2
By, =
48. (d) The Pulitzer Prize is an award for 77 W1 W2
achievement in newspaper, magazine 54. (d) Area of rectangle
50 × 40 × 2 = (50 + x) × 20 × 2
and online journalism, literature and = Length × Breadth
musical composition in United States. ⇒ 50 × 2 = 50 + x
= 28 × 21 = 588 cm2
It was established in 1917. ⇒ x = 50
56 AFCAT-I ~ Solved Paper 2017

. )2 = 025
58. (c) (05 . = 07
. , 049 . 62. (d) Let the CP each article be ` 100. Rate downstream = (2 × 4) km/h
Then, CP of 16 articles = ` (100 × 16) = 8 km/h
3
0008
. = 02 . = 023
. , 023 .
= ` 1600
. > (05
049 . )2 > 023
. > 3
0008
. Rate of stream
SP of 15 articles = ` 1600 ×
135
 1
= (rate downstream − rate upstream)
∴Hence, 3 0008
. is the least.  100
2
59. (b) He sells 20 toffees for 1 rupee, then = ` 2160 1
SP = ` 1 , Loss = 4% 2160 = (8 − 4) = 2 km/h
SP of each article = ` = ` 144 2
100 15
∴ CP = × SP 66. (c)
(100 − Loss%) If SP is ` 96, marked price = ` 100 Animal

=` × 1 = `
100 25 If SP is ` 144, marked price
 96  24
= ` 
100
× 144 = ` 50 Dog Pet
25  96 
Now, CP = ` , Gain = 20%,
24 ∴ Marked price = 50% above CP
(100 + Gain%) CP All the dogs are animal and some dogs
SP = 63. (d) ∵ CP of 15 mangoes = ` 12 are pet animal and some pet animal may
100
12 be dogs.
 120 25 5 ∴ CP of 1 mango = ` = ` 08
.
=` × =`
100 24 4
15 67. (b) Body
5 and selling price of 12 mangoes = ` 15
For ` , toffees sold = 20, Nose Hand
4 ∴ Then, selling price of 1 mango
For ` 1, toffees sold = 20 ×  = 16
4
15
 5 =` = ` 1. 25
12
∴He must sell 16 for a rupee. SP − CP
Nose and Hand are two entirely different
7 7 So, his gain per cent = × 100 parts of body.
60. (c) Gold in A = = , CP
7+2 9 1. 25 − 08. 68. (c) Ocean
= × 100 = 56.25%
7 7 .
08
Gold in B = = Ship
7 + 11 18 2
64. (c) Let ` x lent at 6 %, then,
7 7 21 7 3 Sailor
Gold in C = + = =
9 18 18 6 Sum lent at % = 8400 − x Ship is in the ocean and the sailor is in
2
Copper in A = = 9, According to the question, the ship.
7+2
SI on x + SI on (8400 − x) = 588 69. (b)
11 11 Degree students
Copper in B = = 2
x × 6 ×1
7 + 11 18 3 (8400 − x) × 8 × 1
⇒ + Art Science
2 11 100 100
Copper in C = + graduates graduates
9 18 = 588
15 5 20
= = ⇒ x + 67200 − 8x = 58800
18 6 3 Both Art graduates and Science graduates
Ratio of Gold and Copper in C 20x − 24x belong to the category of degree student
7 5 ⇒ = 58800 − 67200
= : =7:5 3 but both are different to each other.
6 6 4
⇒ − x = − 8400 70. (c) Planets
61. (a) Let he took ‘x’ wickets before last 3
match, then according to the question, 8400 × 3
Total runs given ⇒ x= = ` 6300
4 Sun
before last match 2
12.4 = Hence, the sum lent at 6 % is 6300. Earth
x 3
∵ average = sum of terms  Earth is a planet. But, Sun is a star.
 65. (a) Let man’s rate upstream be x km/h.
number of terms  71. (d) President and Governor are the
⇒12.4 × x = Total runs given before Then, man’s rate downstream = 2x km/h nominal heads of the country and the
last match … (i) Man’s rate in still water = 6 state respectively.
After last Match, 1 72. (b) Mirage is an illusion caused by hot air
Total runs given ⇒ (rate upstream + rate downstream)
2 conditions making one see something
before last match + 26 =6 that is not there especially the
12.4 − 04
. =
x+ 5 1 appearance of a sheet of water on a hot
⇒ (2x + x) = 6
⇒12(x + 5) = 12.4x + 26 [from Eq. (i)] 2 road or in a desert. Rainbow is an arch of
3x seven colours formed in the sky when
⇒ 12x + 60 = 12.4x + 26 ∴ =6 the sun shines through rain.
⇒ 34 = 04. x 2
34 (2 × 6) 73. (d) Radio was invented by Marconi in
⇒ x= × 10 ⇒ x= =4
4 3 1901. Similarly, Television was invented
= 85 ∴ Rate upstream = 4 km/h, by JL Baird in 1926.
AFCAT-I ~ Solved Paper 2017 57

74. (d) Perpetual and irregular are 82. (b) Answer figure (b) will complete the 91. (a) From figures I to II the outer figure is
antonymous to each other. Similarly, problem figure. replaced by a figure with one lesser
prevent and allow are antonymous to 83. (b) Answer figure (b) will complete the number of sides. The circles present
each other. problem figure. inside comes outside and new black
75. (b) If we rotate direction indicator circle is introduced inside.
84. (b) Answer figure (b) will complete the
through 135° in clockwise direction the problem figure. 92. (a) The whole figure rotates 180° and the
pointer indicating South would show inner and outer elements interchange
North-West direction. their places.
N 85. (c) Answer figure (c) will
93. (c) The inner circle changes to square
N-W N-E and vice-versa. The arrow rotates by
complete the problem figure.
180°. Also, a dot appears inside the
W
135°
E 86. (c) In each successive figure, an arrow is inner figure.
added on the left hand side of existing
94. (b) Whole figure rotates 180°.
arrow/arrows. The head of the arrow is in
S-W 95. (b) The number of circles increases from
S-E a direction opposite to adjacent arrow.
S figure 1 to 2 and one of the circle is
87. (b) In the first step, the number of arrows
Similarly, if we rotate the direction inverted. An additional line appears in
increases by one and in the second step,
indicator through 135° in clockwise each.
the number of circle decrease by one.
direction North will become South-East. These two processes are repeated 96. (d) Except figure (d), in all other
76. (a) Except flute, all others are stringed alternately while in each step the entire figures, the number of lines inside ‘L’
musical instruments. figure rotates 90° clockwise. and the number of small circles are
77. (b) Except ‘sap’ all are related to ‘pushing same.
88. (d) In each successive figure, starting
something’ while ‘sap’ is related to from left an arrow is inverting vertically. 97. (c) Only in figure (c), the angle between
‘weaken something’. both the arrows is 90°.
89. (b) In every alternate figure, the outer
78. (a) Trousers are lower garments, while design is circle. Thus, in the answer 98. (b) Except figure (b), in all other figures,
others are upper garments. figure, the outer design would be circle. the direction of two adjacent arrows is
79. (c) All except cashew are fruits. Again, in each subsequent figure, the opposite.
80. (c)All except brother are relations based outer design of the previous figure is 99. (c) Except figure (c), in all other figures,
on parents’ brothers and sisters. reduced and becomes the inner design. both the lines are intersecting each other
90. (a) Lower figure is same in two at the centre of the circle.
81. (a) Answer figure will complete consecutive figures. Upper figure is 100.(d) Except figure (d), all other figures are
the problem figure. same in every alternate figure. same when rotated.
58 AFCAT-II ~ Solved Paper 2016

INDIAN AIR FORCE


AFCAT -II

Solved Paper 2016


Time 2 Hrs MM : 300
INSTRUCTIONS
■ The set contains a total of 100 questions, Comprising Verbal Ability in English, General Awareness, Numerical Ability and Reasoning and
Military Aptitude Test.
■ Each correct question carry 3 Marks and there will be negative marking of 1 Mark for each incorrect attempt.
■ Total time duration will be 2 hrs (120 minutes).
■ No marks will be deducted for unattempted questions.

Directions (Q.Nos. 1-5) Which word Directions (Q.Nos. 6-9) Choose the 10. (a) work (b) wrench
or words explains the meaning of the word which is nearly opposite in (c) press (d) squeeze
following idioms? meaning to the given word. 11. (a) mango (b) lemon
1. All Agog 6. Beguile (c) fruit (d) banana
(a) Almighty (a) Deceive (b) Honest 12. (a) usual (b) ordinary
(b) Unmoved (c) Charm (d) Cheat (c) common (d) normal
(c) Praise someone 7. Ameliorate 13. (a) dissolves (b) carries
(d) Full of interest and excitement (a) Deteriorate (b) Procrastinate (c) contains (d) includes
2. To the manner born (c) Stagnate (d) Deviate
14. (a) continued (b) made
(a) One of low birth 8. Feckless (c) got (d) kept
(b) Naturally at ease (a) Spotted (b) Fatuous
(c) Place of one’s birth (c) Strong (d) Fawning Directions (Q.Nos. 15-18) Choose the
(d) Caesarean birth correctly spelt word.
9. Cacophonous
3. Lose one’s marbles (a) Tamed (b) Harmonious 15. (a) Ascendancy (b) Ascendncy
(a) Lose something dear to you (c) Silent (d) Domestic (c) Ascandency (d) Acsendancy
(b) Suffer a setback Directions (Q.Nos. 10-14) Select the 16. (a) Hegamony (b) Hegemony
(c) Become insane most appropriate word from the options (c) Hegemoney (d) Hegamoney
(d) Become drunk against each number. 17. (a) Parpetuate (b) Perpetuete
4. Bolt from the blue About sixty percent of the human body (c) Perpetuate (d) Perpatuate
(a) Sea swimming is water. If you could ...(10)... out a
18. (a) Apocaleptic (b) Apocalaptic
(b) Complete surprise human being like a ...(11)... you would
(c) Reckless obtain about fifty litres of water. This (c) Apocalyptic (d) Apacalyptic
(d) Careless water which is not like ...(12)... water Directions (Q.Nos. 19-22) Choose the
5. Be like a bear with a sore head because of the substance it ...(13)... is word which is nearest in meaning to the
(a) Be in a foul mood necessary to the life of the human given word.
(b) Have headache being. About a gallon of it is in the 19. Preposterous
(c) Powerful and arrogant blood vessels and is ...(14)... circulating (a) Formal (b) Judicious
(d) Restless by temperature. (c) Ridiculous (d) Ceremonious
AFCAT-II ~ Solved Paper 2016 59

20. Dissemble 25. What is the effect of training of the 33. Highest Civilian Award, Bharat
(a) Disagree (b) Shake wrong mind? Ratna was conferred in 2015 to
(c) Impeccable (d) Conceal (a) We have become perfect in all (a) Atal Bihari Vajpayee
21. Raucous aspects (b) LK Advani
(b) Art of action is too much (c) Jai Prakash Narain
(a) Flavourful (b) Jarring
(c) Boisterous (d) Evil emphasised (d) Madan Mohan Malviya
(c) Each of us could become a master 34. The indigenously developed
22. Abrogate artist
(a) Put an end to (b) Elope Navigational Satellite system is
(d) None of the above known as
(c) Gatecrash (d) Send away
26. The book, ‘On My Terms: From the (a) IRGPS (b) Galileo
Directions (Q. Nos. 23-25) Read the Grassroots to the Corridors of (c) IRNSS (d) GLONASS
following passage carefully and answer Pawar’ is the autobiography of 35. Bajirao was appointed as Peshwa at
the questions given below it. (a) Lal Thanhawla the age of 20 years under the reign
The great Acharyas have said that (b) Ram Jethmalani of
having discovered a great goal, (c) Sharad Pawar (a) Sambhaji
surrender yourself to that goal and act (d) Okram Ibobi Singh (b) Chhatrapati Shahu
towards it, drawing your inspiration (c) Rajaram II of Satara
27. 21st Conference of Parties (COP21)
from that goal whereby you will get a (d) Rajaram Chhatrapati
on Climate Change was held in
new column of energy. Do not allow
Nov-Dec 2015 at 36. Which cricketer has been conferred
this energy to be dissipated in the futile
(a) London (b) Singapore with Col. CK Naidu Lifetime
memories of past regrets or failures, Achievement Award in 2015?
(c) Paris (d) New York
nor in the excitement of the present
(a) Bishen Singh Bedi
and thus bring that entire energy 28. Artificial rain is produced by
(b) Kapil Dev
focussed into activity. That is the seeding clouds with
(c) Mohinder Amarnath
highest creative action in the world (a) Silver Iodide
(d) Syed Kirmani
outside. Thereby the individual who is (b) Potassium Nitrate
till now considered as most inefficient (c) Copper Sulphate 37. Besides the Qutab Mosque and
finds his way to the highest (d) Silver Nitrate Qutab Minar, Qutab-ud-din Aibak
achievement and success. This is said is said to have built
29. Which of the following parts of (a) Adhai-din ka Jhonpra mosque at
very easily in a second. But in order to India receives rainfall from
train our mind to this attitude needs Ajmer
retreating monsoon?
considerable training because we have (b) Khirki Masjid at Jahan-Panath
(a) North East India
already trained the mind wrongly to (c) Mausoleum of Khan-i-Jahan
(b) Tamil Nadu Coast
such an extent that we have become (c) Mahanadi Delta Tilangani
perfect in imperfections. Not knowing (d) Malabar Coast (d) Kila-i-Kuhna mosque at Delhi
the art of action, we have become 30. Who was honoured with the Order of 38. Indian ruler who defeated British in
master artist in doing wrong things. Australia, the highest civilian award their early stage of rule in India was
The totality of activity will bring the in January 2016? (a) Maharaja Ranjit Singh
country to a wrong end, indeed. The (a) Yashwant Sinha (b) Mahadaji Scindhia
point is, intellect is very powerful and (b) Vashu Bhagnani (c) Hyder Ali
everyone is driving but nobody seems (c) Sajeev Koshy (d) Tipu Sultan
to know how to control the mental (d) Sanjeev Kapoor 39. Who among the following will head
energy and direct it properly, or guide the Judicial Committee on ‘One Rank
it to the proper destination. 31. Who among the following was
awarded India’s highest peacetime One Pension (OROP)’ scheme?
23. Which of the following is the source gallantry award, Ashok Chakra (a) Y V Reddy
of energy? posthumously on 26th January, 2016? (b) L Narasimha Reddy
(a) A column that supports a (a) Lance Naik Mohan Nath Goswami (c) Sanjay Mishra
building (d) N S Reddy
(b) Lance Naik Albert Ekka
(b) Stimulation obtained from a set
(c) Lance Naik Hanumanthappa 40. The mascot of the South Asian Games
aim
Koppad conducted in February 2016 was
(c) Highest creative action
(d) Lance Naik Hemraj (a) Chikor (b) Shera
(d) Proper training of the mind to (c) Tikhor (d) Appu
achieve perfection 32. Which one of the following best
explains the occurrence of the solar 41. Who among the following shooters
24. The author’s chief concern is eclipse? clinched the 50 m pistol gold at the
(a) Establishment of socialistic ISSF World Cup in Bangkok in
(a) Position of the Moon between
pattern March 2016?
Sun and Earth
(b) Discovery of great goal in the life (b) Position of the Moon between Sun (a) Jitu Rai
(c) Regulation of energy in the proper and Earth on a new Moon (b) Pang Wei
direction (c) Both (a) and (b) (c) Wang Zhiwei
(d) Training of the mind (d) None of the above (d) Potent Warren
60 AFCAT-II ~ Solved Paper 2016

42. Which country will host 52. In an examination, a student was 60. The mean of 50 observations was
Commonwealth Games in 2018? 3 36. It was found later that an
asked to find of a certain
(a) New Zealand (b) Australia 14 observation 48 was wrongly taken
(c) Scotland (d) Malaysia 3 23. The corrected new mean is
number. By mistake, he found of
43. The largest concentration of 4 (a) 35.2 (b) 34.1 (c) 36.5 (d) 39.1
Harappan sites has been found it. His answer was 150 more than 61. The average score of a cricketer for
along the the correct answer. Find the ten matches is 38.9 runs. If the
(a) Sutlej number. average for the first six matches is
(b) Ghaggar-Hakra (a) 180 (b) 280 (c) 380 (d) 480 42, the average for the last four
(c) Indus 53. An amount of ` 735 was divided matches is
(d) Ravi between A, B and C. If each of them (a) 33.25 (b) 33.5 (c) 34.25 (d) 35
44. Heavy water is called heavy had received ` 25 less, their shares 62. On a certain sum, the simple
because would have been in the ratio of 1
1 : 3 : 2. The money received by C was interest at the end of 6 yr
(a) it is denser than ordinary water 4
(b) it is an oxide of deuteron (a) ` 195 (b) ` 200 3
(c) ` 225 (d) ` 245 becomes th of the sum. The rate
(c) it has a heavy (or bad) taste 8
(d) it has a heavier isotope of 54. 20 L of a mixture contains milk and per cent is
hydrogen water in the ratio 5 : 3. If 4 L of this (a) 7% (b) 6%
mixture is replaced by 4 L of milk, the 1
45. Which of the following soils is (c) 5% (d) 5 %
most conducive for the growth of ratio of milk to water in the new 2
cotton? mixture would be 63. A horse is sold at a profit of 25%. If
(a) Alluvial (b) Red (a) 2 : 1 (b) 7 : 3 (c) 8 : 3 (d) 4 : 3 both the cost price and selling price
(c) Laterite (d) Black 55. A man rows upstream a distance of are ` 200 less, the profit will be 5%
46. India has inked a deal with France 9 km or downstream a distance of more. The cost price is
to acquire fighter aircraft, Dassault 18 km taking 3 h each time. The (a) ` 1100 (b) ` 1200
speed of the boat in still water is (c) ` 1000 (d) ` 900
Rafale. Which is the other French 1 1
fighter aircraft in the inventory of (a) 7 km/h (b) 6 km/h 64. Two trains travel in opposite
Indian Air Force? 2 2
1 1 directions at 36 km/h and 45 km/h
(a) MIG 21 (b) Mirage 2000 (c) 5 km/h (d) 4 km/h respectively. A man sitting in
2 2
(c) AJT Hawk (d) Su-30 Mkl slower train passes the faster train
56. A can do a piece of work in 10 days. in 8 s. The length of the faster train
47. Resolution of Purna Swaraj was
He works at it for 4 days and then B is
passed on 26th January, 1930 at finishes it in 9 days. In how many
(a) Haripur (b) Lahore (a) 80 m (b) 120 m
days can A and B together finish (c) 160 m (d) 180 m
(c) Karachi (d) Calcutta the work?
48. Deepika Padukone has received (a) 6 days (b) 8 days 65. By selling a chair for ` 368, a man
Filmfare award 2016 for the best 1 1 lost 8%. For how much should he
(c) 8 days (d) 7 days have sold it to gain 15%?
actress for the movie 2 2
(a) Bajirao Mastani (a) ` 450 (b) ` 475
57. A crate of mangoes contains one (c) ` 460 (d) ` 500
(b) Happy New Year bruised mango for every 30
(c) Tamasha mangoes in the crate. If 3 out of 66. A train 110 m long is running with
(d) Piku every 4 bruised mangoes are a speed of 60 km/h. In what time
considered unsalable and there are will it pass a man who is running at
49. The largest oil field of Gujarat is in
12 unsalable mangoes in the crate, 6 km/h in the direction opposite to
(a) Sanand (b) Kadi
how many mangoes are there in the that of train?
(c) Ankleshwar (d) Kalol
crate? (a) 5 s (b) 6 s
50. Fundamental Rights are enshrined (a) 480 (b) 500 (c) 420 (d) 520 (c) 7 s (d) 10 s
in the Constitution of India in 67. If 3 men or 4 women can plough a
(a) Part I (b) Part III
58. A man can row 30 km upstream
and 44 km downstream in 10 h. He field in 43 days, in how many days
(c) Part IV (d) Part V
can also row 40 km upstream and 7 men and 5 women can plough the
51. A worker may claim ` 1.5 for 55 km downstream in 13 h. Find same field?
each km which he travels by taxi the rate of current. (a) 18 days (b) 10 days
and 50 paise for each km he drives (a) 3 km/h (b) 2 km/h (c) 12 days (d) 15 days
his own car. If in one week he (c) 4 km/h (d) 5 km/h
claimed ` 50 for travelling 80 km, 68. A man sold two watches for ` 3750
59. How long will it take for a sum of each, on one he gained 5% and on
how many kms did he travel by
money invested at 5% per annum at the other he lost 5%. What was his
taxi? simple interest to increase its value
(a) 20 km (b) 14 km total gain or loss as a percentage?
by 40%? (a) 0.25% loss (b) 2.5% loss
(c) 12 km (d) 10 km
(a) 5 yr (b) 6 yr (c) 7 yr (d) 8 yr (c) 25% gain (d) 12.5 gain
AFCAT-II ~ Solved Paper 2016 61

Directions (Q.Nos. 69-73) In each of 76. Which of the following diagram Directions (Q.Nos. 86-90) In each of
the following question, figure (X) is indicates the best relation between the following question, select a figure
given followed by four alternative Graduate, Teacher and Player? from amongst the four alternatives,
figures (a), (b), (c) and (d). Figure (X) is which satisfies the same conditions
embedded in one of the alternative (a) (b) of placement of the dots as in
figures. Trace out the alternative which figure (X).
contains figure (X) as its part. 86.
69. (c) (d)
(X) (a) (b) (c) (d)
(X) (a) (b) (c) (d) 87.
77. Which of the following diagram
70. indicates the best relation between
Leaf, Seed and Root? (X) (a) (b) (c) (d)

(X) (a) (b) (c) (d) (a) (b) 88.

71. (X) (a) (b) (c) (d)


(c) (d)
89.
(X) (a) (b) (c) (d)
Directions (Q.Nos. 78-80) In each (X) (a) (b) (c) (d)
72. problem, out of the four figures marked
(a), (b), (c) and (d), three are similar in a 90.
certain manner. However, one figure is
(X) (a) (b) (c) (d) not like the other three. Choose the figure (X) (a) (b) (c) (d)
73. which is different from the rest.
78.
Directions (Q.Nos. 91-95) Each
of the following question consists
(X) (a) (b) (c) (d) of five figures marked (1), (2), (3), (4)
and (5) called problem figures followed
Directions (Q.Nos. 74-77) Each of (a) (b) (c) (d) by four other figures marked (a), (b)
these questions given below contains (c) and (d) called the answer
79.
three elements. These elements may or figures. Select a figure from amongst
may not have some inter-linkage. Each the answer figures which will
group of elements may fit into one of continue the same series as
these diagram at (a), (b), (c) or (d). You (a) (b) (c) (d)
establish by the five problem
have to indicate the group of elements 80. figures.
which correctly fit into the diagram.
91. Problem Figures
74. Which of the following diagram
indicates the best relation between (a) (b) (c) (d)
Mountains, Earth and Forests?
Directions (Q.Nos. 81-85) In each of
the following question, four words have (1) (2) (3) (4) (5)
(a) (b) been given out of which three are alike Answer Figures
in some manner, while the fourth one is
different. Choose the word which is
different from the rest.
(c) (d) 81. (a) Bogota (b) Sydney (a) (b) (c) (d)
(c) Doha (d) Brussels
92. Problem Figures
82. (a) Retina (b) Pupil
75. Which of the following diagram (c) Vision (d) Cornea
indicates the best relation between S
Sun, Planets and Earth? 83. (a) Udayagiri and Khandagiri
(b) Badami (1) (2) (3) (4) (5)
(c) Elephanta (d) Doddabetta
(a) (b) Answer Figures
84. (a) Tibia (b) Cortex
(c) Cranium (d) Cerebellum
(c) (d) 85. (a) Saltoro Kangri (b) Kongka #
(c) Zoji La (d) Kumbharli Ghat (a) (b) (c) (d)
62 AFCAT-II ~ Solved Paper 2016

93. Problem Figures Answer Figures alternative which will replace the
question mark.
96. Anatomy : Zoology :: Paediatrics :?

S
(a) (b) (c) (d) (a) Chemistry (b) Medicine
(1) (2) (3) (4) (5) (c) Palaeontology (d) Mechanics
Answer Figures 95. Problem Figures 97. Eccrinology : Secretions ::
X C P C T X C
T X C P C T Selenography : ?
S

S S S P T S X (a) Sun (b) Mantle (c) Crust (d) Moon


S P T S X S P
(1) (2) (3) (4) (5) 98. Virology : Virus :: Semantics : ?
(a) (b) (c) (d)
Answer Figures (a) Amoeba (b) Language
(c) Nature (d) Society
94. Problem Figures X
T
C X
T
C X
T
X C
S P S P S P S P 99. Tectonics : Building :: Taxidermy: ?
C T (a) Classification (b) Conserving
(a) (b) (c) (d) (c) Stuffing (d) Collecting
(1) (2) (3) (4) (5) Directions (Q.Nos. 96-100) In each of 100. Annihilation : Fire :: Cataclysm : ?
the following question, find out the (a) Emergency (b) Tribulation
(c) Anxiety (d) Flood

Answers
1. (d) 2. (b) 3. (c) 4. (b) 5. (a) 6. (b) 7. (a) 8. (c) 9. (b) 10. (d)
11. (b) 12. (d) 13. (c) 14. (d) 15. (a) 16. (b) 17. (c) 18. (c) 19. (c) 20. (d)
21. (c) 22. (a) 23. (b) 24. (c) 25. (d) 26. (c) 27. (c) 28. (a) 29. (b) 30. (c)
31. (a) 32. (b) 33. (*) 34. (c) 35. (b) 36. (d) 37. (a) 38. (c) 39. (b) 40. (c)
41. (a) 42. (b) 43. (b) 44. (d) 45. (d) 46. (b) 47. (b) 48. (d) 49. (c) 50. (b)
51. (d) 52. (b) 53. (d) 54. (b) 55. (d) 56. (a) 57. (a) 58. (a) 59. (d) 60. (c)
61. (c) 62. (b) 63. (b) 64. (d) 65. (c) 66. (b) 67. (c) 68. (a) 69. (d) 70. (c)
71. (d) 72. (d) 73. (d) 74. (c) 75. (b) 76. (d) 77. (c) 78. (a) 79. (c) 80. (d)
81. (b) 82. (c) 83. (b) 84. (a) 85. (a) 86. (d) 87. (a) 88. (c) 89. (d) 90. (c)
91. (d) 92. (c) 93. (a) 94. (a) 95. (d) 96. (b) 97. (d) 98. (b) 99. (c) 100. (d)

Hints and Solutions


1. (d) Idiom ‘All Agog’ means ‘excited, in 7. (a) ‘Ameliorate’ means ‘to improve or 16. (b) The correctly spelt word is
high spirits or in eager expectation’. make something better’. So, ‘Deteriorate’ ‘Hegemony’. Hegemony is political or
Hence, ‘full of interest and excitement’ is is its correct answer as it means ‘to make cultural dominance or authority over
its correct answer. or become worse’. others.
2. (b) Idiom ‘To the manner born’ means 8. (c) ‘Feckless’ means ‘weak or ineffective’. 17. (c) The correctly spelt word is
‘naturally at ease in a specified way of Among the given options, ‘strong’ is ‘Perpetuate’. ‘Perpetuate’ means ‘to make
life, job or situation’. Hence, option (b) is nearly opposite in meaning to it. something continue indefinitely’.
its correct answer. 9. (b) ‘Cacophonus’ means ‘involving or 18. (c) The correctly spelt word is
3. (c) Idiom ‘Lose one’s marbles’ means ‘to producing a harsh and unpleasant ‘Apocalyptic’. ‘Apocalyptic’ means
become insane’. Hence, option (c) is a sounds’. So, ‘harmonius’ is its correct ‘showing or describing the total
suitable choice. antonym as it means ‘producing pleasant destruction and end of the world’.
4. (b) ‘Bolt from the blue’ means ‘a sudden sounds’. 19. (c) ‘Preposterous’ means ‘contrary to
and unexpected event. Hence, ‘a 10. (d) squeeze reason; utterly absurd or ridiculous’.
complete surprise’ is its correct answer. 11. (b) lemon Hence, ‘ridiculous’ is its correct answer.
5. (a) Idiom ‘Be like a bear with sore head’ 12. (d) normal 20. (d) ‘Dissemble’ means ‘to hide or conceal
means ‘to be in a bad mood that causes one’s true feelings or beliefs. Hence,
13. (c) contains
you to treat other people badly’. Hence, ‘conceal’ is its correct answer.
‘be in a foul mood’ is its correct answer. 14. (d) kept
21. (c) ‘Raucous’ means ‘harsh, loud and
6. (b) ‘Beguile’ means ‘to deceive someone 15. (a) The correctly spelt word is unpleasant sound. So, among the given
in a cunning way’. So ‘honest’ is its ‘Ascendancy’. ‘Ascendancy’ means options, ‘Boisterous’ is nearest in
correct antonym as it means ‘free from ‘occupation of a position of dominant meaning to the given word as it also
deceit, truthful and sincere’. power or influence’. means ‘noisy and full of energy’.
AFCAT-II ~ Solved Paper 2016 63

22. (a) ‘Abrogate’ means ‘to end a law, 32. (b) A solar eclipse occurs when the Moon 40. (c) The mascot of the South Asian
agreement or custom formally’. So, ‘put passes between Earth and Sun, thereby Games conducted in February 2016 was
an end to’ is nearest in meaning to the blocking out the sunlight and casting a Tikhor, the baby Indian Khino. Tikhor
given word. shadow on parts of Earth. A solar eclipse carries the message of peace, progress
23. (b) Stimulation obtained from a set aim, can only take place at the phase of new and prosperity in the South Asian region.
is the source of energy. moon. South Asian Games are a biennial
24. (c) The author’s chief concern is 33. (*) India’s highest civilian award, Bharat multi-sport event held among the athletes
regulation of energy in the proper Ratna was conferred in 2015 to Former from South Asia region.
direction. Indian Prime Minister Atal Bihari 41. (a) The International Shooting Sport
Vajpayee and Madan Mohan Malviya Federation (ISSF) World Cup was held in
25. (d) According to the passage, none of the
(posthumously) by former Indian Bangkok in March 2016. Indian Shooter
options are correct.
President Pranab Mukherjee. Jitu Rai clinched the 50 m pistol gold at
26. (c) The book, ‘On My Terms : From the Established in 1954, the award is the ISSF World Cup.
Grassroots to the Corridors of Power’ is conferred ‘‘in recognition of exceptional
the autobiography of Sharad Pawar. 42. (b) The 2018 Commonwealth Games,
service/performance of the highest were an inter-national multi-sport event
He is an Indian politician from order’’, without distinction of race, sex
Maharashtra and leader of Nationalist held on Golds Coast, Queensland
occupation or position. Australia between 4th and 15th April
Congress Party (NCP).
34. (c) The Indian Regional Navigation 2018. The 2022 Commonwealth games
27. (c) The 21st Conference of Parties System (IRNSS) is indegenously is scheduled to be held in Birmingham,
(CoP-21) or CMP11 on Climate Change developed Navigation Satellite System England.
was held in November-December, 2015 that is designed to provide geospatial
in Paris, France. The conference 43. (b) The largest concentration of
positioning information on within the Harappan sites has been found along the
negotiated the Paris Agreement, a global Indian sub-continent with operational
agreement on the reduction of climate Ghaggar-Hakra river that flowed through
name NAVIC, the regional satellite
change. The 25th Conference of North-West India and Eastern Pakistan.
navigation system provides accurate real
Parties (CoP-25) to the UNFCCC will Harappan Civilisation is one of the oldest
time positioning and timing services.
convene from 2nd-13th December, civilisations which was a part of India.
35. (b) Bajirao was appointed as Peshwa at
2019. 44. (d) Heavy water is the water in which
the age of 20 years under the reign of
28. (a) Artificial rain, also known as artificial deuterium, a heavy isotope of hydrogen
Chhatrapati Shahu. He is also known by
precipitation is the act of attempting to takes the place of hydrogen. Heavy water
the name Bajirao Ballal. Maratha empire
artificially induce or increase has high freezing and boiling points as
reached its zenith later on under reign of
precipitation, usually to stave off drought compared to ordinary water.
Chhatrapati Shahu and Bajirao. He was
or the wider global warming. Artificial one of the major contributor in expansion 45. (d) The best soil for the growth of cotton
rain is produced by seeding clouds with over the Indian subcontinent. are black soil. The clayey nature of black
silver iodide. This process is known as soil is much required for growing cotton.
36. (d) India’s former wicket-keeper Syed
‘cloud seeding’. This can be done Black soil is very fertile and can produce
Kirmani has been conferred with Col CK
using airplanes or rockets to sow to high agricultural yields with its high
Naidu Lifetime Achievement Award in
the clouds. moisture storage capacity. It is mainly
2015. He was conferred with the award
29. (b) Tamil Nadu Coast receives rainfall concentrated over the Deccan lava tract
for his contribution to Indian cricket.
from retreating monsoon during months which includes parts of Maharashtra,
37. (a) Qutab-ud-din Aibak was the founder Madhya Pradesh, Gujarat, Andhra
of October and December. The state of
of Slave dynasty and the first Sultan of Pradesh, Karnataka, Rajasthan, Uttar
Tamil Nadu receives almost half of its
Delhi Sultanate. Along with Qutab
annual rainfall during this time. Pradesh and certain parts of Tamil Nadu.
Mosque and Qutab Minar, Qutab-ud-din
30. (c) Sajeev Koshy was honoured with the Aibak is famous for building Adhai-din ka 46. (b) India has inked a deal with France to
Order of Australia, the highest civilian Jhonpra at Ajmer. acquire fighter aircraft, Dassault Rafale. The
award in January, 2016 for his service to other French fighter aircraft in the inventory
38. (c) Hyder Ali defeated British in their
dentistry in Victoria, Australia. The order of Indian Air Force is Mirage 2000. The
early stage of rule in India. He was the
of Australia is an order of chivalry Mirage 2000 is a french multirole, single
Sultan and de-facto ruler of the kingdom
established in 1975 to recognise engine fourth generation jet fighter.
of Mysore in Southern India.
Australian citizens and other persons for 47. (b) The resolution of Purna Swaraj
achievement or meritorious service. He offered strong resistance against the
(complete independence) was passed on
military advances of the British East India
31. (a) Lance Naik Mohan Nath Goswami 19th December, 1929 at Lahore session
company during the First and Second
was awarded India’s highest peacetime of INC. The resolution marked the
Anglo-Mysore Wars.
gallantry award, Ashok Chakra beginning of a large-scale political
posthumously on 26th January, 2016. 39. (b) The Judicial Committee on ‘One Rank movement against colonial rule.
While serving with the 9th battalion of One Pension (OROP)’ scheme was
It was also decided at this session that
the Parachute regiment, Goswami was headed by L. Narasimha Reddy, former
Congress would boycott the Round Table
martyred in an ambush operation while Chief Justice of Patna High Court. The
Conference being held in London and
fighting four terrorists in Kupwara district, committee, with headquarters in Delhi,
observe 26th January, 1930 as first
Jammu and Kashmir. had submitted its report in October 2016.
Independence Day of India.
64 AFCAT-II ~ Solved Paper 2016

48. (d) Deepika Padukone has received 10 + 4 40 55


Ratio of milk and water = and + = 13
Filmfare Award 2016 for the best actress 6 x− y x+ y
for the movie ‘Piku’. The filmfare award 14 80 110
= =7:3 or + = 26 …(ii)
ceremony is one of the oldest and most 6 p q
prestigious film events in India. So, the required ratio of milk and water (multiplying by 2 in both sides)
The Filmfare Award 2019 for best actress =7:3 On solving Eqs. (i) and (ii),
is given to Alia Bhatt for the movie ‘Raazi’. 55. (d) Let the speed of boat in still water be 75 110
+ = 25
49. (c) The largest oil field of Gujarat is x km and speed of current be y km. p q
located in Ankleshwar. The town is According to the condition, 80 110
known for its industrial township called 9 + = 26
x + y= =3 …(i) p q
GIDC (Gujarat Industrial Development 3 − − −
Corporation). 18
and x − y= =6 …(ii) −
5
= −1
50. (b) The Fundamental Rights are 3 p
guaranteed and protected by the Solving Eqs. (i) and (ii), we get
p=5
Constitution to all persons without any 1
2x = 9 ⇒ x = 45 . km/h or 4 km/h and q = 11
discrimination. The Fundamental Rights 2
On putting the values of p and q, we get
have been enshrined in Part III (Article So, the speed of boat in still water x − y=5
12-35) of the Constitution of India. There 1
= 4 km/h x + y = 11
are total six Fundamental Rights in the 2
Constitution of India. 1 2x = 16
56. (a) A’s work in 1 day =
51. (d) Let x and (80 − x) km are the distance 10 ∴ x=8
4 2 and y=3
covered by travelling by taxi and car A’s work in 4 days = =
respectively. 10 5 So, the speed of current = 3 km/h
2 3 59. (d) Let the principal = P
According to the question, Remaining part of work = 1 − =
. x + 050
15 . (80 − x) = 50 5 5
Then, the amount
⇒ 15 . x + 050
. × 80 − 050. x = 50 According to the question, B finishes it in
P × (100 + 40) 140P
⇒ x + 40 = 50 9 days. = =
3 1 100 100
∴ x = 10 km ∴B ’s work in 1 day = =
5 × 9 15 So, the simple interest
52. (b) Let the required number = x 140P 40P
A and B’s work in 1 day = amount − principal = − P=
According to the question, 1 1 3+ 2 1 100 100
3x 3x 21x − 6x = + = =
− = 150 ⇒ = 150 10 15 30 6 P × R×T 40P P × 5 × t
4 14 28 ∵ SI = ⇒ =
∴A and B together finish the work 100 100 100
⇒ 15x = 150 × 28 ⇒15x = 4200 1
4200 = = 6 days ∴ t = 8 yr
∴ x= = 280 1
15 6 60. (c) Average of 50 observations = 36
53. (d) Let the shares of A, B and C are x, 3x 57. (a) Let the bruised mangoes and Total sum of 50 observations
and 2x respectively after less ` 25. unsalable mangoes are 4x and 3x = Average × Total observations
According to the question, respectively, in 30 mangoes. = 50 × 36 = 1800
(x + 25) + (3x + 25) + (2x + 25) = 735 According to the condition, But 48 was wrongly taken 23.
⇒ 6x + 75 = 735 3x = 12 ⇒ x = 4 So, difference = 48 − 23 = 25
⇒ 6x = 660 ⇒ x = 110 Number of bruised mangoes = 4 × 4 = 16 Hence, total correct sum of 50
∴Share of C = 2x + 25 = 2 × 110 + 25 So, the required number of mangoes in a observations = 1800 + 25 = 1825
= 220 + 25 = 245 crate = 16 × 30 = 480 ∴ Required mean
Sum of observations
Hence, C received the money is ` 245. 58. (a) Let the speed of boat in still water =
Number of observations
54. (b) Quantity of mixture = 20 L, = x km/h 1825
and speed of current = y km/h = = 36.5
milk : water = 5 : 3 50
5 According to the question,
∴ Quantity of milk = × 20 = 12.5 L 61. (c) The average of runs of 10 matches
8 30 44
+ = 10 = 38. 9
3
and quantity of water = × 20 = 7.5 L x− y x+ y
8 ∴Total number of runs
Now, Let x − y = p, x + y = q,
Since, quantity of milk in 20 L = 12.5 L = Average × Number of matches
30 44
12.5 × 4 + = 10 = 10 × 389
.
∴Quantity of milk in 4 L = = 2.5 L p q
20 = 389
15 22
Similarly, quantity of water in 4 L of or + =5 Number of runs for the first 6 matches
p q
mixture = 15. L = 42 × 6 = 252
75 110
So, remaining quantity of milk in mixture + = 25 …(i) Number of runs for last 4 matches
= 12.5 − 2.5 = 10 L p q = 389 − 252
and water = 7.5 − 15 . =6L [∵ multiplying by 5 in both sides] = 137
AFCAT-II ~ Solved Paper 2016 65

So, the required mean/average 66. (b) According to the question, 71. (d) Figure (X) is embedded in figure (d).
=
Total runs Length of the train = 110 m
Total matches 60 × 5 2
Speed = m/s = 16 m/s
137 18 3
=
4 5 2
Speed of man = 6 × m/s = 1 m/s
= 3425
. 18 3 72. (d) Figure (X) is embedded in figure (d).
2 2 1
62. (b) Let the principal amount is P, ∵ Relative speed = 16 + 1 = 18
1 3 3 3 3
rate = r%, time = 6 yr and SI = P
4 8 [∵ both run in opposite direction]
P × R×T Hence, required time
∴ SI = Distance (length of train) 73. (d) Figure (X) is embedded in figure (d).
100 =
3 1 r Speed
P= P×6 ×
8 4 100 110 110 × 3
= = =6s
3 25 × r 1 55
⇒ = 18
8 400 3 74. (c)
∴ r = 3 × 2 = 6% 67. (c) According to the question, Earth
Mountains
63. (b) Let the cost price of horse be x. 3 men = 4 women Forests
∴ Selling price 4 × 7 28
∴7 men = = women
cost price × (100 + profit) 3 3
=
100 Then, 7 men + 5 women Mountains and forests both are present
28 43
125 5x = + 5= women on Earth.
=x× = 3 3
100 4 75. (b) Pl
Planets
∵ 4 women can plough a field = 43 days
According to the question, Sun
SP − CP ⇒ 1 women can plough a field Earth
× 100 = New profit percentage = 43 × 4 days
CP
43
 5x − 200 − (x − 200) ∴ women can plough a field Earth is a Planet but Sun is not a planet.
  3
 
⇒ 4 × 100 = 30 43 × 4 76. (d) Player Teacher
x − 200 = = 12 days
43
. x − 200 − x + 200 3
125
⇒ = 3
x − 200 10 SP × 100
025
. x 3 68. (a) CP of first watch =
⇒ = 100 + gain%
x − 200 10 3750 × 100 Graduate
= = 357142
.
⇒ 2.5x = 3x − 600 105 Some Graduates are Teachers and
⇒ . x = 600
05 SP × 100 vice-versa.
CP of second watch =
⇒ x =1200 100 − Loss% Some Teachers are Players and
So, the cost price of horse = ` 1200 3750 × 100 vice-versa.
= = 3947.36
64. (d) The relative speed of the train 95 Some Players are Graduates and
5 vice-versa. Some Graduates are Teachers
= (36 + 45) × The cost price of both watches
18 = 357142
. + 3947.36 = ` 75188 . as well as Players.
[∵ trains run in opposite direction] 77. (c) Seed
and selling price = ` 7500, loss = ` 188
. Root
= 22.5 m/s Loss
Hence, loss per cent = × 100
Time = 8 s Cost price
So, the length of faster train (distance) 188. × 100
= speed × time = 22.5 × 8 = 180 m = = 025
. % Leaf
75188 .
65. (c) Let the cost price of a chair be ` x. Alternate Method
According to the question, ∴Required loss percentage Leaf, Seed and Root are different entities.
CP × (100 − loss) 2 2
=   =   = = 0.25%
= SP r 5 1 78. (a) In figure (a), the design ( ) is
100  10  10 4 different from others.
x × 92
⇒ = 368 69. (d) Figure (X) is embedded in figure (d). 79. (c) Only in figure (c) rectangle is used in
100 place of ‘+’ sign.
368 × 100
⇒ x= = 400 80. (d) Black dot should be at the top in
92
So, the selling price at the profit of 15% figure (d) according to the given figural
series.
= 115% of cost price 70. (c) Figure (X) is embedded in figure (c).
115 81. (b) Except Sydney, all remaining are the
= × 400
100 capitals of their respective countries.
= ` 460 82. (c) Except (c), all remaining are the
internal part of an eye.
66 AFCAT-II ~ Solved Paper 2016

83. (b) Badami was the capital of chalukyas, 89. (d) In question figure one dot is placed 93. (a) Symbols present on either sides of the
while all remaining are mountains or its in the region common to circle, triangle lines interchange positions, one of the
region. and square, and the other dot is placed symbols rotates 45° clockwise one by
84. (a) Tibia is the larger and stronger of the in the region common to triangle and one in each figure and both the symbol
two bones in the leg, while all remaining square. So, from answer figure (d), along on line are replaced by new
are the part of human brain. symbols after every two figures.
85. (a) Except Saltoro Kangri, all remaining are Common 94. (a) In two consecutive figures, the
the mountain passes. Saltoro Kangri is the to circle, Common to number of arrows is same. The arc with
triangle and square and in the square appears in every alternate
peak of Saltoro mountain.
square triangle figure, also it is rotated 90° clockwise.
86. (d) In question figure, one dot is placed in
The number of arcs is increasing in every
the region common to circle and rectangle, 90. (c) In question figure, one dot is placed
alternate figure. All the outer arcs are
and the other dot is placed in the region in the region common to circle and
pointing in the same direction either
common to circle, triangle and square. So, triangle, and the other dot is
clockwise or anti-clockwise.
from answer figure(d). placed only in circle. So, from answer
figure (c), 95. (d) The elements change their positions
Common as follow
Common to Common
to circle,
circle and to circle
triangle Only
rectangle and
and square circle
triangle
87. (a) In question figure, one dot is placed in Figure (1) to (2) Figure (2) to (3)
Figure (3) to (4) Figure (4) to (5)
the region common to circle and triangle, 91. (d) In each successive figure, symbol Figure (5) to
and the other dot is common to triangle ‘ψ’ is moving from top to bottom, from Answer figure

and square. So, from answer figure (a), bottom to centre and from centre to top.
96. (b) As, Anatomy is the branch of
Common to circle Also, at the bottom position the symbol
Zoology, similarly Paediatrics is the
and triangle is inverted vertically. The symbol ‘=’ is
branch of Medicine.
moving from centre to right, from right
Common to square to left and from left to centre. The 97. (d) As Eccrinology is the branch of
and triangle symbol ‘↑’ is moving from right to left, Physiology that deals with Secretions
from left to centre and from centre to and the secretory glands. In the
88. (c) In question figure, one dot is placed in same way Selenography related to
right.
the region common to square and circle, Moon.
1
second dot is placed only in circle, and the 92. (c) Symbols ‘O’ and ‘=’ are moving
2 98. (b) As, Virology related with the effect of
third dot is placed in the region common Virus, similarly Semantics deals with the
1
to triangle and circle. So, from answer arm and 1 arm in anti-clockwise effects of language.
figure (c), 2
direction. 99. (c) Tectonics is the science dealing with
Common to circle the art of building. Similarly, Taxidermy
Only ‘ ’ is moving 1 arm in anti-clockwise
circle and triangle is the art of stuffing animals.
Common to the direction. The first symbol from
100.(d) As, Annihilation is the result of Fire.
square and circle clockwise direction changes with new
Similarly, Cataclysm is the result of
symbol in every figure.
flood.
AFCAT-I ~ Solved Paper 2016 67

INDIAN AIR FORCE


AFCAT-I

Solved Paper 2016


Time : 2 Hrs MM : 300
INSTRUCTIONS
■ The set contains a total of 100 questions, Comprising Verbal Ability in English, General Awareness, Numerical Ability and Reasoning and

Military Aptitude Test.


■ Each correct question carry 3 Marks and there will be negative marking of 1 Mark for each incorrect attempt.

■ Total time duration will be 2 hrs (120 minutes).

■ No marks will be deducted for unattempted questions.

Directions (Q. Nos. 1-3) Read the Today small-pox is no longer a threat to Directions (Q.Nos. 4-9) In the
passage carefully and choose the best humanity. Routine vaccinations have following passage some of the words
answer to each question out of the four been stopped worldwide. have been left out. First read the
alternatives. 1. Which of the following is the best passage over and try to understand
In May 1966, the World Health title for the passage? what it is about. Then, fill in the blanks
Organisation was authorised to initiate (a) The World Health Organisation with the help of the alternatives given.
a global campaign to eradicate (b) The Eradication of Small-pox Science means knowledge, but not all
small-pox. The goal was to eradicate (c) Small-pox Vaccinations knowledge is science. I know from my
the disease in one decade. Because (d) Infectious Diseases own eyesight that our dog Chippy likes
similar projects for malaria and yellow papaya; I know from a book that Akbar
2. What was the goal of the campaign was the ……(4) of Babur and died in
fever had failed, few believed that against small-pox?
small-pox could actually be eradicated, 1605; and I know ……(5) the radio that
(a) To decrease the spread of India did not do well in the latest Test
but eleven years after the initial small-pox worldwide
organisation of the campaign, no cases matches. We can call these ……(6) of
(b) To eliminate small-pox worldwide knowledge facts but they are not
were reported in the field. in ten years science.
The strategy was not only to provide (c) To provide mass vaccinations
mass vaccinations, but also to isolate Science ……(7) with facts, but not with
against small-pox world wide
patients with active small-pox in order facts which have ……(8) to do with
(d) To initiate worldwide projects for
each other, like the facts about our dog,
to contain the spread of the disease and small-pox, malaria and yellow
cricket and the Mughal ruler, those
to break the chain of human fever at the same time
facts are not related ……(9) and so
transmission. 3. According to the paragraph what have nothing to do with science.
Rewards for reporting small-pox was the strategy used to eliminate Science starts with observation.
assisted in motivating the public to aid
the spread of small-pox? 4. (a) grandson (b) grandfather
health workers. One by one, each
(a) Vaccination of the entire village
small-pox victim was sought out, (c) grand nephew (d) son
(b) Treatment of individual victims
removed from contact with others and 5. (a) on (b) in (c) since (d) from
(c) Isolation of victims and mass
treated. At the same time, the entire
vaccinations 6. (a) pieces (b) peace
village where the victim had lived was
(d) Extensive reporting of outbreaks
vaccinated. (c) whole (d) block
68 AFCAT-I ~ Solved Paper 2016

7. (a) starts (b) stops Directions (Q. Nos. 22-26) Group of 32. Kalinga Award is given for
(c) passes (d) drives four words are given below. In each (a) Art (b) Science
group, one word is correctly spelt. Find (c) Literature (d) Sports
8. (a) no (b) neither the correctly spelt word.
(c) nor (d) nothing 33. Which State Government has
22. (a) Collaborate (b) Comemorate announced to set-up bio-ethanol
9. (a) by that way (b) in any way (c) Colate (d) Choclate refinery in the State to tackle the
(c) from the side (d) in addition to menace of wheat and paddy straw
23. (a) Circuiteous (b) Clairvoyant
Directions (Q. Nos. 10-13) Out of the burning?
(c) Chivelery (d) Cavelcade (a) Haryana (b) Punjab
four alternatives, choose the one which
expresses the meaning of the given words. 24. (a) Severety (b) Sovereignity (c) Uttar Pradesh (d) Bihar
(c) Superiorty (d) Serenity 34. National Tribal Crafts Mela
10. Luxuriant
(a) Luxury-loving (b) Lovely 25. (a) Parapharnelia (b) Parsimonious ‘Aadishilp’ which provide tribal
(c) Rich (d) Abundant (c) Peacadilo (d) Peadiatrics artisans an important platform to
showcase their talents through
11. Cantankerous 26. (a) Measureable (b) Manageable their products to urban India is
(a) Cancerous (b) Ferocious (c) Marriagable (d) Manoevrable observed in November, 2015 at
(c) Quarrelsome (d) Fissiparous which place?
Directions (Q. Nos. 27-30) Four
12. Onus alternatives are given for the (a) Tripura (b) Silchar
(a) Sadness (b) Happiness underlined Idiom/Phrase. Choose the (c) Agartala (d) New Delhi
(c) Responsibility (d) Criticism alternative which best expresses the 35. Indigenous Terra Madre (ITM), a
13. Derision meaning of the underlined conclave centered on the
(a) Humiliation (b) Embarrassment Idiom/Phrase. philosophy of Slow Food, Agro
(c) Ridicule (d) Condemnation 27. I joined college late and found it biodiversity, Food Sovereignty,
difficult to catch up with other Agro-ecology, amongst others, was
Directions (Q. Nos. 14-17) Choose the held at which place?
word opposite in meaning to the given students.
(a) to compete with (a) Shillong (b) Kohima
words.
(b) to come to their level (c) Dispur (d) Tripura
14. Advanced
(c) to overtake them 36. A Bench headed by NGT
(a) Progressed (b) Outpaced
(d) to hold them and stop Chairperson Justice Swatanter
(c) Receded (d) Retarded
28. They have made many changes in Kumar directed the Delhi
15. Enlighten the policy, but how many of these Government and the which
(a) Slander (b) Bemoan changes are going to affect the man northern States to stop the age-old
(c) Darken (d) Befog in the street? practice of straw burning recently?
16. Devious (a) the homeless man (a) Punjab (b) Haryana
(b) the ordinary man (c) Rajasthan (d) All of these
(a) Straight (b) Obvious
(c) Simple (d) Superficial (c) the man who works on the street 37. Union Government issued a
(d) the man who repairs roads notification regarding the long-
17. Evanescent
29. The students wanted a holiday, but pending One Rank-One Pension
(a) Imminent (b) Permanent scheme for which category people?
(c) Pervasive (d) Immanent the Principal put his foot down and
said, ‘No’. (a) Physically handicapped
Directions (Q. Nos. 18-21) Out of the (a) asserted his authority (b) Ex-servicemen
four alternatives, choose the one which (c) Socially backward
can be substituted for the given (b) kicked them
(c) stepped out (d) None of the above
words/sentence.
(d) came downstairs 38. Chief Minister and JD (U) leader
18. One who hides away on a ship to Nitish Kumar won the Bihar
obtain a free passage 30. Why should you read between the
lines whenever I say this to you? Assembly Election 2015 and
(a) Compositor (b) Stoker sworn-in as the CM of Bihar. How
(c) Stowaway (d) Shipwright (a) read the lines with great speed
many times he placed in this
(b) interpret the lines wrongly
19. Clues available at a scene position including this time?
(c) find more meaning than the
(a) Circumstantial (b) Derivative (a) 3 (b) 4
words appear to express (c) 5 (d) 6
(c) Inferential (d) Suggestive (d) read a text line-by-line slowly
20. An unexpected piece of good 39. Tata Sons Chairman Emeritus
31. AADHAAR is a unique Ratan Tata inaugurated the T-Hub
fortune identification number, which the as the new face of India at which
(a) Windfall (b) Philanthropy Unique Identification Authority of place?
(c) Benevolence (d) Turnstile India (UIDAI) will issue for all (a) Andhra Pradesh
21. Detaining and confining someone residents in India. How many digits (b) Telangana
(a) Interruption (b) Interrogation does this number consists of?
(c) Karnataka
(c) Interment (d) Internment (a)10 (b) 8 (c) 14 (d) 12 (d) Tamil Nadu
AFCAT-I ~ Solved Paper 2016 69

1
40. The Annual Hundred Drums Wangala 50. Which motor company became the 59. A tree increases annually by th of
Festival observed on 12th November, first manufacturer to test drive its 8
its height. By how much will it
2015. It is a cultural fervour where driverless vehicle on 15th 1
ten Wangala Dance Troupes from November, 2015? increase after 2 yr if it stands
various parts of which hills? (a) Mercedes (b) Ferrari 2
today 8 m high?
(a) Garo hills (b) Khasi hills (c) Tata (d) Ford
(a) 10.75 m (b) 15.60 m
(c) Mizo hills (d) Jaintia hills 51. Union Government has approved (c) 11.85 m (d) 12.25 m
41. Which country has rescinded its Atal Mission for Rejuvenation and
60. The difference between 63% of a
law in November, 2015 banning Urban Transformation (AMRUT)
number and 45% of the same number
many families from having more Action Plans for 81 cities in which
is 342. What is 78% of that number?
than one child? State(s)?
(a) 1342 (b) 1482
(a) Indonesia (b) Bangladesh (a) Tamil Nadu (c) 1558 (d) 1670
(c) India (d) China (b) Madhya Pradesh
(c) Jharkhand 61. What would be the compound
42. India has signed two MoUs i.e. interest obtained on an amount of
co-operation in new and renewable (d) All of the above
`4000 at the rate of 5% per annum
energy and renewal of the cultural 52. The students in three classes are in after 3 yr?
exchange programme 2015-18 with each class in the ratio 2 : 3 : 5. If 40 (a) ` 612 (b) ` 578
which country? students are increased in each class the (c) ` 525.5 (d) ` 630.50
(a) Indonesia (b) France ratio change to 4 : 5 : 7. Originally the
(c) The UK (d) None of these total number of student was 62. What is the least number to be added
(a) 100 (b) 180 (c) 200 (d) 400 to 7700 to make it a perfect square?
43. Maldivian Parliament Majlis (a) 131 (b) 121
impeached whom in an 53. The ratio of incomes of two persons (c) 77 (d) None of these
extraordinary sitting where is 5 : 3 and that of their expenditure
61 Parliamentarians out of the is 9 : 5. The income of each person 63. The average age of a man and his
85 present voted in favour of the if they save ` 1300 and ` 900 son is 40 yr. The ratio of their ages is
motion? respectively, is 7 : 3. What is the man’s age?
(a) Adeeb Ghafoor (b) Mohd Jameel (a) ` 4000, ` 2400 (b) ` 3000, ` 1800 (a) 70 yr (b) 63 yr
(c) Adeeb Jameel (d) Mohd Ghafoor (c) ` 5000, ` 3000 (d) ` 4500, ` 2700 (c) 56 yr (d) 49 yr
44. Which country declared a State of 54. How much quantity of water must 64. A canteen requires 28 dozen bananas
emergency for a period of 30 days? be added to 48 mL of alcohol to for a week. How many dozen
(a) Mauritius (b) Maldives make a solution that contain 25% bananas will it require for 47 days?
(c) New Zealand (d) West Indies alcohol? (a) 2256
(a) 48 mL (b) 64 mL (b) 322
45. Which country has been declared
(c) 144 mL (d) 192 mL (c) 196
Ebola-free by the World Health
(d) None of these
Organisation (WHO) recently? 55. Dilip, Ram and Amar started a shop
(a) Liberia by investing ` 2700, ` 8100 and 65. A and B can do a piece of work in
(b) Sierra Leone ` 7200 respectively. At the end of 1 72 days, B and C in 120 days and A
(c) Guinea yr, the profit was distributed. If and C in 90 days. In what time can
(d) Both (a) and (b) Ram’s share was ` 3600, their total A alone do it?
profit was (a) 90 days (b) 120 days
46. India and UK announced deals
(a) ` 10800 (b) ` 11600 (c) 150 days (d) 180 days
worth £ 9 billion in the area of
(a) Defence (b) Cyber security (c) ` 8000 (d) None of these Directions (Q. Nos. 66-70) Each of
(c) Railway (d) All of these 56. Find the largest number which these questions given below contains
when subtracted from 10000, the three elements. These elements may or may
47. The G-20 Leaders’ Summit, hosted in
Antalya on 15th November, 2015, remainder is divisible by 32, 36, 48 not have some inter-linkage. Each group
alongwith G-20 members which and 54. of elements may fit into one of these
other countries are present there? (a) 8272 (b) 7408 (c) 9136 (d) 8674 diagram at (a), (b), (c) and (d). You
have to indicate the group of elements
(a) Azerbaijan (b) Spain 57. A shopkeeper offers his customers
(c) Malaysia (d) All of these which correctly fits the diagram.
10% discount and still makes a profit
48. India and China decided to deepen of 26%. What is the actual cost to 66. Which of the following diagram
bilateral defence ties and maintain him of an article marked ` 280? indicates the best relation between
peace and tranquility at which area? (a) ` 175 (b) ` 200 (c) ` 225 (d) ` 215 Teacher, Poet and Graduate?
(a) Ladakh (b) Leh 58. A 150 m long train crosses a mile-
(c) LAC (d) None of these stone in 15 s and a train of same (a) (b)
49. Former RBI Deputy Governor Subir length coming from opposite direction
Gokarn was appointed in in 12 s. The speed of other train is
(a) IMF (b) World Bank (a) 36 km/h (b) 54 km/h (c) (d)
(c) UNSC (d) None of these (c) 50 km/h (d) 45 km/h
70 AFCAT-I ~ Solved Paper 2016

67. Which of the following diagram 74. Clock : Time :: Thermometer : ? 83. Problem Figure
indicates the best relation between (a) Heat (b) Radiation
Teachers, Men and Authors? (c) Energy (d) Temperature
75. Paw : Cat :: Hoof : ?
(a) (b) (a) Horse (b) Lion
(c) Lamb (d) Elephant ?

(c) (d) Directions (Q. Nos. 76-80) In each of (X)


the following question, four words have Answer Figures
been given out of which three are alike in
68. Which of the following diagram same manner, while the fourth one is
indicates the best relation between different. Choose the word which is
Tennis fan, Cricket players and different from the rest. (a) (b) (c) (d)
Students? 76. (a) Room (b) Chamber
(c) Veranda (d) Cabin 84. Problem Figure
(a) (b) 77. (a) Mouth Organ (b) Electric Guitar
(c) Keyboard (d) Sonata
78. (a) Heat (b) Light
(c) (d) ?
(c) Bulb (d) Electricity
79. (a) Paper (b) Pencil (X)
69. Which of the following diagram (c) Pen (d) Stationary
indicates the best relation between Answer Figures
Rose, Flower and Lotus? 80. (a) Inch (b) Metre
(c) Yard (d) Quart
(a) (b)
Directions (Q. Nos. 81-85) In each of (a) (b) (c) (d)
the following questions, select a figure
85. Problem Figure
(c) (d) from amongst the four alternatives,
which when placed in the blank space
70. Which of the following diagram of figure (X) would complete the
indicates the best relation between pattern.
Tiger, Four-footed and Elephant? 81. Problem Figure ?
(X)
(a) (b)
Answer Figures

(c) (d) ?
(X) (a) (b) (c) (d)
Directions (Q. Nos. 71-75) In each of Answer Figures Directions (Q. Nos. 86-90) Each of the
the following question, there is a certain following question, consists of five
relationship between two given words figures marked (1), (2), (3), (4) and (5)
on one side of (::) and one word is given called the problem figure followed by
on another side of (::) while another (a) (b) (c) (d) four other figures marked (a), (b), (c)
word is to be selected from the given and (d) called answer figures. Select a
alternatives, having the same 82. Problem Figure figure from amongst the answer figures
relationship with this word as the which will continue the same series as
words of the given pair bear. Choose the ? established by the five problem figures.
correct alternative.
86. Problem Figures
71. Melt : Liquid :: Freeze : ?
(a) Ice (b) Condense
(c) Solid (d) Crystal (X)
72. Forecast : Future :: Regret : ? (1) (2) (3) (4) (5)
Answer Figures
(a) Present (b) Atone Answer Figures
(c) Past (d) Sins
73. Fear : Threat :: Anger : ?
(a) Compulsion (b) Panic (a) (b) (c) (d)
(a) (b) (c) (d)
(c) Provocation (d) Force
AFCAT-I ~ Solved Paper 2016 71

87. Problem Figures (a), (b), (c) and (d) constitute the answer Answer Figures
set. There is a definite relationship
between figures (1) and (2).
Establish a similar relationship
(1) (2) (3) (4) (5) between figures (3) and (4) by selecting (a) (b) (c) (d)
Answer Figures a suitable figure from the answer set
that would replace the question mark 95. Problem Figures
(?) is figure (4).
91. Problem Figures ?
(a) (b) (c) (d)
(1) (2) (3) (4)
88. Problem Figures ?
Answer Figures
S (1) (2) (3) (4)
T S S C C
C
C Answer Figures
(1) (2) (3) (4) (5)
Answer Figures (a) (b) (c) (d)
Directions (Q. Nos. 96-100) In each
C C (a) (b) (c) (d) problem, out of the four figures marked
(a), (b), (c) and (d), three are similar in
(a) (b) (c) (d) 92. Problem Figures a certain manner. However, one figure
is not like the other three. Choose the
89. Problem Figures ? figure which is different from the rest.
96.
(1) (2) (3) (4)
Answer Figures
(1) (2) (3) (4) (5)
(a) (b) (c) (d)
Answer Figures
97.
(a) (b) (c) (d)

(a) (b) (c) (d) 93. Problem Figures


(a) (b) (c) (d)
90. Problem Figures ? 98.

(1) (2) (3) (4)


(1) (2) (3) (4) (5) Answer Figures (a) (b) (c) (d)

Answer Figures 99.

(a) (b) (c) (d)


(a) (b) (c) (d)
(a) (b) (c) (d) 94. Problem Figures
100.
Directions (Q. Nos. 91-95) Each of the
following question, consist of two sets of ?
figures. Figures (1), (2), (3) and (4)
(1) (2) (3) (4) (a) (b) (c) (d)
constitute the problem set, while figures

Answers
1. (b) 2. (b) 3. (c) 4. (a) 5. (d) 6. (a) 7. (a) 8. (d) 9. (b) 10. (d)
11. (c) 12. (c) 13. (c) 14. (c) 15. (c) 16. (a) 17. (b) 18. (c) 19. (a) 20. (a)
21. (d) 22. (a) 23. (b) 24. (d) 25. (b) 26. (b) 27. (b) 28. (b) 29. (a) 30. (c)
31. (d) 32. (b) 33. (b) 34. (d) 35. (a) 36. (d) 37. (b) 38. (c) 39. (b) 40. (a)
41. (d) 42. (a) 43. (a) 44. (b) 45. (d) 46. (d) 47. (d) 48. (c) 49. (a) 50. (d)
51. (d) 52. (c) 53. (a) 54. (c) 55. (c) 56. (c) 57. (b) 58. (b) 59. (a) 60. (b)
61. (d) 62. (d) 63. (c) 64. (d) 65. (b) 66. (b) 67. (a) 68. (a) 69. (a) 70. (c)
71. (c) 72. (c) 73. (c) 74. (d) 75. (a) 76. (c) 77. (d) 78. (a) 79. (d) 80. (d)
81. (a) 82. (b) 83. (b) 84. (c) 85. (c) 86. (d) 87. (b) 88. (a) 89. (b) 90. (a)
91. (b) 92. (a) 93. (c) 94. (b) 95. (d) 96. (d) 97. (d) 98. (b) 99. (d) 100. (b)
72 AFCAT-I ~ Solved Paper 2016

Hints and Solutions


1. (b) ‘The Eradication of Small-Pox’ is the 19. (a) Clues available at a crime scene is written explicitly or openly’. So, option (c)
best title for the passage. called ‘circumstantial’. ‘find more meaning than the words
2. (b) To eliminate small-pox world-wide in 20. (a) ‘Wind fall’ is an amount of money appear to express’, is its correct answer.
ten years, was the goal of the campaign that you win or receive from someone 31. (d) Aadhaar is a 12-digit unique identity
against small-pox. unexpectedly. So, option (a) ‘windfall’ is number that can be obtained voluntarily by
3. (c) According to the paragraph, the correct one word substitution for the residents of India, based on their biometric
strategy to eliminate the spread of given sentence. and demographic data. An Aadhaar card is
small-pox was isolation of victims and 21. (d) Detaining and confining someone is issued to every citizen in India, by the Unique
mass vaccinations. called ‘Internment’. Identification Authority of India (UIDAI).
4. (a) grandson 22. (a) Among the given options, the 32. (b) The Kalinga award is given by UNESCO
5. (d) from correctly spelt word is ‘Collaborate’. to a person(s) who make outstanding
Comemorate → Commemorate contribution to the interpretation of Science
6. (a) pieces
Colate → Collate and Technology to the general public. It was
7. (a) starts
Choclate → Chocolate created in 1951, following a donation
8. (d) nothing from Biju Patnaik, founder President of
9. (b) in any way 23. (b) Among the given options, the
the Kalinga foundation trust in India.
correctly spelt word is ‘Clairvoyant’. The
10. (d) ‘Luxuriant’ means ‘abundant or lush 33. (b) The state government of Punjab has
other options are
in growth as vegetation’. Hence, signed a Memorandum of Understanding
Circuiteous → Circuitous
‘Abundant’ express the meaning of the (MoU) to set up bio-ethanol refinery in
Chivelery → Chivalry
given word. ‘Abundant’ means ‘existing the state to tackle the menace of wheat
Cavelcade → Cavalcade
or occuring in large amounts’. and paddy straw burning. It will help in
24. (d) Among the given options, the
11. (c) ‘Cantankerous’ means ‘bad-tempered, containing the loss of fertility and damage
correctly spelt word is ‘Serenity’. The
argumentative or uncooperative’. to environment and in generating co-products
other options are
‘Quarrelsome’ is its correct synonym as it of biogas, pellets and compost through
Severety → Severity
also means ‘argumentative, disputative etc’. processing bio-refinery effluents and pellets.
Sovereignity → Sovereignty
12. (c) ‘Onus’ means ‘something that is one’s Superiorty → Superiority 34. (d) National Tribal Crafts Mela ‘Aadishilp’
duty or responsibility’. Hence, held at New Delhi in November 2015.
25. (b) Among the given options, the
‘responsibility’ is the correct meaning of Organised by Tribal Cooperative
correctly spelt word is ‘Parsimonious’.
the given word. Marketing Development Federation of
The other options are
13. (c) ‘Derision’ means ‘a state of being India Ltd. (TRIFED), ‘Aadishilp’ aims to
Parapharnelia → Paraphernalia
laughed at or ridiculed’. So, ‘ridicule’ is give tribal artisans an opportunity to
Peacadilo → Peccadillo
its correct answer as it means ‘to laugh at showcase and sell their traditional art
Peadiatrics → Pediatrics
someone in an unkind way’. and craft works directly to the customers.
26. (b) Among the given options, the The National Crafts Mela 2019 held at
14. (c) ‘Advanced’ means ‘to move forward or correctly spelt word is ‘Manageable’. The Bhubaneswar, Odisha.
make progress’. So, ‘Receded’ is opposite other options are
in meaning to ‘Advanced’ as it means ‘to 35. (a) Indigenous Terra Madre (ITM), 2015 a
Measureable → Measurable
move back or further away from a conclave centered on the philosophy of
Marriagable → Marriageable
previous position’. slow food, agro biodiversity, food
Manoevrable → Manoeuvrable
sovereignty and agro-ecology, was held
15. (c) ‘Enlighten’ means ‘to inform or to give 27. (b) Idiom ‘To catch up with’ means ‘to at Shillong. ITM is a network of
someone greater knowledge and get to the same level, standard, or status indigenous communities, partners and
understanding about a subject or as something or someone’. Hence, ‘to organisations. It encourages
situation’. Its opposite is ‘darken’ which come to their level’ is its correct answer. communities, including youth to increase
means ‘to keep someone in dark or
28. (b) The underlined idiom, ‘the man in the their consumption of local foods, both
uninformed.’
street’ means ‘the ordinary person or the cultivated and collected, to keep people
16. (a) ‘Devious’ means something or average citizen’. Hence, option (b) is its healthy and nutritionally secure and
someone that is not straight-forward or correct answer. encourages chefs to use native plants.
‘dishonest’. So, ‘straight’ is its correct
29. (a) The given underlined idiom, ‘put his 36. (d) A bench headed by NGT chairperson
opposite word as it means ‘honest and
foot down’ means ‘to use your authority Justice Swatanter Kumar directed the
frank’.
to stop something happening’. So, Delhi government and the four northern
17. (b) ‘Evanescent’ means ‘quickly fading or ‘asserted his authority’ is its correct states of Punjab, Haryana, Rajasthan
disappearing’. Hence, ‘Permanent’ is its answer. and Uttar Pradesh to stop the age-old
correct antonym. practice of straw burning. The tribunal
30. (c) The underlined idiom ‘read between
18. (c) ‘Stowaway’ is one who hides away on the lines’ means ‘to try to understand also announced a fine of ` 2,500 to
a ship to obtain a free passage. what is meant by something that is not ` 15,000 on farmers found indulging in
straw burning.
AFCAT-I ~ Solved Paper 2016 73

37. (b) Union government issued a notification 44. (b) Maldivian Government declared a The AMRUT Scheme was launched in
regarding the long pending One Rank state of emergency for a period of 30 days 2015 to establish infrastructure that
One Pension (OROP) scheme for ahead of a planned anti government rally. could ensure adequate robust
ex-servicemen. The scheme benefits over A state of emergency given sweeping transformation by implementing urban
25 lakh veterans and war widows. The powers to security forces to arrest suspects. revival projects.
notification said pension will be re-fixed 45. (d) At the time question asked, the World 52. (c) Let the number of student in three
for all pensioners on the basis of the Health Organisation (WHO) declared classes be 2x, 3x and 5x respectively.
average of minimum and maximum Sierra Leone and Liberia as Ebola free in Due to increase of 40 students in each
pension of personnel retiring in 2013 in November 2015. Sierra Leone is one of class, we have,
the same rank and with the same length the three worst affected countries due to 2x + 40 4
of service. =
Ebola virus. The other two are Liberia and 3x + 40 5
38. (c) Chief Minister and JD (U) leader Nitish Guinea. Liberia achieved Ebola free status
⇒ 10x + 200 = 12x + 160
Kumar won the Bihar assembly Election on September, 2015. As per the recent
⇒ 2x = 40
2015 and sworn in as the CM of Bihar report, Guinea has also been declared
for a record fifth time. He again resumed Ebola free by WHO. ⇒ x = 20
the office in 2017 and has served that ∴Total number of students initially
46. (d) During the Prime Minister Narendra = 2x + 3x + 5x = 10x = 200
role six times. Modi’s visit to the UK in November
39. (b) Tata Sons Chairman Emeritus Ratan 2015, India and UK announced deals 53. (a) Let the income of two persons are
Tata inaugurated the T-Hub as the new worth 9 billion pounds in the areas of ` 5x and ` 3x, and the expenditure of
face of India at Hyderabad, Telangana. defence, cyber security and railways. two persons are ` 9y and ` 5y respectively.
T-Hub (Telangana Hub) is India’s largest ∴ 5x − 9y = 1300 …(i)
47. (d) The 2015 G20 Leaders’ Summit was
incubator for startups. The first phase of and 3x − 5y = 900 …(ii)
held in Antalya, Turkey on 15th-16th
T-HUB world house 300 start-ups and On solving Eqs. (i) and (ii), we get
November, 2015. The 2015 G20
can seat 800 employees. The Indian Antalya summit was the tenth annual x = 800, y = 300
school of Business, International Institute meeting of the G20 leaders of Income of first person
of Information Technology (Hyderabad) government. Alongwith G-20 members, = 5 × 800 = ` 4000
and National Law University, (Nalsar) are Azerbaijan, Spain, Malaysia, Senegal, and income of second person = 3 × 800
also associated with the hub. Singapore and Zimbabwe were present = ` 2400
40. (a) The Annual Hundred Drums Wangala as the guest countries. The 2019 G20 54. (c) Let x mL must be added.
Festival is a harvest festival celebrated by summit was held on 28th-29th June According to the question,
the Garo tribe from various parts of Garo 2019 in Osaka, Japan. It was the first 25
hills, who live in Meghalaya, Nagaland G20 summit to be hosted by Japan. (48 + x) × = 48
100
and Assam. Wangala is traditionally 48. (c) India and China decided to deepen 48 25
celebrated in the second week of ⇒ =
bilateral defence ties and maintain peace x + 48 100
November for two to three days or upto a and tranquility at the Line of Actual
week. The celebration is committed to 48 1
Control (LAC). The decision came after ⇒ =
the sun divine force of ripeness. the visit of 26 member delegation led by x + 48 4
41. (d) China has rescinded its nearly 40-years Vice Chairman of China’s Central Military ⇒ x + 48 = 192
old law in November, 2015 banning Commission of India. ⇒ x = 192 − 48 = 144 mL
many families from having more than one The two nations also agreed to
55. (c) Ratio of their investment
child. China’s decision to lift its one-child strengthen cooperation against terror.
= 2700 : 8100 : 7200 = 3 : 9 : 8
policy is expected to diversify the country’s 49. (a) Subir Gokarn was one of the four deputy Let the total profit = ` x
increasing ageing population. governors of the Reserve Bank of India. Then, Ram’s share
42. (a) India has signed two MoUs i.e He was appointed as an Executive Total profit
co-operation in new and renewable Director in International Monetary Fund = × Ram’ s ratio
Sum of ratio
energy and renewal of the cultural (IMF). He has also served as the former 9
exchange programme 2015-18 with chairperson of the Economic Affairs 3600 = ×x
20
Indonesia. Committee of the ASSOCHAM. Dr. 3600 × 20
Gokarn was one of the youngest Deputy ∴ x= = ` 8000
43. (a) Maldivian Parliament Majlis 9
impeached Vice-President Adeeb Ghafoor Governor of RBI.
56. (c) LCM of 32, 36, 48, 54
in an extraordinary sitting where 61 50. (d) Ford Motor Company has become the
parliamentarians out of the 85 present first manufacturer to test drive its driver 2 32, 36, 48, 54
voted in favour of the motion. He has less vehicle in November, 2015. 2 16, 18, 24, 27
been charged with plotting to assassinate 51. (d) Union Government has approved Atal 2 8, 9, 12, 27
the President Yameen Abdul Gayoom. He Mission for Rejuvenation and Urban
is the second Vice-President to be 2 4, 9, 6, 27
Transformation (AMRUT) for 81 cities in
impeached in Maldives after 5 states including Tamil Nadu, Madhya 3 2, 9, 3, 27
impeachment of his predecessor Pradesh, Jharkhand, Odisha and 3 2, 3, 1, 9
Mohamed Jameel. Mizoram.
2, 1, 1, 3
74 AFCAT-I ~ Solved Paper 2016

=2 ×2 ×2 ×2 ×2 × 3× 3× 3 342 × 100 1 1 1 1
⇒ x= = 1900 ∴ + + =
= 864 18 x y z 60
78 1 1 1
∴Required number Hence, 78% of 1900 = 1900 × ∵ + =
= 10000 − 864 100 y z 120
= 9136 = 19 × 78 = 1482 1  1 1 1  1 1
∴ = + +  − + 
57. (b) Let the cost price = ` x and marked 61. (d) Compound interest x x y z  y z
price = ` 280  t 
= P  1 +
r  1 1 1
Marked price × (100 − Discount)  − 1 = − =
SP =  100  60 120 120
100  
90  3  Hence, A alone will complete the work in
= 90% of ` 280 = × 280 5 
= 4000  1 +  − 1 120 days.
100  100 
  66. (b) Some Teachers can be Poet and
= ` 252
CP × (100 + Profit)   105 3  vice-versa. Some Graduates can be
Given, = SP = 4000    − 1
  Teachers or Poets or both. Hence, the
100  100 
diagram will be
⇒ 126% of x = 252  21 21 21 
= 4000  × × −1
∴ x=
252 × 100
= ` 200   20 20 20 
126 Teacher Poet
= 4000  − 1
9261
Length of train  8000 
58. (b) Speed of first train =
Time −
= 4000   = 4000 × 1261
9261 8000
=
150
= 10 m/s,  8000  8000
15 = ` 630.50 Graduate
150 + 150
Second case, = 12 62. (d) 89 67. (a) Some Teachers can be Authors and
x + 10
vice-versa. Some Teachers can be men
Where, x = speed of other train 8 77 00
and vice-versa. Also, some Authors can
180 64
⇒ 300 = 12x + 120 ⇒ x = = 15 m/s be men and vice-versa. So, the diagram
12 169 1300
of these partly related classes will be
∴ Required speed of other train 1521
18
= 15 × = 54 km/h Authors Men
5 ∴ Required number = 1521 − 1300 = 221
59. (a) Height of tree after 1 yr [∵ 7700 + 221 = 7921 = 89]
1
= 8 + 8 × = 9m 63. (c) Let the age of a man and his son be Teachers
8 7x and 3x respectively.
Height of tree after 2 yr 68. (a) Some Students can be Tennis fans or
1 72 + 9 81 Then, total age of a man and his son Cricket players or both. Even some Tennis
= 9+ 9× = = m
8 9 8 = Average × Number of person fans can be Cricket players and
1 = 40 × 2 = 80 yr vice-versa. Some cricket players can be
Height of tree after 2 yr
2 students and vice-versa.
⇒ 7x + 3x ⇒10x = 80
81 81 1 81  1 Cricket players Tennis fans
= + × = 1 + 
8 8 16 8  16  ∴ x=8

=
81 17 1377
× = = 1075
. m Hence, man’s age = 7 × 8 = 56 yr
8 16 128 64. (d) ∵ In 7 days, a canteen requires
Alternate Method = 28 × 12 bananas
1 25 1 Students
P = 8, r = × 100 = %, n = 2 yr [∵ 1 dozen = 12 units]
8 2 2 ∴In 1 day, a canteen requires 69. (a) Rose and Lotus are different type of
∴Required height of tree 28 × 12 Flowers.
n 2 = bananas
= P1 +
r   25   25  7
 = 8 1 +  1 + 
 100  200  400 ∴In 47 days, a canteen requires
81 17 28 × 12 Lotus Rose
= 8× × = 10.75 m = × 47 = 2256 bananas
64 16 7 Flower
60. (b) Let the number is x. = 188 dozen
70. (c) Tigers and Elephants are four-footed
According to the question, 65. (b) Let A, B and C can do a piece of
and are different animals.
63% of x − 45% of x = 342 work in x, y and z days respectively.
63 45  1 1 1 1 1 1
⇒ x× −x× = 342 ∴ 2 + +  = + +
100 100 x y z  72 120 90 Elephant Tiger

63x 45x
− = 342 1 1 1 1  5 + 3 + 4
⇒ + + =
100 100 x y z 2  360  Four-footed
63x − 45x 18x
⇒ = 342 ⇒ = 342 ⇒
1 1 1
+ + =
12 71. (c) On Melting, Liquid is formed. Like
100 100 x y z 2 × 360 this, on Freezing, Solid is formed.
⇒ 18x = 342 × 100
AFCAT-I ~ Solved Paper 2016 75

72. (c) Forecast is related to Future actions 85. (c) Answer figure (c) will complete the 91. (b) One line is added at the bottom of the
or happenings. Like this, Regret is problem figure pattern. first figure to arrive at the second figure.
related to Past actions. 86. (d) In the subsequent figures Similar logic is followed for figure which will
73. (c) First is the result of the second, i.e., respectively one, two, three,four, five replace the ‘?’.
when second happens, first arises as a ... smaller designs shift to the other 92. (a) One more arm/side is added to the first
result. Due to Threat, Fear arises. Like side of line segment in a set order. figure to arrive at the second figure. The half
this, due to Provocation, Anger arises. 87. (b) After four figures the designs move circle at the corner of the sides, which were
74. (d) Clock is used to measure Time and one step in clockwise direction and at the outside, comes in.
Thermometer is used to measure designs are inverted laterally and 93. (c) First design of first pair of problem figure
Temperature. horizontally in alternate manner. is inverted and then it is placed on the
75. (a) Foot of the Cat is called Paw. 88. (a) From first figure to second figure original to get the second figure of the pair,
Similarly, the foot of the Horse is called one design moves to the central similar concept is used in second pair to get
Hoof. position and the central design moves the right answer.
76. (c) Except ‘Veranda’, all others have to the adjacent sector while the other 94. (b) The symbol 'O’ is moving from one
door. two designs are replaced with new corner to other in clockwise direction in
designs. Similar changes would occur each step and one line is removed while
77. (d) Sonata is a long piece of classical
from problem figure (3) to (4) and second figure is formed.
music that’s usually made up of several
from problem figure (5) to answer 95. (d) One line is added and the inner design
parts and other are musical instruments.
figure. is rotated 90° to the first figure to get the
78. (a) Due to others, Heat evolve, so Heat
89. (b) From problem figure (1) to (2) the second one.
is different here.
lowermost design moves to the second 96. (d) All other figures are made of six line
79. (d) Paper, Pencil and Pen come in the position and the first design is replaced segments. Only figure (d) is made of five
class of Stationary items. with a new design. From problem line segments.
80. (d) Inch, Metre and yard are different figure (2) to (3) the first design moves
97. (d) The difference in number of lines of the
units of measuring distance. But Quart is to the third position and the lowermost
figure and the dot/dots in it is two, except in
not a unit of measuring distance. design is replaced with a new design.
figure (d), where the difference is one.
81. (a) Answer figure (a) will complete the These two steps are repeated
alternately in the subsequent figures. 98. (b) All the figures, except figure (b) consist of
problem figure pattern.
‘×’ signs only. But figure (b) consist of ‘×’
82. (b) Answer figure (b) will complete the 90. (a) In each subsequent figures, the
cross and ‘+’ (plus) signs as well.
problem figure pattern. first design shows no change, the
other two designs move one and half 99. (d) None of the figures, except figure (d),
83. (b) Answer figure (b) will complete the have a small figure inside it.
steps in anti-clockwise direction and
problem figure pattern.
shaded part of one of these two 100.(b) All the figure except figure (b), are
84. (c) Answer figure (c) will complete the designs moves anti-clockwise. divided in two equal parts.
problem figure pattern.
76 AFCAT-II ~ Solved Paper 2015

INDIAN AIR FORCE


AFCAT-II

Solved Paper 2015


Time : 2 Hrs MM : 300
INSTRUCTIONS
■ The set contains a total of 100 questions, Comprising Verbal Ability in English, General Awareness, Numerical Ability and Reasoning and
Military Aptitude Test.
■ Each correct question carry 3 Marks and there will be negative marking of 1 Mark for each incorrect attempt.
■ Total time duration will be 2 hrs (120 minutes).
■ No marks will be deducted for unattempted questions.

Directions (Q. Nos. 1-3) Read the 2. Which one is the most likely Directions (Q. Nos. 4-9) Select the
following passage carefully and answer explanation, among the following, most appropriate word from the options
the questions given below it. of the author’s metaphorical against each number.
Patience is better than wisdom: An statement that “it is not every Life is an …(4)… series of challenges
ounce of patience is worth a pound of garden that grows the herbs to
and opportunities to be seized. You
brains. All men praise patience, but few make it with”?
have to plan for exercising the right
can practise it. It is a medicine which is (a) Patience is a must for solving all
career choices and …(5)… the right
good for all diseases, but it is not every our problems in life
opportunities. Planned …(6)… rather
garden that grows the herbs to make it (b) Patience is a rare herb that cures
all diseases than a hasty decision is …(7)… as far
with. Many people are born crying, live as your career is concerned. You need
complaining and die disappointed. (c) Patience is virtue of a highest order
(d) It is only a small number of to …(8)… what occupational groups,
They think every other person’s burden
people that are found observing …(9)… and type of organisations are
to be light and their own feathers to be
patience in life suitable for you.
heavy as lead and yet if the truth were
known, it is their fancy rather than 3. The writer’s remarks, “They think 4. (a) enticing (b) exciting
their fate that makes things go so hard every person’s burden to be light (c) encharming (d) enhancing
with them. Many would be well off and their own feathers to be as heavy 5. (a) catching (b) offsetting
than what they think of. as lead” is very significant. It means
(c) grabbing (d) conceiving
1. Which of the following is the most (a) They are always worried and
dejected 6. (a) delay (b) hindrance
suitable explanation to the author’s
remark that an “ounce of patience (b) They consider their own problems (c) application (d) execution
is worth a pound of brains”? to be difficult to solve as 7. (a) desirable (b) deciphered
compared with problems of other
(a) Wisdom can only be attained by (c) inevitable (d) acceptable
people
practising patience
(c) They feel that they alone face 8. (a) check (b) classify
(b) Patience comes first, wisdom next
serious problems while others (c) divide (d) analyse
(c) Patience is essential for every
have a nice time
achievement in life 9. (a) specifications (b) limitations
(d) They remain very much worried
(d) Without patience wisdom cannot (c) qualifications (d) identification
about their own problem
be used properly
AFCAT-II ~ Solved Paper 2015 77

Directions (Q. Nos. 10-13) Choose the Directions (Q. Nos. 22-26) Choose the 33. Which city will host the 2022
word which is nearest in meaning to the correctly spelt word. Commonwealth Games?
given word. 22. (a) Dolorous (b) Dolurous (a) Gold Coast, Australia
10. Nebulous (b) Birmingham, England
(c) Doloreus (d) Delorious
(a) Tiny (b) Vague (c) Lusaka, Zambia
23. (a) Condiut (b) Conduit (d) Nairobi, Kenya
(c) Insignificant (d) Inadequate
(c) Connduit (d) Condeut
11. Debilitate 34. What is the name of ‘Innovative
24. (a) Gragarious (b) Gragerious Council’ proposed in the Rail
(a) Weaken (b) Attack
(c) Gregarious (d) Grigareous Budget 2015 to promote
(c) Surmount (d) Destroy
innovation?
12. Anathema 25. (a) Intrensic (b) Intrinsic
(a) Rail Tech (b) Sankalp
(a) Religious Chant (c) Entrensic (d) Entrinsec (c) Kayakalp (d) Navachar
(b) Pun 26. (a) Sepulchral (b) Sepalchrle 35. Which of the following Asian
(c) Musical Subject (c) Sepalchral (d) Sepulchrle countries celebrates its
(d) Curse
Directions (Q. Nos. 27-30) Which word Independence Day on 4th
13. Penchant or words explains the meaning of the February?
(a) Liking (b) Eagerness following idioms. (a) Sri Lanka (b) Bahrain
(c) Disability (d) Keenness (c) Cambodia (d) Bangladesh
27. To flog a dead horse
Directions (Q. Nos. 14-17) Choose the (a) To act in a foolish way 36. Animals active at night are called
word which is nearly opposite in (b) To waste one’s efforts (a) Diurnal (b) Nocturnal
meaning to the given word. (c) To revive interest in an old subject (c) Parasites (d) Nacto-diurnal
14. Refractory (d) To revive old memories 37. Think-tank of Government of India
(a) Refreshing (b) Burdensome 28. A bull in a China shop that replaced Planning Commission
(c) Privileged (d) Manageable (a) A person who is very ugly but on 1st January, 2015 is
15. Insolent loves the beautiful things of life (a) NITI Vakya (b) NITI Dharma
(a) Sullen (b) Affable (b) A person who takes a sadistic (c) NITI Vajpayee (d) NITI Aayog
(c) Determined (d) Rich delight in harming innocent 38. The book, ‘Born Again on the
people Mountain’ is authored by
16. Acrimonious
(c) A person who becomes too excited (a) Pooja Rani (b) Arunima Sinha
(a) Legal (b) Severe where no excitement is warranted
(c) Cursive (d) Harmonious (c) Bachendri Pal (d) Urbashi Lal
(d) A rough and clumsy person at a
17. Sceptic place where skill and care are 39. The present Chairman of ISRO is
(a) Cryptic (b) Believer required (a) MYS Prasad
(c) Support (d) Eminent (b) Shailesh Nayak
29. The alpha and the omega (c) AS Kiran Kumar
Directions (Q. Nos. 18-21) Choose the (a) The beginning and the end (d) K Radhakrishnan
word that best defines the given (b) A nice act
phrases. (c) The stars and the Moon 40. Natural radioactivity was
(d) Very costly discovered by
18. To hold something in leash (a) Marie Curie
(a) To restrain (b) To disappoint 30. A fool’s errand (b) Ernest Rutherford
(c) To dismiss (d) To discourage (a) A blunder (c) Henri Becquerel
(b) An impossible task
19. To talk through one’s hat (d) Enrico Fermi
(c) A useless undertaking
(a) To speak fluently 41. Sir CV Raman was awarded Noble
(d) None of the above
(b) To talk nonsense Prize for his work connected with
(c) To talk wisdom 31. Serena Williams defeated ……… to which of the following
(d) To speak at random win the Women’s Singles Title of phenomenon of radiation?
Australian Open 2015.
20. To throw up the sponge (a) Scattering (b) Diffraction
(a) Venus Williams (c) Interference (d) Polarisation
(a) To surrender or give up a contest (b) Sania Mirza
(b) To offer a challenge (c) Maria Sharapova 42. In which atmospheric layer are the
(c) To become utterly disappointed (d) Ana Ivanovic communication satellites located?
(d) To maintain grit and enthusiasm (a) Stratosphere (b) Exosphere
32. Vinod Mehta an eminent Indian (c) Troposphere (d) Mesosphere
until the end journalist and founder editor-in-chief
21. To get into hot waters of ‘Outlook’ published his 43. Who was the Viceroy when the
(a) To be impatient autobiography by the name of Simon Commission visited India?
(b) To suffer a huge financial loss (a) Lucknow Boy: A Memoir (a) Lord Chelmsford
(c) To get into trouble (b) One Life is Not Enough (b) Lord Reading
(d) To be in a confused state of (c) Accidental Prime Minister (c) Lord Irwin
mind (d) The Substance and the Shadow (d) Lord Wellington
78 AFCAT-II ~ Solved Paper 2015

1
44. Mahatma Gandhi left South Africa 54. A can do of a piece of work in 62. A man lent a sum of money at the
to return to India in 2 rate of simple interest of 4%. If the
1
(a) 1911 (b) 1915 (c) 1917 (d) 1919
8 days while B can do of the same interest for 8 yr is ` 340 less than
3
the principal, the principal is
45. Todar Mal was associated with …… work in 8 days. In how many days
(a) ` 500 (b) ` 520
in Akbar’s darbar.
can both do it together? (c) ` 540 (d) ` 560
(a) music (b) literature
(a) 9.6 (b) 10.5 (c) 11.2 (d) 16 63. A sum was invested for 3 yr at
(c) finance (d) law reforms
1
46. Lines joining places of equal 55. A does as much work as B and C simple interest at a certain rate.
temperature are called 1 2 Had it been invested at 4% higher
does as much work as A and B rate of interest, it would have
(a) isotherms (b) isohyets 2
together. If C alone can finish the fetched ` 600 more. The sum is
(c) isomers (d) isobars
(a) ` 4000 (b) ` 4950
47. Who has been awarded the Dada work in 40 days, then together all (c) ` 5000 (d) ` 5150
Saheb Phalke Award for the year
2014? will finish the work in 64. The average weight of 50 boys in a
1 class is 45 kg. When one boy leaves
(a) Pran (a) 30 days (b)13 days
3 the class, the average reduces by
(b) Shashi Kapoor
(c) 15 days (d) 20 days 100 g. Find the weight of the boy
(c) Dilip Kumar
who left the class.
(d) Amitabh Bachchan 56. A train 150 m long is running with (a) 50 kg (b) 50.8 kg
48. How many days Moon takes to a speed of 68 km/h. In what time
(c) 49 kg (d) 49.9 kg
revolve around the Earth? will it pass a man who is running at
1 1 2 1 8 km/h in the same direction in 65. In a mixture of 35 L, the ratio of
(a) 26 (b) 27 (c) 28 (d) 29 which the train is going? milk and water is 4 : 1. If 7 L of
3 3 3 2
(a) 8 s (b) 8.5 s (c) 9 s (d) 9.5 s water is added to the mixture, the
49. Indian-American music composer, ratio of milk and water of the new
Ricky Kej has won the Grammy 57. A train covers a distance of 12 km mixture will be
Award for the album in 10 min. If it takes 6 s to pass a (a) 2 : 1 (b) 1 : 2
(a) In the Lonely Hour telegraph post, the length of the (c) 4 : 5 (d) 5 : 4
(b) Morning Phase train is
(a) 90 m (b) 100 m Directions (Q. Nos. 66-70) Each of
(c) Beyonce
(c) 120 m (d) 140 m these questions given below contains
(d) Winds of Samsara
three elements. These elements may or
50. Recently tested Agni-V missile is 58. A’s salary is 50% more than B’s. may not have some inter-linkage. Each
capable of carrying nuclear payload How much per cent is B’s salary group of elements may fit into one of
of less than A’s? these diagrams at (a), (b), (c) and (d).
1 1
(a) 1000 kg (b) 1500 kg (a) 63 % (b) 43 % You have to indicate the group of
(c) 2000 kg (d) 2500 kg 4 4 elements which correctly fits into the
1 1
(c) 53 % (d) 33 % diagrams.
51. The Russian name of INS 3 3
Vikramaditya is 66. Which of the following diagrams
(a) Admiral Gorshkov 59. The price of an item is decreased by indicates the best relation between
(b) Admiral Gorbachev 25%. What per cent increase must Females, Doctors and Patients?
(c) Admiral Nakhimov be done in new price to get the
(d) Admiral Petr Bezobrazov original price?
1 3 (a) (b)
(a) 33 % (b) 43 %
52. A boat travels upstream from B to A 3 4
and downstream from A to B in 3 h. 2 1
If the speed of the boat in still (c) 55 % (d) 65 %
3 4
water is 9 km/h and the speed of (c) (d)
the current is 3 km/h, the distance 60. Anmol sold two items for ` 1000
between A and B is each. On one, he gained 10% and
(a) 9 km (b) 10 km on other, he lost 10%. How much 67. Which of the following diagrams
did he gain or loss in the whole indicates the best relation between
(c) 11 km (d) 12 km
transaction? Detectives, Spies and Men with
53. A steamer moves with a speed of Beards?
(a) Profit 0.95% (b) Loss 1%
4.5 km/h in still water to a certain
(c) Profit 10% (d) Loss 10%
upstream point and comes back to
(a) (b)
the starting point in a river which 61. If the selling price is doubled, the
flows at 1.5 km/h. The average speed profit triples. The profit per cent is
of steamer for the total journey is 1
(a) 66 % (b) 150%
(a) 12 km/h (b) 9 km/h 3 (c) (d)
(c) 6 km/h (d) 4 km/h (c) 100% (d) 90%
AFCAT-II ~ Solved Paper 2015 79

68. Which of the following diagrams 75. Capriciousness : Reliability :: ? Directions (Q. Nos. 86-90) Each of the
indicates the best relation (a) Heated : Boiling following questions consists of five figures
between Males, Fathers and (b) Tenacious : Practicality marked 1, 2, 3, 4 and 5 called the problem
Human Being? (c) Arbitrary : Whimsical figures followed by four other figures
(d) Unreliable : Inhuman marked (a), (b), (c) and (d) called the
answer figures. Select a figure from
(a) (b) Directions (Q. Nos. 76-80) In each of amongst the answer figures which will
the following questions, four words
continue the same series as established
have been given out of which three are by the five problem figures.
(c) (d) alike in some manner, while the fourth
one is different. Choose the word which 86. Problem Figures
is different from the rest.
69. Which of the following diagrams 76. (a) Niagara (b) Alamere
indicates the best relation (c) Kempty (d) Andes (1) (2) (3) (4) (5)
between Steel, Wood and Building
Material? 77. (a) Small Betrayals Answer Figures
(b) We Indians
(c) Shooting from the Hip
(a) (b)
(d) Surviving Men
(a) (b) (c) (d)
78. (a) Brass (b) Bronze
(c) Silver (d) Steel 87. Problem Figures
(c) (d)
T C = S C S +
79. (a) Documentary (b) Puppet
X T = C S
(c) Commentary (d) Feature S = C + S +
70. Which of the following diagrams 80. (a) Deccan (b) Kaas (1) (2) (3) (4) (5)
indicates the best relation (c) Chhota Nagpur (d) Nanda Devi Answer Figures
between Doctors, Teachers and X +
Engineers? Directions (Q. Nos. 81-85) In each of
the following questions, select a figure
(a) from amongst the four alternatives, (a) (b) (c) (d)
(b)
which when placed in the blank space
of figure (X) would complete the 88. Problem Figures
pattern.
(c) (d) 81. Problem Answer Figures
Figure (1) (2) (3) (4) (5)
Directions (Q. Nos. 71-75) In each of Answer Figures
the following questions find out the
alternative which will replace the ?
question mark. (X) (a) (b) (c) (d) (a) (b) (c) (d)
71. Blizzard : Freeze :: ?
(a) Insult : Humiliation 82. 89. Problem Figures
(b) Bad : Immoral
(c) Caution : Careless ?
(d) Jealousy : Respect
(X) (a) (b) (c) (d) (1) (2) (3) (4) (5)
72. Graceful : Clumsy :: ?
(a) Horror : Sympathy 83. Answer Figures
(b) Laugh : Cry
(c) Recapitulate : Synopsis ?
(d) Encumber : Burden
(X) (a) (b) (c) (d) (a) (b) (c) (d)
73. Zenith : Nadir :: ?
84. 90. Problem Figures
(a) Indigent : Poverty ?
(b) Zeal : Eagerness
C S S C
(c) Success : Peak
(d) Genuine : Phony (1) (2) (3) (4) (5)
(X) (a) (b) (c) (d)
74. Pedagogy : Teaching :: ? Answer Figures
(a) Telepathy : Emotions
85.
(b) Radiology : Sound # #
(c) Mycology : Fossils ?
(d) Entomology : Insects (X) (a) (b) (c) (d) (a) (b) (c) (d)
80 AFCAT-II ~ Solved Paper 2015

Directions (Q. Nos. 91-95) Each of the 93. Problem Figures Directions (Q. Nos. 96-100) In each
following questions consists of two sets problem, out of the four figures
of figures. Figures 1, 2, 3 and 4 marked( a), (b), (c) and (d), three are
constitute the problem set while figures ? similar in a certain manner. However,
(a), (b), (c) and (d) constitute the answer one figure is not like the other three.
(1) (2) (3) (4)
set. There is a definite relationship Choose the figure which is different
between figures (1) and (2). Establish a Answer Figures from the rest.
similar relationship between figures (3) 96.
and (4) by selecting a suitable figure
from the answer set that would replace
the question mark (?) in fig.(4). (a) (b) (c) (d) (a) (b) (c) (d)
91. Problem Figures
94. Problem Figures 97.
?
?
(1) (2) (3) (4) (a) (b) (c) (d)
(1) (2) (3) (4)
Answer Figures 98.
Answer Figures

(a) (b) (c) (d)

(a) (b) (c) (d) 99.


(a) (b) (c) (d)
92. Problem Figures
95. Problem Figures
(a) (b) (c) (d)
? S
? 100.
(1) (2) (3) (4)
(1) (2) (3) (4)
Answer Figures
Answer Figures (a) (b) (c) (d)

S S S
(a) (b) (c) (d) S
(a) (b) (c) (d)

Answers
1. (d) 2. (d) 3. (b) 4. (b) 5. (c) 6. (d) 7. (a) 8. (d) 9. (a) 10. (b)
11. (a) 12. (d) 13. (a) 14. (d) 15. (b) 16. (d) 17. (b) 18. (a) 19. (b) 20. (a)
21. (c) 22. (a) 23. (b) 24. (c) 25. (b) 26. (a) 27. (b) 28. (d) 29. (a) 30. (c)
31. (c) 32. (a) 33. (b) 34. (c) 35. (a) 36. (b) 37. (d) 38. (b) 39. (c) 40. (c)
41. (a) 42. (b) 43. (c) 44. (b) 45. (c) 46. (a) 47. (b) 48. (*) 49. (d) 50. (b)
51. (a) 52. (d) 53. (d) 54. (a) 55. (b) 56. (c) 57. (c) 58. (d) 59. (a) 60. (b)
61. (c) 62. (a) 63. (c) 64. (d) 65. (a) 66. (c) 67. (d) 68. (c) 69. (a) 70. (b)
71. (a) 72. (b) 73. (d) 74. (d) 75. (b) 76. (d) 77. (b) 78. (c) 79. (b) 80. (d)
81. (d) 82. (c) 83. (b) 84. (c) 85. (d) 86. (c) 87. (c) 88. (a) 89. (d) 90. (a)
91. (b) 92. (a) 93. (c) 94. (d) 95. (c) 96. (c) 97. (a) 98. (b) 99. (d) 100. (b)
AFCAT-II ~ Solved Paper 2015 81

Hints and Solutions


1. (d) ‘‘An ounce of patience is worth a 19. (b) Phrase ‘To talk through one’s hat’ He died in New Delhi on 8th March,
pound of brains’’ means ‘without means ‘to talk foolishly, wildly or 2015 after a prolonged illness.
patience wisdom cannot be used ignorantly’. So, ‘to talk nonsense’ is its 33. (b) The 2022 Commonwealth Games is
properly’. So, option (d) is the most correct meaning. scheduled to be held in Birmingham,
suitable explanation to the given author’s 20. (a) Phrase ‘To throw up the sponge’ England. This will be the third
remark. means ‘to give up a contest or Commonwealth Games to be hosted in
2. (d) The meaning of the given author’s acknowledge defeat. So, option (a) ‘To England following London 1934 and
metaphorical statement is that, only a surrender or give up a contest’ is its Manchester 2002.
small number of people are found correct answer. 34. (c) An ‘Innovative Council’ named
observing patience in life. 21. (c) Phrase ‘To get into hot waters’ means ‘Kayakalp’ was proposed in the Rail
3. (b) The given writer’s remark is very ‘to get into trouble or in a difficult Budget 2015 to promote innovation.
significant. It means that people consider situation.’ So, option (c) is a suitable
The council has been set-up by the then
their own problems to be difficult to solve choice.
Railway Minister Suresh Prabhu with
as compared with problems of other 22. (a) The correctly spelt word is ‘Dolorous’. noted industrialist Shri Ratan Tata as its
people. Dolorous means ‘feeling or expressing head.
4. (b) exciting 5. (c) grabbing great sorrow or distress.’
35. (a) Sri Lanka celebrates its
6. (d) execution 7. (a) desirable 23. (b) The correctly spelt word is ‘Conduit’. Independence Day every year on 4th
8. (d) analyse 9. (a) specifications Conduit is a pipe or passage for water or February. The day commemorates the
electrical wires to go through. country’s political independence from
10. (b) ‘Nebulous’ means ‘unclear, vague or
ill-defined’. So, ‘Vague’ is its correct 24. (c) The correctly spelt word is ‘Gregarious’ British rule in 1948.
answer as it also means ‘unclear or and it means ‘a person fond of company 36. (b) Animals who remain active at night
indefinite’. or sociable.’ and sleeps during the day are called
11. (a) ‘Debilitate’ means ‘to make someone 25. (b) The correctly spelt word is ‘Intrinsic’. ‘Nocturnal’. Bats, Owls, Skunks etc. are
weak and infirm’. So, ‘Weaken’ is nearest ‘Intrinsic’ means ‘belonging naturally or some of the examples of Nocturnal
essential.’ animals.
in meaning to the given word.
26. (a) The correctly spelt word is 37. (d) The National Institution for
12. (d) ‘Anathema’ is a formal curse by a
‘Sepulchral’. ‘Sepulcharal’ means ‘of or Transforming India, also called ‘NITI
pope or a council of the church or
relating to burial.’ Aayog’ is a Policy Think Tank of
denouncing a doctrine. Hence, ‘Curse’ is
nearest in meaning to the given word. 27. (b) Idiom ‘To flog a dead horse’ means ‘to Government of India. It was established
waste effort on something that has no on 1st January, 2015 to replace the
13. (a) ‘Penchant’ means ‘a strong or
chance of succeeding’. Hence, option (b) planning commission instituted in 1950.
habitual liking for something or tendency
‘to waste one’s efforts’ is its correct 38. (b) Arunima Sinha is the author of the
to do something’. Hence, ‘Liking’ is its
answer. book ‘Born Again on the Mountain’
correct answer.
28. (d) Idiom, ‘a bull in a China shop’ means launched by Indian Prime Minister
14. (d) ‘Refractory’ means ‘stubborn or
‘a rough and clumsy person at a place Narendra Modi in December 2014.
unmanageable’. Hence, ‘Manageable’ is
where skill & care are required’. Hence, Arunima Sinha is a former Indian
its correct antonym.
option (d) is its correct answer.
15. (b) ‘Insolent’ means ‘lacking respect or Volleyball player, and the first female
29. (a) Idiom ‘The alpha and the omega’ amputee to climb Mount Everest.
rude’. So, ‘Affable’ is its correct opposite
means ‘the beginning and the end’. So,
meaning word as it means ‘friendly or 39. (c) The Chairman of ISRO in 2015 (at
option (a) is a suitable choice.
good natured’. the time question asked) was AS Kiran
30. (c) Idiom ‘A fool’s errand’ means ‘an effort Kumar. In 2014, he was awarded the
16. (d) ‘Acrimonius’ means ‘bitter or harsh in
that is unlikely to be successful’. Hence, Padma Shri, India’s fourth highest
speech or manner’. Its correct opposite
option (c) ‘a useless undertaking’ is its Civilian Award, for his contributions to
meaning word is ‘Harmonious’.
correct answer.
‘Harmonius’ means ‘agreement or the fields of Science and Technology.
discussion that is friendly and peaceful.’ 31. (c) Serena Williams won the Women’s
K. Sivan is the current Chairman (As per
Singles Titles of Australian Open 2015 by
17. (b) ‘Sceptic’ is a person who doubts the 2019) of ISRO who assumed the office
defeating Russian Tennis Player Maria
truth or value of an idea or belief. So, on 15th January, 2019.
Sharapova.
‘Believer’ is its correct antonym as it 40. (c) Henri Becqueral was a French
means ‘a person who believes in the truth In Australian Open 2019, Naomi Osaka
Physicist who discovered natural
or existence of something.’ of Japan defeated Petra Kvitova in the
radioactivity. He won the 1903 Nobel
18. (a) Phrase ‘To hold something in leash’ final to win the Women’s Singles Titles.
Prize in Physics with Pierre and Marie
means ‘to maintain strict or tight control 32. (a) Outlook’s founder editor-in-chief and Curie.
over someone’. Hence, ‘to restrain’ is its renowned journalist, Vinod Mehta’s
He died on 25th August 1908, at the
correct meaning because it means ‘to autobiography ‘Lucknow Boy : A Memoir’
age of 55 in Le Croisic, France.
keep someone or something under was published in New Delhi in
control or within limits.’ November, 2011.
82 AFCAT-II ~ Solved Paper 2015

41. (a) Sir CV Raman was an Indian 49. (d) In 2015, Indian American music 1
55. (b) Given, C’s work of 1 day = part
Physicist who won the Nobel Prize in composer, Ricky Kej has won the Best 40
Physics in 1930 for his work on the New Age Trophy for his album ‘Winds of ∵ (A + B)’ work of 1 day = 2 × C’s work
phenomenon of scattering of light and for Samsara’ at 57th Annual Grammy 1
of 1 day = part
the discovery of Raman effect. Award. 20
National Science Day is celebrated every 50. (b) Agni-V is an intercontinental ballistic Therefore, 1 day work of A, B and C
missile developed by the Defence 1 1 1+ 2 3
year on 28th February in India to = + = = part
commemorate the discovery of ‘Raman Research and Development Organisation 40 20 40 40
(DRDO) of India. 1 40
effect’ by CV Raman. The required time = =
The missile having a strike range of 3 3
42. (b) Communication satellites are located
5000 km is capable of carrying 1500 kg 40
in Exosphere. This is the outermost layer 1
of atmosphere. The exosphere extends to of nuclear payload. = 13 days
51. (a) INS Vikramaditya is a modified 3
10,000 km above the Earth’s surface. In
this layer, atoms and molecules escape Kiev-class aircraft carrier which entered 56. (c) Length of train = 150 m
into space. into service in 2013. 5
Speed = 68 km/h = 68 × = 18.89 m/s
43. (c) The Simon Commission visited India The Russian name of INS Vikramaditya 18
is ‘Admiral Gorshkov’. 5
in 1928 during the viceroyalty of Lord Speed of man = 8 km/h = 8 × m/s
Irwin. The commission was strongly 52. (d) Speed of boat = x = 9 km/h 18
opposed by the Congress and other = 2.22 m/s
Speed of stream of river = y = 3 km/h
nationalist leaders because there was no Relative speed of man and train
Speed of boat in downstream = 9 + 3
Indian representative in it. = 18.89 − 2.22
= 12 km/h
44. (b) At the request of Gopal Krishna [∵ same direction]
Speed of boat in upstream = 9 − 3
Gokhale, Mahatma Gandhi left South = 16.66 m/s
= 6 km/h Length of train
Africa to return to India in 1915. Time taken by boat in upstream and ∴The required time =
During his stay in South Africa, Gandhiji Relative speed
downstream = t1 + t2 = 3 h
fought against injustice and class 15
Let the distance covered by boat = D = = 9s
division. 16.66
∵ t1 + t2 = D  + 
1 1
45. (c) Todar Mal was one of Navaratnas in  12 6  57. (c) The train covers 12000 m in 600 s.
Akbar’s darbar. He was the Finance  distance  [∵ 1 km = 1000 m and 1h = 60 s]
Minister of the Mughal empire during ∵ time = speed  Speed of train =
Distance
Akbar’s reign. As a Finance Minister, he  
1 + 2 D×3 Time
⇒ 3= D 
transformed the land revenue collection  ⇒ 3= =
12000
= 20 m/s
and assessment into a systematic process.  12  12
600
46. (a) Isotherms are imaginary lines joining ∴ D = 12 km Now, train takes 6 s to pass a telegraph
places of equal temperature. 53. (d) Speed of steamer = 4.5 km/h post.
Isobars are imaginary lines joining places Speed of stream = 1.5 km/h Length of train
So, 6 =
of equal atmospheric pressure. Speed of steamer in upstream 20
Isohyets are imaginary lines joining
= 4.5 − 1.5 = 3 km/h = v1 ∴ Length = 6 × 20 = 120 m
places having equal rainfall. Therefore, the length of train = 120 m
Speed of steamer in downstream
Isomers are molecules with same
= 4.5 + 1.5 = 6 km/h = v2 58. (d) The required loss percentage
molecular formulae but different
∴Average speed =
2v1v2  a 
structures. = × 100
v1 + v2  100 + a 
47. (b) Shashi Kapoor was an Indian actor
2 × 3 × 6 36
who got awarded the Dada Saheb Phalke = =  50 
3+ 6 9 = × 100
Award in 2014 for his contribution to the  100 + 50 
Indian Cinema. = 4 km/h
50
The Award is India’s highest award in 1 = × 100
54. (a) A’s work of 8 days = part 150
cinema, presented annually at the 2 100 1
National Film Award ceremony. 1 = = 33 %
A’s work of 1 day = part 3 3
Amitabh Bachchan got honoured with 16
59. (a) The required increase percentage
Dadasaheb Phalke Award for the year 1
B’s work of 8 days = part  a 
2019. 3 value =  × 100
1 1  100 − a 
48. Moon takes 27.32 or 27 days to B’s work of 1 day = part
3 24  25 
1 1 5 = × 100
revolve around the Earth as per sidereal 1 day work of A and B = + =  100 − 25 
1 16 24 48
month and takes 29.5 or 29 days as 25
2 So, the required days =
48
= 96
. days = × 100
5 75
per synodic month. 100 1
= = 33 %
3 3
AFCAT-II ~ Solved Paper 2015 83

60. (b) If a person sells two different articles Total weight of remaining students 70. (b) Some doctors can be techers and
at the same selling price, one at a gain of = 44900 × 49 = 2200100 g vice-versa. Some engineers can be
a% and another at a loss of a%, then the Weight of student who left teachers and vice-versa.
seller always incurs a loss which is given = (2250000 − 2200100) g
2
= 49900 g = 499
by loss % =   %
a . kg
Doctors Teachers Engineers
 10 65. (a) Given, Milk : Water = 4 : 1
2 Total quantity of given mixture = 35 L
Hence, required loss% =   %
10
4
 10 Quantity of milk = × 35 = 28 L 71. (a) First is the cause and second is its
5 effect.
= 1% ∴ Quantity of water = 35 − 28 = 7 L 72. (b) As, graceful and clumsy are opposite
61. (c) Let the selling price and profit be x to each other. Similarly, Laugh and Cry
andy respectively. Quantity of water in new mixture are opposite to each other.
Then, profit = selling price − cost price =7+ 7
73. (d) As, Zenith and Nadir are opposite
= 14 L
y = x − cost price …(i) to each other. Similarly, Genuine and
[∵ By adding 7 L more water]
Again, 3y = 2x − cost price …(ii) Phony are opposite to each other.
Milk = 28 L
From Eqs. (i) and (ii), we get 74. (d) The method and practice of teaching
New ratio of milk and water = 28 : 14
x − y = 2x − 3y ⇒ 2 y = x is called Padagogy. The study of Insects
= 2 :1
x 2 is known as Entomology.
∴ = 66. (c) Some females are doctors and
y 1 75. (b) As, capriciousness and Reliability are
vice-versa. Some doctors are patients
synonyms. Similarly, Tenacious and
Hence, required profit percentage and vice-versa. Some patients are
Practicality are synonyms.
Profit 1 females and vice-versa. Some doctors
= × 100 = × 100 can be patients and females. 76. (d) Except Andes, all others are
Cost price 2 −1 waterfalls.
= 100% 77. (b) Except We Indians all others are the
Female Doctor
62. (a) Let the required sum is P. books written by ‘Shobha De’.
R = 4%, Time period = 8 yr [Given] 78. (c) Except Silver all others are alloys.
and interest = P − 340 Patients 79. (b) Except Puppet all others are related to
PTR
Simple interest = media and news.
100
P × 8× 4 67. (d) Some detectives can be spies and 80. (d) Except Nanda Devi all others are
⇒ P − 340 = vice-versa. Some spies can be men and plateaus.
100
vice-versa. Some men can be detectives 81. (d) Answer figure (d) will complete the
⇒ 100P − 34000 = P × 32
and vice-versa. Some men can be pattern.
⇒ 68P = 34000
34000 detectives as well as spies. 82. (c) Answer figure (c) will complete the
∴ P=
68 pattern.
= ` 500 Spies 83. (b) Answer figure (b) will complete the
63. (c) Time = 3 yr, r = r1%, Detectives pattern.
Men 84. (c) Answer figure (c) will complete the
Second rate = (r1 + 4)%
pattern.
Interest = ` 600
P × 3 (r1 + 4) P × 3 × r1 85. (d) Answer figure (d) will complete the
∵ − = 600 68. (c) All fathers are males. All males are
pattern.
100 100 human being.
∵ SI = P × R × T  86. (c) The symbol ‘ ’ is rotating 135°
 100  anti-clockwise and 45° clockwise
3P (r1 + 4) − 3Pr1 Father alternatively, also ‘ ’. is inverted in
⇒ = 600 each step. The other element rotates
100
⇒ 3Pr1 + 12P − 3Pr1 = 60000 Males 135° anti-clockwise from figure 1 to 2,
from 3 to 4 and from 5 to answer figure.
⇒ 12P = 60000 Human Being
Also, the head of the element gets
∴ P = ` 5000
inverted.
64. (d) Given, Average of 50 students in a 69. (a) Steel and wood are different
87. (c) In each successive step the elements
class = 45 kg materials.
change their positions as follows
Total weight
Building
= Average × Number of students Material
= 45 × 50 = 2250 kg
= 2250000 g [∵ 1 kg = 1000 g] Steel Wood N
Now, one boy leaves the class.
So, new average = 449 . kg = 44900 g N = New element
84 AFCAT-II ~ Solved Paper 2015

88. (a) All the four elements change their 91. (b) The whole figure rotates 180° to top in figure (1) gets reversed in
positions as follows obtain the second figure. figure (2).
92. (a) The shaded petal rotates 135°
RS RS RS RS RS = Reverse clockwise and the unshaded petal rotates
135° anti-clockwise.
93. (c) From figure (1) to (2), all the
From figure (1) to From figure (2)
(2), (3) to (4), (5) to
elements shift one position downward 96. (c) Except figure (c) in all other figures
to (3), (4) to (5)
answer figure and the bottom element becomes the top the marked are equal.
most elements. Also, each element is
97. (a) Except figure (a) in all other figures,
89. (d) Smaller arrow is rotating 45° and 90° inverted laterally. In figure (1),
two shaded and two unshaded halves of
alternatively in anit-clockwise and the the heads of the first and last elements
the petals are attached to the centre of
larger arrow is rotating 135° clockwise in also get reversed in figure (2).
the figure.
each step. 94. (d) From figure (1) to (2), the four corner
98. (b) Except figure (b), in all other figures
90. (a) From figure (1) to (2), (3) to (4) and elements changes their positions as follows
the dots are placed at the middle of the
(5) to answer figure, the right column
N edges, while in figure (b) all the four dots
becomes left and vice-versa. Also, the top
are placed at the four comers of the
and bottom elements interchange their
square.
positions, the middle element remains as
it is. From figure (2) to (3) and (4) to N = New element 99. (d) Except figure (d) are other figures are
(5), in each column top and bottom same when rotated.
95. (c) The four elements change their
elements interchange their positions, also positions from figure (1) to (2) as 100.(b) Except figure (b) in all other figures,
the middle element changes to new follows. Also, the bottom element all the three unshaded leaves are placed
element. in figure (1), gets shaded in figure right angled to each other.
(2) and the third element from
AFCAT-I ~ Solved Paper 2015 85

INDIAN AIR FORCE


AFCAT-I

Solved Paper 2015


Time : 2 Hrs MM : 300
INSTRUCTIONS
■ The set contains a total of 100 questions, Comprising Verbal Ability in English, General Awareness, Numerical Ability and Reasoning and
Military Aptitude Test.
■ Each correct question carry 3 Marks and there will be negative marking of 1 Mark for each incorrect attempt.
■ Total time duration will be 2 hrs (120 minutes).
■ No marks will be deducted for unattempted questions.

Directions (Q.Nos.1-4) Which word or of the figures. Films could be viewed in 13. Intransigent
words explains the meaning of the the ……(8)...... of the home and a (a) Authoritative (b) Impersonal
following Idioms. variety of shows are also available. One (c) Strenuous (d) Unbending
1. In a jiffy of the advantages of travel programmes 14. Intimidate
(a) Outstanding (b) Suddenly is the ……(9)...... of faraway places (a) Mislead (b) Misplace
(c) In a fix (d) Appropriate which many viewers would not (c) Frighten (d) Demoralise
……(10)...... see.
2. Upto the hilt Directions (Q. Nos. 15-18) Choose the
(a) Completely 5. (a) interesting (b) popular word which is nearly opposite in
(b) Upto the mark (c) powerful (d) purposeful meaning to the given word.
(c) Upto the final decision 6. (a) could (b) would 15. Clemency
(d) None of these (c) might (d) shall (a) Corporal (b) Intolerance
3. Man of letters 7. (a) of (b) with (c) as (d) in (c) Compromise (d) Sensibility
(a) Who writes too many letters
8. (a) surroundings (b) assistance 16. Cajole
(b) An important person
(a) Nestle (b) Secede
(c) A politician (c) comfort (d) privilege
(c) Bully (d) Moisten
(d) A literary person 9. (a) glimpses (b) image
17. Malevolent
4. Sangfroid (c) portrait (d) picture
(a) Kindly (b) Vacuous
(a) Composure 10. (a) possible (b) rather (c) Ambivalent (d) Primitive
(b) Go on leave
(c) else (d) otherwise
(c) Changed suddenly 18. Purgatory
(d) Make an attempt Directions (Q.Nos. 11-14) Choose the (a) Reward (b) Celestial
word which is nearest in meaning to the (c) Flawless (d) Proximity
Directions (Q. Nos. 5-10) Select the given word.
most appropriate word from the options Directions (Q.Nos. 19-21) Read the
against each number. 11. Sporadic following passage carefully and answer
As home entertainment, television is (a) Epidemic (b) Whirling the questions given below it.
rapidly becoming more ……(5)...... than (c) Occasional (d) Stagnant In spring, polar bear mothers emerge
any other form. A news broadcast 12. Genesis from dens with three month old cubs.
becomes more immediate when people (a) Style (b) Beginning The mother bear has fasted for as long
……(6)...... actually see the scene (c) Movement (d) Relevant as eight months but that does not stop
……(7)...... question and the movement the young from demanding full access
86 AFCAT-I ~ Solved Paper 2015

to her remaining reserves. If there are Directions (Q.Nos. 26-29) Choose the 36. A man travelled from a point A to B
triplets, the most persistent stands to correctly spelt word. at the rate of 25 km/h and walked
gain an extra meal at the expense of 26. (a) Konnoisseur (b) Conoisseur back at the rate of 4 km/h. If the
others. The smallest of the cubs forfeits whole journey took 5 h 48 min, the
(c) Connoisseur (d) Konoisseur
many meals to stronger siblings. distance between A and B is
Females are protective of their cubs but 27. (a) Munifisent (b) Muneficent (a) 30 km (b) 24 km
tend to ignore family rivalry over food. (c) Munificent (d) Munificient (c) 20 km (d) 51.6 km
In 21 years of photographing polar 28. (a) Equanimity (b) Equannimity 37. A train travelling at a uniform
bears, I have only once seen the speed clears a platform 200 m long
(c) Equanimmisty (d) Equinimity
smallest of triplets survive till autumn. in 10 s and passes a telegraph post
29. (a) Vetarinary (b) Veteninary
19. With reference to the passage, the in 5 s. The speed of the train is
following assumptions have been (c) Veterinary (d) Vetniary (a) 36 km/h (b) 39 km/h
made. 30. A, B and C can do a piece of work in (c) 72 km/h (d) 78 km/h
I. Polar bears fast as long as eight 20 days, 30 days and 60 days, 38. The price of sugar increases by 20%
months due to non-availability respectively. In how many days can due to the festive season. By what
of prey. A do work, if he is assisted by B and percentage should a family reduce
II. Polar bears always give birth to C on every third day? the consumption of sugar, so that
triplets. (a) 12 (b) 15 (c) 16 (d) 18 there is no change in the
Which of the assumptions given 1 expenditure?
above is/are true? 31. A man can row 9 km/h in still 1 2 1
3 (a) 20 (b)18 (c)16 (d)16
(a) Only I water and finds that it takes him 3 3 3
(b) Only II
(c) Both I and II thrice as much time to row up than 39. A’s salary is 20% lower than B’s
(d) Neither I nor II salary, which is 15% lower than C’s
as to row down the same distance salary. By how much per cent is C ’s
20. Female polar bears give birth during in the river. The speed of the salary more than A’s salary ?
(a) Spring (b) Summer (a) 44.05 (b) 45.05
(c) Autumn (d) Winter current is
1 1 (c) 46.05 (d) 47.05
21. Mother bear (a) 3 km/h (b) 3 km/h
3 9 40. The average weight of 5 men is
(a) takes sides over cubs 2 1 increased by 2 kg when one of the
(b) lets the cubs fend for themselves (c) 4 km/h (d) 4 km/h men whose weight is 60 kg is
3 3
(c) feeds only their favourites replaced by a new man. The weight
(d) sees that all cubs get an equal 32. The speed of a boat in still water is of the new man is
share 10 km/h. If it can travel 26 km (a) 50 kg (b) 65 kg
downstream and 14 km upstream (c) 68 kg (d) 70 kg
Directions (Q.Nos. 22-25) Choose the in the same time, the speed of the
word that best defines the given stream is 41. A and B can do a piece of work in
phrases. (a) 2 km/h (b) 2.5 km/h 18 days, B and C can do it in 24
days, A and C can do it in 36 days.
22. A curtain lecture (c) 3 km/h (d) 4 km/h
In how many days B alone can
(a) To speak plainly 33. If a sum become double in 16 yr, then finish the work?
(b) Vulgar ideas how many times will it be in 8 yr? (a) 48 (b) 45
(c) Private scolding of a husband by 1 1 4
his wife (a)1 times (b)1 times (c) 28 (d) 144
2 3 5
(d) Hate others 3 1
(c)1 times (d)1 times 42. The opposition party status is
23. Square pegs in round holes 4 4 accorded to a political party in the
(a) A genuinely helpful person 34. In how many years will a sum of Lok Sabha only if it captures atleast
(b) A clever person ` 800 at 10% per annum (a) 5% seats (b) 10% seats
(c) People in the wrong jobs compounded semi-annually (c) 15% seats (d) 20% seats
(d) To be perplexed become ` 926.10? 43. Who amongst the following is the
24. In weal and woe (a)1
1
(b)1
1
author of the book ‘A Bend in the
(a) By hook or crook 3 2 River’?
(b) During illness 1 1 (a) Chetan Bhagat (b) VS Naipaul
(c) 2 (d) 2
(c) In prosperity and adversity 3 2 (c) Kiran Desai (d) Anita Desai
(d) During the operation
35. A sells 2 TV sets, one at a loss of 44. ‘Long Walk to Freedom’ is a book
25. Globetrotters 15% and another at a profit of 15%. written by
(a) People against global philosophy Find the loss/gain percentage in the (a) Sonia Gandhi
(b) People indulging in treachery overall transaction. (b) LK Advani
(c) Intelligent minds (a) 2.25% loss (b) 3% profit (c) Nelson Mandela
(d) Travellers around the world (c) 4% loss (d) No profit, no loss (d) Benazir Bhutto
AFCAT-I ~ Solved Paper 2015 87

45. Which sports personality has been 54. In which city is headquarters 62. Alauddin Khalji did not build
awarded the honorary rank of of Asian Development Bank (a) Siri Fort
Group Captain by the IAF? located? (b) Tomb of Jalaluddin
(a) Kapil Dev (a) Manila (b) Singapore (c) Hauz-i Alai
(b) Sania Mirza (c) Bangkok (d) Jakarta (d) Jamaat Khana Masjid
(c) Saina Nehwal 55. K-15 missile is 63. Which of the following dynasties
(d) Sachin Tendulkar was ruling over North India at the
(a) Sub-marine Launched Ballistic
46. Which country among the Missile (SLBM) time of Alexander’s invasion?
following has been declared (b) Inter-Continental Ballistic Missile (a) Nanda (b) Maurya
Ebola-free by WHO? (ICBM) (c) Sunga (d) Kanva
(a) Sierra Leone (b) Liberia (c) Medium Range Ballistic Missile 64. Which of the following Inter
(c) Nigeria (d) Guinea (MRBM) Continental Ballistic Missiles (ICBMs)
47. How many Gold medals did India (d) Short Range Ballistic Missile is under development in India?
win in the Incheon Asian Games (SRBM) (a) Agni-I
held in October 2014? 56. India agreed to UN Chief Ban (b) Agni-II
(a) 10 (b) 11 Ki-Moon’s offer to remain as a (c) Agni-IV
(c) 12 (d) 8 member of the advisory board of (d) Agni-VI
48. Who has been appointed as the one of the following recently 65. Who among the following was
new Finance Secretary of India? (a) UNCCT (b) UNICEF adjudged as the Most Valuable
(a) Arvind Mayaram (c) UNEP (d) UNCTAD Player of the 17th Asian Games
(b) Rajiv Mehrishi 57. Kaziranga National Park is famous held at Incheon, South Korea?
(c) Kaushik Basu for (a) Mary Kom of India
(d) Dinesh Gupta (a) One-horned Rhinos (b) Kosuke Hagino of Japan
49. Which among the following is (b) Tigers (c) Ning Zetao of China
India’s first long range subsonic (c) Swamp Dears (Barasingha) (d) None of the above
cruise missile? (d) Elephants
Directions (Q.Nos. 66-70) In each of
(a) Agni II (b) Prithvi 58. Who was the first Indian to win an the following question, find out the
(c) Dhanush (d) Nirbhay individual medal in Olympics? alternative which will replace the
50. The branch of Science that studies (a) Milkha Singh question mark(?).
cells is called (b) PT Usha
66. East : Orient :: ? : ?
(a) Cytology (b) Entomology (c) Karnam Malleshwari
(a) North : Polar
(c) Homoplastic (d) Hormonology (d) KD Jadhav
(b) North : Tropic
51. How many Vice-Presidents are 59. Who among the following was the (c) South : Capricorn
elected at the start of its each Congress President at Madras (d) West : Occident
regular session of UN General Session of 1927 when it boycotted
67. Ignominy : Disloyalty :: ? : ?
Assembly? the Simon Commission?
(a) Death : Victory
(a) Nine (b) Fifteen (a) Maulana Abul Kalam Azad
(b) Martyr : Man
(c) Two (d) Twenty one (b) MA Ansari
(c) Fame : Heroism
(c) Lala Lajpat Rai
52. The highest civilian award of India (d) Subhash Chandra Bose
(d) Destruction : Victory
‘Bharat Ratna’ has been awarded 68. Loath : Coercion :: ? : ?
to only two foreigners so far. One 60. Why did Kalinga prove to be
(a) Detest : Caressing
of them is Nelson Mandela. The a turning point in the life of
Ashoka? (b) lrritate : Caressing
other is (c) Irate : Antagonism
(a) Marshal Tito (a) Ashoka annexed Kalinga
(d) Reluctant : Persuasion
(b) Mikhail Gorbachev (b) It was the starting point of the
(c) Khan Abdul Ghaffar Khan expansion of his empire 69. Trilogy : Novel :: ? : ?
(d) Abdul Wali Khan (c) Ashoka became a zealous (a) Rice : Husk
Buddhist (b) Milk : Cream
53. Sir CV Raman was awarded Nobel (d) It enabled Mauryan Empire to (c) Serial : Episode
Prize for his work connected with reach its climax (d) Geen : Cartridge
which of the following
phenomenon of radiation? 61. Which of the following wings was 70. Wife : Marriage :: ? : ?
(a) Scattering not part of the espionage system (a) Bank : Money
(b) Diffraction described by Kautilya? (b) Nationality : Citizenship
(c) Interference (a) Crime Branch (b) Special Branch (c) Service : Qualification
(d) Polarisation (c) Political Branch (d) None of these (d) Attendance : Register
88 AFCAT-I ~ Solved Paper 2015

Directions (Q.Nos. 71-75) Each of Directions (Q.Nos. 76-80) In each of figures mark (a), (b), (c) and (d) called the
these question given below contains the following question, select a figure answer figures. Select a figure from
three elements. These elements may or from amongst the four alternatives, which amongst the answer figures which will
may not have some interlinkage. Each when placed in the blank space of prblem continue the same series as established by
group of elements may fit into one of figure (X) would complete the pattern. the five problem figures.
these diagrams at (a), (b), (c) or (d). 81. Problem Figures
You have to indicate the group of 76. Problem Figure
elements which correctly fits into the ?
diagrams

S
71. Which of the following diagrams (1) (2) (3) (4) (5)
indicates the best relation between (X) Answer Figures
Doctors, Human Beings and Answer Figures

S
Married People?

S
(a) (b) (a) (b) (c) (d)
(a) (b) (c) (d)
77. Problem Figure
82. Problem Figures
(c) (d)
?

72. Which of the following diagrams


(X) (1) (2) (3) (4) (5)
indicates the best relation between
Children, Naughty and Studious? Answer Figures Answer Figures

(a) (b) 78. Problem Figure


(a) (b) (c) (d)
(c) (d)
? 83. Problem Figures
(X)
73. Which of the following diagrams
Answer Figures
indicates the best relation between (1) (2) (3) (4) (5)
Thief, Criminal and Police?
Answer Figures
(a) (b) (c) (d)
(a) (b) C S C
79. Problems Figures
C
(c) (d) ? (a) (b) (c) (d)
84. Problem Figures
(X)
74. Which of the following diagrams
indicates the best relation between Answer Figures
Man, Worker and Garden?
(1) (2) (3) (4) (5)
(a) (b) (c) (d) Answer Figures
(a) (b)
80. Problems Figures
?
(c) (d)
(a) (b) (c) (d)
(X)
75. Which of the following diagrams 85. Problem Figures
Answer Figures
indicates the best relation
between Males, Cousins and
Nephews? (a) (b) (c) (d)
(1) (2) (3) (4) (5)
(a) (b) Directions (Q.Nos. 81-85) Each of the Answer Figures
following question consists of five figures
marked (1), (2), (3), (4) and (5) called
(c) (d) the problem figures followed by four other
(a) (b) (c) (d)
AFCAT-I ~ Solved Paper 2015 89

Directions (Q.Nos. 86-90) In each of 93. Answer Figures


the following question, four words have
been given out of which three are alike
in some manner, while the fourth one is (a) (b) (c) (d)
different. Choose the word which is (a) (b) (c) (d)
different from the rest. 94.
86. (a) Othello 98. Problem Figures
(b) King Lear
(a) (b) (c) (d)
(c) Oliver Twist ?
(d) Macbeth 95.
87. (a) Nimitz (1) (2) (3) (4)
(b) Yamamoto Answer Figures
(c) Nelson (a) (b) (c) (d)
(d) Montgomery
Directions (Q.Nos. 96-100) Each of
88. (a) Biaze the following questions consists of two
(b) Glint sets of figures. Figures (1) (2), (3) and (a) (b) (c) (d)
(c) Simmer (4) constitute the problem set while
(d) Shimmer
figures (a), (b), (c) and (d) constitute the 99. Problem Figures
89. (a) Aravalli hills answer figures. There is a definite
(b) Shivalik hill relationship between figures (1) and (2). ?
(c) Mole hills Establish a similar relationship
(d) Satpura hills between figures (3) and (4) by selecting (1) (2) (3) (4)
90. (a) Beaver a suitable figure from the answer
(b) Alpaca figures that would replace the question Answer Figures
(c) Walrus mark (?) in figure (4).
(d) Koala 96. Problem Figures
Directions (Q.Nos. 91-95) In each
problem, out of the four figures marked (a) (b) (c) (d)
?
(a), (b), (c) and (d) three are similar in a
100. Problem Figures
certain manner. However, one figure is (1) (2) (3) (4)
not like the other three. Choose the
figure which is different from the rest. Answer Figures ?
91.
(1) (2) (3) (4)

Answer Figures
(a) (b) (c) (d)
(a) (b) (c) (d)
92. 97. Problem Figures

? (a) (b) (c) (d)


(a) (b) (c) (d)
(1) (2) (3) (4)

Answers
1. (b) 2. (a) 3. (d) 4. (a) 5. (b) 6. (a) 7. (d) 8. (c) 9. (a) 10. (d)
11. (c) 12. (b) 13. (d) 14. (c) 15. (a) 16. (c) 17. (a) 18. (a) 19. (d) 20. (d)
21. (b) 22. (c) 23. (c) 24. (c) 25. (d) 26. (c) 27. (c) 28. (a) 29. (c) 30. (b)
31. (c) 32. (c) 33. (a) 34. (b) 35. (a) 36. (c) 37. (*) 38. (c) 39. (d) 40. (d)
41. (c) 42. (b) 43. (b) 44. (c) 45. (d) 46. (c) 47. (b) 48. (a) 49. (d) 50. (a)
51. (d) 52. (c) 53. (a) 54. (a) 55. (a) 56. (a) 57. (a) 58. (d) 59. (b) 60. (c)
61. (c) 62. (b) 63. (a) 64. (d) 65. (b) 66. (d) 67. (c) 68. (d) 69. (c) 70. (c)
71. (d) 72. (c) 73. (a) 74. (d) 75. (a) 76. (b) 77. (d) 78. (d) 79. (c) 80. (a)
81. (d) 82. (c) 83. (c) 84. (c) 85. (a) 86. (c) 87. (d) 88. (c) 89. (c) 90. (c)
91. (c) 92. (d) 93. (*) 94. (b) 95. (c) 96. (a) 97. (d) 98. (c) 99. (c) 100. (c)
90 AFCAT-I ~ Solved Paper 2015

Hints and Solutions


1. (b) Idiom ‘In a jiffy’ means ‘in a very short 18. (a) ‘Purgatory’ means ‘a place or state of 31. (c) Let speed of the current = x km/h
time or quickly’. So, option (b) ‘Suddenly’ being where souls exists temporarily to D D
Then, 3 × =
explains the meaning of the given idiom. admit their sins or received punishment.  28 + x   28 − x 
2. (a) Idiom ‘upto the hilt’ means So, ‘Reward’ is its correct antonym as it  3   3 
‘something that is done completely or means ‘something that is given in return
∵ time = distance 
for good behaviour, service or hard work. 
fully’. Hence, option (a) ‘Completely’ is its speed 
correct answer. 19. (d) With reference to the passage, none
⇒  28 + x  = 3 ×  28 − x 
of the given assumptions are true.
3. (d) Idiom ‘Man of letters’ means a writer,  3   3 
scholar, editor etc especially one whose 20. (d) Female polar bears give birth to cubs
⇒  28 + 3x  = 3 ×  28 − 3x 
work is in the field of literature. So, during winter season.
 3   3 
option (d) ‘A literary person’ is its correct 21. (b) Mother bear lets the cubs fend or look 28 + 3x
answer. after for themselves. ⇒ = (28 − 3x)
3
4. (a) ‘Sangfroid’ means ‘the ability to stay 22. (c) Phrase ‘A curtain lecture’ means ‘a ⇒ 28 + 3x = 84 − 9x
calm in a difficult or dangerous situation’. scolding or rebuke given in private ⇒ 12x = 56
So, ‘Composure’ is its correct answer as it especially by a wife to her husband’. So, 56
means ‘the state or feeling of being calm option (c) ‘private scolding of a husband ⇒ x=
and in control of oneself.’ 12
by his wife’ is its correct answer. 14 2
5. (b) popular 6. (a) could ⇒ x= = 4 km/h
23. (c) Phrase ‘Square peg in a round hole’ 3 3
7. (d) in 8. (c) comfort means ‘someone who does not fit in 32. (c) Let speed of the current = x km/h
9. (a) glimpses 10. (d) otherwise particular place or situation’. So, option
26 14
(c) ‘People in the wrong jobs’ is the Then, =
11. (c) ‘Sporadic’ means ‘occuring at irregular 10 + x 10 − x
intervals’, or happening sometimes. So, correct answer.
24. (c) ‘In weal and woe’ refers to the ∵ speed of the boat = 10km/h and
‘Occasional’ is nearest in meaning to the
given word as it also means ‘not ‘prosperity and misfortune or the joys and 
happening often or regularly’. sorrows’. Hence, option (c) ‘in prosperity distance 
Time =
12. (b) ‘Genesis’ means ‘the origin of and adversity’ is its correct answer. speed 
something, when it is begun or starts to 25. (d) ‘Globetrotter’ is a person who travels ⇒ 260 − 26x = 140 + 14x
exist’. So, ‘Beginning’ is nearest in to many different countries. Hence, ⇒ 40x = 120
meaning to the given word. option (d) ‘travellers around the world’ is 120
⇒ x= = 3 km/h
13. (d) ‘Intransigent’ means ‘unwilling or its correct answer. 40
refusing to change one views or to agree 26. (c) The correctly spelt word is ‘Connoisseur’ 33. (a) Let principal = ` P
about something’. So, ‘Unbending’ is which means ‘a person who knows a lot
Then, amount = ` 2P
nearest in meaning to the given word as about art, good food, music etc’.
Principal × Rate × Time
it also means ‘unwilling to change or 27. (c) The correctly spelt word is Then, SI =
100
having very strict beliefs and attitudes’. ‘Munificent’. ‘Munificent means
P × r × 16
14. (c) ‘Intimidate’ means ‘to frighten or ‘extremely generous’. ⇒ 2P − P =
100
threaten someone, usually in order to 28. (a) The correctly spelt word is P × r × 16
persuade them to do something’. Hence, ‘Equanimity’ and it means ‘calmness and ⇒ P=
‘Frighten’ is its correct answer. 100
composure especially in a difficult
100 25
15. (a) ‘Clemency’ means ‘kindness when situation’. ⇒ r= ⇒r = %
giving a punishment or an act of mercy’. 16 4
29. (c) The correctly spelt word is ‘Veterinary’
P× r ×t
So, ‘Corporal’ is its correct antonym as it and it means ‘of or relating to medical Again, SI =
means ‘punishment that involves treatment of sick or injured animals.’ 100
physically harming someone’. P × 25 × 8
30. (b) Pattern of A, B and C to do a work ⇒ x= [let SI = ` x]
16. (c) ‘Cajole’ means ‘to persuade someone 4 × 100
together
‘to do something by flattery or promises’. P
A, A, A + B + C, A, A, A + B + C ⇒ x=`
So, ‘Bully’ is its correct opposite meaning 2
word, as it means ‘someone’ who hurts (A + B + C)’s 3 days work
P
3 1 1 9 + 2 + 1 12 1 ∴ Required times = P + x = P +
or frightens others and often forcing them = + + = = = 2
to do something that they don’t want to do’. 20 30 60 60 60 5
3P
[∵ A is assisted by B and C every third day] =
17. (a) ‘Malevolent’ means ‘having or showing 2
a wish to do evil to others’. Hence, ∴(A + B + C)’s can do the whole work
1
1 =1 × P
‘Kindly’ is its correct antonym as it means = = 3 × 5 = 15 days 2
‘warm-hearted and of a sympathetic or 1
generous nature’. 3× 5
AFCAT-I ~ Solved Paper 2015 91

34. (b) Given, rate of interest = 10% annually 39. (d) Let C’s salary = ` 100 44. (c) ‘Long Walk to Freedom’ is an
C × (100 − 15) autobiography of former South African
= 5% half-yearly Then, B’s salary =
and time (t) = 2t half-yearly 100 President Nelson Mandela.’ First
100 × 85 published in 1994, the book profiles his
Now, compound amount = Principal = = ` 85
Time 100 early life, coming of age, education and
× 1 +
Rate 
 B × (100 − 20) 27 years in prison.
 100  and A’s salary =
2t
100 45. (d) The Indian Airforce (IAF) has
85 × 80
. = 800 1 +
5 
92610  = = ` 68 awarded the honorary rank of Group
 100 100 Captain to famous sports personality,
2t C−A

.
92610
=  21 ∴Required percentage = × 100 Sachin Tendulkar to honour his
800  20 A achievement in the sport. He is the first
100 − 68
9261  21
2t = × 100 = 47.05% sports person to be conferred a rank by
⇒ =  68 IAF and the first personality with no
8000  20 40. (d) Let the average weight of 5 men be aviation background to receive the
3 2t
⇒   =  
21 21 ‘x’. Then, honour.
 20  20 Total weight of 5 men 46. (c) At the time of the question asked,
⇒ 2t = 3 [∵ on comparing] = Average × Number of men Nigeria has been declared Ebola-free
3 1 = x × 5 = 5x country by World Health Organisation
⇒ t = = 1 yr
2 2 According to the question, (WHO) in 2014, after no new active
5x − 60 + New man' s weight cases were reported in the country.
35. (a) When an article is sold at the same =x+2
per cent of profit and loss then always 5 Nigeria was the first African country to be
becomes a loss. ⇒ New man’s weight declared Ebola-Virus free. As per the
(r)2 (15)2 225 = 5x + 10 − 5x + 60 recent report, the countries Sierra Leone,
∴Loss percentage = = = Liberia and Guinea also have been
100 100 100 ∴New man’s weight = 70 kg
declared as Ebola-free by WHO.
= 2. 25% Alternate Method
47. (b) Asian Games is a continental
36. (c) Let distance between A and B = x km Total increase weight = 5 × 2 kg = 10 kg
multi-sport event held every four years
Then,
x x
+ =5
48 ∴Weight of the new man = (60 + 10) among athletes from all over Asia.
25 4 60 = 70 kg Asian Games held in Incheon, South
∵ time = distance  1
41. (c) (A + B)’s 1 day work = Korea in 2014. India Won 57 medals at
 speed  18 2014. Incheon Asian Games including
4x + 25x 29 29x 29 (B + C)’s 1 day work =
1 11 gold, 10 silver and 36 bronze.
⇒ = ⇒ =
100 5 100 5 24
The 2018 Asian Games held in Jakarata
100 1
∴ x= = 20 km (C + A)’s 1 day work = and Palembang, Indonesia.
5 36
India won total of 70 medals in Asian
37. (*) Let length of train = x m 2 (A + B + C)’s 1 day work
1 1 1 Games 2018.
x + 200 x  distance  = + +
Then, = ∵ time = 18 24 36 48. (a) At the time of the question asked,
10 
5 speed  4+ 3+ 2 9 1 Arvind Mayaram has been appointed as
= = =
Speed of train is same in both cases, 72 72 8 the New Finance Secretary of India.
⇒ 2x = x + 200 ⇒ x = 200 m 1 1 1 He is an Indian Administrative Service
(A + B + C) ’s 1 day work = × =
Length of train 8 2 16 Officer of 1978 batch.The current
∴ Speed of the train =
Time ∴ B’s 1 day work = (A + B + C)’s 1 day Finance Secretary (As per 2018) of India
200 1 1 is Rajiv Kumar.
= m/s work − (C + A)’s 1 day work = −
5 16 36 49. (d) Nirbhay is India’s first long range
= 40 m/s 9− 4 5
= = sub-sonic cruise missile. It is designed
18 144 144 and developed in India by the Defence
= 40 × km/h
5 Hence, B alone can finish the work in Research and Development Organisation
144 4
= 144 km/h days or 28 days. (DRDO). The missile can be launched
5 5 from multiple platforms and is capable of
38. (c) Reduce consumption percentage
r 42. (b) For a political party to be accorded the carrying conventional and nuclear
= × 100 warheads.
(100 + r) opposition party status in the Lok Sabha, it
20 has to capture atleast 10% seats. 50. (a) The branch of science that studies
= × 100 cells is called Cytology. It deals with the
(100 + 20) 43. (b) The author of the book ‘A Bend in the
20 100 River’ is VS Naipaul. Published in 1979, structure, function, multiplication,
= × 100 = the novel is one of Naipaul’s best known pathology and life history of cells.
120 6
50 works and was widely praised. 51. (d) The United Nations General Assembly
=
3 VS Naipaul was a British novelist and consist of a President and twenty one
2 travel writer who won the Nobel Prize for Vice-President elected at the starts of its
= 16 %
3 literature in 2001. each regular session. The United Nations
92 AFCAT-I ~ Solved Paper 2015

General Assembly is one of the six 59. (b) Mukhtar Ahmed Ansari was the 70. (c) As a female becomes wife after
principle organs of United Nations, in Congress President at Madras Session of marriage same as we are eligible for
which all member nations have equal 1927. The session was a milestone in service after qualification with the same.
representation. the independence movement since in 71. (d) All the doctors and married people
52. (c) The Bharat Ratna is the highest this session, decision was taken to belong to the group of human beings.
boycott of Simon Commission.
civilian award of India. It was instituted While, some doctors can be married
in 1954. The award has been awarded 60. (c) Ashoka was the greatest emperor of people. Venn diagram of such relation is
to only two foreigners so far. One of them ancient India. He conquered Kalinga in as,
is Nelson Mandela and other is Khan 261 BC. The conquest of Kalinga
Abdul Ghaffar Khan. marked a turning point in Ashoka’s life
and reign. Horrified at the slaughter of Married
Doctors
53. (a) CV Raman was an Indian physicist. People
the defenders of Kalinga, Ashoka decided
He won the 1930 Nobel prize in Physics to renounced warfare. He started
for his work on the scattering of light and Human Beings
propagating dhamma and Buddhism.
for the discovery of Raman effect. He
was the first person in Asia to obtain 61. (c) Political Branch was the wing that
72. (c) Some children are naughty and
Nobel prize in science. He discovered was not the part of Espionage System
studious same as some naughty are
that when light traverses a transparent described by Kautilya. The system by
studious and children, while some
material, some of the deflected light Kautilya had to be used for two reasons
studious are children as well as naughty.
changes wavelength and amplitude. This in the Mauryan empire i.e. internal
Venn diagram of such relation is as,
phenomenon, is known as Raman policing and war. For this, he classified
Scattering. spies into two categories-fixed (samstha)
and wanderer (sancara) spies. Children Naughty
54. (a) The Asian Development Bank (ADB)
is a regional development bank 62. (b) Alauddin Khalji was the second and
established in 1966. It’s headquarters is the most powerful ruler of the Khalji
Studious
located in Manila, Philippines. The aim dynasty. He reigned from 1296 to 1316.
of the bank is to accelerate economic and The famous moments built by Alauddin
social development in Asia and pacific Khailji include Alia Darwaza, Siri Fort, 73. (a) Thief belong to the category of
region. Currently, ADB is composed of Hauz-i-Alai and Jamaat Khana Masjid. criminal, while police is a separate entity.
68 members, of which 49 are from Asia 63. (a) At the time of Alexander’s invasion Venn diagram of such relation is as,
and Pacific region. during 327-325 BCE, Nanda dynasty
55. (a) K-15 missile, also known by the was ruling over North India. The Nanda
Thief
dynasty ruled in Northern part of the Police
name Sagarika, is a nuclear-capable
Sub-marine Launched Ballistic Missile Indian subcontinent during the 4th Criminal
(SBLM). Its range is 750 kilometers. It century BCE and possibly during the 5th
belongs to the K missile family and forms century BCE. 74. (d) Some man can be worker, while
a part of India’s nuclear thiad. 64. (d) Agni-VI is an intercontinental ballistic garden is a separate entity. Venn
56. (a) India agreed to UN Chief Ban-ki missile being developed by Defence diagram of such relation is as,
Moon’s offer to remain as a member of Research Development Organisation
the advisory board of the United Nations (DRDO) of India. Agni VI will be a four Man Worker
counter Terrorism centre (UNCCT) for an stage missile developed for the strategic
additional three years. The UNCCT was force command of Indian Armed Forces.
established in September 2011 to 65. (b) Japanese swimmer Kosuke Hagino Garden
promote international counter-terrorism was adjudged as the Most Valuable
cooperation and support Member States Player (MVP) of the 17th Asian Games
held at Incheon, South Korea. He is a 75. (a) All nephews are male, while some
in the implementation of the Global cousins can be are male and Nephew
Counter terrorism strategy. four time Olympic medalist and won
medals in all seven events he competed also. Venn diagram of such relation is as,
57. (a) Kaziranga National Park is a at 17th Asian Games.
protected area and a world Heritage site
located in the Golaghat, Karbi Anglong 66. (d) The orient is a historical term of the
Nephew
East, traditionally comprising anything
and Nagaon districts of the state of Cousins
related to Eastern world. Similarly,
Assam. The park is famous for Males
occidental may refers to Western world.
one-horned Rhinos. Kaziranga is also a
home to the highest density of tigers 67. (c) Ignominy is the result of
76. (b) Answer figure (b) will complete the
among protected areas in the world. Disloyalty. Similarly, Fame is the result of
pattern.
Heroism.
58. (d) KD Jahdav was an Indian athlete. He 77. (d) Answer figure (d) will complete the
is best known as a wrestler. He won a 68. (d) Loath and Reluctant are synonyms.
Similarly, Coercion and Persuasion are pattern.
Bronze medal at the 1952 Summer
Olympics in Helsinki. He was the first synonyms. 78. (d) Answer figure (d) will complete the
Indian to win an individual medal in the 69. (c) As, novel is the part of trilogy same as pattern.
Olympics. episode is the part of serial.
AFCAT-I ~ Solved Paper 2015 93

79. (c) Answer figure (c) will complete the position. Also, all of the symbols a get other. Or, except figure (a) in all other
pattern. inverted vertically at their place. figures, half arrow is inside the square
80. (a) Answer figure (a) will complete the 85. (a) In each successive figure the dot (.) and half is outside.
pattern. is moving from one corner to other in 94. (b) Except figure (b), in all other figures
81. (d) For , the figure is rotating 45° anti-clockwise direction and an arrow the sequence of the elements is
clockwise in each step. After rotating, in head from slanting arrows is removed in , either
1st step ‘∩’ is inverted and the other a set pattern. After removing all the moving from top to bottom or from
element changes to new element, in 2nd arrow heads from slanting arrows, new bottom to top.
step, both the end element interchange arrow head will get appear. 95. (c) Except figure (c) in all other figures,
their positions, in 3rd step new elements 86. (c) Othello, King Lear and Macbeth are either two ‘T’ or two ‘↑’ are parallel to
appear at both ends. This process repeats the leading characters of story of William each other.
from next figure. Now, for , the figure Shakespear. While, Oliver Twist was not 96. (a) The elements change their positions
X

is rotating 45° clockwise in each step. the leading character of the story. as follows,
After rotating, in Ist step both the elements 87. (d) All except Montgomery were
interchange their positions, in 2nd step admirals. Nimitz was U.S Admiral, N D
new elements appear at both ends. This Yamamoto was Japanese Admiral and
process repeats from next figure. Nelson was British Admiral, while N D
82. (c) In each step, the whole figure is Montgomery was British, Field Marshal. N = New element
rotating 45° anti-clockwise and a new 88. (c) All except simmer are related to the D = Old element deleted
line segment with a new symbol at the flame of fire while simmer is to cook 97. (d) Whole figure rotates 90°
top is added to the figure in front of the something by keeping it almost boiling anti-clockwise and the inner element
figure. point. goes outside and the outer element goes
83. (c) In each successive figure, the 89. (c) Except mole hills all others are inside.
elements are moving 135° and 90° located in India. 98. (c) The whole figure is rotating 180° and
anti-clockwise alternatively. Also, a new 90. (c) All except walrus are for bearing a small line segment also a small circle is
symbol is added alternatively in front and animals. increasing.
in the end.
91. (c) Except figure (c), all other figures are 99. (c) The upper half of the figure is rotating
84. (c) From figure (1) to (2), (3) to (4), and same when rotated. 180° at its position.
(5) to answer figure, the left most
elements and the right most elements 92. (d) Except figure (d), all other figures are 100.(c) The figure gets laterally inverses and
interchange their positions and the same when rotated. its water image appears below it.
middle element remains at the same 93. (*) Except figure (d), in all other figures
both arrow and dot are opposite to each
AFCAT ~ Practice Set 1 03

AFCAT
AIR FORCE COMMON ADMISSION TEST (ONLINE )

Practice Set ~ 01
Time : 2 Hrs MM : 300

INSTRUCTIONS
1. The set contains a total of 100 questions, Comprising Verbal Ability in English, General Awareness, Numerical Ability and Reasoning and
Military Aptitude Test.
2. Each correct question carry 3 Marks and there will be negative marking of 1 Mark for each incorrect attempt.
3. Total time duration will be 2 hrs (120 minutes).
4. No marks will be deducted for unattempted questions.

Directions (Q. Nos. 1-4) In the 4. I .............. his contribution to Directions (Q. Nos. 10-13) In the
following questions, sentences are given my book ............ it was following questions, four alternatives
with blanks to be filled in with substantial. are given for the idiom/phrase.
appropriate words. Out of the four (a) admitted, although Choose the correct alternative that
alternatives suggested choose (b) confessed to, but expresses the meaning of given
the correct alternative as your (c) debunked, and idiom/phrase.
answer. (d) acknowledged, because 10. Not my cup of tea
1. Mr. Naimi, the chief secretary was Directions (Q. Nos. 5-9) Find out (a) a refreshing drink
unceremoniously ...... after party’s which part of the sentence has an error. (b) a routine work
........ defeat in general election. If there is no error select option (d) ‘No (c) not what I like
(a) exiled, predictable error’ as your answer. (d) not liked by me
(b) ousted, disastrous
5. I am told (a)/John is ill (b)/ since 11. To have second thoughts
(c) recommended, unexpected
Monday. (c)/No error (d) (a) to change decision
(d) honoured, stupendous
(b) to plan carefully
6. I shall leave school (a)/as soon
2. He reads ........ that are ........ to (c) to take someone
as (b)/ I shall find a job. (c)/ No
his profession. (d) to reconsider
error (d)
(a) weeklies, appropriate
7. He is a rich man (a)/ he is owning 12. To eat a humble pie
(b) papers, apparent
(b)/ a number of buildings in (a) to feel downtrodden
(c) periodicals, pertinent
(d) magazines, acceptable Bombay. (c)/No error (d) (b) to accept defeat
(c) to be humiliated
3. The press is supposed to be a 8. We were able to make it (a)/ in
(d) to accept abuse
means of ............ between spite of (b)/ a bad weather. (c)/
the government and ....... . No error (d) 13. To break the ice
(a) communication, people 9. Neither of the two letters of (a) to start doubting
(b) help, society credit (a)/ were useful (b)/ (b) to start a quarrel
(c) confrontation, terrorist when he needed it. (c)/ No (c) to start a conversation
(d) propaganda, private sector error (d) (d) to break a friendship
04 AFCAT ~ Practice Set 1

Directions (Q. Nos. 14-16) Read the 21. COLOSSAL 30. Which plateau is known as the
following passage carefully and answer (a) Fragile (b) Small ‘Mineral Heart Land of India’?
the questions given below it. (c) Colourful (d) Impressive (a) Bhander Plateau
In a free country, the man who reaches (b) Chota Nagpur Plateau
22. INDISPENSABLE (c) Deccan Plateau
the position of leader is usually one of (a) Tolerable (b) Superfluous
outstanding character and ability. (d) Tibetan Plateau
(c) Expensive (d) Hostile
Moreover, it is usually possible to foresee 31. Which of the following is not
that he will reach such a position, since Directions (Q. Nos. 23-25) In the matched correctly?
early in life one can see his qualities of following passage, some words have (a) Pankaj Advani : Billiards
character. But this is not always true in been omitted. For each blank, four (b) Anjali Bhagwat : Shooting
the case of a dictator, often he reaches alternatives are provided. Select the (c) Muhammed Anas Yahiya : Boxing
his position of power through chance, suitable alternative to fill the given (d) Deepa Malik : Athletics
very often through the unhappy state blanks.
of his country. It is possible that 32. The propagation of sound waves in
When I go into stranger’s library I a gas involves
Hitler would never have become the
...(23)... round the bookshelves. This (a) adiabatic compression and
leader of Germany if that country had
is to know the type of person he is rarefaction
not been driven to despair.
and I feel that I know the (key) to his (b) isothermal compression and
14. In a free country, one who mind. rarefaction
becomes a leader A house without books is a ...(24)... (c) isochoric compression and
(a) must be of outstanding character house, no matter how rich the carpets rarefaction
and ability are. These only tell you whether he (d) isobaric compression and rarefaction
(b) must show qualities of character (has) a lot of money, but the books 33. Which one of the following is
from an early age
tell you whether he has a mind as radioactive?
(c) is generally of a remarkable well. It is not a ...(25)... of money that (a) Cesium (b) Platinum
character and ability
we do not buy books. (c) Strontium (d) Thorium
(d) must see that his country is free
from despair 23. (a) wonder (b) run 34. Free-throw is given in which sport?
(c) look (d) wander (a) Volleyball (b) Basketball
15. Hitler became a leader because
24. (a) characterless (b) bleak (c) Badminton (d) Cricket
(a) he exhibited leadership qualities
(b) Germany was a free country (c) unlucky (d) bad 35. Which space agency has developed
(c) Germany was to despair 25. (a) reason (b) question an ultraviolet telescope to study
(d) Germans wanted a dictator (c) issue (d) cause short flashes in Earth’s atmosphere?
(a) NASA (b) Russian
16. The passage seems to suggest that 26. Which one of the following was
(c) ISRO (d) JAXA
(a) despair sometimes leads to the last Buddhist text produced in
dictatorship India? 36. ‘Subroto Cup’ is associated with
(b) Hitler was no leader (a) Divya Vandana which game/sports?
(c) a leader is chosen only by a free (b) Dohakosa (a) Hockey (b) Football
country (c) Vajrachedika (c) Basketball (d) Badminton
(d) a leader foresees his future position (d) Vamsathapakasini 37. Who among the following is
Directions (Q. Nos. 17-19) Choose 27. The sultans of which dynasty known as ‘Metro Man of India’?
the word similar in meaning to the ruled for the longest time? (a) PV Chandran
given word. (a) Khilji Dynasty (b) E. Sreedharan
(b) Tughlaq Dynasty (c) Tessy Thomas
17. VORACIOUS
(c) Slave Dynasty (d) Man Mohan Sharma
(a) Very bad (b) Insatiable
(c) Stingy (d) Malicious (d) Lodhi Dynasty 38. Green Biotechnology is applied to
28. Who among the following (a) agricultural process
18. STRICTURE (b) medical sciences
(a) Strictness (b) Stinging introduced Zamindari System of (c) sea resources
(c) Discipline (d) Censure
India? (d) industrial process
(a) Lord Cornwallis
19. OBEISANCE (b) Thomas Munro
39. What is the name of Indigenous
(a) Homage (b) Pilgrimage Aircraft Carrier (IAC) will be
(c) Warren Hasting
(c) Subjugation (d) Obligation delivered to the Indian Navy in
(d) Lord Macaulay
2021 for advanced trials?
Directions (Q. Nos. 20-22) Choose 29. Which of the following rocks are (a) Rafael (b) Vikrant
the word opposite in meaning to the known as ‘Primary Rocks’? (c) Tejas (d) MG-21
given word. (a) Igneous Rocks
40. Which article deals with the
20. PERTINENT (b) Metamorphic Rocks establishment of Panchayati Raj?
(a) Eloquent (b) Distant (c) Sedimentary Rocks (a) Article 39 (b) Article 40
(c) Relevant (d) Irrelevant (d) None of the above (c) Article 41 (d) Article 42
AFCAT ~ Practice Set 1 05

41. Which was the first indigenously 52. If A and B together do a job in 60. Two numbers are respectively
built missile boat of India? 7.5 days and if A could do the job 10% and 25% more than a third
(a) INS Vinash (b) INS Khukri in 10 days if he worked alone. number. What per cent is the first
(c) INS Vibhuti (d) INS Ajay How many days would B take to of the second?
do the job if he worked alone? (a) 75 (b) 95 (c) 68 (d) 88
42. Which among the following is not
(a) 30 days (b) 40 days 61. ` 800 becomes ` 956 in 3 yr at a
a subsidiary of RBI?
(c) 25 days (d) 50 days certain rate of simple interest. If
(a) DICGC 5
(b) IFTAS 53. If p = , then the value of the rate of interest is increased by
8 4%, what amount will ` 800
(c) Bhartiya Reserve Bank Note 1 9 1
Mudran Pvt Ltd 27 p3 − − p2 + p is become in 3 yr?
216 2 4
(d) SIDBI 4 5 (a) ` 1020.80 (b) ` 1025
(a) (b)
27 27 (c) ` 1052 (d) ` 1050
43. How many e-cars have launched
8 10
in the Indian Army’s pilot project (c) (d) 62. A jeep is chasing a car which is
to combat air pollution? 27 27 5 km ahead. Their respective
(a) 12 (b) 10 54. A train is running at 36 km/h. If speeds are 90 km/h and 75 km/h.
(c) 8 (d) 4 it crosses a pole in 25s, its After how many minutes will the
length is jeep catch the car?
44. Which among the following pairs
(a) 248 m (b) 250 m (a) 18 (b) 20 (c) 24 (d) 25
of gas are the major contributors
to smog and acid rain? (c) 255 m (d) 260 m 63. Alcohol and water in two vessels
(a) NO2 , SO2 (b) SO2 , CO2 55. Average marks of 40 students is A and B are in the ratio 5 : 3 and
(c) CO, N 2 O (d) SO2 , CO 34. A number is entered as 62 5 : 4, respectively. In what ratio,
instead of 26. So, now what is the the liquids in both the vessels be
45. Which is a type of Electrically mixed to obtain a new mixture in
Erasable Programmable Read new average after correction?
(a) 33.1 (b) 34.1 vessel C in the ratio 7 : 5 ?
Only Memory (EEPROM)?
(c) 35.1 (d) 36.1 (a) 2 : 3 (b) 3 : 2
(a) Flash (b) Flange (c) 3 : 5 (d) 2 : 5
(c) Fury (d) FRAM 56. In one litre of mixture of alcohol
and water, water is 30%. The 64. How much per cent above the
46. Earth Hour is organised by cost price should a shopkeeper
amount of alcohol that must be
(a) UNESCO mark his goods so as to earn a
added to the mixture, so that the
(b) Earth Hour Foundation profit of 32% after allowing a
part of water in the mixture
(c) World Wildlife Fund discount of 12% on the marked
becomes 15%, is
(d) UNs Environment Programme
(a) 1000 mL (b) 700 mL price?
47. Who has been appointed the (c) 300 mL (d) 900 mL (a) 50% (b) 40% (c) 60% (d) 45%
first-ever Chief Financial Officer 65. A reduction of 20% in the price of
57. In the following, find the value of
(CFO) of the RBI? sugar enables Mr Bhadra to buy a
x.
(a) Ashoke Sen
45 − [28 − {37 − 15 + x}] = 58 extra 5 kg of it for ` 320. The
(b) Sudha Balakrishnan original rate of sugar is
(a) 18 (b) 19
(c) Jayant Narlikar (a) ` 12 per kg (b) ` 15 per kg
(c) 20 (d) 21
(d) Raghuram Rajan (c) ` 16 per kg (d) ` 20 per kg
48. Jnanpith Award is conferred to 58. The value of
2 3 66. The difference between simple
those in the field of × is and compound interest
1 5 3 1
(a) Literature (b) History 1+ of ÷ 1 compounded annually, on a
1 6 2 4
(c) Drama (d) Dance 1− certain sum of money for 2 yr at
2 4% per annum is ` 1. The sum
49. The first Indian Satellite, Aryabhatta (a) 6 (b) 8 (in `) is
was launched in the year (c) 4 (d) 2 (a) 650 (b) 630 (c) 625 (d) 640
(a) 1972 (b) 1975
(c) 1976 (d) 1979 59. The sum of numerator and 67. The greatest number of four digits
denominator of a fraction is 13. If which when divided by 3, 5, 7 and
50. The SAARC Secretariat is located 1 is added to the numerator and
at 9 leaves remainders 1, 3, 5 and 7,
2 is subtracted from the respectively is
(a) New Delhi (b) Columbo 1
denominator, the fraction is . (a) 9763 (b) 9764
(c) Kathmandu (d) Karachi 2
(c) 9766 (d) 9765
51. What least number must be The value of the original fraction
subtracted from 518, so that is 68. The perimeter of a semi-circular
the sum is completely divisible 7 3 path is 36 m. Find the area of this
(a) (b)
by 13? 12 10 semi-circular path.
(a) 11 (b) 10 13 (a) 42 sq m (b) 54 sq m
(c) 3 (d)
(c) 9 (d) 12 50 (c) 63 sq m (d) 77 sq m
06 AFCAT ~ Practice Set 1

Directions (Q. Nos. 69-71) In the 80. Which of the following is a leap 84. Which one of the following
following questions choose the term year? diagram best depicts the
which is different from others. (a) 2800 relationship among Nose, Hand,
69. (a) Sky (b) Star (b) 1800 Body?
(c) Planet (d) Comet (c) 2600
(d) 3000
70. (a) Rigveda (b) Yajurveda
(c) Atharvaveda (d) Ayurveda 81. Point A is 4 km to the West of (a) (b) (c) (d)
point C. Point C is 2 km to the
71. (a) Teeth (b) Tongue
North of point D. Point B is 2 km 85. Which of the following options
(c) Palate (d) Chin
to the South of point A. What is indicates the relation among
Directions (Q. Nos. 72-74) In the the distance between point B and Circle, Square and Triangle?
following questions, select the pair that point D?
has the same relation as the original (a) 2 km (b) 6 km
(or given) pair of words. (c) 4 km (d) 8 km
72. Dearth : Surplus 82. Two statements followed by two
(a) Simple : Complicated conclusions are given. Read the
(b) True : Unbelievable conclusions and then decide (a) (b)
(c) Touch : Repulsion which of the conclusions, if any,
(d) Dream : Fantasy logically follows from the two
given statements, disregarding the
73. Disobedience : Punishment known facts.
(a) Teenager : Dynamic Statements
(b) Prayer : Salvation All fathers are sons. (c) (d)
(c) Bravery : Appreciation No sons are educated.
(d) Patience : Listening Conclusions Directions (Q. Nos. 86-88) In the
I. All sons are educated. following questions, choose the figure
74. Lotus Temple : Delhi which is different from others.
II. No fathers are educated.
(a) Jama Masjid : Patna
(b) Hawa Mahal : Kolkata
(a) Only Conclusion I follows 86.
(b) Only Conclusion II follows
(c) Char Minar : Hyderabad
(c) Neither I nor II follows
(d) Amarnath Cave : Ahmedabad
(d) Both Conclusions I and II follow (a) (b) (c) (d)
Directions (Q. Nos. 75-77) What 83. A statement followed by two
comes in place of question mark (?) in conclusions is given. You have to
87.
the series given below? take the given statement to be
75. CEG, IKM, OQS, ? true even, if it seems to be at
(a) (b) (c) (d)
(a) VXZ (b) TVX variance from commonly known
(c) TUV (d) UWY facts. Read the conclusions and 88.
then decide which of the
76. A, R, C, S, E, T, G, ?, ? conclusions, if any, logically
(a) X, Z (b) U, I follows from the given statement,
(c) W, Y (d) V, B disregarding the known facts. (a) (b) (c) (d)

77. AEI, BFJ, CGK, ? Statement Women’s


organisations in India have Directions (Q. Nos. 89-90) In the
(a) DHL (b) DLH following questions, one or more dots are
welcomed the amendment of the
(c) EIM (d) LPT Industrial Employment Rules placed in the figure marked as (A). This
78. In a certain code, the words 1946 to curb sexual harassment figure is followed by four alternatives
‘COME AT ONCE’ were written at the work place. marked as (a), (b), (c) and (d). One out of
as XLNV ZG LMXV. In the same Conclusions these four options contains region(s)
code which of the following I. Sexual harassment of women at common to circle, square, triangle,
would be ‘OK’? work place is more prevalent in similar to that marked by the dot in
India as compared to other figure (A). Choose the correct figure.
(a) LM (b) LP (c) KM (d) KL
developed countries.
79. If ‘orange’ is called ‘blue’, ‘blue’ is II. Many organisations in India
89.
called ‘white’, ‘white’ is called will stop recruiting women to
‘yellow’, ‘yellow’ is called ‘black’, avoid such problems. (A) (a) (b) (c) (d)
‘black’ is called ‘red’ and ‘red’ is (a) Only Conclusion I follows
called ‘brown’, then what is the (b) Only Conclusion II follows 90.
colour of milk? (c) Neither Conclusion I nor II
(a) Orange (b) Blue follows (A) (a) (b) (c) (d)
(c) White (d) Yellow (d) Both Conclusions I and II follow
AFCAT ~ Practice Set 1 07
Directions (Q. Nos. 91-93) In each of Directions (Q. Nos. 94-96) In each 96. Problem Figure
the following questions, choose the answer of the following questions, a part
figure which is embedded in the of problem figure is missing. Find ?
problem figure. out from the given answer figures
91. Problem Figure (a), (b), (c) and (d), that can
replace the ‘?’ to complete the problem
figure.
94. Problem Figure Answer figures

Answer Figures

(a) (b) (c) (d)


?
(a) (b) (c) (d) Directions (Q. Nos. 97-100) Choose
the correct code for the uncoded term.
Answer Figures
92. Problem Figure 97.

EX FY GZ FX ?
(a) (b) (c) (d) (a) EZ (b) FZ (c) GX (d) FX
Answer Figures 98.
95. Problem Figure

? PS WM PR DM ?
(a) (b) (c) (d) (a) PS (b) WS (c) WM (d) DR

93. Problem Figure 99.

Answer Figures
VE VK TF TK ?
(a) VF (b) TK (c) VK (d) OF
Answer Figures 100.
(a) (b) (c) (d)
ZS AM TR ZM ?
(a) TS (b) ZR (c) TM (d) ZS
(a) (b) (c) (d)

Answers
1 (b) 2 (c) 3 (a) 4 (d) 5 (b) 6 (c) 7 (b) 8 (d) 9 (b) 10 (c)
11 (a) 12 (d) 13 (c) 14 (c) 15 (c) 16 (a) 17 (b) 18 (d) 19 (a) 20 (d)
21 (b) 22 (b) 23 (c) 24 (d) 25 (b) 26 (d) 27 (b) 28 (a) 29 (a) 30 (b)
31 (c) 32 (a) 33 (d) 34 (b) 35 (b) 36 (b) 37 (b) 38 (a) 39 (b) 40 (b)
41 (c) 42 (d) 43 (b) 44 (a) 45 (a) 46 (c) 47 (b) 48 (a) 49 (b) 50 (c)
51 (a) 52 (a) 53 (c) 54 (b) 55 (a) 56 (a) 57 (b) 58 (d) 59 (b) 60 (a)
61 (c) 62 (b) 63 (a) 64 (a) 65 (c) 66 (c) 67 (a) 68 (d) 69 (a) 70 (d)
71 (d) 72 (a) 73 (b) 74 (c) 75 (d) 76 (b) 77 (a) 78 (b) 79 (d) 80 (a)
81 (c) 82 (b) 83 (c) 84 (b) 85 (d) 86 (d) 87 (c) 88 (d) 89 (a) 90 (a)
91 (c) 92 (d) 93 (c) 94 (d) 95 (a) 96 (a) 97 (b) 98 (b) 99 (a) 100 (a)
08 AFCAT ~ Practice Set 1

Hints and Solutions


1. (b) The suitable pair of words to be filled show qualities at an early age. The word 29. (a) Igneous rocks are known as ‘primary
in the blanks are ‘ousted’ and ‘must’ is nowhere used in the passage rocks’. These rocks are referred to as
‘disastrous’. ‘Ousted’ means ‘to force or in the sense as is given in the other primary rocks as they begin the rock cycle
somebody out of job or position of three options. and represent the rocks which directly or
powers’ and ‘disastrous’ means 15. (c) The last two lines of the passage indirectly provided materials for the
‘unsuccessful or devastating’. make it clear that Hitler became the formation of other types of rocks.
2. (c) The suitable choice for the given leader of Germany because of the 30. (b) Chota Nagpur Plateau is known as the
blanks is option (c) i.e. ‘periodicals’ and atmosphere of despair, prevalent in that ‘Mineral Heart Land of India’. The plateau
‘pertinent’. ‘Periodicals’ means ‘a country. is a store house of minerals like mica,
magazine or newspaper published at 16. (a) Among the given options, option (a) bauxite, copper, limestone, iron ore and
regular intervals’ and ‘pertinent’ means appears to be the appropriate as the coal. The Damodar Valley in the plateau
‘relevant to a particular subject’. other options find no consonance in the is rich in coal and it is considered as the
3. (a) In the context of the sentence, option context of the passage. Moreover, the prime centre of coking coal in the country.
(a) i.e. ‘communication’ and ‘people’ are answer can be traced from the lines “in 31. (c) Muhammed Anas Yahiya from Kerala
suitable words to be filled in the given the case of a dictator, often he reaches is an Indian sprinter who specialises in
blanks. his position of power through chance, the 400 metres distance. He holds the
4. (d) The suitable pair of words to be filled very often through the unhappy state of national record in 400 meters.
in the blanks are ‘acknowledged’ and his country”. 32. (a) The propagation of sound waves in a
‘because’. ‘Acknowledged’ means ‘to 17. (b) ‘Voracious’ and ‘Insatiable’ have the gas involves the adiabatic compression
accept, admit or recognise something’. same meaning which means and rarefaction. An adiabatic process is
5. (b) Use ‘John has been ill’ in place of ‘exceedingly fond of eating’ or ‘avid’. one that occurs without transfer of heat or
‘John is ill’ to make the syntax correct as matter between a thermodynamic system
18. (d) ‘Stricture’ and ‘Censure’ have the same
there is a time reference. and its surroundings and rarefaction is the
meaning which means ‘a remark’ or
reduction of an item’s density.
6. (c) Remove ‘shall’ from the part of the ‘comment’, especially an adverse criticism.
sentence to make it correct. The 33. (d) Thorium is a naturally occurring
19. (a) ‘Obeisance’ and ‘Homage’ are
following syntax should be used: radioactive element with the symbol (Th)
synonyms as they have same meaning
Sub+shall/will+…+sub+v1 / v 5… and atomic number 90. It was discovered
i.e. ‘to express deep respect’.
in 1828 by the Swedish chemist Jons
7. (b) ‘He is owning’ is to be replaced by 20. (d) ‘Pertinent means ‘to the point’ or Jacob Berzelius. It was named after Thor,
‘He owns’. ‘Own’ is not used in ‘relevant’. Hence, ‘irrelevant’ is its correct the Norse God of Thunder.
progressive form. antonym.
34. (b) In basketball, free throws or foul shots
8. (d) No error 21. (b) ‘Colossal’ means ‘gigantic’ or ‘huge’. are unopposed attempts to score points
9. (b) ‘Neither’ is followed by singular verb. Hence, ‘small’ is its correct antonym. from a restricted area on the court. These
So, use ‘was’ in place of ‘were’. 22. (b) ‘Indispensable’ means ‘necessary’. are generally awarded after a foul on the
10. (c) ‘Not my cup of tea’ means not what ‘Superfluous’ means more than is needed. shooter by the opposing team. Each
somebody likes or is interested in. So, ‘no Hence, ‘superfluous’ is its correct antonym. successful free-throw is worth one point.
what I like’ is its correct answer. 23. (c) look 35. (b) The Russian Space Agency has
11. (a) ‘To have second thoughts’ means to 24. (d) bad developed an Ultra-violet telescope along
change your opinion after thinking about with international participation to study
25. (b) question
something again. Hence, ‘to change short flashes in Earth’s atmosphere. The
decision’ is its correct answer. 26. (d) Vamsathapakasini was the last ultra-violet telescope is placed on the
Buddhist text produced in India. The text International Space Station.
12. (d) ‘To eat a humble pie’ means to say
gives us information about the origin of
and show that you are sorry for a 36. (b) Subroto Cup is associated with
mistake that you made. Hence, ‘to the Mauryas. Football. This is a prestigious international
accept abuse’ is its correct answer. 27. (b) The sultans of Tughlaq dynasty ruled inter-school football tournament in India.
13. (c) ‘To break the ice’ means to do or say for the longest time i.e. between It is named after the Indian Air Force
something to relieve tension or get 1320-1414. Muhammad Bin Tughlaq Air Marshal Subroto Mukherjee. The first
conversation going in a strained and Firozshah Tughlaq were the tournament was held in 1960, with
situation. So, option (c) ‘to start a prominent rulers of this dynasty. participation of about 50 school teams.
conversation’ is its correct answer. 28. (a) Zamindari System was introduced by 37. (b) E. Sreedharan is a retired Indian
14. (c) Refer to the starting lines of the Lord Cornwallis in 1793 through Engineering Service (IES) Officer who is
passage – In a free country, the man Permanent Settlement Act. It was popularly known as ‘Metro Man of India’.
introduced in provinces of Bengal, Bihar, He is largely credited for changing the face
who reaches the position of leader is
Odisha and Varanasi. The system was of public transport in India by his
usually one of outstanding character and
leadership in building Konkan Railway and
ability. It is not stated that he must be of also known as Permanent Settlement
Delhi Metro.
outstanding character and ability or must System.
AFCAT ~ Practice Set 1 09

38. (a) Green Biotechnology is the erased and reprogrammed. It is 5


53. (c) Given, p =
biotechnology applied to ‘agricultural commonly used in USB flash drives, 18
processes’. Three main areas of digital cameras and solid-state drives. 1 9 1
27p 3 − − p2 + p
applications are plant tissue culture, plant 46. (c) Earth Hour is a Word Wide movement 216 2 4
genetic engineering and plant molecular organised by the World Wildlife Fund 3
= (3p)3 −   − 3 × (3p)2 ×
1 1
marker-assisted breeding. It is used to (WWF) for nature. The event is held  6 6
make plants pest and drought tolerant. annually encouraging individuals,
1
39. (b) India’s first Indigenous Aircraft Carrier communities and businesses to turn + 3 × 3p ×
(IAC) Vikrant will be delivered to the off non-essential electric lights for one 36
3
Indian Navy in 2021 for advanced trials. hour.
=  3p − 
1
IAC Vikrant is in advanced stage of 47. (b) Former NSDL Vice-President, Sudha  6
construction and it is being built by Balakrishnan has been appointed as the
Cochin Shipyard Ltd. [∵(a − b)3 = a3 − b3 − 3a2 b + 3ab2 ]
first Chief Financial Officer (CFO) of
5
40. (b) Article 40 of Indian Constitution deals Reserve Bank of India (RBI). She has On putting the value of p =
with the establishment of Panchayati been appointed for a 3-year term and will 18
3
Raj. The Article includes that state will be responsible for reporting of financial
=  3 ×
5 1
− 
form the structure of panchayat. But it information of the Central Bank,  18 6 
doesn’t explain its power, who will be the establishing accounting policies and 3 3
=  −  =  
members, their qualifications, etc. So, ensuring compliance with regulations. 5 1 4
the 73rd Constitutional Amendment was  6 6  6
48. (a) The Jnanpith Award is an Indian
done in 1992. The amendment provided literary award presented annually by the 3
=   =
2 8
a constitutional status to the Panchayati Bharatiya Jnanpith to an author ‘for their  3 27
Raj institutions. Outstanding Contributions towards
41. (c) INS Vibhuti was India’s first literature’. 54. (b) Given, speed of train = 36 km/h
36 × 5
=   m/s 
5
indigenously built missile boat launched Leading Contemporary English writer,  m/s ∵1 km = 
at Mazgaon docks in Mumbai.  18   18
Amitav Ghosh has been awarded the
42. (d) The Reserve Bank of India (RBI) is 54th Jnanpith Award. = 10 m/s
India’s Central Bank, which controls the By formula,
49. (b) The first Indian Satellite, Aryabhatta
issue and supply of the Indian Rupee. Distance = Speed × Time
was launched on 19th April, 1975. The
Currently RBI has four subsidiaries viz. satellite was named after the prominent Length of train = 10 × 25 = 250 m
Deposit Insurance and Credit Guarantee Indian astronomer and mathematician 55. (a) Average marks of 40 students = 34
Corporation of India (DICGC), Bharatiya Aryabhatta. ∴Sum of marks of 40 students
Reserve Bank Note Mudran Private 50. (c) The SAARC Secretariat is based in = Average × Number of Students
Limited (BRBNMPL), Reserve Bank Kathmandu, Nepal. It coordinates and = 34 × 40 = 1360
Information Technology Pvt. Ltd. (ReBIT) monitors implementation of activities, But, marks of one student had been
and Indian Financial Technology and prepares for and services meetings, and wrongly entered as 62 instead of 26.
Allied Services (IFTAS). serves as a channel of communication ∴Original sum of marks
43. (b) The Indian Army launched 10 e-cars between the association and its member = 1360 − 62 + 26 = 1324
in its pilot batch for its officials in New states as well as other regional 1324
∴Required average = = 33.1
Delhi. The main aim behind the move is organisations. 40
ensure minimal emission and boost 51. (a) 518 = 13 × 39 + 11 56. (a) In 1 L i.e. 1000 mL of mixture,
efficiency in an attempt to combat 30
worsening air quality in New Delhi. [∵ divident = quotient × divisor Water = 1000 × = 300 mL
100
44. (a) NO2 and SO2 (Nitrogen Oxide and + remainder]
Alcohol = 1000 − 300 = 700 mL
Sulphur Dioxide) are the major So, 11 must be subtracted from 518, so Let x mL of alcohol is mixed.
contributors to smog and acid rain. Both that the sum is completely divisible by
According to the question,
these gases react with volatile organic 13. 300
1 × 100 = 15
compounds to form smog which cause 52. (a) Work done by (A + B) in 1 day = 1000 + x
respiratory problems in humans. Acid 7.5
1 ⇒ 1000 + x = 2000
rain can harm vegetation, change the Work done by A in 1 day = ⇒ x = 1000 mL
chemistry of river and lake water by 10
lowering pH which is harmful to animal 1 1 57. (b) By using VBODMAS,
∴Work done by B in 1 day = −
life and also decompose the marbles and 7.5 10 45 − [28 − {37 − 15 + x}] = 58
statues. 2.5 45 − [28 − {22 + x}] = 58
=
45. (a) Flash is a type of Electrically Erasable 75 45 − [28 − 22 − x] = 58
Programmable Read Only Memory 1 ⇒ 45 − [6 − x] = 58
=
(EEPROM). Flash Memory is an 30 ⇒ 45 − 6 + x = 58
electronic non-volatile computer memory Hence, B alone can complete the work in ⇒ 39 + x = 58
storage medium that can be electrically 30 days. ⇒ x = 58 − 39 = 19 ⇒ x = 19
10 AFCAT ~ Practice Set 1

58. (d) By using VBODMAS, 63. (a) By Alligation rule, 68. (d) Perimeter of semi-circular path
2
×
3 Mixutre I Mixture II = 36 m
1 5 3 1 5 5 πr + 2r = 36
1+ of ÷ 1 Alcohol = Alcohol =
1 6 2 4 8 9 [r is the radius of path]
1−
7
r  + 2 = 36
2 22

=
2
×
3 12  7 
1+ 2 5 × 3 ÷ 5 7 5 1 5 7 1 22 + 14
− = − = ⇒ 
12 9 36 8 12 24 r  = 36
6 2 4  7 
2 3 2 1 1
= × = × 3=2 Required ratio = : =2:3 36 × 7
3 5× 4 3 36 24 ⇒ r= =7m
36
4 5 64. (a) Let the CP be ` 100 and the marked πr2
59. (b) Let numerator of the fraction = x price be ` x. Area of semi-circular path =
2
Then, denominator of fraction = 13 − x Selling price = 132 1 22
MP × (100 − discount%) = × × 7 × 7 = 77 sq m
∴Original fraction =
x = SP 2 7
13 − x 100
69. (a) Stars, Planets and comets have their
88
According to the question, ∴ x× = 132 existence in the 5 kg.
x +1 1 100
= 70. (d) ‘Ayurveda’ is the branch of medicine,
13 − x − 2 2 132 × 100
⇒ x= = 150 all others are Vedas.
⇒ 2(x + 1) = (11 − x) 88
150 − 100 71. (d) Except ‘Chin’, all other parts are inside
⇒ 2x + 2 = 11 − x Required per cent = × 100
100 the mouth.
⇒ 3x = 9 ⇒ x = 3
50 72. (a) Dearth means lack of something and
∴Original fraction = × 100 = 50%
x 3 3 100 surplus means excess so there is opposite
= = = word relationship.
13 − x 13 − 3 10 65. (c) Let original price of sugar be
` x per kg. 73. (b) Disobedience means refusal to obey
60. (d) Suppose third number = x rules and as a result of this one gets
The reduce price of sugar
∴First number = x + 10% of x 20x 80x 4x punishment.
10 110 =x− = =`
=x+ x=x× = 1.1x 100 100 5 74. (c) Former is the site and latter is the
100 100 location.
According to the question,
and second number = x + 25% of x 320 320
− =5 75. (d) The pattern is as follows
25
=x+ x 4x x 3 5 7 9 11 13 15 17 19 21 23 25
100 5
125 C E G I K M O Q S U W Y
=x× = 1.25 x ⇒
1600 320
− =5
100 4x x +2 +2 +2 +2 +2 +2 +2 +2 +2 +2 +2
∴Required percentage 1600 − 1280 ∴ = UWY
11. ⇒ =5
= × 100 = 88% 4x 76. (b) The pattern is as follows
.
125 ⇒ 20x = 320 1 18 3 19 5 20 7 21 9
61. (c) Given, A = ` 956, P = ` 800, t = 3 yr 320 A R C S E T G U I
⇒ x= = ` 16 per kg
SI = A − P = 956 − 800 = ` 156 20 +1 +1 +1
SI × 100 2 +2 +2 +2 +2
∴ Rate = 66. (c) Difference = Sum 
r 
Principal × Time  First ? = U, Second ? = I
 100
156 × 100 ∴Required answer = U, I
= 2 2
⇒ 1 = Sum 
4  1
800 × 3  ⇒1 = Sum   77. (a) The pattern is as follows
 100  25
= 6.5% per annum +1 +1 +1
∴Sum = (25) 2 = ` 625
∴ New rate = Old rate + 4%
= 10.5% 67. (a) LCM of 3, 5, 7 and 9 = 315 AEI BFJ CGK DHL
+1
Principal × Time × Rate Largest four digit number = 9999
+1 +1
Now, SI =
100 315) 9999 (31 +1 +1 +1
800 × 3 × 105. ∴ ? = DHL
= = ` 252 945
100
549 78. (b) The code has been generated by
∴ Amount = 800 + 252 = ` 1052
315 taking opposite letters:
62. (b) Relative speed = 90 − 75 = 15 km/h As,
234
By Formula, A B C D E F G H I J K L M Pairs of
Distance ∴Number divisible by 315 = 9999 − 234 opposite
Required ratio =
Relative speed = 9765 Z Y X W V U T S R Q P O N letters

5 5 By technique, Therefore, O K L P
= h= × 60 min
15 15 Required number = 9765 − 2 = 9763
= 20 min [∵(3 − 1) = (5 − 3) = (7 − 5) = (9 − 7) = 2 ]
AFCAT ~ Practice Set 1 11

79. (d) The colour of milk is white. But 85. (d)


according to given condition white is G Z
called yellow. So, the colour of milk is
yellow.
∴ FZ
80. (a) The century year which is completely
divisible by 400 is a leap year. Thus, the All shapes are different.
year 2800 is a leap year. 86. (d) All other figures are divided into equal 98. (b)
parts. P S
81. (c) The direction diagram is as follows
A C N 87. (c) Except (c), all have undivided leave
4 km on top of the line. W M
2 km 2 km W E
88. (d) In figure (d), the length of both the
B D
S arrows is same. D R
Here, AC = BD 89. (a) Common to the
∴Required distance, BD = 4 km triangle and square ∴ WS
82. (b)
× 99. (a)
Fathers Educated V E
90. (a) Common to the
Sons circle and triangle

Conclusions I. (×) II. (P) T K


Common to the
Hence, only Conclusion II follows. triangle and square
83. (c) The amendment in the rule has been F
91. (c) Answer figure (c) is embedded in
welcomed in India does not imply that
problem figure.
the problem is more prevalent here.
So, I does not follow. Also, the 92. (d) Answer figure (d) is embedded in ∴ VF
amendment seeks to discourage problem figure.
only sexual harassment of women 93. (c) Answer figure (c) is embedded in 100.(a)
problem figure. Z S
and shall in no way discourage
employment of women. So, II also 94. (d) Answer figure (d) will complete the
does not follow. problem figure.
Body 95. (a) Answer figure (a) will complete the A M
84. (b)
problem figure.
Nose Hand 96. (a) Answer figure (a) will complete the
T R
problem figure.
97. (b) E X
Nose and hand are two entirely different TS
parts of the body. F Y
12 AFCAT ~ Practice Set 2

AFCAT
AIR FORCE COMMON ADMISSION TEST (ONLINE )

Practice Set ~02


Time : 2 Hrs MM : 300

INSTRUCTIONS
1. The set contains a total of 100 questions, Comprising Verbal Ability in English, General Awareness, Numerical Ability and Reasoning and
Military Aptitude Test.
2. Each correct question carry 3 Marks and there will be negative marking of 1 Mark for each incorrect attempt.
3. Total time duration will be 2 hrs (120 minutes).
4. No marks will be deducted for unattempted questions.

Directions (Q. Nos. 1-5) Read the 2. According to the author, no man can Directions (Q. Nos. 6 and 7) In the
following passage carefully and answer be discontented with the world if he following questions, sentences are given
the questions given below it. (a) is determined to be happy with blanks to be filled in with an
It is no doubt true that we cannot go (b) is sincere in discharging his duties appropriate word(s). Four alternatives
through life without sorrow. There can (c) has a healthy attitude to life are suggested for each question. Choose
(d) likes sunshine the correct alternative out of the four.
be no sunshine without shadow, we
must not complain that roses have 3. The expression ‘Life is a tragedy 6. Sonia disliked eating in the ………
thorns but rather be grateful that thorns to those who feel’ means that it is restaurant.
bear flowers. Our existence here is so a tragedy to those who (a) noisily (b) noiseful
complex that we must expect much (a) think about the world (c) noisy (d) noised
sorrow and suffering. Yet, it is certain (b) believe in fate 7. There’s a cinema near the station,
that no man was ever discontented (c) do not understand the world ……… .
with the world who did his duty in it. (d) are sensitive and emotional (a) aren’t it (b) isn’t there
The world is like a looking glass; if 4. The author says, “There are some (c) weren’t it (d) isn’t it
you smile, it smiles; if you frown, it persons whose very presence Directions (Q. Nos. 8-10) In these
frowns back. Always try, then, to look seems like a ray of sunshine and questions, four alternatives are given for
at the bright side of things. There are brightens the whole room”. The the idiom/phrase. Choose the
some persons whose very presence reason for this is that, they alternative which best expresses the
seems like a ray of sunshine and (a) have the capacity to love meaning of the idiom/phrase.
brightens the whole room. Life has (b) talk more of roses and less of
thorns
8. To foam at one’s mouth
been described as a comedy to those
(c) are happy and spread happiness (a) to brush properly
who think and a tragedy to those who
(d) look good and behave well (b) to get very angry
feel.
(c) to salivate on seeing food
1. The author says that we cannot go 5. What is the author’s message in (d) None of the above
through life without sorrow this passage?
because (a) Look at the bright side of things 9. To feel like a fish out of water
(a) it is our fate (b) Our existence is so complex (a) disgusted
(b) we are always discontented (c) The world is a looking glass (b) uncomfortable
(c) life is a tragedy (d) Expect much sorrow and (c) disappointed
(d) human life is very complex suffering (d) homeless
AFCAT ~ Practice Set 2 13

10. To burn one’s fingers 20. (a) expansive (b) extensive 34. The spoon dropped by an
(a) to get hurt physically (c) extended (d) expended astronaut in a satellite will
(b) to suffer financial losses (a) fall to the floor
21. (a) open (b) opened
(c) to find work (b) remain stationary
(c) opening (d) opined
(d) to suffer nervous breakdown (c) continue to follow the motion of
Directions (Q. Nos. 11 and 12) Find 22. (a) urban (b) village the satellite
out which part of the sentence has an (c) rural (d) country (d) move tangentially away
error. If there is no error, select option 23. (a) in (b) inside 35. The first UN-Habitat Assembly
(d) ‘No error’ as your answer. (c) on (d) into held in
11. The pile of books(a)/ are(b)/ (a) Sudan (b) Nigeria
24. (a) lead (b) led
(c) Congo (d) Kenya
missing. (c)/ No error (d)
(c) conformed (d) caused
12. The operation of this machine(b)/ 36. NASA unveiled which space
Directions (Q. Nos. 25-27) Choose the mission for 2024?
is different(b)/ to that of other.
correct synonym for the following
(c)/ No error (d) (a) Mass Mission
words.
Directions (Q. Nos. 13-16) Four (b) Venus Mission
words are given in each question, out of 25. Acquaint (c) Moon Mission
which only one word is correctly spelt. (a) Withhold (b) Conceal (d) None of the above
Find the correctly spelt word. (c) Familiarise (d) Risky
37. Who is known as ‘the Father of
13. (a) Liabertarian (b) Libertarian 26. Coalition Indian Missile Technology’?
(c) Liebertarain (d) Liberterian (a) Alone (b) Disintegration (a) Dr. UR Rao
(c) Alliance (d) Perfection (b) Dr. APJ Abdul Kalam
14. (a) Emphetic (b) Emphattic (c) Dr. Chidambaram
27. Melancholy
(c) Emphatick (d) Emphatic (d) Dr. Homi Bhabha
(a) Animated (b) Carefree
15. (a) Mountainer (b) Mountaineer (c) Depression (d) Forgivable 38. ‘Life Divine’ is a book written by
(c) Mounteener (d) Mountineer Directions (Q. Nos. 28-30) Choose (a) MK Gandhi
16. (a) Happened (b) Happenned the correct antonym for the following (b) Rabindranath Tagore
(c) Hapened (d) Hapenned words. (c) S Radhakrishnan
28. Benevolent (d) Sri Aurobindo
Directions (Q. Nos. 17-19) Out of the
four alternatives, choose the one which (a) Caring (b) Humane 39. Which of the following national
can be substituted for the given (c) Gracious (d) Selfish park is the natural habitat of
word/sentences. Kashmiri stag?
29. Blustered
(a) Nandankanan National Park
17. A medicine to nullify the effect of (a) Angry (b) Gentle
poison. (b) Padmaja Naidu Himalayan
(c) Neglect (d) Impede Zoological Park
(a) Antibody (b) Antigen
30. Autonomy (c) Dachigam National Park
(c) Antidote (d) Anticlimax
(a) Slavery (d) Kaziranga National Park
18. An assembly of worshippers. (b) Subordination 40. Indian satellite series which is
(a) Congregation (b) Conflagration (c) Dependence being used for telecommunication
(c) Configuration (d) Confrontation (d) Submissive system is
19. A person who lives by himself. 31. Kalibangan, the Indus Valley site (a) Rohini (b) IRS
(a) Monk (b) Recluse is in (c) CARTOSAT (d) INSAT
(c) Extrovert (d) Prophet (a) Rajasthan 41. Which capital city of the world
Directions (Q. Nos. 20-24) In the (b) Gujarat included Hindi as ‘Third Official
following passages, some of the words (c) Madhya Pradesh Court Language’?
have been left. Try to understand what it (d) Uttar Pradesh (a) Tehran (b) Abu Dhabi
is about. Then fill in the blanks with the 32. Which one of the following is an (c) Baghdad (d) Tokyo
help of the alternatives given. unpredictable natural disaster? 42. The Supreme Court at Calcutta
One of the oldest cities of Asia, Srinagar (a) Earthquake (b) Cyclone was established by
is known for its unique wooden (c) Tornado (d) Hurricane (a) Regulating Act of 1773
architecture. Its ...(20)... network of
33. In terms of area, which one of the (b) Pitts India Act of 1784
narrow streets amid multi-storey wooden following Indian states has the (c) Charter Act of 1793
houses ...(21)... out its waterways, make largest coverage of forests? (d) Charter Act of 1813
it look like medieval Islamic ...(22)... (a) Andhra Pradesh
centers. The large-scale demolition of 43. Who built the famous temples at
(b) Chhattisgarh
traditional buildings and bazaars ...(23)... Khajuraho?
(c) Madhya Pradesh
the quest for modernity have, however (a) Chandelas (b) Gahadvalas
(d) Odisha
...(24)... to their disappearance. (c) Palas (d) Senas
14 AFCAT ~ Practice Set 2

44. The Mohiniattam dance form was 53. A can do a piece of work in 62. A car travelling at a speed of
developed in 12 days and B in 15 days. They 40 km/h can complete a journey in
(a) Andhra Pradesh (b) Karnataka work together for 5 days and then 9 h. How long will it take to travel
(c) Kerala (d) Tamil Nadu B left. The days taken by A to the same distance at 60 km/h?
finish the remaining work is (a) 6 h (b) 3 h
45. The distance of a Marathon run is 1
(a) 3 (b) 5 (c) 10 (d) 12 (c) 4 h (d) 4 h
(a) 26 miles 385 yards
54. Average age of 6 sons of a family 2
(b) 27 miles 385 yards
(c) 28 miles 389 yards is 8 yr. Average age of sons 63. Out of given response, one of the
together with their parents is
(d) 29 miles 390 yards factors of 3 x3 − 6 x2 − 3 x + 6 is
22 yr. If the father is older than
46. Which of the following describes the mother by 8 yr, then the age (a) 3x 2 + 3x + 6
India as a secular state? of mother (in years) is (b) 3x 2 − 3x + 6
(a) Fundamental Rights (a) 44 (b) 52 (c) 60 (d) 68 (c) 3x 2 − 3x − 6
(b) Directive Principles of State Policy
55. A dealer buys an old cooler listed (d) 3x 2 + 2x − 6
(c) Fifth Schedule
at ` 950 and gets successive 5
(d) Preamble of the Constitution
discounts of 20% and 10%. He 64. The value of 4 − is
1
47. Smog is caused due to spends ` 66 on painting. He sells 1+
1
(a) emissions from vehicles the cooler at a profit of 25%. Find 3+
1
(b) from incinerators it’s selling price. 2+
(a) ` 937.50 (b) ` 935 1 1 4
(c) oil paints (a) (b)
(d) All of the above (c) ` 940 (d) ` 936.50 8 32
1 1
48. Greatest number of sweat glands 56. The marked price of an item is (c) (d)
are present in which part of the twice the cost price. For a gain of 64 16
human body? 15%, the discount should be 65. A certain sum, invested at 4%
(a) Forehead (a) 7.5% (b) 20.5% per annum compound interest,
(b) Forearm (c) 32.5% (d) 42.5% compounded half-yearly, amounts
(c) Palm of the hand to ` 7803 at the end of 1 yr. The
57. 20% of a = b, then b% of 20 is the
(d) Back sum is
same as ........ of a.
(a) ` 7000 (b) ` 7200
49. Which bank has launched the (a) 8% (b) 40%
artificial intelligence chatbot (c) 4% (d) 80% (c) ` 7500 (d) ` 7700
named ‘ABHi’?
58. A number is increased by 10% 66. ‘Doctor’ is related to ‘Stethoscope’,
(a) Axis Bank and then it is decreased by 10%. in the same way as ‘Painter’ is
(b) Andhra Bank The net change in the number is related to
(c) Allahabad Bank (a) 1% decrease (a) Painting (b) Brush
(d) State Bank of India (b) 2% decrease (c) Exhibition (d) Art
50. Who has delivered Chetak (c) 1% increase 67. Complete the second pair in the
helicopter to the Indian Navy (d) Neither increase nor decrease same way as first pair.
ahead of its schedule? DRIVEN : NEVIRD :: BEGUM : ?
59. In a mixture of 60 L, the ratio of
(a) Hindustan Aeronautics Limited acid and water is 2 : 1. If the ratio (a) MEUBG
(b) Defence Research and of acid and water is to be 1 : 2, then (b) BGMUE
Development Organisation the amount of water (in litres) to (c) EBGMU
(c) Bharat Heavy Electricals Limited be added to the mixture is (d) MUGEB
(d) Oridnance Factory Board (a) 55 (b) 60 68. Choose the correct number that
1 (c) 50 (d) 45 will replace the question mark.
51. If th part of a number is 100
7 1 60. The difference between the 8 : 56 :: 9 : ?
more than th part of a number, (a) 10 (b) 63 (c) 7 (d) 9
11 simple interests received from
then the number is two different banks on ` 500 in 69. Complete the second pair in the
(a) 770 (b) 1925 2 yr is ` 2.5. The difference same way as first pair.
(c) 1825 (d) 1200 between their per annum rates of ABC : 123 : : EFG : ?
interest is (a) 567 (b) 456
52. 40 men can complete a work in
(a) 0.10% (b) 0.25% (c) 678 (d) 987
40 days. They started the work
(c) 0.50% (d) 1.00%
together. But at the end of each Directions (Q. Nos. 70-72) In the
10th day, 5 men left the job. The 61. A 120 m long train takes 10 s to following questions, choose the word
work would have been completed in cross a man standing on platform. which is different from others.
2
(a) 56 days
1
(b) 53 days
What is the speed of the train?
3 3 (a) 12 m/s (b) 10 m/s 70. (a) Basket (b) Barrel
(c) 52days (d) 50 days (c) 15 m/s (d) 20 m/s (c) Bag (d) Barrow
AFCAT ~ Practice Set 2 15

71. (a) Cricket (b) Baseball given statements, disregarding the Directions (Q. Nos. 83-86) Choose
(c) Football (d) Billiards known facts. the correct alternative that will
Statements complete the figure.
72. (a) Genius (b) Geyser No coin is a dollar.
(c) Gesture (d) Revenge Red token is a coin. 83. Problem Figure
73. Choose the correct missing terms Conclusions
in the following series. I. Red token is not a dollar. ?
a_bb_c_a_ _ c_ aa II. Red token may not be a dollar.
(a) acabbc (b) abcabc (a) Only Conclusion I follows
(c) acbbac (d) abcbbc (b) Only Conclusion II follows
74. When decoded, OPTRRE reads as (c) Neither Conclusion I nor II follows
(d) Both Conclusions I and II follow Answer Figures
PORTER. In the same way, what
does the following read as? 81. A statement followed by two
EROPTR conclusions is given. You have to
(a) ROPE (b) PROPER take the given statement to be
(c) PORT (d) REPORT true even, if it seems to be at (a) (b) (c) (d)
variance from commonly known
75. If MATHEMATICS 84. Problem Figure
facts. Read the conclusions and
= 12 3 4 512 3 6 78,
then decide which of the
then MAHATHMA = ? conclusions, if any, logically follows
(a) 12423412 (b) 12345123 from the given statement,
(c) 12345676 (d)12425341 disregarding the known facts.
76. Deepak starts walking straight Statement Our securities ?
towards East. After walking 75 m, investments carry market risk.
he turns to the left and walks Consult your investment advisor
or agent before investing. Answer Figures
25 m straight. Again he turns to
the left, walks a distance of 40 m Conclusions
straight, again he turns to the left I. One should not invest in
and walks a distance of 25 m. securities.
How far is he from the starting II. The investment advisor
(a) (b) (c) (d)
point? calculates the market risk with
certainty.
(a) 35 m (b) 50 m 85. Problem Figure
(c) 115 m (d) 140 m (a) Only Conclusion I follows
(b) Only Conclusion II follows
77. If 15th June falls 3 days after (c) Either Conclusion I or II follows
tomorrow and tomorrow is Friday, (d) Neither Conclusion I nor II
on what day will the last day of follows
the month fall? ?
(a) Monday (b) Tuesday 82. One statement followed by two
(c) Wednesday (d) Thursday assumptions numbered I and II is
Answer Figures
given. You have to consider the
78. Which of the following diagram statement and the following
correctly represents the relation assumptions and decide which of
among Animal, Dog and Pet? the assumptions is implicit in the
statement.
(a) (b) (c) (d)
Statement
‘Use ‘X’ brand shoes. They are 86. Problem Figure
(a) (b) (c) (d) available in all sizes and last
longer’—an advertisement in the
79. Which of the following figures newspaper ‘A’. ?
best describes the relationship Assumptions
among Ocean, Ship and Sailor? I. Very few people read
advertisement in a newspaper.
II. Very few people read the
newspaper ‘A’. Answer Figures
(a) (b) (c) (d) (a) Only Assumption I is implicit
80. Two statements followed by two (b) Only Assumption II is implicit
conclusions are given. Read the (c) Neither Asumption I nor II is
conclusions and then decide implicit (a) (b) (c) (d)
which of the conclusions, if any, (d) Both Assumptions I and II are
logically follows from the two implicit.
16 AFCAT ~ Practice Set 2

Directions (Q. Nos. 87-90) Choose Directions (Q.Nos. 91-94) In the Answer Figures
the correct answer figure in which the following questions, choose the
problem figure is embedded. correct answer figure that will continue
87. Problem Figure the series given in problem figures.
91. Problem Figures (a) (b) (c) (d)

95. Two positions of a dice are shown.


When 4 is at the bottom, what
Answer Figures 1 2 3 4 5 number will be on the top?
1 1
Answer Figures
2 6
3 5
(a) (b) (c) (d)
(i) (ii)
88. Problem Figure
(a) (b) (c) (d) (a) 1 (b) 2
(c) 5 (d) 6
92. Problem Figures
Directions (Q.Nos. 96-100) In the
following questions, choose the figure
Answer Figures which is different from others.
1 2 3 4 5 96.
Answer Figures
- + - + - +
(a) (b) (c) (d) - (a) (b) (c) (d)
89. Problem Figure +
97.
(a) (b) (c) (d)

93. Problem Figures


(a) (b) (c) (d)
Answer Figures
98.
1 2 3 4 5
Answer Figures
(a) (b) (c) (d)
(a) (b) (c) (d)
90. Problem Figure 99.
(a) (b) (c) (d)

94. Problem Figures (a) (b) (c) (d)

Answer Figures 100.

1 2 3 4 5
(a) (b) (c) (d)
(a) (b) (c) (d)

Answers
1 (d) 2 (d) 3 (d) 4 (c) 5 (a) 6 (c) 7 (b) 8 (b) 9 (b) 10 (b)
11 (b) 12 (c) 13 (b) 14 (d) 15 (b) 16 (a) 17 (c) 18 (a) 19 (b) 20 (b)
21 (a) 22 (a) 23 (a) 24 (b) 25 (c) 26 (c) 27 (c) 28 (d) 29 (b) 30 (c)
31 (a) 32 (a) 33 (c) 34 (c) 35 (d) 36 (c) 37 (b) 38 (d) 39 (c) 40 (d)
41 (b) 42 (a) 43 (a) 44 (c) 45 (a) 46 (d) 47 (d) 48 (c) 49 (b) 50 (a)
51 (b) 52 (a) 53 (a) 54 (c) 55 (a) 56 (d) 57 (c) 58 (a) 59 (b) 60 (b)
61 (a) 62 (a) 63 (c) 64 (a) 65 (c) 66 (b) 67 (d) 68 (b) 69 (a) 70 (d)
71 (d) 72 (b) 73 (a) 74 (d) 75 (a) 76 (a) 77 (b) 78 (c) 79 (c) 80 (a)
81 (b) 82 (c) 83 (d) 84 (d) 85 (d) 86 (b) 87 (b) 88 (b) 89 (a) 90 (d)
91 (b) 92 (d) 93 (a) 94 (c) 95 (a) 96 (b) 97 (a) 98 (b) 99 (c) 100 (a)
AFCAT ~ Practice Set 2 17

Hints and Solutions


1. (d) We can’t go through life without 18. (a) Congregation is an assombly of release of energy in the Earth’s
sorrow because human life is very persons gathered for religious worship. lithosphere that creates seismic waves.
complex. 19. (b) A recluse is a person who lives alone The suddenness associated with
2. (d) No man can be discontented with the and deliberately avoids other people. earthquakes is the main reason behind
world if he likes sunshine. 20. (b) extensive severe damage to life and property.
3. (d) It is a tragedy to those, who are 21. (a) open 33. (c) According to Forest Report 2017, the
sensitive and emotional. 22. (a) urban largest coverage of forest is in Madhya
4. (c) Quite obvious from the tone of the 23. (a) in Pradesh. Next to Madhya Pradesh is
passage that they are happy and spread 24. (b) led Arunachal Pradesh, Chattisgarh, Odisha
happiness. and Maharashtra.
25. (c) ‘Acquaint’ means make someone
5. (a) The meaning of the passage suggests aware of or familiar with and ‘familiarise’ 34. (c) The spoon dropped by an astronaut in
that the author’s message is to look at also means the same. So, option (c) is a satellite will continue to follow the
the bright side of things. correct synonym of ‘Acquaint’. motion of the satellite.
6. (c) ‘Noisy’ is the proper adjective to use 26. (c) ‘Coalition’ is the union of two or more 35. (d) The first session of the UN-Habitat
here to make the sentence meaningful. political parties that allows them to form Assembly held from 27th to 31st May,
a government or fight an election 2019 at UN-Habitat at Nairobi, Kenya.
7. (b) The use of ‘isn’t there’ is appropriate
together. ‘Alliance’ is a group of The special theme for the UN-Habitat
here. When there is a positive statement
countries, political parties, or people who Assembly is ‘Innovation for better quality
then the question tag is used in negative
have agreed to work together because of of life in cities and communities.’
and when there is a negative statement
then the question tag is used in positive. shared interests or aims. India has been elected to the Executive
So, ‘Alliance’ is the synonym of Board of the first UN-Habitat Assembly.
8. (b) ‘To foam at one’s mouth’ means to
‘Coalition’. 36. (c) National Aeronautics and Space
have a mass of small bubbles in and
around its mouth, especially because it is 27. (c) ‘Melancholy’ is an intense feeling of Administration (NASA) unveiled schedule
sick or angry. Hence, to be very angry is sadness which lasts for a long time and for 2024 Moon Mission ‘Artemis’ on
the correct answer. which strongly affects your behaviour 26th May, 2019. The Mission is named
and attitude. So, ‘Depression’ is its ‘Artemis’ after the Greek mythological
9. (b) To feel like a fish out of water means
synonym as it means the state of feeling Goddess of the Moon and twin sister to
a person who feels uncomfortable or
unhappy and without hope for the future. Apollo. This Mission has committed to
awkward because he or she is in
surrounding that are not familiar. 28. (d) ‘Benevolent’ is kind and helpful and take a female astronaut to the surface of
‘Selfish’ is caring only about oneself or the moon for the first time.
10. (b) To burn one’s fingers means to suffer
lacking consideration for other people. 37. (b) Former President of India, Dr. APJ
as result of doing something without
So, option (d) ‘selfish’ is an antonym of Abdul Kalam is known as ‘The Father of
realising. Usually referred to the loss of
‘benevolent’. Indian Missile Technology’. He played a
money. Hence, ‘to suffer financial loses’ is
its correct answer. 29. (b) ‘Blustered’ is to talk in a loud, crucial role in the development of
aggressive way and ‘gentle’ is having a ballistic missile and launch vehicle
11. (b) ‘Is’ will be used in place of ‘are’. Here
mild or kind way of talking. Therefore, technology.
subject ‘The pile of books’ is singular.
option (b) gentle is the opposite of In 1997, Kalam has been awarded the
12. (c) Replace ‘to’ with ‘from’ to make the blustered. Bharat Ratna, India’s highest civilian
sentence grammatically correct. award.
30. (c) ‘Autonomy’ is the ability to make your
13. (b) The correctly spelt word is own decisions without being controlled 38. (d) ‘Life Divine’ is a book written by
‘Libertarian’. Libertarian means one who by anyone else. Hence, ‘Dependence’ is Sri Aurobindo. The book is Sri Aurobindo’s
advocates liberty either generally or on a its correct opposite. principal philosophical work in which he
specific issue. presents a theory of spiritual evolution. Sri
31. (a) Kalibangan, was excavated between
14. (d) The correctly spelt word is ‘Emphatic’. 1960-61 and 1968-69. This ancient site Aurobindo was an Indian nationalist,
Emphatic means special attention or of Indus Valley Civilisation is located on philosopher, and a poet who actively took
prominence given to something. the bank of the dry bed of the Ghaggar in part in Indian Freedom Struggle.
15. (b) The correctly spelt word is Rajasthan. 39. (c) The Kashmir stag (Cervus canadensis
‘Mountaineer.’ The pre-historic and pre-mauryan hanglu), is a subspecies of elk native
Mountaineer means someone, who character of Indus Valley Civilisation was to India. It is found in dense riverine
climbs mountains for sport or pleasure. first identified at this site. forests in the high valleys and mountains
16. (a) The correctly spelt word is 32. (a) Earthquake is nature’s most of the Kashmir Valley and Northern
‘Happened.’ Happened means ‘to occur unpredictable and one of the most Chamba district of Himachal Pradesh. In
or take place.’ devastating natural disasters. An Kashmir, it is found in the Dachigam
17. (c) Antidote is a medicine that stops or earthquake is the shaking of the surface National Park where it receives
controls the effect of poison. of the Earth resulting from a sudden protection.
18 AFCAT ~ Practice Set 2

40. (d) The Indian National Satellite System mixture of air pollutants like sulphur Remaining work
(INSAT) is a series of multipurpose dioxide, nitrogen oxides, ozone and 15 20 − 15 5 1
=1− = = =
geo-stationary satellites launched by ISRO. other volatile organic compounds. 20 20 20 4
These satellites are used for 1
48. (c) Palm of the hand has greatest ∴Time taken by A to complete work
telecommunication systems, broadcasting, number of sweat glands in human 4
meteorology and search and rescue 1
body. The human body has = × 12 = 3 days
operations. approximately 2-4 million sweat glands 4
41. (b) In a landmark decision, the capital city found all over the body except on the 54. (c) Let the mother’s age = x yr
of UAE, Abu Dhabi has included Hindi as nails, ears and lips. ∴Father’s age = (x + 8) yr
the third official language used in its court. 49. (b) Andhra bank has launched an Average age of 6 sons = 8 yr
Rest of the two languages used at court Artificial Intelligence (AI) Sum of ages of 6 sons = 8 × 6 = 48 yr
premises are Arabic and English. Chatbot-ABHi, integrated with its core Sum of ages of 6 sons and parents
This will help Hindi speakers to learn banking servers to digitally engage and = 22 × 8 = 176 yr
about litigation procedures, their rights and automate customer support for its ∴Sum of ages of parents
duties without a language barrier. 5 crore customers. The virtual assistant = 176 − 48 = 128 yr
42. (a) The Regulating Act of 1773 has been crafted by Bangalore based According to the question,
AI start up Floatbot.
established a Supreme Court of Fort x + x + 8 = 128 ⇒ 2x = 120
William, Calcutta. This Supreme Court 50. (a) Hindustan Aeronautics Limited ∴ x = 60
consisted one Chief Justice and three other (HAL) delivered a Chetak Helicopter ∴Mother’s age = 60 yr
regular judges. ahead of schedule to the Indian Navy.
55. (a) Given, marked price = ` 950
Sir Elijah Impey was the first Chief Justice HAL entered into a contract with Indian
Navy in August 2017 for the supply of First discount = 20%
of this Supreme Court.
eight Chetak helicopters with the Second discount = 10%
43. (a) Khajuraho Temples are among the
delivery schedule of the first two in CP of the cooler (100 − 20)% of
most beautiful medieval monuments in the
country. These temples were built by the
August 2019 and the rest in August (100 − 10)% of 950 = 80% of 90% of
2020. 80 90
Chandella rulers over a span of 100 years 950 = × × 950 = ` 684
between 950 AD and 1050 AD. The 51. (b) Let the number = x 100 100
temples of Khajuraho are popular for their x x 11x − 7x Total CP including the expenditure on
∴ − = 100 ⇒ = 100
stunning architecture and has been 7 11 77 painting = 684 + 66 = ` 750
designated as a UNESCO World Heritage 100 × 77 To get a profit of 25%, then
⇒ x= = 1925
site. 4 SP = 125% of 750 = ` 937.50
44. (c) Mohiniyattam is one of the eight 52. (a) Remaining part of work after 50 56. (d) Let amount (CP) = ` 100
classical dances of India that developed days MP = ` 200
= 1 −  +
and remained popular in the state of 1 7 3 5 2 15% gain on CP
+ + +  100 × 115
Kerala. It was originated in 16th century  4 32 16 32 16  i.e. SP = = ` 115
CE and is performed by women in honour 1 100
=
of the Hindu God Vishnu in his incarnation 16 Discount = MP− SP = 200 − 115 = ` 85
as the enchantress Mohini. Kathakali Discount
Similarly, part of work done by 15 men Discount per cent = × 100
is another classical dance form of Kerala. 15 × 10 3 MP
in 10 days = = 85
45. (a) The Marathon is a long-distance race 1600 32 = × 100 = 42.5%
with an official distance of 42.195 km 3 200
∵ part of the work is done by 15 a
(approximately 26 miles 385 yards), 32 57. (c) 20% of a = b ⇒ = b
usually run as a road race. The event was men in 10 days 5
1 b a a
instituted in commemoration of the fabled ∴ part of the work is done in ∴b% of 20 = = = × 100%
run of the Greek soldier Pheidippides, a 16 5 25 25
messenger from the Battle of Marathon to 32 1 20 2 = 4% of a
10 × × = = 6 days
Athens, who reported the victory. 3 16 3 3 58. (a) Given that, r = 10%
46. (d) Secularism in India means equal ∴Total number of days to complete the According to the formula, we know that
2 2
treatment of all religions by the state. With work = 50 + 6 = 56 days change in the number is decrease.
the 42nd Amendment of the Constitution 3 3 (10)2
of India enacted in 1976, the Preamble to 1 ∴Decrease percentage = = 1%
53. (a) 1 day’s work of A = 100
the Constitution asserted that India is a 12
secular nation. And we as citizens of India 1 59. (b) Quantity of acid in the mixture
1 day’s work of B = 2
must abide by it. 15 = × 60 = 40 L
3
47. (d) In present time’s smog is caused due 1 day’s work of A and B Quantity of water = 60 − 40 = 20 L
1 1 5+ 4 9 3
to vehicular emissions, industrial fumes, = + = = = Let required quantity of water = x L
oil paints and incinerators which react in 12 15 60 60 20 40 1
15 ∴ =
atmosphere along with sunlight to form 5 day’s work of A and B = 20 + x 2
photochemical smog. This contains toxic 20
⇒ 80 = 20 + x ⇒ x = 60 L
AFCAT ~ Practice Set 2 19

60. (b) Let the two rates be R1% and R2 % per 65. (c) Let the sum be ` P. 76. (a) The direction diagram is as follows,
annum. As, the interest is compounded D 40 m C
Then, R1 > R2 half-yearly,
P × R1 × T P × R2 × T ∴ r = 2% and t = 2 half years
Difference = − t 25 m 25 m N
A = P1 +
r 
100 100 ∴ 
According to the question,  100
W E

2 E B
500 × 2 × R1 500 × 2 × R2 Starting A 75 m S
⇒ 7803 = P1 +
2 
− = 2.5  End
100 100  100 point
point
⇒ 10(R1 − R2 ) = 2.5 2
⇒ 7803 = P1 +
1
2.5  ∴Required distance, AE = AB – EB
⇒ R1 − R2 =  50
10 51 51 = (75 − 40) m = 35 m
= 025
. % per annum ⇒ 7803 = P × ×
50 50 [∵ EB = DC = 40 m ]
61. (a) Given, length of the train = 120 m 7803 × 50 × 50 77. (b) Given, tomorrow = Friday
⇒ P= = ` 7500
and time = 10 s 51 × 51 ∴ Day on 15th June
Length of train 120
Speed of train = = 66. (b) ‘Stethoscope’ is used by a ‘Doctor’ as = Friday + 3 = Monday
Time 10
= 12 m/s
a tool to perform his work. Similarly, a ∴ Day on 22nd and 29th = Monday
‘Painter’ uses a ‘Brush’ as a tool to
62. (a) Given, speed = 40 km/h, Time = 9 h ∴ Last day of the month i.e. 30th
perform his work.
∴Total distance covered = speed × time = Tuesday
67. (d) As, D R I V E N N E V I R D
= 40 × 9 = 360 km 1 2 3 4 5 6 6 5 4 3 2 1 78. (c) The relation can be shown through
The required time to cover 360 km at the diagram.
Similarly, B E G U M MUGE B
360
60 km/h = =6h 1 2 3 4 5 5 4 3 2 1
Animal
60 68. (b) As, 8 × 7 = 56
63. (c) Here given expression, Pet Dog
Similarly, 9 × 7 = 63
f(x) = 3x 3 − 6x 2 − 3x + 6
69. (a) As, A B C
Put x = 1
Positional Value
f(1) = 3(1)3 − 6 (1)2 − 3(1) + 6 Clearly, all the dogs are animal and some
1 2 3
= 3 − 6 + 6 − 3= 0 dogs are pet animal and some pet
∴x − 1 is a factor of given expression. Similarly, E F G animals may be dog.
Positional Value Ocean
Now, dividing expression by (x − 1) 79. (c)
(x − 1)) 3x 3 − 6x 2 − 3x + 6(3x 2 − 3x − 6 5 6 7
Ship
3x 3− 3x 2 70. (d) ‘Barrow’ is a man-driven cart,
Sailor
− + whereas others are usual containers.
− 3x 2 − 3x + 6 71. (d) Except ‘Billiards’ all others are Sailor is a part of ship and ship is a part
outdoor games. of ocean.
− 3x + 3x 2
72. (b) Only ‘Geyser’ is visible, whereas all 80. (a)
+ − others are invisible.
− 6x + 6 73. (a) aa/bb/cc/aa/b b/cc/aa ⇒ acabbc ×
− 6x + 6 Red Token Dollar
74. (d) As, O P T R R E
+ −
Coin
×
∴(3x 2 − 3x − 6) is also a factor of P O R T E R
Conclusions I. (P) II. (×)
expression 3x 3 − 6x 2 − 3x + 6. Similarly, E R O P T R
Sailor is a part of ship and ship is a part
5
64. (a) 4 − of ocean.
1
1+ 81. (b) The statement advises one to consult
1 R E P O R T
3+ investment advisor before investing. So,
1
2+ 75. (a) As, Conclusion II follows. Investment in
4
5 5 securities involves risk. This does not
= 4− = 4− M A T H E M A T I C S
1 9 mean that one should not invest in
1+ 1+ securities. So, I does not follow.
4 31
3+ 1 2 3 4 5 1 2 3 6 7 8
9 82. (c) Neither Assumption I nor II is implicit.
5 × 31 160 − 155 Many people read advertisement in a
= 4− = Therefore, M A H A T H M A
40 40 newspaper. And, the number of readers
5 1 of newspaper ‘A’ depends upon the
= = 1 2 4 2 3 4 1 2 popularities of the newspaper.
40 8
20 AFCAT ~ Practice Set 2

83. (d) Figure (d) will complete the problem three shift one step anti-clockwise in a 95. (a) It is clear from the figures that
figure. cyclic order. In the next step all the 2, 3, 5 and 6 lie adjacent to 1. So,
84. (d) Figure (d) will complete the problem elements which are present in the horizontal 4 lies opposite to 1. Hence, when 4 is
figure. line move one position downwards while at the bottom, then 1 must be on the
elements present at corners move 90° top.
85. (d) Figure (d) will complete the problem
anti-clokwise. Now, the whole process 96. (b) In all the other figures, both the
figure.
repeats except that the shifting of three overlapping figures are identical having
86. (b) Figure (b) will complete the problem elements is now clockwise. same area.
figure.
92. (d) In each step, the clockwise end 97. (a) In all other figures, the lower figure is
87. (b) The problem figure is embedded in element moves to the anti-clockwise end obtained by inverting the upper figure and
answer figure (b). elements. shading it.
88. (b) The problem figure is embedded in 93. (a) From problem figure (1) to (2), whole 98. (b) In all figures, except figure (b), triangle
answer figure (b). figure rotates anti-clockwise direction is formed on the same side in which the
89. (a) The problem figure is embedded in and changes its position with a certain arrow is directed. In figure (b), triangle
answer figure (a). rule. Similar rule follows from problem and arrow are directed opposite to each
90. (d) The problem figure is embedded in figure (3) to (4) and problem figure (5) other and hence, does not belong to this
answer figure (d). to answer figure. group.
91. (b) In the 1st step two lower elements 94. (c) Problem figure (5) is same as figure (1) 99. (c) In all the other figures except (c) the
interchange places while the other three with shifting of shaded portion in opposite shaded portions lie opposite to each other.
shift anti-clockwise in cyclic order. In the direction. Therefore, answer figure will be
100.(a) In all figure except (a) square is
next step, the upper two elements same as figure (2) with shifting of shaded
divided into seven parts and one part is
interchange places while the remaining portion.
shaded.
AFCAT ~ Practice Set 3 21

AFCAT
AIR FORCE COMMON ADMISSION TEST (ONLINE )

Practice Set ~03


Time : 2 Hrs MM : 300

INSTRUCTIONS
1. The set contains a total of 100 questions, Comprising Verbal Ability in English, General Awareness, Numerical Ability and Reasoning and
Military Aptitude Test.
2. Each correct question carry 3 Marks and there will be negative marking of 1 Mark for each incorrect attempt.
3. Total time duration will be 2 hrs (120 minutes).
4. No marks will be deducted for unattempted questions.

Directions (Q. Nos. 1-4) Read the 2. According to the author, science make a cool, shady ...(7)... where the
following passage carefully and answer and technology should be Arabs live in easily movable tents.
the questions given below it. (a) tabooed and eliminated from life They move from place to place in
We are tempted to assume that (b) used in a controlled and careful ...(8)... of food. They load the ...(9)...
technological progress is real progress manner and move for miles as if he is the
and that material success is the (c) encouraged and liberally used ...(10)... of the desert.
criterion of civilisation. If the Eastern (d) made compulsory in education
5. (a) narrow (b) vast
people become fascinated by machines 3. From the passage, one gathers
(c) great (d) broad
and techniques and use them, as that the Eastern people must
Western nations do, to build huge (a) appreciate scientific achievements 6. (a) around (b) below
industrial organisations and large (b) build huge industrial (c) above (d) across
military establishments, they will get organisations
7. (a) garden (b) park
involved in power politics and drift (c) avoid being controlled by
machines and techniques of (c) oasis (d) pond
into the danger of death. Scientific and
technological civilisation brings great industrial production 8. (a) want (b) search
opportunities and great rewards, but (d) be fascinated by machines (c) lack (d) shortage
also great risks and temptations. 4. According to the author, science 9. (a) horse (b) donkey
Science and technology are neither and technology are
good nor bad. They are not to be (c) camel (d) cattle
(a) totally harmless
tabooed but tamed and assigned their (b) extremely dangerous 10. (a) ship (b) boat
proper place. They become dangerous (c) to be treated as idols (c) car (d) aeroplane
only if they become idols. (d) useful, if they are not worshipped Directions (Q. Nos. 11 and 12)
1. According to the author, people blindly Choose the word which is nearest in
think that real progress lies in Directions (Q. Nos. 5-10) Select the meaning to the given word.
(a) material success and most appropriate word from the options 11. Mordant
technological growth against each number. (a) Stupid (b) Pensive
(b) imitating Western nations
Arabia is a ...(5)... sandy desert. At day (c) Senseless (d) Sarcastic
(c) having large industries and
political power time, the sand becomes fiery. There are 12. Pragmatic
(d) taking risks and facing springs here and there ...(6)... which (a) Theoretical (b) Realistic
temptations grass, fig and palm trees grow and (c) Perfect (d) Simple
22 AFCAT ~ Practice Set 3

Directions (Q. Nos. 13-15) Choose 24. The millionaire who was caught 34. Deficiency of Vitamin-B6 in man
the word which is nearly opposite in stealing was found to be suffering causes
meaning to the given word. from ........... (a) rickets (b) scurvy
13. Minuscule (a) Klaptomania (b) Kleptomania (c) beri-beri (d) anaemia
(a) Menial (b) Minute (c) Klepptomania (d) Kleptemania 35. Nehru Cup is associated with
(c) Massive (d) Impressive 25. The script was delivered to the (a) Hockey (b) Football
14. Wary director ahead of ........ (c) Kabaddi (d) Table Tennis
(a) Schedule (b) Schedulle
(a) Free (b) Kind 36. The first World Cup in cricket was
(c) Careless (d) Watchful (c) Schdule (d) Schedulee held in
15. Transience 26. Who was popularly known as (a) 1975 (b) 1976
‘Nana Saheb’? (c) 1983 (d) 1980
(a) Eternity (b) Rest
(a) Baji Rao I
(c) Slow (d) Shallow 37. The 3-tier Panchayati Raj System
(b) Balaji Baji Rao in India was proposed by the
Directions (Q. Nos. 16-20) Choose (c) Balaji Vishwanath
(a) Balwant Rai Mehta Committee
the word that best describe the given (d) Madhav Rao
phrase. (b) Ashok Mehta Committee
27. When did the British make (c) Royal Commission
16. Anything written in a letter after English the medium of (d) None of the above
it is signed. instruction in India?
(a) Transcription (b) Addendum 38. ISRO launched the world’s first
(a) 1813 (b) 1833
(c) Endorsement (d) Postscript satellite dedicated to education, is
(c) 1835 (d) 1844
(a) GSAT 1 (b) GSAT 2
17. Deviation from the common rule 28. What is the creative art in which (c) GSAT 3 (d) None of these
or standard. Sonal Mansingh distinguished?
(a) Analogue (b) Anagram 39. Which of the following is the first
(a) Dance
(c) Anathema (d) Anomaly missile which has been developed
(b) Vocal music (Hindustani) in India?
18. A style in which a writer (c) Painting
(a) Akash (b) Prithvi
makes display of his (d) Instrumental music
(c) Agni (d) Trishul
knowledge. 29. Which one of the following
(a) Ornate (b) Pedantic 40. The headquarters of the Food and
countries is known as the ‘Land
(c) Artificial (d) Showy Agricultural Organisation (FAO) is at
of Thunderbolt’?
(a) New York (b) Paris
19. A situation that stops an activity (a) Belgium (b) Nepal
(c) Geneva (d) Rome
from progressing. (c) Bhutan (d) Bolivia
(a) Bypass (b) Breach 41. ‘National Youth Day’ is celebrated
30. Cultivation of wheat requires on
(c) Bottleneck (d) Blockhead (a) moderate temperature and heavy
(a) 15th January (b) 9th January
20. Filled with excessive enthusiasm rains
(c) 18th January (d) 12th January
especially in religion. (b) humid temperature and heavy
(a) Hydrophobia (b) Fanatic rains 42. INTERPOL stands for
(c) Henchman (d) Votary (c) humid temperature and moderate (a) International Criminal Police
rains Organisation
Directions (Q. Nos. 21-25) Choose (d) moderate temperature and (b) International Political Conference
the correctly spelt word. moderate rains (c) International Association of Police
21. A true friend is known in the day 31. ‘Nagoya Protocol’ is related to Officers
of ...... (d) None of the above
(a) International Finance
(a) Advercity (b) Adversity (b) Biodiversity 43. The author of the book titled ‘The
(c) Adverisity (d) Advercety (c) Pharmaceutical Industry Future of India’ is
22. Don’t think you are going to (d) Global Warming (a) Bimal Jalan (b) Deepak Chopra
appease me with your ......... 32. The frequency of ultrasound wave (c) Anurag Mathur (d) Amitav Ghosh
attitude. is typically 44. Which country became the first to
(a) Connillatory (b) Concilletry (a) above 20 KHz declare ‘Climate Emergency’ in its
(c) Conciliatory (d) Consiliatory (b) above 20,000 KHz Parliament?
23. The price for the holiday includes (c) below 20 KHz (a) UK (b) USA
......... and flights. (d) below 02 KHz (c) Russia (d) Japan
(a) accomodation 33. Which of the following is not a 45. An astronaut in outer space will
(b) accommodation form of carbon? observe sky as
(c) acommodation (a) Soot (b) Hematite (a) white (b) black
(d) acomodation (c) Graphite (d) Charcoal (c) blue (d) red
AFCAT ~ Practice Set 3 23

46. ‘Lemru Elephant Reserve’ is 55. If a train, with a speed of 60 km/h 64. If the sum of the digits of any
located in which state? crosses a pole in 30 s, then integer lying between 100 and
(a) Chhattisgarh (b) Assam the length of the train 1000 is subtracted from the
(c) Odisha (d) Haryana (in metres) is number, the result always is
(a) 1000 (b) 900 (a) divisible by 2 (b) divisible by 9
47. Which Institute developed a
unique robot named ‘Grasp Man’? (c) 750 (d) 500 (c) divisible by 5 (d) divisible by 6
1 1 1  3 1  65. The sum of numerator and
(a) IIT Kanpur (b) IIT Delhi 56. Simplify 1÷  + + ÷ −  .
(c) IIT Madras (d) IIT Kharagpur 2 3 6  4 3   denominator of a fraction is 13. If
30 37 1 is added to the numerator and 2
48. In the term GIS, ‘G’ stands for (a) (b)
is subtracted from the
37 30
(a) Global (b) Geographic 7 1
(c) Goodness (d) Geological (c) 1 (d) denominator, the fraction is .
37 2
49. The resources which are The value of the original fraction
57. The third proportional of the is
obtained from biosphere and
numbers 3, 4 is 7 3 13
have life are 9 3 (a) (b) (c) 3 (d)
(a) potential resources (a) (b) 12 10 50
4 2
(b) renewable resources
(c) abiotic resources (c)
16
(d)
12 Directions (Q. Nos. 66-70) The
(d) biotic resources 3 2 following questions consist of two words
each, that have a certain relationship
50. Total internal reflection cannot 58. If a dining table with marked
with each other, followed by four
take place when light goes from price, ` 6000 was sold to a
lettered pairs of words. Select the letter
(a) glass to water (b) water to glass customer for ` 5500, then the
pair that has the same relationship as
rate of discount allowed on the
(c) water to air (d) glass to air the original pair of words.
table is
51. Anuradha sold a bicycle at a gain (a) 10% (b) 8% 66. Emollient : Soothe
of 8%. Had it been sold for ` 75 1 (a) Dynamo : Generate
more, the gain would have been (c) 8 % (d) 9%
3 (b) Elevation : Level
14%. The cost price of the bicycle (c) Hurricane : Track
was 59. What is the greatest four digit
number which when divided by (d) Precipitation : Fall
(a) ` 1200 (b) ` 1250
(c) ` 1350 (d) ` 1500
10, 15, 21 and 28 leaves 67. Disabuse : Error
remainders 4, 9, 15 and 22 (a) Persevere : Dereliction
52. When the average age of respectively? (b) Discredit : Reputation
husband, wife and their son was (a) 9654 (b) 9666 (c) Rehebilitate : Addiction
42 yr, the son got married and (c) 9664 (d) 9864 (d) Belittle : Imperfection
a child was born just one year
after the marriage. When child 60. 60% of a number is 24 less than 68. Inspiration : Poetry
3
turned to be 5 yr, then the th of that number. What is the (a) Music : Notes
average age of family became 36 4 (b) Dirt : Disease
yr. What was the age of number? (c) Brush : Painting
daughter-in-law at the time of (a) 160 (b) 200 (d) Mind : Thought
marriage? (c) 150 (d) 140
69. Hockey : Game
(a) 26 yr (b) 25 yr 61. If ( x − 4) ( x2 + 4 x + 16) = x3 − P, (a) King : Rule
(c) 24 yr (d) 23 yr then P is equal to (b) Constitution : Assembly
53. 6 men and 8 women can do a (a) 27 (b) 8 (c) Book : Read
piece of work in 10 days. Then, (c) 64 (d) 0 (d) Latin : Language
3 men and 4 women can do the 62. If the amount received at the
same work in 70. Letter : Word
end of 2nd and 3rd year at
(a) 24 days (b) 5 days (a) Homework : School
compound interest on a certain
(c) 12 days (d) 18 days (b) Club : People
principal is ` 1800 and ` 1926,
respectively. What is the rate of (c) Product : Factory
54. The simple interest on a sum of (d) Page : Book
4 interest?
money is of the principal and (a) 7.5% (b) 7% 71. Find the odd word.
9
the number of years is equal to (c) 6% (d) 6.5% (a) Silver (b) Iron
the rate per cent per annum. The 63. The wheel of a motor car makes (c) Gold (d) Hydrogen
rate per annum is 1000 revolutions in moving 72. Find the odd word.
2 440 m. The diameter (in m) of (a) Bhagwad Geeta
(a) 5% (b) 6 %
3 the wheel is (b) Quran
1 (a) 0.44 (b) 0.14 (c) Ramayana
(c) 6% (d) 7 %
5 (c) 0.24 (d) 0.34 (d) Mahabharata
24 AFCAT ~ Practice Set 3

73. If ‘TYPEWRITER’ is written as 80. A statement followed by two Assumptions


GBKVDIRGVI, how ‘STENO’ conclusions is given. You have to I. The clerk may not obey Mr.
can be written in that code? take the given statement to be true A’s instructions.
(a) LMVGH (b) HGVML even, if it seems to be at variance II. The clerk may not inform his
(c) LMHGV (d) HVLGM from commonly known facts. Read late coming unless instructed.
the conclusions and then decide (a) Only Assumption I is implicit
74. In a code language, ‘FORGE’ is which of the conclusions, if any,
written as FPTJI. How will (b) Only Assumption II is implicit
logically follows from the given (c) Neither I nor II is implicit
‘CULPRIT’ be written in that code?
statement, disregarding the (d) Both I and II are implicit
(a) CVNSVNZ (b) CSJNPOT known facts.
(c) CVMQSTU (d) CXOSXYU Statement Directions (Q. Nos. 83-86) In each of
75. Rohan walks a distance of 3 km The national norm is 100 beds per the following questions, four figures are
towards North, then turns to his thousand populations but in this given. Three are similar in a certain way
left and walks for 2 km. He again state, 150 beds per thousand are and so form a group, find out which one
turns left and walks for 3 km. At available in the hospitals. of the figures does not belong to that group.
this point he turns to his left and Conclusions 83.
walks for 3 km. How many I. Our national norm is
kilometres is he from starting point? appropriate.
(a) 1 km (b) 2 km II. The state’s health system is (a) (b) (c) (d)
(c) 3 km (d) 5 km taking adequate care in this
76. Which of the following represents regard. 84.
Father, Women and Human? (a) Only Conclusion I follows
(b) Only Conclusion II follows
(c) Neither I nor II follows (a) (b) (c) (d)
(d) Both I and II follow
85.
(a) (b) (c) (d) 81. One statement followed by two
assumptions numbered I and II is
77. Which of the following Venn given. You have to consider the
diagrams correctly represents statement and the following (a) (b) (c) (d)
Planet, Earth and Sun? assumptions and decide which of
the assumptions is implicit in the 86.
statement.
Statement
“Our bank provides all your (a) (b) (c) (d)
(a) (b) (c) (d) banking requirements in one
location”— an advertisement of a Directions (Q. Nos. 87-89) In each of
78. Which of the following figures bank. the following questions, find the answer
represents, Degree students, BA Assumptions figure in which problem figure is
students and B.Sc. students? I. Customers prefer to carry out embedded.
all banking transactions at one 87. Problem Figure
place.
II. People may get attracted by the
(a) (b) (c) (d)
advertisement and carry out their
transactions with this bank.
79. Two statements are given (a) Only Assumption I is implicit Answer Figures
followed by two conclusions. (b) Only Assumption II is implicit
Read the conclusions and then (c) Neither I nor II is implicit
decide which of the conclusions, (d) Both I and II are implicit
if any, logically follows from the (a) (b) (c) (d)
two given statements, 82. One statement followed by two
disregarding the known facts. assumptions numbered I and II 88. Problem Figure
Statements All horses are dogs. is given. You have to consider the
All dogs are mice. statement and the following
Conclusions assumptions and decide which of
I. All horses are mice. the assumptions is implicit in the
II. All mice are dogs. statement. Answer Figures
(a) Only Conclusion I follows Statement
(b) Only Conclusion II follows ‘Give this packet to Mr. “X” at
(c) Neither I nor II follows his residence and return
immediately. In case you are
(d) Both Conclusions I and II follow (a) (b) (c) (d)
likely to be late inform me.
AFCAT ~ Practice Set 3 25

89. Problem Figure 92. Problem Figure (a) CS (b) TD


(c) PS (d) TA
96. Choose the correct code for the
uncoded term.
Answer Figures ?

Answer Figures

(a) (b) (c) (d)


AC BR GA BC AB ?
Directions (Q. Nos. 90-93) In each (a) BC (b) GA
of the following questions, a part (a) (b) (c) (d) (c) RG (d) AB
of question figure is missing. 97. Choose the correct code for the
Choose the correct alternative
93. Problem Figure
uncoded term.
that will complete the problem
figure. ?
90. Problem Figure
PR TS UP

? Answer Figures
SR ?
(a) UT (b) ST
(a) (b) (c) (d) (c) UR (d) SR
Answer Figures
Directions (Q. Nos. 98-100) In the
94. Choose the correct code for the following questions, choose the term
uncoded term. that will complete the series.
(a) (b) (c) (d) 98. AZ, CX, EV, ?
AP CR EP AT ER ? (a) GT (b) AC
91. Problem Figure (a) CR (b) CT (c) CP (d) AR (c) HT (d) GU

95. Choose the correct code for the 99. A, E, I, O, ?


uncoded term. (a) B
(b) T
(c) U
? (d) P
PA SD PC 100. PWX, QVY, RUZ, STA, ?
Answer Figures (a) TVT
(b) TSB
(c) SBT
TC SA ? (d) RST
(a) (b) (c) (d)

Answers
1 (a) 2 (b) 3 (c) 4 (d) 5 (b) 6 (a) 7 (c) 8 (b) 9 (c) 10 (a)
11 (d) 12 (b) 13 (c) 14 (c) 15 (a) 16 (d) 17 (d) 18 (b) 19 (c) 20 (b)
21 (b) 22 (c) 23 (b) 24 (b) 25 (a) 26 (b) 27 (c) 28 (a) 29 (c) 30 (d)
31 (b) 32 (a) 33 (b) 34 (d) 35 (b) 36 (a) 37 (a) 38 (c) 39 (b) 40 (d)
41 (d) 42 (a) 43 (a) 44 (a) 45 (b) 46 (a) 47 (c) 48 (b) 49 (d) 50 (b)
51 (b) 52 (b) 53 (b) 54 (b) 55 (d) 56 (a) 57 (c) 58 (c) 59 (a) 60 (a)
61 (c) 62 (b) 63 (b) 64 (b) 65 (b) 66 (a) 67 (b) 68 (d) 69 (d) 70 (d)
71 (d) 72 (b) 73 (b) 74 (a) 75 (a) 76 (d) 77 (c) 78 (b) 79 (a) 80 (b)
81 (d) 82 (b) 83 (c) 84 (a) 85 (d) 86 (b) 87 (a) 88 (a) 89 (a) 90 (d)
91 (d) 92 (c) 93 (d) 94 (b) 95 (b) 96 (c) 97 (a) 98 (a) 99 (c) 100 (b)
26 AFCAT ~ Practice Set 3

Hints and Solutions


1. (a) According to the author, people think 20. (b) ‘Fanatic’ is a person exhibiting 31. (b) The ‘Nagoya Protocol’ on Access and
that real progress lies in material success excessive enthusiasm especially in Benefit-sharing is a global agreement that
and technological growth because people religion or politics. implements the access and benefit-sharing
become fascinated by machines and 21. (b) The correctly spelt word is ‘adversity’ obligations of the convention on Biological
techniques they use. which means ‘a difficult or unpleasant Diversity. It was adopted in Nagoya,
2. (b) According to the author science and situation’. Japan in October 2010, after six years of
technology should be used in a negotiations.
22. (c) The correctly spelt word is ‘conciliatory’
controlled and careful manner. which means ‘behaviour or actions to end 32. (a) Ultrasound waves are the sound
3. (c) Eastern people are fascinated by a disagreement with someone.’ waves with frequencies higher than the
machines and techniques and they have upper audible limit of human hearing
23. (b) The correctly spelt word is
build huge industrial organisations, but it range i.e. above 20 kHz. In general,
‘accommodation’ which means ‘a place
is suggested with this passage that they humans can hear sounds with a
to live or stay, especially on a holiday’.
must avoid being controlled by machines frequency between 20 Hz and 20 kHz.
24. (b) The correctly spelt word is
and techniques of industrial production. 33. (b) Soot, Graphite and Charcoal all are
‘Kleptomania’ which is an addiction to
4. (d) According to the author, science and forms of carbon but, Hematite is the oxide
steal.
technology are useful, if they are not of iron. It is a red, gray or black mineral.
25. (a) The correctly spelt word is ‘Schedule’ It is found in sedimentary, metamorphic
worshipped blindly.
and it means ‘a list of thing planned to and igneous rocks. It is one of the chief
5. (b) vast happen at a particular time’. sources of iron.
6. (a) around 26. (b) Balaji Baji Rao, popularly known as 34. (d) Vitamin-B 6 or Pyridoxal Phosphate (PLP)
7. (c) oasis ‘Nana Saheb’, was a Peshwa of the
8. (b) search is a B-complex vitamin. The deficiency of this
Maratha empire in India. He was the son
vitamin may lead to the anaemia. The other
9. (c) camel of Baji Rao who succeeded his father at
symptoms are depression, dermatitis and
10. (a) ship the age of 20. He contributed heavily to
high blood pressure.
11. (d) ‘Mordant’ means ‘very sarcastic and the development of the city of Pune,
India. Nana Saheb lost his cousin, 35. (b) Nehru Cup is associated with football.
painful wit or speech’. Hence, option (d)
Sadashivrao Bhau and his eldest son, The Nehru Cup is an International
‘sarcastic’ is nearest in meaning to the
Vishwasrao at the Third Battle of Association Football Tournament
word ‘mordant’.
Panipat. organised by the All India Football
12. (b) ‘Pragmatic’ means ‘of or relating to a Federation (AIFF). It was launched in 1982.
practical point of view or practical 27. (c) English as a medium of instruction
was introduced in India by the English 36. (a) The inaugural Cricket World Cup was
considerations’ and ‘realistic’ means
Education Act, 1835. On Lord held in 1975 in England. England was
‘able to see things as they really are and
Macaulay’s recommendations, English the only nation able to put forward the
to deal with them in a practical way’. So,
was made the medium of higher resources to stage an event of such
option (b) ‘realistic’ is nearest in meaning
education by the then Governor General magnitude at that time. The first Cricket
to ‘pragmatic’.
of India, Lord William Bentinck in 1835. World Cup was won by West Indies.
13. (c) ‘Minuscule’ means ‘extremely small’
and ‘massive’ means ‘very large’. So, 28. (a) Sonal Mansingh is an eminent Indian 37. (a) The Balwant Rai Mehta Committee
option (c) ‘massive’ is opposite in classical dancer and choreographer was a committee appointed by the
meaning to ‘minuscule’. Government of India in 1957. The
renowned for her Odishi dancing style.
function of this Committee was to
14. (c) ‘Wary’ means ‘feeling or showing She is also proficient in Bharatanatyam,
examine the working of the community
caution about possible dangers or Kuchipudi and Chhau. She has received
Development Programme (1952) and the
problems.’ So, option (c) ‘careless’ is various awards including Sangeet Natak
National Extension Service (1953).
opposite in meaning to ‘wary’. Akademi Award in 1987 and the Padma
15. (a) ‘Transience’ means ‘the state or fact Vibhusan in 2003. The committee submitted its report in
of lasting only for a short time’ and November, 1957 and recommended for
29. (c) Bhutan is known as the ‘Land of introduction of a three-tier Panchayati Raj
‘eternity’ means ‘lasting or existing
Thunderbolt’ because of the violent and system in India which includes Gram
forever’. Hence, option (a) ‘eternity’ is
large thunderstorms that whip Panchayat at village level, Panchayat
opposite in meaning to ‘transience’.
down through the valleys from the Samiti at the block level and Zila Parishad
16. (d) Anything written in a letter after it is Himalayas.
signed is called ‘Postscript’. at the district level.
30. (d) Wheat is a Rabi crop. The crop 38. (c) EDUSAT or GSAT-3 is a communication
17. (d) ‘Anomaly’ is a deviation from the
requires moderate temperature and satellite which was launched in 2004 by
common rule, type, arrangement or form.
moderate rainfall during growing season Indian Space Research Organisation (ISRO).
18. (b) ‘Pedantic’ is a style in which a writer and bright sunshine at the time of
makes display of his knowledge. EDUSAT is world’s first satellite built
harvest. The two important wheat exclusively to serve educational sector.
19. (c) ‘Bottleneck’ is a situation that stops a growing zone are Ganga and Black soil
process or an activity from progressing. It has revolutionised classroom teaching
region of Deccan.
through IP based technology.
AFCAT ~ Practice Set 3 27

39. (b) Prithvi missile is India’s first Called the ‘Grasp Man’, this new class of (H + 6) + (W + 6) + (S + 6)
indigenously developed ballistic missile. robot can be used for industrial purposes, + (D + 6) + 5
Now, = 36
The Prithvi missile is a family of tactical search and rescue operations and other 5
surface to surface Short-Range Ballistic applications that involve climbing, ⇒ (H + W + S) + D + 29 = 180
Missiles (SRBM). It was first test-fired on holding and assembling. ⇒ 126 + D + 29 = 180
25th February, 1988 from Sriharikota, 48. (b) GIS means Geographic Information ⇒ D = 180 − 155 = 25 yr
SHAR Centre, Potti Sreeramulu Nellore, System. It is a computer system designed 53. (b) 6 men and 8 women can do a work
Andhra Pradesh. It has a range of upto to capture, store, manipulate, analyse, in 10 days.
150 to 350 km. manage and present all types of spatial Let the efficiency of each man be ‘m’ and
40. (d) The Food and Agriculture or geographical data. GIS applications each woman be ‘w’.
Organisation of the UN (FAO) is a are used in a number of fields.
∴ 6m + 8 w = 10
specialised agency that leads 49. (d) The resources which are obtained
or 2(3m + 4 w) = 10
international efforts to defeat hunger. from biosphere and have life are biotic
or 3m + 4 w = 5
It serves both developed and developing resources. Biotic resources also known as
countries. The headquarters of FAO is living resources, are renewable resources ∴3 men and 4 women will complete the
including plants and animals. work in 5 days.
located in Rome, Italy.
4
41. (d) National Youth Day is celebrated Abiotic resources are non-living 54. (b) Given, SI = P, R = T
resources that fall into the larger category 9
every year on 12th January to PRT
of natural resources, which occurs SI =
commemorate the birth anniversary of ∵
naturally in the environment and are not 100
Swami Vivekananda. 4 P× R× R
created by humans or human activity. ∴ P=
The day is observed all over India at Such as soil, water, minerals. 9 100
schools and colleges with processions, A renewable resource is an organic 400
speeches, music, seminars, recitations etc. ∴ R =
2
natural resource which can replenish to 9
42. (a) International Criminal Police overcome usage and consumption, 400 20 2
through naturally recurring processes. or R= = =6 %
Organisation or INTERPOL is an 9 3 3
intergovernmental organisation facilitating Potential resources are resources which
exist in a region and which can be used 55. (d) Given, speed of train = 60 km/h
international police cooperation and crime
5 50
control. It was established in 1923. in the future, these are not being used at = 60 × = m/s
present. 18 3
43. (a) Former Governor of RBI, Bimal Jalan
50. (b) Total internal reflection occur when a ∴Length of train = Speed × Time
is the author of the book ‘The Future of 50
India’. wave strikes from denser to lighter = × 30 = 500 m
medium at an angle larger than the 3
The book focuses on 3 forces-politics,  1 1 1  9 − 4 
critical angle with respect to the normal 56. (a) 1 ÷ + + ÷
economy and governance, which will
surface.  2 3 6  12  
jointly determine future of India.
Total internal reflection cannot occur
=1÷  + + ÷ 
1 1 1 5
44. (a) On 1st May, 2019 the UK Parliament when light goes from water (n = 1.33) a  2 3 6 12 
by passing a motion declared a climate lighter medium to glass (n = 1.5) a
=1÷  + + ×
emergency, making the UK, the first denser medium. 1 1 1 12 
country to officially declare a climate  2 3 6 5 
51. (b) Let the CP of cycle be ` x, then
=1÷  + + 
emergency. 1 1 2
(100 + profit%) 108 x
45. (b) Astronaut in outer space will observe SP = CP =`  2 3 5
100 100
15 + 10 + 12 
=1÷ 
sky as black because there are no
According to the question,
substances or objects to reflect back any 108x 114x  30 
colour. The light from the sun is reflected + 75 =
100 100 37 30 30
back as the blues, greens and browns as =1÷ =1× =
⇒ 108x + 7500 = 114x 30 37 37
we see.
⇒ 114x − 108x = 7500 57. (c) Let the third proportional be x.
46. (a) Lemru Elephant Reserve of ⇒ 6x = 7500 ∴ 3: 4:: 4: x
Chhattisgarh will be one of its kind
∴ x=
7500
= ` 1250 3 4 4 × 4 16
elephant reserve in the world. It will not ⇒ = ⇒ x= =
6 4 x 3 3
only serve as permanent habitat for wild
elephants but also prevent damage 52. (b) Let the ages of husband, wife, son 58. (c) According to the question,
caused by the movement of elephants. It and daughter-in-law at the time of son’s Discount = MP − SP
will contribute towards saving wild life marriage be denoted by H, W, S and D, = 6000 − 5500
respectively.
and preserving of biodiversity in the = ` 500
H+ W+ S
region. Given, = 42 yr If discount = x%, then
3 6000 × x
47. (c) Researchers at the Indian Institute of = 500
or H + W + S = 126 yr
Technology, Madras (IIT MADRAS) have 100
When child’s age is 5 yr, age of each of 500 25 1
developed a robot with grasping and ⇒ x= = =8 %
the other family members has increased
locomotion abilities like a human hand. 60 3 3
by 6 yr.
28 AFCAT ~ Practice Set 3

59. (a) LCM of 10, 15, 21 and 28 63. (b) Distance covered by wheel in the Therefore,
2 10, 15, 21, 28 revolution CU L P R I T C VN S V N Z
Total distance
2 5, 15, 21, 14 = +0
Total number of revolutions +1
3 5, 15, 21, 7 440 +2
= = 0.44 m +3
5 5, 5, 7, 7 1000 +4
7 1, 1, 7, 7 Now, revolution distance +5
= Circumference +6
1, 1, 1, 1 .
044 .
044
0.44 = 2πr ⇒ 2r = = ×7 75. (a) According to the question, the
∴LCM = 2 × 2 × 3 × 5 × 7 = 420 π 22
direction diagram is as follows,
Greatest number of 4-digits = 9999 Diameter, 2r = 0.14 m
C 2 km B
9999 339
Now, = 23 64. (b) Such number is always divisible by 9.
420 420 To make it clear, we can take some N
3 km 3 km
∴ Remainder = 339 example. End W E
point
∴4-digit number divisible by 10, 15, 21 e.g., 496 − (4 + 9 + 6) = 477,
D A S
and 28 = 9999 − 339 = 9660 which is divisible by 9. 3 km
E

Here, k = 10 − 4 = 15 − 9 = 21 − 15 971 − (9 + 7 + 1) = 954


Starting point

= 28 − 22 = 6 which is divisible by 9. ∴Required distance,


∴Required number 65. (b) Let the numerator of the fraction = x AE = DE – DA
= (9660 − k) = 9660 − 6 = 9654 = (3 − 2) km = 1 km
Then, denominator of fraction = 13 − x
60. (a) Let the required number be x, then (∵ DA = BC = 2 km)
According to the question,
by given condition, x +1 1 76. (d)
3 = ⇒ 2(x + 1) = (11 − x)
60% of x = x − 24 13 − x − 2 2 Human
4
60 3x ⇒ 2x + 2 = 11 − x ⇒ 3x = 9 ⇒ x = 3
⇒ ×x= − 24
100 4 ∴Original fraction Father Women
 3x − 3x  = 24 x 3 3
⇒ = = =
  13 − x 13 − 3 10
 4 5
⇒ 15x − 12x = 480 ∴ x = 160 66. (a) As ‘Emollient’ is used to ‘Soothe’ the
skin, similarly a ‘Dynamo’ serves to Father and women are different from each
61. (c) (x − 4) (x 2 + 4x + 16) = x 3 − P other but both are human.
‘Generate’ electricity.
⇒ x 3 + 4x 2 + 16x 77. (c)
67. (b) First indicates the lack of second.
− 4x 2 − 16x − 64 = x 3 − P Planet
68. (d) As ‘Poetry’ originates from
⇒ x − 64 = x − P
3 3
‘Inspiration’, similarly ‘Thought’ Earth
On comparing both sides, we get originates from ‘Mind’.
⇒ P = 64
69. (d) ‘Hockey’ is a ‘Game’ and ‘Latin’ is a Sun
62. (b) Let principal = ` P and rate = r% ‘Language’.
∴Amount received at the end of 2nd year 70. (d) ‘Letter’ is a part of ‘Word’ and in the Earth is a planet, but Sun is entirely
= ` 1800 same way, ‘Page’ is a part of ‘Book’. different.
T
Amount = P1 +
R  71. (d) All except ‘Hydrogen’, are metals.
∵ 
 100 72. (b) All except ‘Quran’, are religious epics 78. (b)
2 Degree students
1800 = P1 +
r  of Hindus.
∴  ...(i)
 100 73. (b) The code has been generated by BA B.Sc.
Amount received at the end of 3rd year taking opposite letters. students students
= ` 1926 As,
3
1926 = P1 +
r  T Y P E W R I T E R
∴  ...(ii) Opposite Both BA students and B.Sc. students
 100
Letters belong to the category of degree students,
On dividing Eq. (ii) by Eq. (i), we get G B K V D I R G V I but both are different from each other.
3
P1 +
r  Therefore, S T E N O 79. (a)

1926  100
= 2
1800
P1 +
r  H G V M L

 100 Horses
1926 r 74. (a) As,
⇒ =1+ F O R G E F P T J I
1800 100 Dogs
+0
126 r Mice
⇒ = +1
+2
1800 100 +3 Conclusions I. (P) II. (×)
∴ r = 7% +4
AFCAT ~ Practice Set 3 29

80. (b) More number of beds per thousand 91. (d) Here, all the three parts have exactly 94. (b) The codes can be decoded as
population are available in the state same design. So, the fourth part with
indicates that the state’s health system is question mark (?) must also contain the A, E, C
taking adequate care in this regard. So, same design to complete the question
Conclusion II follows. Whether the national figure as shown in the following figure.
P, R and T
norm is appropriate or not cannot be
said. So, Conclusion I does not follow.
81. (d) Both Assumptions I and II are implicit. From the above codes, we see that the
It is reasonable to think that the customers code for the last symbol i.e. is CT,
prefer to carry out all banking transactions where C specifies shape and T specifies
at one place. Also, advertisement is meant its shade.
Now, it is clear from the above figure that
to attract more customers. Hence, option (b) is correct.
answer figure (d) will complete the
82. (b) Assumption I is not implicit. The clerk problem figure. 95. (b) The codes can be decoded as
should be obedient to his manager.
92. (c) Here, all the three equal parts have
Assumption II is implicit, because until P, S, T,
similar design. So, we can obtain the
clerk is not asked to do a job, he is not
answer figure for the missing portion by A, D and C
responsible for that job.
rotating the original given figure by 90°
83. (c) Except figure (c), in all other figures, there clockwise. From the above codes, we see that the
are three shapes which are not identical. code for the last term is TD, where T
84. (a) In all other figures, shaded triangle 90° clockwise specifies the shape of line and D
and line segment attached to straight line rotation specifies the symbol at the ends of lines.
lie on the same side.
? ? Hence, option (b) is correct.
85. (d) In all the other figures, no small circle 96. (c) The codes can be decoded as
is in between the parallel lines. (X) (X′)
A, C, B,
86. (b) Except in figure (b), all the other Now, by comparing figure (X) with (X′),
figures comprise of 6 line segments. we get
87. (a) The problem figure is embedded in G and R
answer figure (a). ? =
From the above codes, we see that
Now, it is clear that answer figure (c) code for the last shape is RG, where R
completes the problem figure. specifies the upper shape and G specifies
88. (a) The problem figure is embedded in the lower shape.
answer figure (a). 93. (d) Here, all the three equal parts have
Hence, option (c) is correct.
similar design. So, we can obtain the
answer figure for the missing portion by 97. (a) The codes can be decoded as
rotating the question figure by 90° P, R, S,
clockwise.
89. (a) The problem figure is embedded in T and U
answer figure (a).
+

90° clockwise From the above codes, we see that the


+ ? rotation
+

code for the last shape is UT, where U


+ specifies the inner shape and T specifies
?
+

+
the outer shape.
(X) (X′) Hence, option (a) is correct.
90. (d) Here, all the three parts have exactly 98. (a) –2 –2 –2
same design. So, the fourth part with Now, by comparing figure (X) with (X′),
question mark (?) must also contain the we get
A Z C X E V G T
same design to complete the
+

=
question figure as shown in the following ? +2 +2 +2
figure
99. (c) The given series consists of continuous
Now, it is clear that answer figure (d)
vowels.
completes the problem figure, which looks
as shown in the below figure. Hence, the missing term is U.
100.(b) P +1 +1 +1 +1
Q R S T
+

–1 –1 –1 –1
+

W V U T S
+

Now, it is clear from the above figure that +1 +1 +1 +1


+

X Y Z A B
answer figure (d) will complete the
problem figure.
30 AFCAT ~ Practice Set 4

AFCAT
AIR FORCE COMMON ADMISSION TEST (ONLINE )

Practice Set ~04


Time : 2 Hrs MM : 300

INSTRUCTIONS
1. The set contains a total of 100 questions, Comprising Verbal Ability in English, General Awareness, Numerical Ability and Reasoning and
Military Aptitude Test.
2. Each correct question carry 3 Marks and there will be negative marking of 1 Mark for each incorrect attempt.
3. Total time duration will be 2 hrs (120 minutes).
4. No marks will be deducted for unattempted questions.

Directions (Q. Nos. 1-5) In the 5. (a) destroy (b) suffocate 7. The proper background of early
following passage, some of the words (c) damage (d) injure education is
have been left out. Read the passage (a) a school (b) a college
carefully and choose the correct answer Directions (Q. Nos. 6-10) Read the (c) a religious institution
to each blank out of the four following passage carefully and answer (d) the home
alternatives. the questions that follows.
8. From the passage, we get an
Many parents greet their children’s When we talk of education in our impression that the highest honour
years with needless dread. While teens present age, we think largely in terms is earned by
(1) assault us with heavy-metal music, of schools and colleges. The man who (a) a man who has received education
flaunt outlandish clothes and spend is well-to-do spends money in sending in a foreign country
(2) time with friends, such behaviour his son to foreign lands, in the belief (b) a man who has struggled against
always adds up to full-scale revolt. that some wonderful process will take adversity
Teenage (3) according to psychologist place there transforming a dull fellow (c) a man who has seen prosperity
into a genius. Yet, the products of alone
Laurence Steinberg, has been greatly (d) the son of a prosperous man
exaggerated, Sociologist Santord expensive schools and universities
Dornbusch agrees, “The idea that often fail to make good. On the other 9. A well-to-do man sends his son to
teenagers inevitably rebel is a reality hand, the poor man who has struggled foreign lands
that has the potential for great family against adversity often earns the (a) because it is the fashion of the day
(4)’’, says Dornbusch. He believes the highest honour. The fact is that the (b) in the belief that his dull son will
true background of early education is be transformed into a genius
notion can (5) communication during
the home. The home, the influence of (c) so that the son may learn the
this critical time for parents to customs of those countries
influence youngsters. the mother, the inspiring examples
(d) in order to make his son familiar
that are held before the child at an age
1. (a) can (b) must while he is impressionable are the true
with the persons and places of
(c) may (d) should those countries
groundwork of character.
2. (a) her (b) his 10. The expression ‘the products of
6. According to the passage, who expensive schools and universities
(c) their (d) our
helps in our character building? often fail to make good’ means
3. (a) rebellion (b) subversion (a) A foreign university (a) they fail to make a mark in life
(c) mania (d) revolution (b) A well-to-do man (b) they fail to become intelligent
4. (a) ruin (b) downfall (c) Examples that inspire (c) they fail to earn proper living
(c) harm (d) defeat (d) A man who has earned honour (d) they do not earn good reputation
AFCAT ~ Practice Set 4 31

Directions (Q. Nos. 11-15) In each of 25. Logical 35. What is the name of the scheme
the following words, choose the correctly (a) Responsive (b) Rational launched by the Government of
spelt word. (c) Educated (d) Improper India with an aim to reduce
11. (a) Inimitable (b) Innimatible preventable maternal and new
26. Who among the following rulers born deaths in India?
(c) Inimitible (d) Inimmitable were the first to introduce the
(a) MAATRITVA (b) JANANI
12. (a) Pitiaeble (b) Pitiable gold coinage in India?
(c) SHISHU (d) SUMAN
(c) Pitiabale (d) Pitiebale (a) The Kushanas (b) The Cholas
13. (a) Occurrance (b) Occurrence (c) The Pandyas (d) The Cheras 36. UPSC is established under which
(c) Occurance (d) Occurence Article of the Constitution of
27. Which among the following
India?
14. (a) Argumant (b) Arguemant Mughal emperors, executed the
(a) 315 (b) 316
(c) Argument (d) Arguement fifth Sikh Guru, Guru Arjan Singh?
(c) 317 (d) 318
(a) Akbar (b) Humayun
15. (a) Commission (b) Commision
(c) Commision (d) Comission (c) Jahangir (d) Shahjahan 37. Under which Article of the
Constitution of India, there is a
Directions (Q. Nos. 16-20) In the 28. Which among the following provision for the impeachment of
following questions, out of the four Governor-Generals of India, called
the President of India?
alternatives, choose the one which can Social Reformer of British India?
(a) Article 60 (b) Article 51 A
be substituted for the given (a) Lord William Bentinck
(c) Article 72 (d) Article 61
words/sentence. (b) Lord Metcalfe
(c) Lord Wellesley 38. What does the word ‘electronic’
16. A house for storing grains imply in the terms e-Governance?
(d) Lord Minto
(a) Cellar (b) Store (a) Excellence driven Governance
(c) Godown (d) Granary 29. The Big Bang Theory is related to
(b) Safeguarded Governance
which one of the universal
17. A name adopted by an author in (c) Technology driven Governance
formations?
his writings (d) None of the above
(a) Lunar Eclipse
(a) Title (b) Nomenclature
(b) Solar Eclipse 39. Joint Military Exercise ‘Vajra
(c) Nickname (d) Pseudonym
(c) Formation of meteors Prahar’ is between which of the
18. Of unknown and unadmitted (d) Formation of galaxies of stars following countries?
authorship (a) India-US
(a) Gullible (b) Anonymous 30. India’s most active volcano is
(b) India-Japan
(c) Unanimous (d) Vexation situated in which part of the
(c) India-UAE
country?
19. A person who is made to bear the (a) Lakshadweep (b) Barren Island
(d) India-Australia
blame due to others 40. Which country’s Prime Minister
(c) Indira Point (d) Kanyakumari
(a) Innocent (b) Scapegoat is awarded with Nobel Peace Prize
(c) Ignoramus (d) Nincompoop 31. Which among the following is the for the year 2019?
normal temperature of a human
20. A person who brings goods body?
(a) Kenya (b) Tanzania
illegally into the country (c) Ethiopia (d) Uganda
(a) 37°C or 98.6°F
(a) Exporter (b) Importer 41. Indian Air Force celebrates its
(b) 39°C or 99.5°F
(c) Smuggler (d) Imposter foundation day on which day?
(c) 37°C or 98.7°F
Directions (Q. Nos. 21-23) In these (d) 39°C or 98.4°F (a) 6th October (b) 8th October
questions, choose the word opposite in (c) 10th October (d) 12th October
32. Which one is not the example
meaning to the given word. of non-renewable natural 42. The museum for all former Prime
21. Efficacious resources? Ministers of India will be
(a) Productive (b) Ineffective (a) Mineral (b) Coal constructed in which city?
(c) Improper (d) Urgent (c) Natural Gas (d) Air (a) Lucknow (b) New Delhi
22. Outrageous (c) Kolkata (d) Mumbai
33. Which symptoms are found
(a) Justifiable (b) Lusty largely in the patient of Ebola 43. Which salt is used as a raw material
(c) Jolly (d) Wicked Virus Disease? for making a large number of useful
23. Garrulous (a) Haemorrhagic fever chemicals in industry?
(a) Verbose (b) Serious (b) Muscle pain and internal and (a) Sodium Chloride
(c) Gaunt (d) Reticent external bleeding (b) Ammonium Hydroxide
(c) Kidney and liver dysfunction (c) Ammonium Chloride
Directions (Q. Nos. 24 and 25) In
(d) All of the above (d) Sodium Nitrate
these questions, choose the word most
similar in meaning to the given word. 34. Which of the following is the 44. Which planet in our solar system
fastest process of heat transfer? has the highest number of Moons?
24. Mastery
(a) Mystery (b) Weighty (a) conduction (b) convection (a) Jupiter (b) Uranus
(c) Authority (d) Weakness (c) radiation (d) insolation (c) Neptune (d) Saturn
32 AFCAT ~ Practice Set 4

45. Golden Revolution is related to 54. A and B are two alloys of gold 63. What is the greatest number which
(a) food grain production and copper prepared by mixing will divide 110 and 128 leaving a
(b) milk production metals in the ratio 5 : 3 and 5 : 11, remainder 2 in each case?
(c) honey and horticulture respectively. Equal quantities of (a) 8 (b) 18 (c) 28 (d) 38
production these alloys are melted to form a
64. The radii of two circles are 5 cm
(d) fish production third alloy C. The ratio of gold
and 12 cm. The area of a third
and copper in the alloy C is
46. Siachen Glacier, the highest battle circle is equal to the sum of the
(a) 25 : 33 (b) 33 : 25 areas of the two circles. The radius
field is in which mountain range?
(c)15 : 17 (d)17 : 15 of the third circle is
(a) Ladakh (b) Jaskar
(c) Karakoram (d) Kailash 55. What per cent of 5 L is 400 mL? (a) 13 cm (b) 21 cm
(a) 10 (b) 8 (c) 12 (d) 6 (c) 30 cm (d) 17 cm
47. Which country launched the
world’s first satellite dedicated to 56. In a class, the average score of 65. If n is even, (6 n − 1) is divisible by
monitoring Greenhouse Gas girls in an examination is 73 and (a) 37 (b) 35 (c) 30 (d) 6
emission? that of boys is 71. The average
66. Which of the following best
(a) Japan (b) Brazil score for the whole class is 71.8.
represents the relation among
(c) India (d) USA Find the percentage of girls.
Tall man, Black haired people and
(a) 40% (b) 50% Indians?
48. The value of (c) 55% (d) 60%
 5 6  5 1 
 6 × 113  ÷ 2 7 ÷ 3 4   is 57. Cost of a CD was reduced by
 
25%. A person paid ` 24 for the
24
(a) (b) 1 CD. What was the cash discount? (a) (b) (c) (d)
35
(a) ` 25 (b) ` 6
35 91 67. Which of the following best
(c) (d) (c) ` 1 (d) ` 8
24 76 represents the relation among
1 58. A fruit seller buys lemons at 2 for Poison, Arsenic and Cyanide?
49. If x + = 2 and x is real, then the
` 1 and sells them at 5 for ` 3.
x
1 His profit per cent is
value of x17 + 19 is
x (a) 10 (b) 15
(a) 1 (b) 0 (c) 20 (d) 25
(a) (b) (c) (d)
(c) 2 (d) − 2 59. The average age of a husband and
a wife was 27 yr, when they 68. Which of the following diagrams
50. Simple interest on ` 500 for 4 yr best depicts the relationship
at 6.25% per annum is equal to married 4 yr ago. The average age
of the husband, the wife and a among Reptiles, Monkey, Turtles?
the simple interest on ` 400 at
5% per annum for a certain new born child is 21 yr now. The
period of time. The period of present age of the child is
time is (a) 4 yr (b) 3 yr
(c) 2 yr (d) 1 yr (a) (b) (c) (d)
(a) 4 yr (b) 5 yr
1 2 60. A water tank has two pipes. The 69. Which of the following diagram
(c) 6 yr (d) 8 yr
4 3 empty tank is filled in 12 min by indicates the best relation
51. A man rows 40 km upstream in the 1st and the full tank is among Teacher, Poet and
8 h and a distance of 36 km emptied by the 2nd in 20 min. Graduate?
downstream in 6 h. Then, speed The time required to fill the 1/2
of the stream is full tank when both pipes are in
action, is
(a) 0.5 km/h (b) 1.5 km/h
(a) 16 min (b) 15 min (a) (b) (c) (d)
(c) 1 km/h (d) 3 km/h
(c) 20 min (d) 30 min
52. Three numbers are in the ratio 2 3 1 70. Which of the following indicates
1 : 2 : 3 and the sum of their cubes 61. th of th of th of 26000 is the best relation among Parrots,
5 4 4
is 4500. The smallest number is equal to Birds, Mice?
(a) 4 (b) 5 (a) 1900 (b) 1950
(c) 6 (d) 10 (c) 1980 (d) 1930
53. A certain distance is covered by a 62. A and B together do a job in (a) (b) (c) (d)
cyclist. If a jogger covers half the 12 days and A could do the job in
distance in double the time, the 20 days if he worked alone. How Directions (Q. Nos. 71-75) In the
ratio of the speed of the jogger to many days would B take to do the following questions, three alternatives
that of the cyclist is job if he worked alone? out of four are same in a certain way
(a)1 : 4 (b) 4 : 1 (a) 30 days (b) 25 days and so form a group. Find the odd word
(c)1 : 2 (d) 2 : 1 (c) 24 days (d) 15 days that does not belong to the group.
AFCAT ~ Practice Set 4 33

71. (a) Tomato (b) Potato Answer Figures Directions (Q. Nos. 86-90) In each of
(c) Rice (d) Rose the following questions, four figures are
72. (a) Crow (b) Vulture given. Three are similar in a certain
(c) Sparrow (d) Bat way and so form a group. Find out
which one of the figures does not belong
73. (a) Graph (b) Chart
(a) (b) (c) (d) to that group.
(c) Model (d) Drawing
86.
74. (a) Mattress (b) Pillow 82. Problem Figure
(c) Bed sheet (d) Curtain
75. (a) Needle (b) Pencil
(c) Spade (d) Candle (a) (b) (c) (d)

Directions (Q. Nos. 76-80) The 87.


?
following questions consist of two words
each, that have a certain relationship
with each other, followed by four Answer Figures (a) (b) (c) (d)
lettered pairs of words. Select the letter
pair that has the same relationship as 88.
the original (or given) pair of words.
76. Milk : Cream (a) (b) (c) (d) (a) (b) (c) (d)
(a) College : Students
(b) Sugar : Sweet 83. Problem Figure 89.
(c) Clay : Pottery
(d) Fruit : Glucose
77. Loathe : Coercion (a) (b) (c) (d)
(a) Detest : Caressing 90.
(b) Irritate : Caressing ?
(c) Irate : Antagonism
(d) Reluctant : Persuasion Answer Figures
(a) (b) (c) (d)
78. Straws : Nest
(a) Water : Stream Directions (Q. Nos. 91-95) Each of
(b) Animals : Zoo the following questions, consist of two
(a) (b) (c) (d)
(c) Threads : Cloth sets of figures (1), (2), (3) and (4)
(d) Wood : Paper 84. Problem Figure constitute the problem set while figures
(a), (b), (c) and (d) constitute the answer
79. Umpire : Game
set. There is a definite relationship
(a) Prodigy : Wonder
between figures (1) and (2) establish a
(b) Chef : Banquet
similar relationship between figures (3)
(c) Legislator : Election
? and (4) by selecting a suitable figure
(d) Moderator : Debate from the answer set that would replace
80. Fly : Walk Answer Figures the question mark (?) is figure (4).
(a) Sit : Sleep 91. Problem Figures
(b) Roast : Bake
(c) Sky : Earth
(d) Pilot : Captain ?
(a) (b) (c) (d)
Directions (Q. Nos. 81-85) In each of (1) (2) (3) (4)
the following questions, a part of problem 85. Problem Figure
Answer Figures
figure is missing. Find out from the given
answer figures (a), (b), (c) and (d), that
can replace the ‘?’ to complete the
problem figure.
81. Problem Figure ? (a) (b) (c) (d)
92. Problem Figures
Answer Figures

? ?
(1) (2) (3) (4)
(a) (b) (c) (d)
34 AFCAT ~ Practice Set 4

Answer Figures Answer Figures 98. Problem Figures

(a) (b) (c) (d) (a) (b) (c) (d) (1) (2) (3) (4) (5)

93. Problem Figures Answer Figures


Directions (Q. Nos. 96-100) In each
of the following questions, select a
? figure from the answer figures which
will continue the series as established
(1) (2) (3) (4) by the five problem figures. (a) (b) (c) (d)
Answer Figures 96. Problem Figures 99. Problem Figures
S

(1) (2) (3) (4) (5)


(a) (b) (c) (d) (1) (2) (3) (4) (5)
Answer Figures
94. Problem Figures Answer Figures
+
#
?
(1) (2) (3) (4) (a) (b) (c) (d)
(a) (b) (c) (d)
Answer Figures 97. Problem Figures
100. Problem Figures
# # #
+ + +
+
(a) (b) (c) (d) (1) (2) (3) (4) (5)
(1) (2) (3) (4) (5)
95. Problem Figures Answer Figures
Answer Figures

?
(1) (2) (3) (4) (a) (b) (c) (d)
(a) (b) (c) (d)

Answers
1 (c) 2 (c) 3 (a) 4 (a) 5 (b) 6 (c) 7 (d) 8 (b) 9 (b) 10 (a)
11 (a) 12 (b) 13 (b) 14 (c) 15 (a) 16 (d) 17 (d) 18 (b) 19 (b) 20 (c)
21 (b) 22 (a) 23 (d) 24 (c) 25 (b) 26 (a) 27 (c) 28 (a) 29 (d) 30 (b)
31 (a) 32 (d) 33 (d) 34 (c) 35 (d) 36 (a) 37 (d) 38 (c) 39 (a) 40 (c)
41 (b) 42 (b) 43 (a) 44 (d) 45 (c) 46 (c) 47 (a) 48 (c) 49 (c) 50 (c)
51 (a) 52 (b) 53 (a) 54 (c) 55 (b) 56 (a) 57 (d) 58 (c) 59 (d) 60 (b)
61 (b) 62 (a) 63 (b) 64 (a) 65 (b) 66 (a) 67 (b) 68 (c) 69 (b) 70 (d)
71 (d) 72 (d) 73 (c) 74 (d) 75 (d) 76 (c) 77 (c) 78 (c) 79 (c) 80 (c)
81 (d) 82 (a) 83 (c) 84 (c) 85 (d) 86 (a) 87 (c) 88 (a) 89 (d) 90 (c)
91 (a) 92 (c) 93 (b) 94 (b) 95 (a) 96 (b) 97 (d) 98 (a) 99 (b) 100 (d)
AFCAT ~ Practice Set 4 35

Hints and Solutions


1. (c) may 23. (d) ‘Garrulous’ means ‘excessively 33. (d) Generally, Ebola virus disease patient
2. (c) their talkative, especially on trivial matters’ complains about muscle pain,
3. (a) rebellion and ‘Reticent’ means ‘inclined to be haemorrhagic fever, headache, sore
silent or unwilling to speak about one’s throat, diarrhoea, kidney and liver
4. (a) ruin thought’. So, ‘reticent’ is opposite in dysfunction and internal and external
5. (b) suffocate meaning to ‘garrulous’. bleeding.
6. (c) According to the passage, inspiring 24. (c) ‘Mastery’ means ‘control or superiority And to cure the disease, officially no
examples helps in building the child’s over someone or something’ and ‘Authority’ licensed vaccines are available, only
character. means ‘the power to control, give order or immune therapies are conducted
7. (d) Proper and true background of early make decisions’. So, option (c) ‘authority’ currently.
education is the home. is similar in meaning to ‘mastery’. 34. (c) Radiation is the fastest process of
8. (b) The highest honour is earned by a 25. (b) ‘Logical’ means ‘reasonable and transfer of heat as it travels at the speed
man who has struggled against adversity. based on logic’ and ‘Rational’ means of light. The slowest mode of transfer of
9. (b) A well-to-do man sends his son to ‘based on or in accordance with reason heat is conduction because it takes place
foreign lands in the belief that his dull or logic’. So, ‘rational’ is similar in from particle to particle.
son will be transformed into a genius. meaning to ‘logical’. 35. (d) The Ministry of Health and Family
10. (a) The given expression means that 26. (a) Kushan emperor Vima Kadphises was Welfare launched the Surakshit Matritva
despite going to expensive schools and first to introduce the gold coinage in Aashwasan (SUMAN) scheme.
universities one fails to make a mark in the India. The coins of this period had the Under the scheme, pregnant women and
life. So, option (a) is a suitable answer. images of rulers along with deities. mothers upto 6 months after delivery and
11. (a) The correctly spelt word is 27. (c) Guru Arjan was the fifth Sikh Guru of all sick newborns will be able to avail
‘Inimitable’, which means ‘incapable of ten gurus of Sikh religion. free healthcare benefits. The scheme will
being copied’. In 1606, on the orders of Mughal ensure to help in bringing down maternal
emperor Jahangir, Guru Arjan was and infant mortality rates in the country.
12. (b) The correctly spelt word is ‘pitiable’
which means ‘deserving or arousing pity’. arrested, tortured and executed after he 36. (a) The Union Public Service Commission
had refused to stop the preachings of (UPSC) is a constitutional body established
13. (b) The correctly spelt word is
Guru Nanak Dev of worshipping God. under Article 315 of the Constitution of
‘occurrence’ which means ‘an instance of
28. (a) Lord William Bentinck, the first India. The commision conducts
something when something happens’.
Governor-General of India (AD 1828-35) prestigious Civil Services examinations for
14. (c) The correctly spelt word is ‘argument’ bureaucratic positions such as the IAS,
is called ‘Social Reformer’ of British
meaning ‘a disagreement’. IPS and IFS. It comprises chairman and
India. He carried out social reforms like
15. (a) The correctly spelt word is ‘commission’ prohibition of Sati (1829) and ten members, who are appointed and
which means ‘an instruction, command elimination of thugs (1830) and on Lord removed by the President.
or role given to person or group’. Macaulay’s recommendations, he made 37. (d) Article 61 of the Indian constitution
16. (d) ‘Granary’ is a house which is used for English the medium of higher education deals with the impeachment of the
storing grains. in India for the first time. president. President can be impeached
17. (d) ‘Pseudonym’ is a fictitious name used 29. (d) Big Bang was an explosion of only on the grounds of violation of
by an author in his writings. concentrated matter in the universe that Constitution.
18. (b) ‘Anonymous’ means ‘of unknown occurred 15 billion years ago, leading to The impeachment procedure can be
authorship or origin’. So, option (b) is a the formation of galaxies of stars and initiated in either House of Parliament.
suitable choice. other heavenly bodies. 38. (c) The word ‘electronic’ in the term
19. (b) ‘Scapegoat’ is a person who is 30. (b) India’s most active volcano is situated ‘e-Governance’ implies ‘technology driven
blamed for something that someone else in the Barren Island. Barren Island, governance’.
has done. located in the Andaman sea, is the only The perspective of the e-Governance is
20. (c) A person who brings goods illegally confirmed active volcano in South Asia ‘the use of the technology that both help
into the country is called smuggler. and the only active volcano along a chain to govern and have to be governed.’
of volcanoes from Sumatra to Myanmar. The National e-Governance Plan (NeGP)
21. (b) ‘Efficacious’ means ‘effective or able
to produce the intented result’ and It is a part of Indian Union Territory of takes a holistic view of e-Governance
‘Ineffective’ means ‘not able to produce Andaman and Nicobar Island, lies about initiatives across the country, integrating
desired result’. 138 km North-East of its capital Port Blair. them into a collective vision.
So, option (b) ‘ineffective’ is the correct 31. (a) The normal temperature of a human 39. (a) ‘Vajra Prahar’ is a joint military
opposite meaning word to ‘efficacious’. body is 37°C or 98.6°F. exercise between India and the US. The
22. (a) ‘Outrageous’ means ‘shocking and 32. (d) Renewable resources, are available in 10th edition of this exercise held at
morally unacceptable’ and ‘Justifiable’ large excess i.e. never ends, e.g. air, Seattle (US) from 13th to 28th October,
means ‘an action, idea or decision that is sunlight, etc. Non-renewable resources 2019.
acceptable or correct’. are available in limited quantity and end 40. (c) The 2019 Nobel Peace Prize has
e.g. mineral, coal, petroleum, natural been awarded to Ethiopean PM Abiy
Hence, option (a) ‘justifiable’ is the
gas, etc. Ahmed who made peace last year
opposite meaning word of ‘outrageous’.
36 AFCAT ~ Practice Set 4

with bitter foe ERITREA. He was 1 56. (a) Let the number of boys be x and that
49. (c) ∵ x + = 2 ⇒ x 2 − 2x + 1 = 0
awarded the prize for his efforts to x of girls be y.
‘achieve peace and international ⇒ (x − 1)2 = 0 ⇒ x = 1 According to the question,
cooperation’.
[∵(a − b)2 = a2 + b2 − 2ab] 71x + 73y = 718 . (x + y)
41. (b) Every year on 8th October, Indian Air 1 ⇒ 718. x − 71x = 73y − 718 . y
Force celebrates its Foundation Day or ∴ x 17 + 19 = 1 + 1 = 2 ⇒ 08. x = 12 . y
Indian Air Force Day. The IAF was x
x 12 . 12 3
50. (c) Let the period of time be T yr. ⇒ = = = =3:2
founded on 8th October, 1932 and the y 08 . 8 2
force has participated in several crucial According to the question, 2
wars. 400 × 5 × T 500 × 4 × 625 . Hence, percentage of girls = × 100
= 5
42. (b) A museum for all former Prime 100 100 = 20 × 2 = 40%
Ministers of the country will be built in ∵ SI = PRT 
57. (d) Let the original cost of CD be ` x.
the Teen Murti Bhavan complex, that  100 
Then, 75% of x = 24
houses the Nehru Memorial Museum 500 × 4 × 625
. 25 1
and Library (NMML) in New Delhi. ⇒T= = = 6 yr ⇒
75
× x = 24 ⇒ x = ` 32
400 × 5 4 4 100
The proposed museum, dedicated to all
Prime Ministers of India is envisaged to 51. (a) Speed of stream But he paid ` 24.
1 Therefore, the cash discount
give visitors a holistic understanding of = (Speed downstream
the continued thread of leadership. 2 − Speed upstream) = 32 − 24 = ` 8
43. (a) Common salt i.e. Sodium Chloride 1  36 40 1 1 1
=  −  = (6 − 5) = 58. (c) CP of 1 lemon =
(NaCl) is used as a raw material for 2 6 8 2 2 2
making a large number of useful = 0.5 km/h 3
SP of 1 lemon =
chemicals in industry. 5
52. (b) Let the three numbers be x, 2x and 3x. SP − CP
44. (d) In 2019, 20 new Moons of Saturn According to the question, ∴Profit per cent = × 100
were discovered using the Subaru CP
x 3 + (2x)3 + (3x)3 = 4500 3 1
telescope which is set up at top of −
⇒ 36x 3 = 4500 ∴ x 3 = 125 ⇒ x = 5 6−5
Hawai’s Mauna Kea Volcano. Thus, = 5 2 × 100 = × 100 = 20%
1 5
making Saturn (82 moons), the planet Hence, smallest number = 5
with the highest number of Moons. 2
53. (a) Let the distance covered by the
Jupiter occupies the 2nd position with cyclist be d and time taken to cover it be 59. (d) Given, 4 yr ago,
79 moons. t. Then, these value for the jogger will be Average age of husband and wife = 27 yr
45. (c) In India, the period between d Sum of the present ages of husband and
and 2 t.
1991-2003 is termed as ‘Golden 2 wife = 2 × 27 + 8 = 62 yr
Revolution’. It focuses on honey and ∴The ratio of the speed of the jogger to Similarly, sum of the present ages of
horticulture production. During this that of the cyclist, husband, wife and child
period, there was a huge increase in the d = 21 × 3 = 63 yr
production of various fruits, vegetables Distance 2 d 1 ∴Present age of the child
Speed = = : = :1 = 1 : 4
and other horticulture products. Time 2t t 4 = 63 − 62 = 1 yr
46. (c) Siachen Glacier is located in the 54. (c) Let 1 kg of each of the alloys A and B 60. (b) Since, 1st pipe takes 12 min to
Eastern Karakoram range in the be mixed together. completely fill tank.
Himalayas at about 35.421226 °N and In alloy A, 1
77.109540 °E. At 76 km (47 m) long, ∴Tank filled by 1st pipe in 1 min =
5 12
Quantity of gold = kg
it is the longest glacier in the Karakoram 8 Now, 2nd pipe takes 20 min to empty the
and second-longest in the world’s 3 full tank.
Quantity of copper = kg
non-polar areas. 8 1
∴Tank emptied by 2nd pipe in 1 min =
47. (a) The Greenhouse Gases Observing 5 20
In alloy B, Quantity of gold = kg
Satellite (GOSAT) is the first satellite 16 Both pipes to fill the complete tank in 1 min
dedicated to monitoring Greenhouse Gas 11 1 1 5− 3 2 1
Quantity of copper = kg = − = = =
emission launched into space by Japan. 16 12 20 60 60 30
It was a global initiative to combat
∴Required ratio =  +
5 5   3 11  Since, total tank will be filled in 30 min.
climate change which will monitor the  : + 
 8 16   8 16  Hence, time taken to fill the half tank
levels of greenhouse gases like carbon 30
=
15 17
: = 15 :17 = = 15 min
dioxide and methane. 2
16 16
(c)  ×  ÷  ÷ 
5 19 19 13 2 3 1
48. 55. (b) Let x% of 5 L = 400 mL 61. (b) of of of 26000
 6 13  7 4 5 4 4
x
∴ × 5000 = 400
=   ÷  × 
95 19 4 2 3 1
100 = × × × 26000 = 325 × 6 = 1950
 78   7 13 5 4 4
[∵1 L = 1000 mL]
=   ÷   =
95 76 95 91 35 400 × 100 62. (a) A can do a job in 20 days.
× = ⇒ x= = 8%
 78   91 78 76 24 ∴1 day work of A =
1
5000
20
AFCAT ~ Practice Set 4 37

Similarly, 1 day wok of A and B together Some teachers can be poet. Some 91. (a) In the second figure, a small figure is
1 graduates can be teachers or poets. added, same as that present inside the
=
12 70. (d) Birds circle and two small lines are added in
1 1
Now, 1 day work of B = − lower part of figure.
12 20 92. (c) Counting from top, the first figure rotates
5− 3 2 1
= = = Parrots 90° anti-clockwise and comes at third
60 60 30 Mice
position, second figure is reversed at the
∴B can complete the work in 30 days. same position and third figure rotates 180°
All parrots are birds. But mice is
63. (b) Required number = HCF of {(110 − 2) entirely different. and after reversing comes at the first
and (128 − 2)}. position.
71. (d) All others are food items.
= HCF of 108 and 126 72. (d) All others fly in the day time.
93. (b) From problem figure (1) to (2), symbols
By Division Method change positions as shown in the diagram
73. (c) Except ‘Model’, all are same and rotate through 180° except the hexagon
108) 126 (1
because all are made on paper. as it rotates through 90°. Same pattern will
108
74. (d) All except ‘Curtain’ are parts of follow from answer figure to figure (4).
18) 108 (6 bed-spread.
108 75. (d) Except ‘Candle’ all other articles
× have one of its ends sharp or pointed.
∴Greatest number = 18 76. (c) ‘Cream’ is made from ‘Milk’.
64. (a) Let the radius of third circle be r. Likewise, ‘Pottery’ is made from ‘Clay’.
94. (b) From problem figure (1) to (2), all the
Area of third circle 77. (c) ‘Loathe’ and ‘Coercion’ are related.
symbols change positions as shown in the
= Area of first circle + Area of second circle In the same way ‘Irate’ and
diagram and the symbol in place of * is
⇒ πr2 = π (5)2 + π (12)2 ‘Antagonism’ are related.
replaced by new symbol.
⇒ πr2 = π(25 + 144) = 169π 78. (c) ‘Nest’ is made up of ‘Straws’ and
‘Cloth’ is made up of ‘Threads’.
⇒ πr2 = 169π ⇒ r = 169 = 13 cm
79. (c) As, ‘Umpire’ is in a ‘Game’, in
65. (b) We have, (6n − 1) the same way ‘Legislator’ is in an
If n is even, then taking n = 2, ‘Election’.
6n − 1 = 62 − 1 = 36 − 1 = 35 80. (c) ‘Sky’ and ‘Earth’ are related to one 95. (a) From figure (1) to (2) both the designs
Here, number 35 is divisible by 35. another, in the same manner as ‘Fly’ interchange their positions and one design
and ‘Walk’. is inverted.
Hence, for any even value of n, (6n − 1) is
divisible by 35. 81. (d) Answer figure (d) will complete the 96. (b) Dark circle moves one step clockwise
problem figure. and addition of a new element takes place
66. (a)
Tall man Black 82. (a) Answer figure (a) will complete the at the centre.
haired problem figure. 97. (d) All elements in the figure move one step
people anti-clockwise.
83. (c) Answer figure (c) will complete the
Indians problem figure. 98. (a) The elements at the face of the cylinder
84. (c) Answer figure (c) will complete the remain the same for two corresponding
Some tall man can be black haired and
problem figure. figures; like problem figures (1) and (2)
some Indians can be tall or black haired.
have same face elements; problem figures
67. (b) 85. (d) Answer figure (d) will complete the
(3) and (4) also have same face elements.
problem figure.
Arsenic Cyanide So, problem figure (5) and the answer figure
86. (a) In all other figures, both the arrows
must also have same face elements. Among
Poison attached to the main figure are in
the two corresponding figures, one is vertical
opposite direction to each other.
Arsenic and Cyanide are different, but while other is horizontal. As problem figure
87. (c) All other figures except (c), are (5) is vertical, the answer figure must be
both are poison. rotated forms of the same figure. horizontal. Also, the number of shaded dots is
68. (c) 88. (a) In all other figures, one of the increased by one in each subsequent figure.
small lines attached to the main line 99. (b) Circle moves one and half and one step
Turtles Monkey segment is perpendicular to it. anti-clockwise alternately while arrow
89. (d) In all other figures, there are only rotates 135° and 90° clockwise alternately.
Reptiles
two dots on the base of the figure. 100.(d) From figure (1) to (2) black dots move
Turtles are Reptiles but Monkey is different. 90. (c) In all other figures, transverse lines one step in anti-clockwise direction. From
69. (b) Teacher between the two parallel lines are figure (2) to (3) white circles move one step
Poet inclined towards the dot on the figure. anti-clockwise and this process goes on in
alternate steps.
Graduate
38 AFCAT ~ Practice Set 5

AFCAT
AIR FORCE COMMON ADMISSION TEST (ONLINE )

Practice Set ~05


Time : 2 Hrs MM : 300

INSTRUCTIONS
1. The set contains a total of 100 questions, Comprising Verbal Ability in English, General Awareness, Numerical Ability and Reasoning and
Military Aptitude Test.
2. Each correct question carry 3 Marks and there will be negative marking of 1 Mark for each incorrect attempt.
3. Total time duration will be 2 hrs (120 minutes).
4. No marks will be deducted for unattempted questions.

Directions (Q. Nos. 1-3) Read the (b) It is a mask looking like the head Directions (Q. Nos. 7-9) In the
following passage carefully and answer of a dragon with its tongue following passage, some words have been
the questions that follow. hanging out omitted. For each blank, four alternatives
(c) It is a mask looking like an are suggested. Select the suitable
Martin had many little tricks highly
animal with horns, wings and a alternative to fill the given blanks.
entertaining to his son. On an evening,
pair of ferocious eyes emitting It is not proper to damn a system without
returning from the market, he would buy fire understanding it. The Indian bureaucracy
a paper mask, the head of a hissing (d) It is a mask looking like the head may be as bad after all, as it is made out
dragon. He would put it on and knock at of a King Cobra to be. Times without number, it has been
the door. On opening the door, the boy
would be terrified for a moment, but only 3. The father and son rolled with (7) that our bureaucrat is a strange creature
laughter after the who has the habit of sitting (8) the files
for a moment, for he would soon remove
(a) father put on the mask and also happens to sleep on the remainders.
it and the two would roll with laughter.
(b) opening of the door What is worse is (9) his own word.
Tom would, then, go out with the mask
(c) son saw the mask 7. (a) declared (b) proclaimed
and knock at the door for his father to
(d) father removed the mask
open. Martin had to act as if he was (c) said (d) pronounced
paralysed with fear. 4. Insert suitable interjection to fill 8. (a) on (b) at
in the blank space.
1. Martin played his little tricks (c) by (d) with
......... ! This place smells so bad.
because 9. (a) minces (b) eats
(a) Eww (b) Argh (c) Wow (d)
(a) he was very much interested in (c) minds (d) shuffles
Shh
them
(b) he wanted to terrify his son 5. Insert proper preposition in the Directions (Q. Nos. 10 and 11) Find
(c) his son got pleasure from them sentence. out which part of the sentence has an
(d) it was his habit to make tricks Our office is situated ...... Agarwal error. If the sentence is free from error,
Road. then choose ‘No error’ as your answer.
2. Which of the following (a) in (b) on (c) of (d) at 10. The members of the family (a)/
statements is the most
6. Fill in the blank with the soon became (b)/ aware what had
appropriate description of the
appropriate word. My cousim is a taken place. (c)/ No error (d)
mask?
(a) It is a mask looking like a dragon ......... smoker. 11. Let us congratulate him (a)/ for his
with long tail and covering the (a) heavy (b) serious success (b)/ in the examination.
whole body of Martin (c) big (d) serial (c)/ No error (d)
AFCAT ~ Practice Set 5 39

Directions (Q. Nos. 12-16) In the Directions (Q. Nos. 21-23) In the 31. Which one was the first
following questions, one word in each following questions, out of the four ‘Biosphere Reserves’ Project
group is misspelt. Choose the misspelt alternatives, choose the one which best scheme?
word. expresses the meaning of the given (a) Sundarbans Biosphere Reserve
word. (b) Nilgiri Biosphere Reserve
12. (a) Trival (b) Trifles
(c) Asert (d) Reason 21. Prolific (c) Nandadevi Biosphere Reserve
(a) Skilful (b) Swift (d) Gulf of Mannar Biosphere Reserve
13. (a) Misanthropic (b) Cynical (c) Fertile (d) Wearisome 32. A dynamo is used to convert
(c) Obsollete (d) Pensive
22. Soporific (a) mechanical energy into electrical
14. (a) Scornful (b) Frosty (a) Flattering (b) Sickening energy
(c) Serene (d) Franchis (c) Exciting (d) Sleep-producing (b) electrical energy into mechanical
energy
15. (a) Capacity (b) Abdicait 23. Conciliatory (c) electrical energy into magnetic
(c) Abandon (d) Quit (a) Foolish (b) Soothing energy
16. (a) Allergy (b) Ascetic (c) Advisory (d) Over proud (d) magnetic energy into mechanical
energy
(c) Athiest (d) Ameliorate Directions (Q. Nos. 24 and 25) In
the following questions, choose the 33. Which of the following
Directions (Q. Nos. 17-20) In the word opposite in meaning to the given organisations has Sarvatra
following questions, four alternatives word. Sarvottam Suraksha as its slogan?
are given for the Idiom/Phrase (a) Border Security Force
underlined in the sentence. 24. Parsimonious
(b) Sikh Light Infantry
Choose the alternative which best (a) Conformity (b) Generous
(c) Indian Air Force
expresses the meaning of the (c) Stiff (d) Convincing
(d) National Security Guard
Idiom/Phrase. 25. Fetter 34. PM Narendra Modi unveiled
17. If you rub him the wrong way, he (a) Liberate (b) Cure world’s largest
is bound to react. (c) Compare (d) Diminish (a) Bhagawad Gita (b) Ramayana
(a) annoy him 26. Which of the following book was (c) Mahabharata (d) Rig Veda
(b) abuse him authored by Vikram Seth?
35. Which of the following agencies is
(c) flatter him (a) My God Died Young associated with UTKARSH 2022?
(d) encourage him (b) Freedom in Exile (a) NITI Aayog
18. Our school is within a (c) A Suitable Boy (b) University Grants Commission
stone’s throw of the railway (d) Look Back in Anger (c) Reserve Bank of India
station. (d) National Informatics Centre
27. Who among the following
(a) within a certain radius founded the All India Muslim 36. The three medals India won in
(b) at a short distance League? Beijing Olympics were in
(c) within a definite circumference (a) Syed Ahmed Khan (a) Shooting, Boxing and Wrestling
(d) very far off (b) Muhammed Ali (b) Badminton, Chess and Shooting
19. Some people have a habit (c) Agha Khan (c) Badminton, Shooting and Boxing
of wearing their heart on their (d) Hamid Ali Khan (d) Shooting, Archery and Wrestling
sleeve. 28. Shiv Kumar Sharma is famous for 37. With which game is Geet Sethi
(a) avoiding being friendly with playing associated?
others (a) Santoor (b) Sitar (a) Lawn Tennis (b) Kabaddi
(b) saying something which is not to (c) Sarod (d) Flute (c) Billiards (d) Squash
be taken seriously 38. When is the International Day for
(c) exposing their innermost feelings 29. The largest delta of the world is
(a) Sunderbans Disaster Risk Reduction (IDDRR)
to others observed?
(d) wasting their time on (b) Amazon basin
(c) Greenland (a) 11th October (b) 13th October
unnecessary details (c) 15th October (d) 14th October
(d) Congo basin
20. There was opposition to the new 39. The Constitution of India
policy by the rank and file of the 30. The place to receive annual describes India as
government. rainfall below 50 cm is (a) a union of states
(a) the majority (a) Meghalaya (b) quasi-federal
(b) the ordinary members (b) Leh in Kashmir (c) a federation of State and Union
(c) the cabinet members (c) Coromondel Coast Territories
(d) the official machinery (d) Konkan Coast (d) a unitary state
40 AFCAT ~ Practice Set 5

2 4 6
40. Which one of the following 52. A sells a cycle to B at a profit of 62. HCF of , and is
correctly describes ‘AGNI’? 10%, B sells to C at a profit of 3 5 7
(a) A fighter plane 20%. If C pays ` 264 for it, how 48 2
(a) (b)
(b) A versatile tank much did A pay for it? 105 105
(c) A long-range missile (a) ` 200 (b) ` 220 16 24
(c) (d)
(d) A long-range gun (c) ` 225 (d) ` 234 105 105
41. Air Force Academy is located in 53. If A earns 10% more than B’s, 63. The difference between the simple
(a) Hyderabad (b) Coimbatore then how many per cent less does and compound interest on a
(c) Kakinada (d) Mumbai B earn than A? certain sum of money at 5% rate
2 1 of interest per annum for 2 yr is
42. Which animal is chosen as the (a) 12 (b)11 (c)13 (d)11
9 9 ` 15. Then, the sum is
Mascot for National Games 2022 (a) ` 6500 (b) ` 5500
that is to be held in Meghalaya? 54. In an alloy, there is 12% of
(c) ` 6000 (d) ` 7000
(a) Sangai copper. To get 69 kg of copper,
(b) Tiger how much alloy will be required? 64. The digit in unit’s place of the
(c) Clouded Leopard (a) 424 kg (b) 575 kg product 81 × 82 × 83 × … × 89 is
(d) Hoolock Gibbon 2 (a) 0 (b) 2
(c) 828 kg (d)1736 kg
43. When was ISRO set up? 3 (c) 6 (d) 8
5
(a) 1965 (b) 1969 55. A’s wealth is times of B’s and 65. 0.142857 + 0.285714 is equal to
(c) 1971 (d) 1976 10 7 1 3
C’s is times of B’s. What is the (a) (b)
44. Indira Gandhi Rashtriya Udan 7 2 7
ratio of C’s wealth to A’s? (c) 2 (d) None of these
Academy is located at
(a) 49 : 100 (b)1 : 2
(a) Secunderabad (b) Rae Bareilly 66. Two statements are given followed
(c) 2 : 1 (d)100 : 49
(c) Jodhpur (d) Delhi by two conclusions. Read the
56. Mohan lent some amount of conclusions and then decide which
45. The lightest radioactive element is money at 9% simple interest and of the conclusions,
(a) deuterium (b) polonium an equal amount of money at if any, logically follows from the
(c) tritium (d) uranium 10% simple interest each for 2 yr. two given statements, disregarding
46. Indira Gandhi Super Thermal If his total interest was ` 760, the known facts.
Power Project is located in which then what amount was lent in Statements All cups are saucers.
of the following? each case? No saucer is a kettle.
(a) Uttarakhand (b) Haryana (a) ` 1700 (b) ` 1800 Conclusions
(c) Punjab (d) Uttar Pradesh (c) ` 1900 (d) ` 2000 I. No cup is a kettle.
II. No kettle is a cup.
47. Rogers Cup is related to which sport? 57. A train 300 m long is running at a (a) Only Conclusion I follows
(a) Cricket (b) Tennis speed of 25 m/s. It will cross a
(b) Only Conclusion II follows
(c) Hockey (d) Badminton bridge of 200 m in
(c) Neither conclusion I nor II follows
(a) 5 s (b) 10 s (c) 20 s (d) 25 s
48. Who is the first lady recipient of (d) Both Conclusions I and II follow
Dadasaheb Phalke Award? 58. Walking at the rate of 4 km an
67. One statement followed by two
(a) Nargis Dutt (b) Uma Devi hour, a man covers a certain
assumptions numbered
(c) Devika Rani (d) Sulochana distance in 3 h 45 min. If he
covers the same distance on cycle, I and II is given. You have to
49. With which sport is the term cycling at the rate of 16.5 km/h, consider the statement and the
‘butterfly stroke’ linked then time taken by him is following assumptions and decide
(a) Boxing (b) Swimming which of the assumptions is
(a) 55.45 min (b) 54.55 min
(c) Karate (d) Judo implicit in the statement.
(c) 55.44 min (d) 45.55 min
Statement
50. A and B can do a piece of work in 59. If a − b = 2 and ab = 24, then what ‘Please do not wait for me, I may
10 days. B and C can do it in is the value of a3 − b3 ? be late, start taking lunch as soon
12 days. A and C can do it in (a) 280 (b) 124 (c) 140 (d) 152
15 days. How long will A take to as the guests arrive’.—a message
5  8 x 1 13 2 x from a director of a company to
do it alone? 60. If  −  + = , then
(a) 24 days (b) 20 days 2  3 2 2 3 his office managers.
what is the value of x? Assumptions
(c) 40 days (d) 30 days
7 8 8 7
(a) (b) (c) − (d) − I. Keeping guests waiting is not
51. The average per day income of 8 7 7 8 desirable.
A, B and C is ` 450. If the average
II. Lunch may not be ready in time.
per day income of A and B is ` 400 10 + 25 + 108 + 154 + 225
and that of B and C is ` 430, then 61. 3
(a) Only Assumption I is implicit
8 (b) Only Assumption II is implicit
the per day income of B is =?
1 (c) Neither I nor II is implicit
(a) ` 300 (b) ` 310 (a) 4 (b) 8 (c) 2 (d) (d) Both I and II are implicit
(c) ` 415 (d) ` 425 2
AFCAT ~ Practice Set 5 41

Directions (Q. Nos. 68-70) Find the Directions (Q. Nos. 78-82) In each of 89. Select from the alternatives, the
missing term in the series given below. the following questions, choose the box that can be formed by folding
68. 26, 37, 50, 65, ?, 101 word which will complete the second the sheet showing in figure (X).
pair in the same way as first pair.
(a) 77 (b) 80 2
(c) 81 (d) 82 78. Pleasure : Sorrow :: Right : ?
(a) Wrong (b) Wonderful 5 1
69. 758, 753, 748, 744, 740, 736, ?
(c) Happy (d) Sure
(a) 732 (b) 733 4
(c) 734 (d) 735 79. Aluminium : Bauxite :: Iron : ?
(a) Pyrite (b) Magnesite 6 3
70. E-5, G-7, I-9, K-11, M-13, ?
(c) Pynolosite (d) Haematite (X)
(a) N-14 (b) N-15
(c) O-15 (d) O-17 80. Truthfulness : Liar :: Loyalty : ?
3 1 2 3
(a) Worker (b) Traitor
71. Ramakant walks northwards. 6
1
3
5
6
4
1
4
After a while, he turns to his right (c) Diligent (d) Faithful
and a little further to his left. 81. House : Door :: Compound : ? (a) (b) (c) (d)
Finally, after walking a distance of (a) Gate (b) Fence
one kilometre, he turns to his left (c) Foundation (d) Wall
Directions (Q. Nos. 90-93) In each of
again. In which direction is he the following questions, a part of
moving now ? 82. Hongkong : China :: Vatican : ? problem figure is missing. Find out
(a) France (b) Mexico from the given answer figures (a), (b),
(a) North (b) South
(c) East (d) West
(c) Canada (d) Rome (c) and (d), that can replace the ‘?’ to
83. In a certain coded language, complete the problem figure.
72. Following diagram represents
school children, artists and “SPRING” is written as 90. Problem Figure
singers. Study the diagram and “ % ? * − $ ÷” and “PRAISE” is
written as “ ? * @ – % ×”. How ?
identify the region which
represents those school children will the word “ASPIRE” is coded?
who are artists and not singers. (a) @ % ? − * × (b) ? @ * − $ ÷
School children (c) @ ? − * × % (d) % * ? − ÷ @
Answer Figures
84. In a certain code ‘bi nic pic’
a means, ‘some good jokes’,
b ‘nic bat lik’ means ‘some real
f stories’ and ‘pic lik tol’ means (a) (b) (c) (d)
d c Artists ‘many good stories’. What is the
code for ‘jokes’ ? 91. Problem Figure
e (a) bi (b) nic (c) pic (d) lik
g
85. What is the angle traced by hour
Singers hand in 40 min?
(a) 20° (b) 15° (c) 40° (d) 35°
(a) a (b) b (c) f (d) e ?
73. Find the odd word. Directions (Q. Nos. 86-88) In each of
(a) Peso (b) Drachma
the following questions, four figures are Answer Figures
(c) Shora (d) Baht
given. Three are similar in a certain way
and so form a group. Find out which one
74. Find the odd pair of words. of the figures does not belong to that group.
(a) Venus : Shukra
(b) Uranus : Indra
86.
(a) (b) (c) (d)
(c) Mars : Mangal
(d) Saturn : Budha 92. Problem Figure
75. Find the odd group of letters. (a) (b) (c) (d)
(a) AOE (b) EUI 87.
?
(c) OUT (d) IEM
76. Find the odd one.
(a) 216 (b) 33 (a) (b) (c) (d)
(c) 8 (d) 64 Answer Figures
88.
77. Find the odd one.
(a) KQ14 (b) AY13
(c) MR11 (d) GW15 (a) (b) (c) (d) (a) (b) (c) (d)
42 AFCAT ~ Practice Set 5

93. Problem Figure Answer Figures 98. Problem Figure

(a) (b) (c) (d)


? 96. Problem Figure (A)
Answer Figures
Answer Figures

Answer Figures (a) (b) (c) (d)


(a) (b) (c) (d) 99. Problem Figure
Directions (Q. Nos. 94-96) In each of
the following questions, find the answer (a) (b) (c) (d)
figure in which problem figure is
embedded. Directions (Q. Nos. 97-100) In the (A)
94. Problem Figure following questions, choose that
figure which contains region(s) Answer Figures
common to circle, square, triangle,
similar to that marked by the dot in
figure (A).
(a) (b) (c) (d)
Answer Figures 97. Problem Figure
100. Problem Figure

(a) (b) (c) (d) (A)


(A)
95. Problem Figure Answer Figures
Answer Figures

(a) (b) (c) (d)


(a) (b) (c) (d)

Answers
1 (c) 2 (b) 3 (d) 4 (a) 5 (d) 6 (a) 7 (a) 8 (c) 9 (b) 10 (c)
11 (b) 12 (c) 13 (c) 14 (d) 15 (b) 16 (c) 17 (a) 18 (b) 19 (c) 20 (b)
21 (c) 22 (d) 23 (b) 24 (b) 25 (a) 26 (c) 27 (c) 28 (a) 29 (a) 30 (b)
31 (b) 32 (a) 33 (d) 34 (a) 35 (c) 36 (a) 37 (c) 38 (b) 39 (a) 40 (c)
41 (a) 42 (c) 43 (b) 44 (b) 45 (c) 46 (b) 47 (b) 48 (c) 49 (b) 50 (a)
51 (b) 52 (a) 53 (d) 54 (b) 55 (c) 56 (d) 57 (c) 58 (b) 59 (d) 60 (d)
61 (c) 62 (b) 63 (c) 64 (a) 65 (b) 66 (d) 67 (a) 68 (d) 69 (b) 70 (c)
71 (d) 72 (b) 73 (c) 74 (d) 75 (c) 76 (b) 77 (c) 78 (a) 79 (d) 80 (b)
81 (a) 82 (d) 83 (a) 84 (a) 85 (a) 86 (c) 87 (d) 88 (d) 89 (d) 90 (a)
91 (c) 92 (c) 93 (c) 94 (b) 95 (d) 96 (d) 97 (b) 98 (c) 99 (b) 100 (c)
AFCAT ~ Practice Set 5 43

Hints and Solutions


1. (c) Martin played his little tricks because 21. (c) ‘Prolific’ means ‘fruitful or highly 30. (b) Leh region in Kashmir is a cold desert.
his son got pleasure from them. productive’ and ‘fertile’ means ‘producing The rainfall here is very less (<50 cm).
2. (b) It is a mask looking like the head of a or capable of producing abundant amount Meghalaya and Konkan Coast receives
dragon with its tongue hanging out, is of vegetation, crop etc’. Hence, ‘fertile’ is heavy rainfall in monsoon (>200 cm)
the most appropriate description of the similar in meaning to ‘prolific’. and Coromondel Coast receives more
mask. 22. (d) ‘Soporific’ means ‘causing or tending to than 100 cm rain due to retreating
cause sleep’. So, option (d) ‘sleep-producing’ monsoon.
3. (d) The father and son rolled with
laughter after the father removed the express the correct meaning of ‘soporific’. 31. (b) Nilgiri Biosphere Reserve was the first
mask. 23. (b) ‘Conciliatory’ means ‘try to make Biosphere Reserve in India which is
someone feel better’ and ‘soothing’ established in 1986. It is an international
4. (a) Interjection ‘Eww!’ is used to express
means ‘to make someone feel calm or biosphere reserve in the Western Ghats.
distaste or disgust, so it is suitable choice
to fill the given blank. less worried’. So, ‘soothing’ best express 32. (a) Dynamo is an electric generator
the meaning of ‘conciliatory’. which converts mechanical energy into
5. (d) Preposition ‘at’ is used to refer to a
24. (b) ‘Parsimonious’ means ‘not willing to electrical energy through the process of
definite location, so option (d) ‘at’ is
spend money or to give or use resources’ electromagnetic induction.
suitable to fill in the blank.
and ‘Generous’ means ‘willing to give money, 33. (d) National Security Guard (NSG) has
6. (a) ‘Heavy’ is the appropriate word to fill
help etc especially more than usual or ‘Sarvatra Sarvottam Suraksha’ as its
the given blank. ‘Heavy smoker’ is the
expected’. So, ‘generous’ is opposite in slogan. NSG is a counter terrorism unit
person who smokes a lot of cigarettes in
meaning to the word ‘parsimonious’. under the Indian Ministry of Home
a day.
25. (a) ‘Fetter’ means ‘to restrict someone’s Affairs. It was raised on 15th October,
7. (a) declared 1984, following operation Blue Star,
freedom or stop them from making
8. (c) by progress’ and ‘liberate’ means ‘to free Akshardham Temple attack and the
9. (b) eats someone from conditions or restrictions’. assassination of Indira Gandhi.
10. (c) Add ‘of’ after ‘aware’, because ‘aware’ Hence, option (a) is a suitable choice. The force is used for tackling the terror
is followed by the preposition ‘of’. 26. (c) ‘A Suitable Boy’ is a novel written by activities and to protect states against
Vikram Seth. Published in 1993, the internal disturbances.
11. (b) Use ‘on’ in place of ‘for’. The verb
‘congratulate’ is followed by the novel covers an engaging story that is set 34. (a) PM Narendra Modi unveiled world’s
preposition ‘on’. in post-independence India. largest Bhagawad Gita at the ISKON
Vikram Seth is an Indian novelist and temple in New Delhi on 26th February,
12. (c) The misspelt word is ‘asert’.
poet who has received several awards 2019. Measuring over 2.8 m and over
Its correct spelling is ‘assert’.
including Padma Shri, Sahitya Akademi 800 kg the giant Bhagawad Gita is one
13. (c) The misspelt word is ‘obsollete’. of the kind in the world.
Award, Crossword Book Award etc.
Its correct spelling is ‘obsolete’.
27. (c) All India Muslim League was founded 35. (c) ‘UTKARSH 2022’ is a three-year
14. (d) The misspelt word is ‘franchis’. Its by Agha Khan. The first conference of the roadmap of Reserve Bank of India (RBI)
correct spelling is ‘franchise’. All India Muslim League was held at to improve regulation and supervision
15. (b) The misspelt word is ‘Abdicait’. Its Amritsar under the Presidentship of Sir among other functions of the Central
correct spelling is Abdicate. Syed Ali Imam. Early genesis of party can Bank. This medium term strategy is in
16. (c) The misspelt word is ‘Athiest’. Its be traced as an aftermath of Literary line with the global Central Banks’ plan
correct spelling is Atheist. Movement led by Syed Ahmed Khan. to strengthen the regulatory and
supervisory mechanism.
17. (a) Idiom ‘rub the wrong way’ means ‘to 28. (a) Shiv Kumar Sharma is a famous
santoor player who is credited with 36. (a) At the 2008 Beijing Olympics,
annoy or to irritate someone’. Hence,
making the santoor a popular classical Abhinav Bindra won the gold medal in
option (a) best express the meaning of
instrument. The Santoor is a folk the men’s 10 m air rifle shooting event,
given idiom.
instrument from Jammu and Kashmir. Sushil Kumar won the wrestling Bronze
18. (b) Idiom ‘within a stone’s throw’ means medal for India in the men’s freestyle
Shiv Kumar Sharma was awarded with
‘at a short distance’. Hence, option (b) is 66 kg. And Vijender Kumar won a
Padma Shri in 1991 and the Padma
a suitable choice. Vibhushan in 2001. Bronze medal in the middle weight 75 kg
19. (c) Idiom ‘wearing one’s heart on one’s boxing category.
29. (a) The Ganges-Brahmaputra Delta is the
sleeve’ means ‘to display one’s feeling 37. (c) Geet Sethi of India is a professional
largest river delta in the world. It is also
openly and habitually. Hence, option (c) player of English Billiards, who
known as the Sunderbans Delta, Ganges
is a suitable choice. dominated the sport throughout much of
Delta, the Brahmaputra Delta or the
20. (b) Idiom ‘rank and file’ means ‘the the 1990s and a notable amateur
Bengal Delta. The Sunderbans Delta is
ordinary members of an organisation as Snooker player.
located in the South Asia region of
opposed to its leaders and Bengal consisting of Bangladesh and He is the recipient of Rajiv Gandhi Khel
managers’. So, option (b) is a suitable Indian state of West Bengal. Ratna Award and Padma Shri Awards.
choice.
44 AFCAT ~ Practice Set 5

38. (b) 13th October is designated as 46. (b) Indira Gandhi Super Thermal Power 52. (a) If the CP of A be ` x, then
International Day for Disaster Risk Project is located at Jharli village in cost price of A
Reduction (IDDRR) by the UN General Jhajjar district of Haryana.  100   100 
Assembly. Began in 1989, this day is =    ×R
The coal based power project was  100 ± x   100 ± x 
celebrated every year to promote a developed by Arawali power company, a 100 100
global culture of disaster, including joint venture company comprising the = × × 264
disaster prevention, mitigation and 100 + 10 100 + 20
NTPC, IPGCL and HPGCL. It is the
100 100
preparedness. largest thermal power plant located in = × × 264
39. (a) Article 1 of the Constitution of India Haryana. 110 120
100
declares that India shall be a Union of 47. (b) Rogers cup presented by National = × 22 = 2 × 100 = ` 200
States. The territory of India shall 11
Bank is an annual Tennis Tournament
consists of the territories of the states, held in Canada. It is part of the US Open 53. (d) Here, R = 10%, then
the Union Territory and any territory that Series of tournaments and is played on Required percentage
may be acquired. hard courts.  R 
= × 100
40. (c) The Agni series of ballistic missiles 48. (c) Devika Rani was an Indian actress  100 − R 
are medium to long range ballistic who won the Dadasaheb Phalke Award 10
missiles developed by the Defence = × 100
in 1969 and become the first recipient of 100 − 10
Research and Development Organisation the award. She was honoured with the 10 1
(DRDO) of India. It was named after one award at the 17th National Film Awards. = × 100 = 11 %
of the five elements of nature. The first 90 9
49. (b) The term ‘Butterfly stroke’ is related
missile of the series, Angi I was 54. (b) ∵ 12 kg copper is contained in
to swimming. It is a swimming stroke
developed under the Integrated Guided 100 kg of alloy
swum on the chest, with both arms
Missile Development Programme and ∴69 kg copper is contained in
moving symmetrically, accompanied by 100
tested in 1991. = × 69 = 575 kg of alloy
the butterfly kick.
41. (a) The Air Force Academy is located at 12
50. (a) According to the question,
Dundigal, Hyderabad. With its inception 1 55. (c) According to the question,
in 1971, the academy was set up to (A + B)’s 1 day’s work = ...(i) 5 A 5 10
10 A= B ⇒ = and C = B
train cadets from all streams at one 1 7 B 7 7
location. (B + C)’s 1 day’s work = ...(ii) C 10
12 ⇒ =
42. (c) The Sports and Youth Ministry in the 1 B 7
(C + A)’s 1 day’s work = ...(iii) C
meeting of Executive Committee of the 15
C
National Games 2022 has chosen the On adding all three equations, ∴Ratio of C : A = = B
Meghalaya’s state animal ‘Clouded 2(A + B + C)’s 1 day’s work A A
Leopard’ for the Mascot. Meghalaya 1 1 1 B
= + + 10 7
will host the 2022 National Games, 10 12 15 = × = 2 :1
which is coinciding with its 50 years of 6 + 5+ 4 7 5
=
statehood. 60 56. (d) Let the sum lent in each case be ` x.
43. (b) The Indian Space Research 15 1 Then, according to the question,
= =
Organisation (ISRO) is the primary space 60 4 Sum of interests = ` 760
agency of the Indian Government. It was 1 x × 9 × 2 x × 10 × 2
∴(A + B + C)’s 1 day’s work = ⇒ + = 760
established in 1969. Headquartered in 8 100 100
Bangalore, ISRO is under the ∴A’s 1 day’s work x ×2
⇒ (9 + 10) = 760
administrative Control of the Department 1 1 3−2
= − = 100
of Space, Government of India. 8 12 24 2 × 19x
1 ⇒ = 760
Its objective is to harness space = 100
technology for national development 24 760 × 100
∴ x= = ` 2000
while pursuing space science research ∴A will complete the work in 24 days. 2 × 19
and planetary exploration. 51. (b) Given, total per day income of A, B
57. (c) Given, length of train = 300 m and
44. (b) The Indira Gandhi Rashtriya Udan and C = 3 × 450 = ` 1350
speed = 25 m/s
Academy is an aviation school under the Total per day income of A and B
In crossing the bridge, the train travels its
Ministry of Civil Aviation of India. = 2 × 400 = ` 800
own length + length of the bridge.
Established in 1985, the institute Total per day income of B and C
Total distance = 300 + 200 = 500 m
is located at Rae Bareilly, Uttar = 2 × 430 = ` 860
Pradesh. Speed = 25 m/s
∴B’s per day income = (Income of B and
Total distance
45. (c) Tritium is a radioactive isotope of C + Income of A and B) − (Income of A, ∴Required time =
hydrogen. It is the lightest radioactive B and C) Speed of train
element. = (800 + 860 − 1350) = ` 310 =
500
= 20 s
25
AFCAT ~ Practice Set 5 45

2
15 = P
58. (b) According to the question, 5  Clearly, he is finally walking towards
⇒ 
Distance covered on foot = Speed × Time  100 West direction.
3
= 4 × 3 = 15 km 2 72. (b) The letter ‘b’ represents those school
15 = P 
1
4 ⇒ children, who are artists but not singers.
Distance  20
∴ Time taken on cycle = 73. (c) All others are currencies, while ‘Shora’
15 = P
1 
Speed ⇒  is the Parliament of Afghanistan.
15 15 × 60  400
= h= min 74. (d) In all pairs except (d), first is a planet
16.5 16.5 ∴ P = 15 × 400 = ` 6000
and second is the correct Indian name of
= 5455
. min 64. (a) Given, 81 × 82 × 83 × … × 89. that planet.
59. (d) Given, a − b = 2, ab = 24 Then, the digit in unit’s place = Unit’s 75. (c) All except ‘OUT’ have a difference of
Now, (a − b) = a + b − 2ab
2 2 2 digit in the product four positions.
(1 × 2 × 3 × 4 × 5 × 6 × 7 × 8 × 9) = 0
⇒ 4 = a2 + b2 − 48 ⇒ a2 + b2 = 52
65. (b) 0.142857 + 0.285714 1 15 5 5 21 9 15 21 20 9 5 13
∴ a3 − b3 = (a − b) (a2 + b2 + ab) 142857 285714
A O E E U I O U T I E M
= +
= 2 (52 + 24) = 2 × 76 = 152 999999 999999 +4 +4 +5 +4
5  8x 1  13 2x 142857 + 285714
60. (d)  −  + = = 76. (b) All except ‘33’ are cubes of natural
2  3 2 2 3 999999
428571 47619 numbers.
5  16x − 3 13 2x = =
  + =
2 6  2 3 999999 111111 77. (c) In all other groups, number at the end
117 9 3 is half of the positions of sum of first and
 80 x − 15 + 13 = 2x = = =
  273 21 7 second letters in the alphabet. Except
 12  2 3
option (c), all are following this rule.
80 x − 15 + 78 2x 66. (d)
= 78. (a) ‘Pleasure’ is opposite of ‘Sorrow’ in
12 3
80 x + 63 × the same way ‘Right’ is opposite of
= 2x Cups Kettle
‘Wrong’.
4
80 x + 63 = 8 x Saucers
79. (d) Latter represents the ore of the
⇒ 63 = − 72x former.
63 7 80. (b) Former cannot be expected from the
x=− =− Conclusions I. (P)
72 8 II. (P) latter.

67. (a) One should not make a visitor wait. If 81. (a) ‘Doors’ are entry to a ‘House’ or
10 + 25 + 108 + 154 + 225 we can get into the house through door.
61. (c) a visitor is to be coming, then lunch must
3 In the same way, gates are entry to a
8 be prepared on time. So, only
Assumption I is implicit. compound or we can get into a
10 + 25 + 108 + 154 + 15 compound through ‘Gate’.
= 68. (d) The pattern is as follows
2 82. (d) As, ‘Hongkong’ is in ‘China’, in the
26 37 50 65 82 101
same way ‘Vatican’ is situated in ‘Rome’.
10 + 25 + 108 + 169
= +11 +13 +15 +17 +19 83. (a) As,
2
∴ ? = 82
10 + 25 + 108 + 13 S P R I N G
= 69. (b) Deduct the middle digit each time to
2 obtain the next number.
758 − 5 = 753, 753 − 5 = 748 % ? ∗ – $ ÷
10 + 25 + 121
= 748 − 4 = 744, 744 − 4 = 740
2 and P R A I S E
740 − 4 = 736, 736 − 3 = 733
10 + 25 + 11 10 + 36
= = ∴ ? = 733
2 2 70. (c) The letters in the given series are ? ∗ @ – % ×
10 + 6 16 4
= = = =2 alternate and the numbers indicate their Therefore,
2 2 2 position in the English alphabetical series A S P I R E
2 4 6 from the beginning.
62. (b) Given fractions = , and
3 5 7 ∴ ? = O –15 @ % ? – ∗ ×
HCF of fractions
HCF of numerators 71. (d) The direction diagram is as follows
= End point
84. (a) Here,
LCM of denominators D
HCF (2, 4, 6) 2 N bi nic pic = some good jokes
= = E
LCM (3, 5, 7) 105 1 km
W E nic bat lik = some real stories
63. (c) We know that, B
C S pic lik tol = many good stories
difference between SI and CI for 2 yr
2 A Starting
= P
R  So, code for jokes is ‘bi’.
 point
 100
46 AFCAT ~ Practice Set 5

85. (a)∵Angle traced by hour hand in 1 min So, these cubes cannot be formed. Hence,
1 °
=  
only the cube given in figure (d) can be
 2 formed.
∴Angle traced by hour hand in 40 min 90. (a) Answer figure (a) will complete the
1° problem figure. 97. (b) Common to the
= 40 × = 20° circle and square
2 91. (c) Answer figure (c) will complete the
86. (c) Except (c), all figure have their legs problem figure.
Common to the
facing in upward direction. 92. (c) Answer figure (c) will complete the triangle and circle
87. (d) In all others, the arrangement of dots problem figure.
is in a sequence i.e. 1-2-3-4-5 dots. 93. (c) Answer figure (c) will complete the
problem figure. 98. (c) Only circle
88. (d) Except figure (d), all the other figures
are same. They are just rotating from 94. (b) The problem figure is embedded in
one position to other. answer figure (b),
89. (d) Common to the
2 circle and triangle

5 1 Opposite 99. (b) Common to the


circle and square
Opposite

4
95. (d) The problem figure is embedded in Common to the
6 3 answer figure (d), triangle and square
Opposite
100.(c)
∴‘2’ is opposite to ‘4’, ‘1’ is opposite to Only triangle
‘6’ and ‘5’ is opposite to ‘3’.
In figure (a), ‘6’ and ‘1’ in figure (b) ‘3’
96. (d) The problem figure is embedded in
and ‘5’ and in figure (c) ‘2’ and ‘4’ lie Common to the
answer figure (d),
adjacent to each other. square and circle
AFCAT ~ Practice Set 6 47

AFCAT
AIR FORCE COMMON ADMISSION TEST (ONLINE )

Practice Set ~06


Time : 2 Hrs MM : 300

INSTRUCTIONS
1. The set contains a total of 100 questions, Comprising Verbal Ability in English, General Awareness, Numerical Ability and Reasoning and
Military Aptitude Test.
2. Each correct question carry 3 Marks and there will be negative marking of 1 Mark for each incorrect attempt.
3. Total time duration will be 2 hrs (120 minutes).
4. No marks will be deducted for unattempted questions.

Directions (Q. Nos. 1-3) In the Directions (Q. Nos. 4-5) Find out One very common brake is regressing—
following passage, some words have been which part of the sentence has an error. looking back every now and then at
omitted. For each blank, four alternatives If there is no error, then choose ‘No something already read. It is like stepping
are provided. Select the suitable error’ as your answer. backwards every few metres as you
alternative to fill the given blanks. 4. Beside (a)/ his mother he has two walk— hardly the way to move ahead
Mass communication is the delivery of aunts (b)/ who stay with him. (c)/ quickly. Regression may arise from a lack
news ideas and entertainment to No error (d) of confidence, vocabulary deficiency, or
thousands or millions of ......(1) 5. This photograph (a)/ appears to actually missing a word or phrase. It
simultaneously; it is a force with be (b)/ the best of the two. (c)/ No makes a long sentence seem even more
incalculable impact on today’s world. error (d) complex as the eyes frequently regress.
The success of mass communication Eye movement photographs of 12000
Directions (Q. Nos. 6 and 7) Fill in
rests upon the skills of the the blanks with correct word. readers in America showed that university
communicator to ......(2) the recipient’s students regress an average of 15 times in
thinking, to encourage-stir emotions, 6. The miser gazed ......... at the pile reading only 100 words. The average
to him or her to action. Mass of gold coins in front of him.
student of class four was found to look
communication is the one-to-one (a) avidly (b) admiringly
back 20 times. In short, regression
impact of one human intelligence upon (c) thoughtfully (d) earnestly
consumes one-sixth of your precious
......(3) carried on thousand fold 7. If you smuggle goods into the reading time. Release this brake and enjoy
simultaneously among individuals who country, they may be ... by the a spurt in reading speed.
have no direct personal contact. customs authority.
(a) possessed (b) punished
8. In the context of the
1. (a) people passage, what does ‘regression’
(b) pictures (c) confiscated (d) fined
mean?
(c) advertisements Directions (Q. Nos. 8-10) Read the (a) Lack of desire to improve the
(d) items following passage carefully and answer reading speed
2. (a) stimulate the questions that follow. (b) Looking back at what is already
(b) agitate How can you improve your reading read
(c) understand speed? By taking off the brakes. You (c) Lack of proper understanding of
(d) minimise what one reads
wouldn’t think of driving a car with the
(d) Comparing the reading speed
3. (a) another (b) people brake on. Yet as a reader you probably
of school and university students
(c) man (d) women have several brakes slowing you down.
48 AFCAT ~ Practice Set 6

9. In order to be a good reader you 17. She is one of the ........ 28. Which among the following
should personalities that you often see is the sacred book of the
(a) regress whenever necessary presenting TV reality shows Buddhists?
(b) be like a careful driver (a) Efervescent (b) Efervesent (a) Upanishad (b) Vedas
(c) not look back frequently while (c) Effervesent (d) Effervescent (c) Tripitaka (d) Jatakas
reading 18. Local companies were asked to 29. The exact spot at which the
(d) test your vocabulary frequently complete a .......... about their earthquake occurs is known as
10. According to the author reading exports. (a) focus (b) core
with regression is like (a) questionnaeir (b) questtionnaire (c) epicentre (d) pole
(c) questionnaire (d) questionaire 30. Amino acids are the building
(a) driving with poor quality brakes
(b) stepping backwards while 19. She was brought up in an ....... blocks of
walking after her parents died. (a) DNA and RNA
(c) using several brakes in order to (a) orphanage (b) orphanege (b) lipids
slow down (c) orphannage (d) orphainage (c) proteins
(d) making sudden spurts in reading 20. The ............ issued a warrant for (d) carbohydrates
speed his arrest. 31. Which among the following is
(a) maegistrate (b) magistrate primarily responsible for green
Directions (Q. Nos. 11-15) In the (c) magestrate (d) magisttrate house effect?
following questions, four alternatives (a) CO (b) CO 2
are suggested for the given idiom/phrase. Directions (Q. Nos. 21 and 22) In the
(c) CFC (d) O 3
Choose the alternative which best expresses following questions, out of the four
the meaning of given idiom/phrase. alternatives, choose the one which best 32. The satellite in vacuum
expresses the meaning of the given (a) is kept in orbit by remote control
11. To make clean breast of word. (b) is kept in orbit by retro-rocket
(a) To gain prominence
21. Facile (c) derives energy from gravitational
(b) To praise oneself field
(c) To confess without reserve (a) Arduous (b) Fashionable
(c) Humorous (d) Easy (d) does not require any energy for
(d) To destroy before it blooms orbiting
12. To keep one’s temper 22. Incongruous
33. The commando unit of the Indian
(a) To become hungry (a) Inconceivable
Air Force is named
(b) To be in a good mood (b) Inevitable
(a) Baaz (b) Garud
(c) To preserve ones energy (c) Inconsistent
(c) MARCOS (d) Ghatak
(d) To be aloof from (d) Incontrovertible
34. The red colour of human blood is
13. To drive home Directions (Q. Nos. 23-25) In the due to
(a) To find one’s roots following questions choose the word (a) myoglobin
(b) To return to place of rest opposite in meaning to the given word.
(b) haemoglobin
(c) Back to original position 23. Venerate (c) immunoglobulin
(d) To emphasise (a) Accuse (b) Abuse (d) None of the above
14. Ball’s in your court (c) Criticise (d) Defame
35. The National Calendar was adopted
(a) your responsibility now 24. Capacious by the Government of India in
(b) playing with balls (a) Changeable (b) Limited (a) 1965 (b) 1954
(c) to favour someone (c) Caring (d) Foolish (c) 1943 (d) 1957
(d) to take advantage of someone 25. Dour 36. The Ghodazari Wildlife Sanctuary
15. To keep at an arm’s length (a) Cheerful (b) Active (GWS) is situated in which Indian
(a) To keep very near (c) Young (d) Radical state?
(b) A safe place (a) Chhattisgarh (b) Tamil Nadu
26. The Dilwara temple at Mount
(c) To keep someone happy Abu in Rajasthan was built by the (c) Rajasthan (d) Maharashtra
(d) To avoid coming in contact followers of 37. Sunil Chhetri is related to which of
Directions (Q. Nos. 16-20) Choose the (a) Buddhism (b) Jainism the following sports?
correctly spelt word to fill the given blank. (c) Hinduism (d) Sikhism (a) Hockey (b) Volleyball
16. They refused to show ........ for 27. Who was the first Governor (c) Football (d) Kabaddi
their crimes. General of Bengal? 38. Which article of the Indian
(a) Repantance (a) Warren Hastings Constitution mentions about
(b) Repantence (b) Lord Cornwallis Financial Emergency?
(c) Repentance (c) Lord Wellesley (a) Article 340 (b) Article 330
(d) Repentence (d) Lord Minto (c) Article 360 (d) Article 350
AFCAT ~ Practice Set 6 49

39. What is the tenure of the Prime 49. A container contains 60 kg of 58. A sells a bicycle to B at a profit of
Minister of India? milk. From this container 6 kg of 20%. B sells it to C at a profit of
(a) Conterminous with the tenure of milk was taken out and replaced 25%. If C pays ` 225 for it, the
the Lok Sabha by water. This process was cost price of the bicycle for A is
(b) Conterminous with the tenure of repeated further two times. The (a) ` 110 (b) ` 125
President amount of milk left in the (c) ` 120 (d) ` 150
(c) As long as he enjoys the support of container is
59. If a boat goes a certain distance at
a majority in the Lok Sabha (a) 34.24 kg (b) 39.64 kg
30 km/h and comes back the same
(d) 5 years (c) 43.74 kg (d) 47.6 kg
distance at 60 km/h. What is the
40. Which country assisted India to 50. The simple interest on a sum for average speed for the total
establish the Koodan-Kulam Nuclear 5 yr is one-fourth of the sum. journey?
Power Station in Tamil Nadu? The rate of interest per annum is (a) 45 km/h (b) 50 km/h
(a) Russia (b) Canada (a) 5% (b) 6% (c) 4% (d) 8% (c) 40 km/h (d) 35 km/h
(c) Germany (d) France 51. A bus moving at a speed of 60. A can do a piece of work in
41. INS Viraat serves the Indian Navy. 45 km/h overtakes a truck 150 m 20 days and B in 15 days. With
It is a ahead going in the same direction help of C, they finish the work in
(a) submarine (b) gunboat of 30 s. The speeds of the truck is 5 days. C can alone do the work in
(c) aircraft carrier (d) freighter (a) 27 km/h (b) 24 km/h (a) 5 days (b) 6 days
(c) 25 km/h (d) 28 km/h (c) 10 days (d) 12 days
42. The capital city London is famous for
(a) the highest concentration of 52. A student is asked to multiply a 61. 5 yr ago, the average age of A, B, C
higher education 8 and D was 45 yr. With E joining
number by . Inspite of it he
(b) Madame Tussauds Museum 17 them now, the average age of all the
(c) global leader in banking and 8 five is 49 yr. How old is E?
divides the number by and get
finance 17 (a) 25 yr (b) 40 yr (c) 45 yr (d) 64 yr
(d) All of the above 225 more than the original
62. By selling at ` 55.80 per quintal a
result. The given number is
43. Which among the following dealer lost 7%. At what price
(a) 8 (b) 17 (c) 64 (d) 136
currencies is the most traded should he have sold it to gain 7%?
currency in the world? 53. 50 persons can do a work in (a) ` 65.40 (b) ` 700
(a) Pound (b) Yen 12 days, working 6 h per day. (c) ` 540 (d) ` 64.20
(c) Rupee (`) (d) Dollar 60 persons can do the same work
in 8 days, working x h per days. 63. The difference between simple
44. ‘Mitra Shakti’ is the joint military and compound interest
The value of x is
exercise between India and which (compounded annually) on a sum
1 2
another country? (a) 15 (b) 7 (c) 5 (d) 5 of money for 2 yr at 10% per
2 3
(a) USA (b) Sri Lanka annum is ` 65. The sum is
1 494
(c) Nepal (d) Russia 54. The value of + 999 × 99 is (a) ` 65650 (b) ` 65065
5 495
45. Gagan Narang is associated with (c) ` 6565 (d) ` 6500
which sports? (a) 90000 (b) 99000 64. ‘a’ divides 228 leaving a remainder
(a) Hockey (b) Football (c) 90900 (d) 99990 18. The biggest two digit value of
(c) Shooting (d) Volleyball 55. If the side of a square is increased ‘a’ is
46. Silent Valley National Park is by 5 cm, the area increased by (a) 21 (b) 35 (c) 30 (d) 70
located in 165 sq cm. What is the side of
65. The smallest number, which when
(a) Kerala the square?
divided by 12, 10 and 8, leaves
(b) Madhya Pradesh (a) 12 cm (b) 13 cm remainder 6 in each case is
(c) Andhra Pradesh (c) 14 cm (d) 15 cm (a) 246 (b) 186 (c) 126 (d) 66
(d) Chandigarh 56. A shopkeeper marks the price of 66. A statement is given followed by
47. Who became the first woman pilot his goods 20% higher than the two conclusions. You have to take
of the Indian Air Force to qualify original price. Due to increase in the given statement to be true
to undertake missions by day on a demand, he further increases the even, if they seem to be at
fighter aircraft? price by 10%. How much profit variance from commonly known
(a) Bhawana Kanth will he get? facts. Read the conclusions and
(b) Kamala Singh (a) 22% (b) 38% (c) 30% (d) then decide which of the
(c) Avni Chaturvedi 32% conclusions, if any, logically
(d) Srimala Das 57. Out of four numbers, the average follows from the two given
1 1 1
48. If x + y + z = 1, + + = 1 and of the first three is 18 and that of statements, disregarding the
x y z the last three is 16. If the last known facts.
xyz = − 1, then x3 + y3 + z3 is equal number is 19, then the first Statement The manager
to number is humiliated Sachin in the presence
(a) − 1 (b)1 (c) − 2 (d) 2 (a) 19 (b) 18 (c) 20 (d) 25 of his colleagues.
50 AFCAT ~ Practice Set 6

Conclusions Directions (Q. Nos. 72-74) In the Answer Figures


I. The manager did not like following questions, find out that
Sachin. answer figure in which the problem
II. Sachin was not popular with figure is embedded.
his colleagues.
(a) Only Conclusion I follows 72. Problem Figure (a) (b) (c) (d)
(b) Only Conclusion II follows
Directions (Q. Nos. 77-79) In each of
(c) Either I or II follows
the following questions, four figures are
(d) Neither I nor II follows
given. Three are similar in a certain way
Directions (Q. Nos. 67-71) In each of Answer Figures and so form a group. Find out which one
the following questions, choose that of the figures does not belong to that group.
figure which contains region(s) 77.
common to circle, square, triangle,
similar to that marked by the dot(s) in (a) (b) (c) (d)
figure (A).
(a) (b) (c) (d)
67. 73. Problem Figure
78.

(A)
Answer Figures (a) (b) (c) (d)

(a) (b) (c) (d)


79.

68.
(a) (b) (c) (d) (a) (b) (c) (d)
74. Problem Figure
(A) Directions (Q. Nos. 80-84) In each of
the following questions, select a figure
from the answer figures which will
continue the series as established by the
(a) (b) (c) (d) five problem figures.
Answer Figures
69. 80. Problem Figures
C D L C Z

(A) C C D D L
(a) (b) (c) (d)
1 2 3 4 5
Directions (Q. Nos. 75 and 76) In the Answer Figures
D C D CD D
following questions, find out that
(a) (b) (c) (d) answer figure which is embedded in the
L Z L Z L Z L Z
problem figure.
70. (a) (b) (c) (d)
75. Problem Figure
(A) 81. Problem Figures

(a) (b) (c) (d) Answer Figures 1 2 3 4 5


Answer Figures
71.

(A)
(a) (b) (c) (d) (a) (b) (c) (d)
76. Problem Figure 82. Problem Figures

(a) (b) (c) (d)

1 2 3 4 5
AFCAT ~ Practice Set 6 51

Answer Figures 88. Bud : Flower : : ? 97. Which of the following best
(a) Clay : Mud represents the relation among
(b) Sapling : Tree Goat, Cow, Milk and Hen?
(c) River : Glacier
(a) (b) (c) (d) (d) Bird : Tree

83. Problem Figures 89. Ideas : Brain


(a) Literature : Author (a) (b)
(b) Clouds : Ocean
(c) Money : Bank
1 2 3 4 5 (d) Planets : Earth
Answer Figures (c) (d)
90. Frankness : Blunt
(a) Rise : Awake (b) Weep : Laugh 98. Which of the following best
(c) Sickness : Death represents the relation among
(a) (b) (c) (d) (d) Rest : Activity Elephant, Carnivorous and Tiger?

84. Problem Figures Directions (Q. Nos. 91-95) In the


following questions, three alternatives
M
out of four are same in a certain way
and so form a group. Find the odd word (a) (b)
1 2 3 4 5 that does not belong to the group.
Answer Figures 91. (a) Papaya (b) Watermelon
(c) Jackfruit (d) Guava
(c) (d)
M
M

92. (a) Beijing (b) Melbourne


99. Which of the following diagrams
(a) (b) (c) (d) (c) Paris (d) Athens
indicates the best relation among
93. (a) Stag (b) Kitten Tennis fan, Cricket players and
85. If Sand is coded as Brick, ‘Brick’ Students?
(c) Colt (d) Fawn
is coded as ‘House’, ‘House’ as
‘Temple’, ‘Temple’ as ‘Palace’, 94. (a) Bitch (b) Horse
then where do you worship? (c) Cow (d) Vixen
(a) Palace (b) Temple 95. (a) Ring (b) Tyre
(c) Brick (d) House
(c) Plate (d) Bangle
Directions (Q. Nos. 86-90) In the (a) (b)
following questions, select the letter 96. Which of the following options
pair that has the same relationship as indicates the relation among
the original pair of words. Truck, Ship and Goods?
86. Horse : Hoof (c) (d)
(a) Man : Foot (b) Dog : Black
(c) Paise : Rupee (d) Pen : Pencil 100. In the following question, some of
(a) (b) the letters of the series are missing.
87. Tagore : Geetanjali Find the correct alternative
(a) Madam Curie : Radium consisting the missing letters.
(b) Shakespeare : Skylark bc_bca_cab_ab_a_ca
(c) Dickens : Oliver Twist (a) abcab (b) cabac
(d) Nobel : Dynamite (c) (d) (c) abccb (d) cabca

Answers
1 (a) 2 (a) 3 (a) 4 (a) 5 (c) 6 (a) 7 (c) 8 (b) 9 (c) 10 (b)
11 (c) 12 (b) 13 (d) 14 (a) 15 (d) 16 (c) 17 (d) 18 (c) 19 (a) 20 (b)
21 (d) 22 (c) 23 (b) 24 (b) 25 (a) 26 (b) 27 (a) 28 (c) 29 (a) 30 (c)
31 (b) 32 (c) 33 (b) 34 (b) 35 (d) 36 (d) 37 (c) 38 (c) 39 (c) 40 (a)
41 (c) 42 (d) 43 (d) 44 (b) 45 (c) 46 (a) 47 (a) 48 (b) 49 (c) 50 (a)
51 (a) 52 (d) 53 (b) 54 (b) 55 (c) 56 (d) 57 (d) 58 (d) 59 (c) 60 (d)
61 (c) 62 (d) 63 (d) 64 (d) 65 (c) 66 (d) 67 (c) 68 (d) 69 (c) 70 (d)
71 (a) 72 (a) 73 (b) 74 (d) 75 (a) 76 (a) 77 (d) 78 (d) 79 (c) 80 (c)
81 (d) 82 (d) 83 (d) 84 (d) 85 (a) 86 (a) 87 (c) 88 (b) 89 (c) 90 (a)
91 (b) 92 (b) 93 (a) 94 (b) 95 (c) 96 (a) 97 (d) 98 (d) 99 (a) 100 (c)
52 AFCAT ~ Practice Set 6

Hints and Solutions


1. (a) people 18. (c) The correctly spelt word is 29. (a) The place of the occurrence of an
2. (a) stimulate ‘Questionnaire’ which means ‘a set of earthquake is called focus. And the place
questions for obtaining statistically useful which experiences the seismic event first
3. (a) another information for individuals’. is called epicentre, which is located on
4. (a) ‘Beside’ should be replaced by 19. (a) The correctly spelt word is the Earth’s surface and is always
‘besides’ because ‘beside’ means ‘next to’ ‘Orphanage’ which means ‘an institution perpendicular to the focus.
and ‘besides’ means ‘except or in for the housing and care of orphans.’ 30. (c) Amino acids are building blocks of
addition to’. 20. (b) The correctly spelt word is proteins. Proteins are composed of 20
5. (c) ‘The best’ should be replaced by the ‘Magistrate’ who is an official entrust different amino acids (encoded by he
‘better’ because there is a comparison with administration of the laws. standard genetic code, construct proteins
between two objects (photographs). 21. (d) ‘Facile’ means ‘easily accomplished in all species). Proteins are made up of
6. (a) In the context of the sentence option (a) or achieved’. So, option (d) ‘easy’ is the amino acid monomers linked together by
‘avidly’ will be used to fill the given blank. correct synonym of ‘Facile’. peptide bonds.
‘Avidly’ is an adverb which means ‘with 22. (c) ‘Incongruous’ means ‘having 31. (b) CO2 (Carbon Dioxide) is the principal
great interest’. inconsistent or inharmonious parts, greenhouse gas and is primarily
7. (c) The word ‘Confiscated’ means ‘taken elements etc’. So, option (c) responsible for the greenhouse effect.
or seized (possession) with authority’. ‘Inconsistent’ best expresses the meaning Other gases responsible for greenhouse
So, in the context of the sentence option of ‘Incongruous’. effect include water vapour, methane,
(c) ‘confiscated’ is the correct choice to 23. (b) ‘Venerate’ means ‘regard with great nitrous oxide and ozone.
fill the given blank. respect’ and ‘abuse’ means ‘to treat 32. (c) The satellite revolves due to the
8. (b) ‘Regression’ means looking back someone rudely or cruelly’. Hence, gravitational force, that acts as a
every now and then at what is already ‘abuse’ is the word opposite in meaning centripetal force. Without the gravitational
read. to the word ‘venerate’. force satellite could not be able to revolve
9. (c) In order to be a good reader you 24. (b) ‘Capacious’ means ‘having a lot of around the Earth.
should not look back frequently while space and able to contain or hold much’ 33. (b) The Garud Commando Force is the
reading. and ‘limited’ means ‘restricted in size, special forces unit of the Indian Air Force.
amount or extent’. So, ‘Limited’ is It was formed in September 2004 and
10. (b) According to the author, reading with
opposite in meaning to ‘Capacious’. has current strength of over 1500
regression is like stepping backwards
while walking. 25. (a) ‘Dour’ means ‘unfriendly, personnel. The unit derives its name from
unhappy and very serious’ and ‘cheerful’ Garuda, divine bird-like creature in
11. (c) Idiom ‘To make clean breast of’ means
means ‘happy and positive’. Hinduism.
to confess without hiding anything.
So, ‘Cheerful’ is opposite in meaning to 34. (b) The red colour of human blood is due
So, option (c) ‘To confess without
‘Dour’. to haemoglobin present in it.
reserve’ is the correct choice.
26. (b) The Dilwara temple located at Mount Haemoglobin is a porphyrin protein
12. (b) Idiom ‘To keep one’s temper’ means present in RBCs of blood. This contains
Abu in Rajasthan was built by the
‘to remain calm’, therefore, option (b) ‘to iron as its central metal atom and
followers of Jainism. This temple built by
be in a good mood’, is the most suitable porphyrin ring around it.
between the AD 11th and 13th centuries
answer.
are world famous for their stunning use Its main function is to carry oxygen from
13. (d) Idiom ‘To drive home’ means to make of marble. The five legendary marble lungs to tissue and brings back CO2 from
clearly understood or to make a point. temples of Dilwara is a sacred pilgrimage tissue to lungs.
Therefore, option (d) ‘to emphasise’ is the
place of Jains. 35. (d) The National Calendar was adopted
most appropriate choice.
27. (a) The Regulating Act of 1773 created from 22nd March, 1957 along with the
14. (a) Idiom ‘Ball’s in your court’ means Gregorian Calendar. It is based on the
the office with title ‘Governor-General of
that ‘something is your responsibility Saka Era with Chaitra being its first
Bengal’. Warren Hasting was the first
now’. So, option (a) is the correct choice. month. It consists of 365 days in a
Governor-General of Bengal.
15. (d) Idiom ‘To keep at an arm’s length’ normal year.
He established India’s first Supreme
means ‘to keep distance from somebody 36. (d) Ghodazari Wildlife Sanctuary (GWS) is
Court in Calcutta and also abolished the
or something’. Hence, option (d) ‘to avoid a wildlife reserve established in 2018 in
dual system of administration.
coming in contact’ is a suitable choice. Chandrapur district, Maharashtra. With an
16. (c) The correctly spelt word is 28. (c) ‘Tripitaka’ is the sacred book of the
area of 159 sq km, the sanctuary is home
‘Repentance’ which means ‘the activity Buddhists. The book was written in Pali
to various animal species including tigers,
of reviewing one’s actions and feeling language and is based on the teachings Sambar, Chital, wild boar etc.
regret for past wrongs’. of Lord Buddha. Tripitaka is divided into
three sections namely Vinaya Pitaka, 37. (c) Sunil Chhetri is an Indian football
17. (d) The correctly spelt word is player who plays as a striker for the
‘Effervescent’ which means ‘active, Sutta Pitaka and the Abhidhamma
Indian National team.
positive and full of energy’. Pitaka.
AFCAT ~ Practice Set 6 53

In 2019, Sunil Chhetri is awarded with The 7th edition of ‘Mitra Shakti’ held 1
50. (a) Simple interest = Principal
the prestigious Padma Shri Award. from 1st December to 14th December, 4
2019 at Foreign Training Node (FTN) in SI × 100
He was also awarded with Arjuna Award ∴Rate =
in 2011 in recognition of his outstanding Pune. Principal × Time
achievement in sports. 45. (c) Gagan Narang is an Indian Shooter, 1 × 100
in Air Rifle shooting. He was the first = = 5% per annum
38. (c) Article 360 of the Indian Constitution, 4× 5
explains Financial Emergency imposed Indian to qualify for the London
Olympics. He won the Bronze Medal in 51. (a) Let the speed of the truck = x km/h
by the President.
the Men’s 10 m Air Rifle Event at the Relative speed of the bus
It is imposed in situation of financial
2012 Summer Olympics in London.
Unstability where the credit of India or = (45 − x) km/h
any part is threatened. 46. (a) Silent Valley National Park, is a Distance
National Park in Kerala. It is located in ∴ Time =
39. (c) The Prime Minister is generally the Relative speed
the Nilgiri hills and has a core area of
leader of a party that has a majority in 150
89.52 km2 , which is surrounded by a
the Lok Sabha, the lower house of the 30
buffer zone of 148 km2 . This national ⇒ = 1000
Parliament of India. 60 × 60 (45 − x)
park has some rare species of flora and
He remains in office till he enjoys the fauna. This area was explored in 1847 1 15
confidence of the house. However, if he ⇒ =
by the botanist Robert Wight. 120 100 (45 − x)
loses the confidence of the Lok Sabha,
47. (a) Bhawana Kanth is one of the first 1 3
he must resign or the President can ⇒ = ⇒ (45 − x) = 18
female fighter pilots of India to qualify to 6 (45 − x)
dismiss him.
undertake missions. She is from the first
40. (a) Russia assisted India to establish the batch of women fighter pilots of the IAF, ∴ x = 45 − 18 = 27 km/h
Koodan-Kulan Nuclear Power (KKNP) who joined the force in July 2016. 52. (d) Let the number = x
which is located in Tirunelveli district of Bhawana joined the fighter squadron in 17 8
Tamil Nadu. It is developed by the Nuclear November 2017 and flew the first solo ∴ x− x = 225
Power Corporation of India (NPCIL). 8 17
on MiG-21 Bison.
KKNP is India’s first pressurised water 289 − 64
48. (b) ∵ x + y + z = 1 ...(i) ⇒ x = 225
reactor belonging to the light water 8 × 17
category. 1 1 1 225 × 17 × 8
+ + =1 ⇒ x= = 136
41. (c) INS Viraat is a Centaur-class aircraft x y z 225
carrier. and xyz = − 1 53. (b) Here, M1 = 50, T1 = 6
INS Virat is currently the second largest We know that, D1 = 12, M2 = 60, D2 = 8
and oldest serving aircraft carrier in the
Indian Navy commissioned in 1959 as x 3 + y3 + z 3 yz = (x + y + z) Let, T2 = x
the Royal Navy’s HMS Hermes. (x 2 + y2 + z2 − xy − yz − zx) By the formula,
It was transferred to India in 1987. ⇒ x + y + z − 3(− 1)
2 2 2 M1T1D1 = M2T2 D2
42. (d) London is the capital of Great Britain, = 1 [x + y + z + 2(xy + yz + zx)
2 2 2 ⇒ 50 × 6 × 12 = 60 × x × 8
a famous European country. London is a 50 × 6 × 12 1
global leader in banking and financial − 3(xy + yz + zx)] x= =7
⇒ x 2 + y2 + z2 + 3 = (x + y + z)2 60 × 8 2
services. With 43 universities, London
has the highest concentration of higher  1 1 1 54. (b)
1
+ 999
494
× 99
− 3(xyz)  + + 
education in Europe. Madame Tussauds x y z 5 495

= +  999 + 1 −
Museum in London is world famous for
⇒ x 2 + y2 + z2 + 3 = 1 − 3(− 1) (1) 1 1 
 × 99
the wax effigy of the famous personalities 5  495
like Mahatma Gandhi, Sachin Tendulkar, ⇒ x 2 + y2 + z2 = 1 + 3 − 3
⇒ x 2 + y2 + z2 = 1 =
1
+ 1000 − 1  × 99
Narendra Modi, etc. from all over the  
world in various fields. 5  495
49. (c) Given, x = 60 kg, n = 3, a = 6
1 99 1 1
43. (d) The US Dollar ($) is the world’s n = + 99000 − = + 99000 −
Amount of left milk = x 1 − 
most-traded currency with about 47% a 5 495 5 5
share of global payments and 87% of the  x = 99000
forex market’s daily turnover. 3 3 55. (c) Let, original side = x cm
= 60 1 −
6  1
44. (b) ‘Mitra-Shakti’ is a joint military  = 60 1 − 
 60  10 Then, (x + 5)2 − x 2 = 165
exercise conducted annually between the
9 9 9 6 × 729 ⇒ x 2 + 25 + 10x − x 2 = 165
armies of India and Sri Lanka. The = 60 × × × =
exercise is a part of the United Nations 10 10 10 100 ⇒ 10x = 140
peacekeeping forces and aims to build 4374
= ∴ x = 14
and promote positive relations between 100
the two countries. ∴ Side of the square = 14 cm
= ` 43.74 kg
54 AFCAT ~ Practice Set 6

56. (d) Given that, x = 20% and y = 10% 62. (d) CP =


100
× SP 70. (d)
100 − Loss% Common to the
According to the formula, triangle, square and circle
100
Required profit = × 55.80 Common to the
(+ 20) × (+ 10) 100 − 7
= + 20 + (+ 10) + triangle and square
100 100
= × 5580
. = 60 71. (a)
= + 30 + 2 = + 32 % 93 Common to the triangle,
(100 + Gain%) circle and square
57. (d) Sum of first three numbers Now, SP = × CP
100 Common to the
= 18 × 3 = 54 triangle and circle
100 + 7
Sum of last three numbers = 16 × 3 = 48 = × 60 = ` 64.20
100 72. (a) Clearly, the problem figure is
∵ Last number is 19.
63. (d) Difference between CI and SI for 2 yr embedded in answer figure (a). The
∴Sum of 2nd and 3rd number portion which question figure occupies in
Pr2
= 48 − 19 = 29 = the alternative figure has been shown in
(100)2 the below figure.
Hence, first number = 54 − 29 = 25
P × 100
58. (d) Cost price for A ⇒ 65 = ⇒ P = ` 6500
100 × 100
100 100
= × ×R
100 ± x 100 ± y 64. (d) Given, divisor = a
100 100 and remainder = 18
= × × 225 73. (b) Clearly, the problem figure is embedded
100 + 20 100 + 25 We know that, in answer figure (b). The portion which
=
100 100
× × 225 Dividend = (Divisor × Quotient) question figure occupies in the alternative
figure has been shown in the below figure.
120 125 + Remainder
10 4
= × × 225 Through options
12 5
228 = (70 × 3) + 18
10
= × 45 Hence, biggest two digit value = 70
3 74. (d) Clearly, the problem figure is
65. (c) Required number = LCM of (12, 10,
= 15 × 10 = ` 150 embedded in figure (d) only.
8) + Remainder
59. (c) Here x = 30 km/h, y = 60 km/h
2xy 2 12, 10, 8
∴ Average speed =
x+ y 2 6, 5, 4
2 × 30 × 60 3, 5, 2
= 75. (a) Clearly, answer figure (a) is embedded
30 + 60
= 40 km/h ∴ Number = 120 + 6 = 126 in problem figure and has been shown in
the below figure.
60. (d) A can do a piece of work in 20 days. 66. (d) It can be possible that the manager
1 humiliated Sachin because of his
∴1 day work of A = mistake or any other negligence in his
20
1 work. So, Conclusion I does not follow.
Similarly, 1 day work of B = Also, nothing about Sachin’s rapport with
15 76. (a) Clearly, answer figure (a) is embedded
his colleagues can not be deduced from
1 in problem figure and has been shown in
and 1 day work of A, B and C = the statement. So, Conclusion II also
5 the adjacent figure.
does not follow.
1 1 1
∴1 day work of C = − − 67. (c)
5 20 15
12 − 3 − 4
= Common to the
60 circle and triangle
5 1 77. (d) In all other figures, except figure (d)
= =
60 12 arrow and black dot are at opposite sides.
68. (d)
∴ C can complete the work in 12 days. 78. (d) In all the figures, except figure (d)
Common to the group of three lines (||| or ≡≡ ) is on the
61. (c) Given, 5 yr ago, the average age of A,
anti-clockwise end.
B, C and D = 45 yr circle and square
79. (c) In all others, there is a pair of laterally
∴Total age 5 yr ago = 45 × 4 = 180
inverted images.
Present total age 69. (c) Common to the
80. (c) Inner circle becomes dark and white
= 180 + 4 × 5 = 180 + 20 = 200 triangle and square
alternately. All four elements are moving
Common to the
Sum of the present ages of A, B, C, D clockwise and a new element replaces the
triangle and circle
and E = 49 × 5 = 245 yr old one at upper corner right hand side.
∴Present age of E = 245 − 200 = 45 yr
AFCAT ~ Practice Set 6 55

81. (d) Line at the bottom in figure (1) is 87. (c) ‘Geetanjali’ is written by ‘Tagore’ and 97. (d) Milk
same as line at the bottom in figure (5). ‘Oliver Twist’ is written by ‘Dickens’.
Therefore, the line in answer figure must 88. (b) As ‘Bud’ grows and becomes a Hen
be same as in figure (2). Main figure is ‘Flower’; similarly, ‘Sapling’ grows and
same for two consecutive figures. Also, becomes a ‘Tree’. Goat Cow
the shaded portion of the circle in figure
89. (c) ‘Ideas’ are stored in ‘Brain’ and
(5) is the reverse of figure (1). Hence, it
‘Money’ is stored in ‘Bank’. Goat and cow both give milk and hen is
will be the reverse of figure (2)in answer
figure. 90. (a) ‘Frankness’ and ‘Blunt’ are synonyms different. So option (d) is correct.
and so are ‘Rise’ and ‘Awake’. 98. (d) Carnivorous
82. (d) Figure rotates in anti-clockwise
91. (b) All others grow on trees, while
direction through angles of 45°, 90°,
‘Watermelon’ grows on creepers. Elephant
45°, 90°,… and so on and every time Tiger
one line is deleted from the figure. 92. (b) All others are capitals of the
countries, while ‘Melbourne’ is the city of
83. (d) Left hand side element comes to right
Australia.
hand side in next figure and vice-versa. Tiger is carnivorous but elephant is different.
93. (a) All others are the young ones of
84. (d) From problem figure (1) to (2),the animals, while ‘stag’ is an adult animal. 99. (a)
symbol on the left rotates 90° clockwise 94. (b) All others are female animals.
and gets shifted to the right and a new Cricket players Tennis fans
95. (c) Except, ‘Plate’, all things have holes.
symbol is added on the left. The same
pattern follows from problem figure (3) to 96. (a) Trucks Students
Goods
(4) and (5) to answer figure.
Some students can be tennis fans or
85. (a) We do worship in temple. But as cricket players. Even some tennis fans
given temple is coded as palace. Ship can be cricket players.
So, we do worship in palace. 100.(c) Series pattern :
86. (a) ‘Hoof ’ is related to ‘Horse’, Truck and ship are entirely different. But
some goods are carried by trucks and bc/abc/abc/abc/abc/abca
in the same way, ‘Foot’ is related to
some by ships. ∴ Required = abccb
‘Man’.
56 AFCAT ~ Practice Set 7

AFCAT
AIR FORCE COMMON ADMISSION TEST (ONLINE )

Practice Set ~07


Time : 2 Hrs MM : 300

INSTRUCTIONS
1. The set contains a total of 100 questions, Comprising Verbal Ability in English, General Awareness, Numerical Ability and Reasoning and
Military Aptitude Test.
2. Each correct question carry 3 Marks and there will be negative marking of 1 Mark for each incorrect attempt.
3. Total time duration will be 2 hrs (120 minutes).
4. No marks will be deducted for unattempted questions.

1. Fill in the blank with the Directions (Q. Nos. 5-10) In the 8. (a) Rich (b) Lucky
appropriate pronoun. following passage, some words have been (c) Poor (d) Bad
My uncle decided to take .......... and omitted. For each blank, four alternatives 9. (a) writing (b) speaking
my sister to the market. are provided. Select the suitable (c) listening (d) reading
(a) I (b) mine alternative to fill the given blanks.
(c) me (d) myself 10. (a) himself (b) ourselves
Happy is the man who ...(5)... the habit
(c) yourselves (d) themselves
2. Fill in the blank with suitable of reading when he is young. He has
preposition. secured a life-long source of pleasure Directions (Q. Nos. 11-15) In the
Soniya was asked to report to office instruction and inspiration. So long as he following questions, four words are
........ 2 pm and 6 pm. has his beloved books he need ...(6)... given in each question, out of which only
(a) in (b) at feel lonely. He always has a pleasant one word is correctly spelt. Find the
(c) during (d) between possession of leisure moments. He is the correctly spelt word.
...(7)... of wealth more precious than 11. (a) Discription (b) Discretion
Directions (Q. Nos. 3 and 4) Find gold. ...(8)... indeed is the man who does (c) Dessemination (d) Dessertation
out the part of the sentence which has
not read and vacuum is his life. ...(9)... 12. (a) Occurence (b) Occassion
an error. If the sentence is free from
gives the highest kind of pleasure. When (c) Occupancy (d) Octogenarean
error, then choose ‘No error’ as your
answer. we are sick it is a healthy recreation to
13. (a) Luminous
lose ...(10)... in the company of books.
3. Either the operator (a)/ or the (b) Humanetarianism
foreman are (b)/ to be 5. (a) owns (b) buys (c) Hulabaloo
(c) acquires (d) takes (d) Hurrecane
blamed for the accident. (c)/
No error (d) 6. (a) always (b) ever 14. (a) Capracious (b) Auspicious
(c) Fallicious (d) Dalicious
4. The article offers (a)/ good advice (c) sometimes (d) never
to (b)/ whomever must accept it. 7. (a) possessor (b) loser 15. (a) Inapropriate (b) Inaccessible
(c)/ No error (d) (c) master (d) heir (c) Infallible (d) Invinscible
AFCAT ~ Practice Set 7 57

Directions (Q. Nos. 16-19) Read the Directions (Q. Nos. 20-24) In the 31. Which one is not a pollutant
following passage carefully and answer following questions, out of the four normally?
the questions that follow. alternatives, choose the one which can (a) Hydrocarbon
Many doctors flatly refused to believe be substituted for the given (b) Carbon dioxide
Jenner when he announced that he had words/sentence. (c) Carbon monoxide
found a preventive against smallpox. 20. Occurring at night (d) Sulphur dioxide
They declared vaccination to be a (a) Nightly (b) Dark 32. Which country is the largest
dangerous practice. But the dread of (c) Neurotic (d) Nocturnal emitter of SO 2, according to
smallpox was in everybody’s heart and 21. Determine the nature of the Greenpeace data?
people flocked to Jenner to be vaccinated. disease (a) Russia (b) India
The Latin word for cow is ‘Vacca’, it is (a) Investigate (b) Determine (c) China (d) Saudi Arabia
the root from which the word vaccination (c) Diagnose (d) Correct 33. Which Indian Armed Force has
was formed. Some of the ‘vacca’ used by launched OASIS (Officers
22. To present opposing arguments or
Jenner were not pure and some harms evidence Automated and Structured
were done; but when supplies of pure (a) Rebut (b) Criticise Information System) software?
vaccine were available, the practice of (c) Rebuff (d) Reprimand (a) Indian Army
vaccinating spread all over England and (b) Indian Air Force
from England to other countries. We 23. To mediate between two parties (c) Indian Navy
hardly hear of outbreaks of smallpox now. in a dispute
(d) Indian Coast Guard
(a) Interfere (b) Interact
16. The passage describes (c) Interrupt (d) Intercede 34. Which space agency has
(a) how smallpox may be treated successfully launched the Raman
(b) how vaccines were manufactured 24. Arrangement in order of Sat2 miniature satellite?
in England occurrence (a) NASA (b) ISRO
(c) the dangers of vaccination (a) Timely (b) Chronological (c) JAXA (d) Roscosmos
especially for children (c) Chronic (d) Temporal
35. Asia is located in which
(d) the gradual acceptance of Directions (Q. Nos. 25-27) In the hemisphere?
vaccination as a preventive following questions, out of the four
against smallpox (a) Eastern and Northern
alternatives. Choose the one which best Hemisphere
17. Vaccination sometimes proved expresses the meaning of the given (b) Western and Northern
harmful because word. Hemisphere
(a) vaccination was a dangerous 25. Impromptu (c) Southern and Northern
practice (a) Offhand (b) Unimportant Hemisphere
(b) some of the vaccines used were of (c) Unreal (d) Effective (d) Northern Hemisphere
a poor hygienic standard
26. Rabble 36. Which is the largest tribe in India?
(c) there are physiological
differences between cows and (a) Mob (b) Noise (a) Bhil (b) Gond
human beings (c) Roar (d) Rubbish (c) Baiga (d) None of these
(d) vaccination is given at a very 27. Troupe 37. PSLV stands for
early age (a) Polar Satellite Launch Vehicle
(a) Fast (b) Group
18. People hastened to get themselves (c) Medium (d) Energetic (b) Polish Satellite Launch Vehicle
vaccinated because (c) Perfect Satellite Launching
(a) many doctors supported Jenner’s Directions (Q. Nos. 28-30) In the Verifier
claims following questions, choose the word (d) Polar Satellite Launching Vehicle
(b) fear of the terrible disease drove opposite in meaning to the given word.
38. What is the full form of NATO?
them to take the risk of 28. Synchronous (a) North African Treaty
vaccination (a) Not in working order Organisation
(c) supplies of pure vaccine had now (b) Not in phase
become available (b) North Atlantic Treaty
(c) Without problems Organisation
(d) the practice of vaccinating had
(d) Surfiet (c) North Asian Treaty Organisation
spread all over the world
29. Vexation (d) North American Treaty
19. Vaccination was intended by Organisation
Jenner to (a) Patience (b) Pleasantness
(a) cure people suffering from (c) Displeasure (d) Dislike 39. ‘Faster, Higher, Stronger’ is the
smallpox 30. Undermine motto of which international
(b) delay the death of smallpox victims games event?
(a) Ensnare
(c) build up a defence against (a) FIFA World Cup
(b) Mollify
smallpox germs (b) Olympic
(c) Terminate
(d) prevent cows from spreading the (c) World Athletic Championship
(d) Bolster
disease (d) Afro-Asian Games
58 AFCAT ~ Practice Set 7

40. Which of the following are 51. The perimeter of a rectangular 59. Pure milk costs ` 16 per litre. After
functional items in the Eleventh field is 480 m and the ratio adding water the milkman sells the
Schedule of the Constitution? between the length and breadth mixture ` 15 per litre and thereby
(a) Public distribution system is 5 : 3. What is the area of the makes a profit of 25%. In what
(b) Small-scale industries field? respective ratio does he mix milk
(c) Fisheries (a) 13500 sq m (b) 14500 sq m with water?
(d) All of the above (c) 15000 sq m (d) 12500 sq m (a) 3 : 1 (b) 4 : 3
(c) 3 : 2 (d) 5 : 3
41. What is the name of the 1st ever
2  6 x 1 1 9 x 60. If 60% of ( x − y) = 20% of ( x + y),
spacecraft that can service 52. If  −  + = , then what
satellites in space? 35 4 3 5 what per cent of x is y?
is the value of x?
(a) RISAT-2B (b) Vostok (a) 80 (b) 50
1 −1
(c) MEV-1 (d) HYSIS (a) (b) (c) 40 (d) 60
6 6
42. When was the first 1 1 61. A shopkeeper bought 15 kg of rice
(c) (d) −
Commonwealth Games held? 5 5 at the rate of ` 29 per kg and
(a) 1930 (b) 1934 (c) 1938 (d) 1948 25 kg of rice at the rate of
53. The simplified value of ` 20 per kg. He sold the mixture of
43. What was the title of India’s first both types of rice at the rate of
newspaper?
5 + 11 + 19 + 29 + 49 is ` 27 per kg. His profit in this
(a) Punjab Mirror transaction is
(b) The Bengal Gazette (a) 3 (b) 2
(a) ` 125 (b) ` 150
(c) Azad Hind (c) 4 (d) 6
(c) ` 140 (d) ` 145
(d) Enlighten India 54. If the cost price of 16 tables be 62. Simple interest on a certain sum
44. At which of the following places is equal to the selling price of 9
the Heavy Vehicles Factory located? 12 tables, the gain per cent is for 6 yr is of the sum. The rate
25
(a) Kanpur (b) Khadki 1
(a) 33 % (b) 20% of interest is
(c) Avadi (d) 24 Paraganas 3 1
(c) 30% (d)15% (a) 6% (b) 6 %
45. At which of the following places 2
was a new Officers’ Training 55. A and B can do a piece of work in (c) 8%
1
(d) 8 %
Academy for training Army 72 days, B and C can do it in 2
officers established in 2011? 120 days and A and C can do it
63. A student rides on bicycle at
(a) Pune (b) Jabalpur in 90 days. When A, B and C
8 km/h and reaches his school
(c) Secunderabad (d) Gaya work together, how much work is
2.5 min late. The next day he
finished by them in 3 days?
46. India’s 1st 3-D Smart Traffic 1 1
increases his speed to 10 km/h and
Signal ‘Intelights’ was launched in (a) (b) reaches school 5 min early. How
40 30 far is the school from his house?
which city? 1 1
(a) New Delhi (b) Bengaluru (c) (d) 5
20 10 (a) km (b) 8 km
(c) Mohali (d) Gurugram 8
56. The average of 11 numbers is 63. (c) 5 km (d) 10 km
47. The outermost layer of the Earth If the average of first six numbers
is 64. The digit in the unit’s place of the
is 60 and the last six numbers is
(a) Magma (b) Mantle 65, then the 6th number is product
(c) Crust (d) Solid ironcore (a) 57 (b) 60 (2464)1793 × (615)317 × (131)491is
48. The first Indian ruler, who (c) 62 (d) 64 (a) 0 (b) 2 (c) 3 (d) 5
established the supremacy of 57. A fruit seller buys 700 oranges at
Indian Navy in the Arabian Sea 3
the rate of ` 500 for 100 oranges, 65. of total students in a class are
was 5 2
and another variety of 500 girls and remaining are boys. If
(a) Rajaraja I (b) Rajendra I 3
oranges at the rate of ` 700 for 1
(c) Rajadhiraja I (d) Kulottunga I of girls and of boys are absent,
100 oranges and sells them at 4
49. Who coined the word ` 84 per dozen. The profit per then which part of number of total
‘Nanotechnology’? cent is students are present?
(a) Eric Drexler (a) 20% (b) 40% 23
(b) Richard Feynman (a)
(c) 30% (d) 10% 30
(c) Sumio Iijima 18
58. Speed of a boat along and against (b)
(d) Richard Smalley
the current are 14 km/h and 49
50. Which country invented table 8 km/h, respectively. The speed of (c)
23
tennis? the current is 36
(a) France (b) Germany (a) 11 km/h (b) 6 km/h (d) None of the above
(c) England (d) Italy (c) 5.5 km/h (d) 3 km/h
AFCAT ~ Practice Set 7 59

Directions (Q. Nos. 66-68) What 72. Which of the following best Directions (Q. Nos. 85-89) In each of the
comes in place of question mark (?) in represents the correct relation following questions, select a figure from the
the series given below? among Children, Serious, Naughty? answer figures which will continue the series
66. AI, BJ, CK, ? as established by the four problem figures.
(a) DC (b) DM 85. Problem Figures
(c) DL (d) CM
(a) (b)
67. AHL, ?, CFJ, DEI
(a) BGK (b) BKG
(c) GKB (d) GBK (1) (2) (3) (4)
68. BCD, BDC, CBD, ?, DCB, DBC Answer Figures
(c) (d)
(a) CDB (b) CCD
(c) CDE (d) DCB Directions (Q. Nos. 73-77) In the
69. Which of the following diagram following questions, three alternatives
indicates the best relation among out of four are same in a certain way (a) (b) © (d)
Leaf, Seed and Root? and so form a group. Find the odd word 86. Problem Figures
that does not belong to the group.
73. (a) Silicon (b) Platinum
(c) Arsenic (d) Antimony
74. (a) Brass (b) Gun metal (1) (2) (3) (4)
(a) (b) (c) Bronze (d) Germanium
Answer Figures
75. (a) Wife (b) Bachelor
(c) Widow (d) Spinner
76. (a) Dilution (b) Distribution
(c) Dispersion (d) Diversion
(a) (b) (c) (d)
(c) (d) 77. (a) Quran (b) Gita
(c) Panchsheel (d) Bible 87. Problem Figures
70. Which of the following diagram
indicates the best relation Directions (Q. Nos. 78-82) In each of
among Mountains, Earth and the following questions, choose the term
Forests? that will complete the second pair in the
(1) (2) (3) (4)
same way as first pair.
Answer Figures
78. Brinjal : Vegetable : : Orange : ?
(a) Fruit (b) Stem
(c) Leaf (d) Root
(a) (b) 79. Contamination : Food : : Infection (a) (b) (c) (d)
:?
(a) Germs (b) Disease 88. Problem Figures
(c) Body (d) Microbes
80. AZCX : BYDW : : HQJO : ?
(a) GREP (b) IPKM
(c) (d) (1) (2) (3) (4)
(c) IPKN (d) GRJP
Answer Figures
71. Which of the following diagram 81. P × Q : 16 × 17 : : L × M : ?
indicates the best relation (a)13 × 12 (b)12 × 13
among Tiger, Four-footed and (c)16 × 13 (d)17 × 13
Elephant?
82. 386 : 383 : : 517 : ? (a) (b) (c) (d)
(a) 514 (b) 571 89. Problem Figures
(c) 715 (d) 512
83. If BAT = CXY, then CAT = ?
(a) (b)
(a) DXY (b) XDY
(c) DVW (d) None of these (1) (2) (3) (4)
84. If ‘PAINT’ is coded as 74128 and Answer Figures
‘EXCEL’ is coded as 93596, how
is ‘ACCEPT’ coded?
(c) (d) (a) 457958 (b) 459758
(c) 455978 (d) 459578 (a) (b) (c) (d)
60 AFCAT ~ Practice Set 7

Directions (Q. Nos. 90-92) In each of 95. Answer Figures


the following questions, four figures are
given. Three are similar in a certain
way and so form a group. Find out ZA YC YB ZD XD ?
which one of the figures does not belong (a) XA (b) YD (c) XC (d) YA (a) (b) (c) (d)
to that group.
96.
90. 99. Problem Figure

DM QP MM
(a) (b) (c) (d)

91. ?

CR DE ? Answer Figures
(a) (b) (c) (d)
(a) CP (b) QP (c) MC (d) DP
92. x Directions (Q. Nos. 97-99) In each of
x
cc xx the following questions, find the
(a) (b) (c) (d)
missing part of the problem figure.
(a) (b) (c) (d) 97. Problem Figure 100. Two statements followed by two
conclusions are given. Read the
Directions (Q. Nos. 93-96) Choose conclusions and then decide which
the correct code for the uncoded of the conclusion, if any logically
term.
follows from the two given
93. ? statements, disregarding the
known facts.
CU AT BS CT
Statements
Answer Figures
All cars are houses.
BR ?
Some horns are houses.
(a) CR (b) AS
Conclusions
(c) TS (d) CS
(a) (b) (c) (d) I. Some cars are horns.
94.
II. No car is horn.
98. Problem Figure
(a) Only Conclusion I follows
VP TO WN WP UM ? (b) Only Conclusion II follows
(a) TP (c) Either I or II follows
(b) VP (d) Both Conclusions I and II
(c) UN ? follow
(d) VO

Answers
1 (c) 2 (d) 3 (b) 4 (c) 5 (c) 6 (d) 7 (d) 8 (c) 9 (d) 10 (b)
11 (b) 12 (c) 13 (a) 14 (b) 15 (c) 16 (d) 17 (b) 18 (b) 19 (c) 20 (d)
21 (c) 22 (a) 23 (d) 24 (b) 25 (a) 26 (a) 27 (b) 28 (b) 29 (b) 30 (d)
31 (b) 32 (b) 33 (a) 34 (a) 35 (a) 36 (a) 37 (a) 38 (b) 39 (b) 40 (d)
41 (c) 42 (a) 43 (b) 44 (c) 45 (d) 46 (c) 47 (c) 48 (a) 49 (a) 50 (c)
51 (a) 52 (a) 53 (a) 54 (a) 55 (c) 56 (a) 57 (a) 58 (d) 59 (a) 60 (b)
61 (d) 62 (a) 63 (c) 64 (a) 65 (d) 66 (c) 67 (a) 68 (a) 69 (b) 70 (a)
71 (c) 72 (b) 73 (b) 74 (d) 75 (b) 76 (a) 77 (c) 78 (a) 79 (c) 80 (c)
81 (b) 82 (a) 83 (a) 84 (c) 85 (c) 86 (d) 87 (d) 88 (d) 89 (c) 90 (d)
91 (a) 92 (d) 93 (b) 94 (b) 95 (d) 96 (a) 97 (b) 98 (a) 99 (d) 100 (c)
AFCAT ~ Practice Set 7 61

Hints and Solutions


1. (c) The blank needs an objective case 18. (b) People hastened to get themselves 31. (b) The common gaseous pollutants are
pronoun. So, ‘me’ is the suitable usage. vaccinated because of the fear of the oxides of carbon (CO and CO2 ), oxides of
2. (d) Preposition ‘Between’ should be used terrible disease which drove them to take nitrogen (NO and NO2 ), oxide of sulphur
here as time interval is given. the risk of vaccination. (SO2 and SO 3). All these together
19. (c) Vaccination was intended by Jenner contribute 90% of the global air
3. (b) ‘Are’ should be replaced by ‘is’
to build up a defence against smallpox pollution. Out of all these CO2 is not a
because conjunction ‘either or’ connects
germs. pollutant normally, the green plants, by
two subjects, but the verb depends on
photosynthesis balance the CO2 and O2
the nearer subject. Here ‘foreman’ is 20. (d) ‘Nocturnal’ means ‘occurring at night’.
ratios in the air to a great extent.
nearer subject which is singular. Hence, option (d) is a suitable choice.
Whereas others like carbon monoxide
4. (c) ‘Whomever must accept it’ should be 21. (c) ‘Diagnose’ means ‘to recognise a (Co), NO2 , etc. are poisonous gases.
replaced by ‘those who accept it’ as disease by signs and symptoms’. Hence,
32. (b) India is the largest emitter of
‘must’ shows the compulsion which is option (c) ‘diagnose is the correct one
anthropogenic SO2 (Sulphur Dioxide) in
wrong to use in this context. word substitution.
the world as per the data released by
5. (c) acquires 22. (a) ‘Rebut’ means ‘to contradict or environmental NGO Greenpeace.
6. (d) never oppose by formal legal argument or Anthropogenic sulphur dioxide is produced
evidence’. So, option (a) ‘Rebut’ is a from burning of coal and it is known to
7. (d) heir
suitable choice. largely contribute to air pollution.
8. (c) poor
23. (d) ‘Intercede’ means ‘to mediate 33. (a) Indian Army has launched the ‘OASIS’
9. (d) reading between two parties in a dispute’. (Officers Automated Structures Information
10. (b) ourselves 24. (b) ‘Chronological’ is the arrangement of System) to digitize its official records. Under
11. (b) The correctly spelt word is ‘Discretion’. order in which a series of events the programme, the records of the army
The correct spelling of other words are happened’. Hence, option (b) officers are to be digitised. The software is
Discription → Description ‘chronological’ is a suitable choice. to be hosted in the Army’s Intranet.
Dessemination → Dissemination 25. (a) ‘Impromptu’ means ‘done without 34. (a) NASA successfully launched the
Dessertation → Dissertation being planned or without advance Raman Sat2 miniature satellite to the
12. (c) The correctly spelt word is thought’ and ‘offhand’ means ‘without edge of space. The miniature satellite
‘Occupancy’. previous thought or preparation’. measuring 4 cm × 4 cm × 4 cm has
The correct spelling of other words are So, ‘offhand’ expresses the correct been designed by 17 years old Indian
student Aabhaas Sikka, during his
Occurence → Occurrence meaning of ‘Impromptu’.
internship with SPACE India. This
Occassion → Occasion 26. (a) ‘Rabble’ means ‘a disorderly crowd or miniature satellite was launched from
Octogenarean → Octogenarian mob’. So, option (a) ‘mob’ expresses the NASA’s Columbia Scientific Balloon
13. (a) The correctly spelt word is correct meaning of ‘Rabble’. Facility in the US state of New Mexico
‘Luminous’. 27. (b) ‘Troupe’ is a group of performers such and achieved an altitude of 38 km.
The correct spelling of other words are as singers or dancers who work and 35. (a) Asia is the Earth’s largest and most
Humanetarianism → Humanitarianism travel together. So, option (b) ‘group’ populous continent, located primarily in
Hulabaloo → Hullabaloo expresses the suitable meaning of the Eastern and Northern Hemisphere.
Hurrecane → Hurricane ‘Troupe’.
It shares the continental landmass of
14. (b) The correctly spelt word is 28. (b) ‘Synchronous’ means ‘happening or Eurasia with the continent of Europe and
‘Auspicious’. existing at the same time’ and ‘Not in the continental landmass of Afro-Eurasia
The correct spelling of other words are phase’ means ‘not happening at the with both Europe and Africa.
Capracious → Capricious same time’. So, option (b) ‘Not in phase’
36. (a) According to the 2011 Census of
Fallicious → Fallacious is the correct choice. India, Bhil is the most populous tribe and
Dalicious → Delicious 29. (b) ‘Vexation’ means ‘the state of being the largest with a total population of
15. (c) The correctly spelt word is ‘Infallible’. annoyed, frustrated or worried’ and 4,618,068. It constitutes 37.7 per cent
The correct spelling of other words are ‘Pleasantness’ means ‘giving pleasure of the total ST population. Gond is the
or enjoyment’. So, ‘pleasantness’ is second largest tribe, with a population of
Inapropriate → Inappropriate
the word opposite in meaning to 4,357,918 constituting 35.6 per cent.
Inaccesible → Inaccessible
‘vexation’. 37. (a) The Polar Satellite Launch Vehicle
Invinscible → Invincible
30. (d) ‘Undermine’ means ‘to gradually (PSLV) is an expendable launch system
16. (d) The passage describes the gradual
make someone or something less strong developed and operated by ISRO. It was
acceptance of vaccination as a preventive
or effective’ and ‘bolster’ means ‘to developed to allow India to launch its
against smallpox.
support someone or something to make it Indian Remote Sensing (IRS) satellites
17. (b) Vaccination sometimes proved stronger’. So, ‘Bolster’ is opposite in into Sun synchronous orbits.
harmful because some of the vaccines meaning to ‘Undermine’. PSLV can also launch small size satellites
used were of a poor hygienic standard. into Geostationary Transfer Orbit (GTO).
62 AFCAT ~ Practice Set 7

38. (b) The North Atlantic Treaty course at the OTA prepares graduates for 53. (a) Expression
Organisation (NATO), is an all branches of the Army, except for the
inter-governmental military alliance Army Medical Corps. It has been set up = 5 + 11 + 19 + 29 + 49
based on the North Atlantic Treaty, at Gaya (Bihar) in 2011.
signed on 4th April 1949. 46. (c) ‘Intelights’, India’s first wireless 3-D = 5 + 11 + 19 + 29 + 7
The organisation constitutes a system of Smart Traffic Signal system, has been
collective defence whereby its member launched by Mohali Traffic police. = 5 + 11 + 19 + 6
states agree to mutual defence in The signal system will regulate traffic = 5 + 11 + 5
response to an attack by any external signals using a wireless sensor system.
party. NATO’s headquarters is in The signal system has been installed = 5+ 4 = 9 = 3
Brussels, Belgium. near the Mohali’s Airport Road. 54. (a) Here, x = 16 and y = 12
39. (b) ‘Faster, Higher, Stronger’ is the motto 47. (c) Earth’s outermost, rigid, rocky layer is
of Olympic games. It was proposed by ∴ Required gain per cent
called the crust. It is composed of
Pierre de Coubertin upon the creation of x − y 
low-density, easily melted rocks. = × 100 %
International Olympic Committee in  y 
The Earth mainly consists of four
1894. 16 − 12
concentric layers; inner core, outer core, = × 100%
40. (d) Eleventh Schedule contains total 29 mantle and crust (the outermost layer). 12
functional items which are placed under 1
the ambit of Panchayats, i.e. public
48. (a) Chola King Rajaraja I created a = × 100%
powerful standing army and a 3
distribution system, fisheries, small-scale 1
considerable Navy. One of the last = 33 %
industries including food processing
conquests of Rajaraja was the naval 3
industries, animal husbandry, dairying
conquest of the ‘old islands of the sea 55. (c) According to the question,
and poultry, agriculture, land
numbering 12,000, the Maldives. 1
improvement, water management, social (A + B)’s 1 day’s work = ...(i)
forestry and farm forestry, khadi, village Chola Navy also had played a major role 72
and cottage industries, drinking water in the invasion of Lanka. 1
(B + C)’s 1 day’s work = ...(ii)
and education including primary and 49. (a) The term ‘Nanotechnology’ was 120
secondary schools, etc. independently coined and popularised by 1
(C + A)’s 1 day’s work = ...(iii)
41. (c) US-based Northrop Grumman Eric Drexler. It referred to a future
90
Corporation, has launched the 1st manufacturing technology based on
molecular machine systems. On adding all three equations
satellite servicing spacecraft robotic
‘Mission Extension Vehicle-1 (MEV-1)’. Nanotechnology (Nanotech) is 2(A + B + C)’s 1 day’s work
manipulation of matter on an atomic, 1 1 1 5+ 3+ 4 1
The MEV-1 is a satellite service = + + = =
molecular and supramolecular scale. 72 120 90 360 30
spacecraft with a mission to provide
50. (c) Table tennis, also known as 1
service to the Intelsat 901 by making ∴ (A + B + C)’s 1 day’s work =
use of its orientation thrusters to put it ping-pong is a sport in which two or four 60
back into an ideal target orbit. players hit a lightweight ball back and 3 1
∴ (A + B + C)’s 3 day’s work = =
Fourth across a table using small 60 20
42. (a) The first Commonwealth Games
rockets.
(formerly known as British Empire 56. (a) Sum of 11 numbers = 11 × 63 = 693
Games) were held in Hamilton, in the Ping pong is a trademark name for table
tennis and associated equipment. Sum of first six numbers = 6 × 60 = 360
province of Ontario in Canada from 16th
The name Ping-pong was invented by Sum of last six numbers = 6 × 65 = 390
to 23rd August, 1930.
the English firm J Jaques and Son at the ∴ 6th number = 390 + 360 − 693 = 57
43. (b) The first newspaper printed in India
was Hicky’s Bengal Gazette or The end of the 1800s. 57. (a) CP of 700 oranges
51. (a) Let the length = 5x 500
Bengal Gazettee. The newspaper was = × 700 = ` 3500
started in 1780 under the British Raj by and breadth = 3x, Then 100
James Augustus Hicky. It was a weekly Perimeter = 2 (l + b) CP of 500 oranges of another variety
English newspaper and was founded in 700
2(5x + 3x) = 480 ⇒ x = 30 = × 500 = ` 3500
Calcutta the capital of British India. 100
∴ I = 150 m and b = 90 m Total CP = 3500 + 3500 = ` 7000
44. (c) The Heavy Vehicles Factory (HVF) is
located at Avadi in Chennai in the Indian ∴ Area = 150 × 90 and total number of oranges purchased
state of Tamil Nadu. HVF was set up in = 13500 sq m = 700 + 500 = 1200
52. (a) Given, 
1961 by the Ordnance Factory Board, 2 6x 1  1 9x Total SP =
84
× 1200 = ` 8400
−  + =
Government of India to manufacture 3 5 4 3 5 12
heavy battlefield equipment. 4x 1 1 9x SP − CP
⇒ − + = ∴ Profit per cent = × 100%
45. (d) The Officers Training Academy (OTA) 5 6 3 5 CP
are training establishments of the Indian − 1 + 2 9x − 4x 8400 − 7000
⇒ = ⇒x =
1 = × 100%
Army that train officers for the Short 6 5 6 7000
Service Commission (SSC). The 49-week = 20%
AFCAT ~ Practice Set 7 63

58. (d) Downstream speed of boat, 64. (a) Given, 69. (b) The best relation between leaf, seed
v = 14 km/h (2464)1793 × (615)317 × (131)491 and root is as follows
Upstream speed of boat, u = 8 km/h Then, unit’s digit of (2464)1793 Seed Root
v−u
∴Speed of current = Remainder of
1793
=1
2 4
14 − 8 Leaf
= = 3 km/h Hence, unit’s digit of (2464)1793 = 4
2
59. (a) ∵ SP of the mixture = ` 15 Unit’s digit of (615)317 = 5 All Leaf, Seed and Root are different from
SP × 100 Unit’s digit of (131)491 = 1 one other.
∴ CP of the mixture =
100 + Gain ∴Required unit’s digit
70. (a) Earth
100
= 15 × = ` 12 = Unit’s digit in (4 × 5 × 1) Forests Mountains
125
= Unit’s digit in 20 = 0
By Alligation Rule
65. (d) Let the total number of students = x
Milk Water Some mountains are forest and some
3x
16 0 ∴Number of girls = forests are mountainous. But, both
12 5 mountains and forests are part of Earth.
3x 2x
12 4 and number of boys = x − = 71. (c)
5 5
∴Ratio of milk and water in mixture 1 2x
Number of absent boys = × =
x Elephant Tiger
= 12 : 4 = 3 : 1
4 5 10
60. (b) ∵ 60% of (x − y) = 20% of (x + y) 2 3x 2x Four-footed
Number of absent girls = × =
60 20 3 5 5
⇒ × (x − y) = × (x + y) Tigers and Elephants are four-footed and
x 2x
100 100 Number of absent students = + are different animals.
⇒ 6(x − y) = 2(x + y) 10 5
x + 4x x 72. (b) Children
⇒ 6x − 6 y = 2x + 2 y = =
10 2
x 8 Naughty Serious
⇒ 4x = 8y ⇒ = = 2 ∴Number of present students = x −
x
y 4 2
x
∴Required percentage =
y
× 100 = Some children are naughty and some are
x 2 serious.
1
1 Hence, part of total students were 73. (b) Silicon, Arsenic and Antimony are
= × 100 = 50% 2
2 metalloids while ‘Platinum’ is an
present.
61. (d) CP of 40 kg of mixture element.
66. (c) The pattern of the series is as follows
= ` (15 × 29 + 25 × 20) 74. (d) Brass, Bronze and Gun metal are
+1 +1 +1
alloys while ‘Germanium’ is an element.
= ` (435 + 500) = ` 935
Al BJ CK DL 75. (b) Wife, Widow and Spinner can only be
SP of 40 kg of mixture
a woman. While either a man or a
= 27 × 40 = ` 1080 woman can be ‘Bachelor’.
+1 +1 +1
∴Gain = 1080 − 935 = ` 145 76. (a) Except ‘Dilution’ all others signifies
9 ∴ ? = DL
62. (a) Given, SI = Principal division. While Dilution is a liquid that
25 67. (a) The pattern of the series is as follows has been diluted with liquid.
SI × 100 1 +1 2 +1 3 +1 4
Rate = A B C D
77. (c) Except ‘Panchsheel’, all are holy
Principal × Time books of different religions.
8 −1 7 −1 6 −1 5
9 100 H G F E 78. (a) ‘Brinjal’ is a ‘Vegetable’, in the same
= × = 6% per annum
25 6 12 −1 11 −1 10 −1 9 way ‘Orange’ is a ‘Fruit’.
L K J I
63. (c) Let x km be the required distance. 79. (c) ‘Food’ gets affected by
Difference in time ∴ ? = BGK ‘Contamination’, in the same way ‘Body’
68. (a) The pattern of the series is as follows gets affected by ‘Infection’.
7.5 1
= 2.5 + 5 = 7.5 min = h= h 80. (c) As, A Z C X
60 8 2 3 4 3 2 4 4 3 2
B C D C B D D C B +1 −1 +1 −1
According to the question,
x x 1 B Y D W
− = B D C C D B D B C
8 10 8 2 4 3 3 4 2 Similarly, H Q J O
5x − 4x 1 40 +1 −1 +1 −1
⇒ = ⇒x = = 5 km ∴ = CDB
40 8 8 I P K N
64 AFCAT ~ Practice Set 7

81. (b) In forward order letter sequence, are placed along the different sides of the From the above codes, we see that code
Position of P = 16, Position of Q = 17 figures. for last figure is CP, where C specifies
∴P × Q = 16 × 17 93. (b) Here, the coded series can be outer figure and P specifies inner figure.
Similarly, Position of L = 12, decoded as Hence, option (a) is correct.
Position of M = 13 97. (b) Here, all the three equal parts have
U, T,
∴ L × M = 12 × 13 exactly same design. So, the fourth part
S,
∴ ? = 12 × 13 R with question mark (?) must also contain
82. (a) As, 386 − 3 = 383 the same design to complete the question
and the dots at either end can be shown as figure as shown in the following figure
Similarly, 517 − 3 = 514 C, A, B
83. (a) As, B A T C X Y
From the above codes, we see that the
+1 code for the last figure is AS, where A
–3
+5
specifies ends of wave form and S
specifies shape of waves.
Similarly, C A T D X Y Hence, option (b) is correct. 98. (a) Here, all the three equal parts have
+1 94. (b) Here, the coded series can be similar design. So, we can obtain the
–3 decoded as 3 white boxes = P, one answer figure for the missing portion by
+5
white box = N, two white boxes = U/M, rotating the question figure by 90°
84. (c) As, P 5 white boxes = O and 2 black boxes clockwise.
A I N T
= V, three black boxes = W, one black
90° clockwise
box = U/M and zero black box = T. rotation
7 4 1 2 8
From the above codes, we see that code ?

?
and E X C E L for last figure is VP, where V specifies
number of black boxes and P specifies (X) (X′)
number of white boxes.
9 3 5 9 6 Now, by comparing figure (X) with (X′),
Hence, option (b) is correct. we get
Therefore, A C C E P T
95. (d) Here, the coded series can be
decoded as ? =
4 5 5 9 7 8
85. (c) The circle moves half-a-side of the A, C, D,
square boundary in clockwise direction 99. (d) Here, all the three equal parts have
in each step. similar design. So, we can obtain the
answer figure for the missing portion by
86. (d) The figure gets laterally inverted and B and for black dots as
rotating the question figure by 90°
the number of arrows increases by one
clockwise.
in each step.
Z, Y, X
87. (d) In each step, the figure gets vertically
inverted and a line segment is added to
the RHS end. i.e. for two black dots = Z
88. (d) Similar figure repeats in every second one black dot = Y
Now, it is clear that answer figure (d)
step. and zero black dot = X
completes the question figure.
89. (c) Half portion of a rectangle is lost in From the above codes, we see that code
100.(c)
one step and one complete rectangle is for the last figure is YA, where Y specifies
lost in the next step. number of dots and A specifies the grid.
90. (d) In all other figures except (d), any Hence, option (d) is correct.
Cars Horns
two of the symbols are identical whereas 96. (a) Here, the coded series can be
in figure (d) all the symbols are different. decoded as Houses
91. (a) In figure (a), two of the cups are For outer/large figures,
placed in the same direction. In all other
D, Q, C and Conclusions I. May be true
figures, all the cups are placed in
or
different directions. For inner/small figures, II. May be true
92. (d) In all the other figures except (d), M, P, R, E ∴ Either I or II follows.
symbols inside and outside the figures
AFCAT ~ Practice Set 8 65

AFCAT
AIR FORCE COMMON ADMISSION TEST (ONLINE )

Practice Set ~08


Time : 2 Hrs MM : 300

INSTRUCTIONS
1. The set contains a total of 100 questions, Comprising Verbal Ability in English, General Awareness, Numerical Ability and Reasoning and
Military Aptitude Test.
2. Each correct question carry 3 Marks and there will be negative marking of 1 Mark for each incorrect attempt.
3. Total time duration will be 2 hrs (120 minutes).
4. No marks will be deducted for unattempted questions.

Directions (Q. Nos. 1-5) Read the 2. ‘They can be ordered by postcard’ Directions (Q. Nos.6-8) Find out
following passage carefully and means that the part of the sentence that has an
answer the questions that follows. (a) the present may only be a error. If the sentence is free from
postcard error, then choose ‘No error’ as your
It is possible to give wedding presents,
(b) the present would be an answer.
birthday and Christmas presents, without
expensive one
any thought of affection at all, they can 6. All scientists agree (a)/ that there
(c) the choice does not involve much
be ordered by postcard; but the should be (b)/ a total ban on
care
unbirthday present demands the nicest nuclear explosions. (c)/ No
(d) the present would not be worth
care. It is therefore the best of all and it is giving error (d)
the only kind to which the golden rule of 7. Tagore was (a)/ one of the greatest
present-giving imperatively applies the 3. The ‘unbirthday’ present is the
best of all because poet (b)/ that ever lived (c)/
golden rule which insists that you must No error (d)
never give to another person anything (a) it cannot be ordered by postcard
(b) it means giving expensive 8. There has been (a)/ a number
that you would not rather keep: nothing
presents of railway accidents (b)/
that does not cost you a pang to part
(c) its choice needs the utmost care during the last month. (c)/
from. It would be better if this rule (d) other occasions are better than No error (d)
governed the choice also of those other birthdays for giving presents
three varieties of gifts, but they can be 9. Insert proper preposition in the
less exacting. 4. A ‘golden rule’ is a rule which sentence.
(a) brings profit He stumbled .......... the rocks
1. The author says that wedding, (b) is very important while crossing the river.
birthday and Christmas presents (c) is very difficult (a) over (b) upon
(a) are always indicators of the (d) is very easy (c) into (d) through
giver’s affection
(b) may not always be given with 5. The writer is of the view that one 10. Fill in the blank with the proper
any thought of affection should give a present that infinitive.
(c) are given only to flatter the (a) one would like to possess onesets We fixed plywood on the
recipient (b) one would like to get rid of windows .... for the storm.
(d) are given only to fulfil an (c) cannot be ordered by mail (a) preparing (b) to prepare
obligation (d) is highly expensive and attractive (c) to prepared (d) preparation
66 AFCAT ~ Practice Set 8

11. Fill in the blank with the suitable Directions (Q. Nos. 21-24) Choose 34. Operation flood refers to
adjective. the correctly spelt word. (a) increase in the production of
Kalpana always buys .......... 21. (a) Fullfil (b) Foulfil milk
chocolates than the rest of us. (b) increase in the production of dairy
(c) Fulfill (d) Fullfill
(a) few (b) less products
(c) lesser (d) fewer 22. (a) Necessary (b) Necessarry (c) controlling flood
(c) Necesary (d) Neccessary (d) increasing the production of
Directions (Q. Nos. 12-15) In the agricultural crops
following questions, out of the four 23. (a) Gaurantee (b) Guarantee
alternatives, choose the one which (c) Garuntee (d) Guaruntee 35. Which of the following sites has
can be substituted for the given been included in UNESCO’s list of
24. (a) Exemple (b) Exampel World Heritage Sites?
words/sentence. (c) Example (d) Exampal (a) Chilika Lake
12. To send an unwanted person out (b) Dal Lake
of the country Directions (Q. Nos. 25-27) In the
following questions, out of the four (c) Negin Lake
(a) Exclude (b) Ostracise (d) Sunderbans National Park
(c) Deport (d) Expatriate alternatives, choose the one which
best expresses the meaning of the 36. A light year is a unit of
13. Indifference to pleasure of pain given word. (a) time (b) distance
(a) Docility (b) Stoicism (c) speed of light (d) intensity
(c) Patience (d) Reticence
25. Rancour
(a) Bitterness (b) Energy 37. Who discovered the atom bomb?
14. The doctrine that human souls (c) Fatigue (d) Tolerance (a) Madam Curie (b) Pierre Curie
pass from one body to another at (c) Otto Hahn (d) Albert Einstein
the time of death 26. Sedate
(a) Metamorphosis(b) Transition (a) Long-delayed (b) Offensive 38. The pH of human blood is
(c) Transmigration (d) Extrapolation (c) Secular (d) Staid (a) 7.2 (b) 7.8
27. Tedium (c) 6.6 (d) 7.4
15. A drawing on transparent paper
(a) Red print (b) Blue print (a) Boredom 39. India’s first ‘Tree Ambulance’
(c) Negative (d) Transparency (b) Magnitude launched in
(c) Joke (a) Delhi (b) Mumbai
Directions (Q. Nos. 16-20) In the (d) Supplementary (c) Chennai (d) Hyderabad
following passage, some words have
Directions (Q. Nos. 28-30) In the 40. Which government organisation
been omitted. For each blank, four released ‘Vision 2021’?
alternatives are suggested. Select the following questions, choose the word
opposite in meaning to the given (a) ISRO (b) DRDO
suitable alternative to fill the given
word. (c) RBI (d) NCERT
blanks.
28. Garrulous 41. The first Indian woman to get an
Some people believe that marriages are
(a) Soft-spoken (b) Reticent Olympic medal is
made in heaven. One cannot say (16)
(c) Peaceful (d) Kind (a) PT Usha
this is true or not. However, in America
(b) Karnam Malleshwari
now many (17) who get married seek to 29. Forbearance (c) Ashwini Nachappa
evolve a fool proof (18) to ensure that (a) Patience (b) Self-control (d) Usha Sachdev
the marriage survives. However, the idea (c) Intolerance (d) Preference
of married persons (19) the burden of 42. Right to Education became a
30. Predilection Fundamental Right on
domestic chores, instead of all the dirty
(a) Confusion (b) Detestation (a) 15th March, 2010
work being dumped on the woman has
(c) Abnormality (d) Desperateness (b) 1st April, 2010
(20) been propagated by the feminist
lobby and it has gradually, if grudgingly, 31. The famous rock-cut temple of (c) 17th July, 2010
been accepted. Kailasa is at (d) 10th October, 2010
(a) Ajanta (b) Badami 43. Right to Vote is mentioned in
16. (a) whether (b) still
(c) Mahabalipuram (d) Ellora the parts of the Constitution
(c) however (d) that
32. Which country has launched the relating to
17. (a) pairs (b) men biggest transport spaceship for (a) Fundamental Rights
(c) couples (d) women space station? (b) Union Legislature
18. (a) method (b) project (a) Israel (b) Japan (c) State Legislature
(c) plan (d) system (c) Germany (d) Poland (d) Election

19. (a) executing (b) dividing 33. The longest river in Asia is 44. What is the name of the Light
(a) river Indus Combat Aircraft developed by
(c) undertaking (d) sharing
(b) river Yangtze India indigenously?
20. (a) long (b) sometimes (c) river Hwang Ho (a) Brahmos (b) Chetak
(c) always (d) occasionally (d) river Ganga (c) Astra (d) Tejas
AFCAT ~ Practice Set 8 67

45. Which one of the following is a 55. A and B together can complete a 63. If 4 x = 5 + 2, then the value of
‘Surface to Air Missile’? work in 8 days and B and C 1
4x − is
(a) Trishul together in 12 days. All of the 16 x
(b) K-15 Sagarika three together can complete the (a) 1 (b) − 1 (c) 4 (d) 2 5
(c) Brahmos work in 6 days. In how much time
will A and C together complete 64. B and C can complete a piece of
(d) Agni
the work? work in 12 days, C and A can do
46. Headquarters of World Health (a) 8 days (b) 10 days it in 8 days. All the three can do it
Organisation (WHO) are situated in 6 days. A and B together can
(c) 12 days (d) 20 days
at complete it in
(a) Hague (b) Oslo 56. Average weight of 25 persons is (a) 4 days (b) 6 days
(c) Geneva (d) New York increased by 1 kg, when one man (c) 8 days (d) 10 days
weighing 60 kg is replaced by a
47. The first woman to get the Bharat new person. Weight of new 65. The cost price of 25 articles is
Ratna Award is person is equal to the selling price of 20 of
(a) Mother Teresa (a) 50 kg (b) 61 kg them. The gain or loss per cent is
(b) Indira Gandhi (c) 86 kg (d) 85 kg given by
(c) Lata Mangeshkar (a) 20% loss (b) 25% gain
(d) Sarojini Naidu
57. If a person purchased 11 books (c) 60% loss (d) 75% gain
for ` 100 and sold 10 books for
48. Ozone Day is celebrated on ` 110, then percentage of profit 66. The sum of a natural number and
(a) 21st April per book sold is its square is 90. Find the number.
(b) 16th September (a) 10 (b) 11.5 (c) 17.3 (d) 21 (a) 8 (b) 9 (c) 10 (d) 11
(c) 25th September 1 1 1
58. An alloy contains zinc, copper and 67. If x + y + z = 1, + + = 1 and
(d) 5th June tin in the ratio 2 : 3 : 1 and another x y z
49. The first woman to swim across contains copper, tin and lead in xyz = −1, then find the value of
seven important seas is the ratio 5 : 4 : 3. If equal weights
(a) Chandini of both alloys are melted together x3 + y3 + z3 .
(b) Bula Choudhary to form a third alloy, then the (a) 0
(c) Mridula Rajiv weight of lead per kg in the new (b) 1
(d) Priya Shanbag alloy will be (c) −1
1 1 3 7 (d) Cannot be determined
50. Bt seed is associated with (a) kg (b) kg (c) kg (d) kg
2 8 14 9 68. In what time will ` 72 become
(a) rice (b) wheat 1
(c) cotton (d) oil seeds 59. A man standing on a platform ` 81 at 6 % per annum simple
finds that a train takes 3 s to pass 4
51. If in 3 yr at simple interest, the him and another train of the same interest?
principal increases by 18%, what length moving in the opposite 1
(a) 1 yr (b)1 yr
will be the compound interest direction takes 4 s. The time 2
earned on ` 25000 in 3 yr at the taken by the trains to pass each 1
same rate? (c) 2 yr (d) 2 yr
other will be 2
(a) ` 4775.4 (b) ` 5775.4 3 3 3 3
(c) ` 4557.4 (d) ` 5575.4
(a) 2 s (b) 3 s (c) 4 s (d) 5 s 69. One statement followed by two
7 7 7 7 assumptions numbered I and II is
1
52. If x + = 2, then what is x 60. If the price of the cooking gas given. You have to consider the
1
1+ increased by 15%. By how many statement and the following
1
2+ per cent should a family reduce its assumptions and decide which of
3 consumption, so as not to exceed the assumptions is implicit in the
equal to? its budget on cooking gas? statement.
7 13 1 1 Statement
(a) (b) (a) 20 (b) 16 (c)13 (d)12
10 10 23 7 The government has withdrawn
11 17 12 % tax levied on air tickets in
(c) (d) 61. A fruit seller had some apples. He
10 10 the domestic sector with
sells 40% apples and still has 420 immediate effect.
53. 9997 × 10003 = ? apples. Originally, he had Assumptions
(a) 9999991 (b) 99999911 (a) 600 apples (b) 760 apples I. There may not be significant
(c) 99999991 (d) 9999911 (c) 700 apples (d) 800 apples change in the domestic sector.
54. What least value must be 62. P travels for 6 h at the rate of II. People may still prefer to
assigned to ‘*’, so that the 5 km/h and for 3 h at the rate of travel by train it is more safe.
number 63576*2 is divisible 6 km/h. The average speed of the (a) Only Assumption I is implicit
by 8? journey in km/h is (b) Only Assumption II is implicit
(a) 1 (b) 2 1 1 2 2 (c) Neither I nor II is implicit
(a) 3 (b) 5 (c)1 (d) 2
(c) 3 (d) 4 2 3 9 5 (d) Both I and II are implicit
68 AFCAT ~ Practice Set 8

Directions (Q. Nos. 70-72) In each 80. If ‘JACOB’ can be written as QZXLY, Answer Figures
of the following questions, choose the then ‘KENDY’ can be written as
word that will complete the second (a) PVWMA (b) PVMWB
pair in the same way as first pair. (c) PUMWB (d) PVMWA
70. King : Throne :: Judge : ? 81. If ‘NEUROTIC’ can be written as (a) (b) (c) (d)
(a) Lawyer (b) Bench ‘TICRONEU’, then how can 86. Problem Figures
(c) Court (d) Trial ‘PSYCHOTIC’ be written ?
(a) TICOHPSY (b) TICCHOPSY
71. Lion : Roar : : Ass : ? ?
(c) TICCOHPSY (d) TICHCOPSY
(a) Bark (b) Trumpet
(c) Howl (d) Bray Directions (Q. Nos. 82-86) Each of (1) (2) (3) (4)

72. Lamb : Frisk : : Mouse : ? the following questions consists of two Answer Figures
(a) Trot (b) Scamper sets of figures. Figures (1), (2), (3) and (4)
(c) Gallop (d) Flit constitute the problem set while figures
(a), (b), (c) and (d) constitute the answer
Directions (Q. Nos. 73-75) In the set. There is a definite relationship
(a) (b) (c) (d)
following questions, three between figure (1) and (2), establish a
alternatives out of four are same in a similar relationship between figure (3) Directions (Q. Nos. 87-91) In each of
certain way and so form a group. and (4) by selecting a suitable figure the following questions, a part of problem
Find the odd word that does not from the answer set that would replace figure is missing. Find out from the
belong to the group. the question mark (?) in figure (4). given answer figures that can replace
the ‘?’ to complete the problem figure.
73. (a) Diamond (b) Ruby 82. Problem Figures
(c) Marble (d) Sapphire 87. Problem Figure
74. (a) Rifle (b) Cannon ?
?
(c) Sword (d) Pistol
(1) (2) (3) (4)
75. (a) Orange (b) Litchi
Answer Figures
(c) Apple (d) Guava
76. Which of the following best Answer Figures
represents the relation among
Profit, Dividend and Bonus? (a) (b) (c) (d)

83. Problem Figures


(a) (b) (c) (d)
(a) (b) (c) (d) ? 88. Problem Figure
77. Which one of the following best (1) (2) (3) (4)
represents the relation among ?
Honey bee, Insect and Housefly? Answer Figures

(a) (b) (c) (a) (b) (c) (d) Answer Figures


(d)

78. Which of the following best 84. Problem Figures


represents the relation among
Herbivorous, Buffalo, Cow, Tiger? ? (a) (b) (c) (d)
(1) (2) (3) (4) 89. Problem Figure
(a) (b) (c) (d) Answer Figures

79. One day, Ravi left home and


cycled 10 km Southwards, turned
right and cycled 5 km and turned ?
(a) (b) (c) (d)
right and cycled 10 km and turned
left and cycled 10 km . How 85. Problem Figures Answer Figures
many kilometers will he have to
cycle to reach his home straight ? ?
(a) 10 km (b) 15 km
(c) 20 km (d) 25 km (1) (2) (3) (4) (a) (b) (c) (d)
AFCAT ~ Practice Set 8 69

90. Problem Figure Directions (Q. Nos. 93-96) In each Answer Figures
of the following questions, four C C C
figures are given. Three are similar in C
a certain way and so form a group.
Find out which one of the figures does (a) (b) (c) (d)
? not belong to that group.
98. Problem Figures
93.
Answer Figures

(a) (b) (c) (d) (1) (2) (3) (4) (5)


Answer Figures
94.
(a) (b) (c) (d)

91. Problem Figure


(a) (b) (c) (d) (a) (b) (c) (d)

95. 99. Problem Figures


?

(a) (b) (c) (d)


(1) (2) (3) (4) (5)
Answer Figures 96. Answer Figures

(a) (b) (c) (d)


(a) (b) (c) (d)
(a) (b) (c) (d)
Directions (Q. Nos. 97-100) In each
92. What will be the number at the 100. Problem Figures
of the following series, select a figure
bottom, if 5 is at the top; the two from the answer figures which will
positions of the dice being as continue the series as established by
given below? the five problem figures. (1) (2) (3) (4) (5)
1 6 97. Problem Figures Answer Figures
4 2 2 3 C
C

(i) (ii) C C
(1) (2) (3) (4) (5)
(a) 1 (b) 2 (c) 3 (d) 6 (a) (b) (c) (d)

Answers
1 (b) 2 (c) 3 (c) 4 (b) 5 (a) 6 (c) 7 (b) 8 (a) 9 (b) 10 (b)
11 (d) 12 (c) 13 (b) 14 (c) 15 (d) 16 (a) 17 (c) 18 (d) 19 (d) 20 (a)
21 (c) 22 (a) 23 (b) 24 (c) 25 (a) 26 (d) 27 (a) 28 (b) 29 (c) 30 (b)
31 (d) 32 (b) 33 (b) 34 (a) 35 (d) 36 (b) 37 (c) 38 (d) 39 (c) 40 (c)
41 (b) 42 (b) 43 (d) 44 (d) 45 (a) 46 (c) 47 (b) 48 (b) 49 (b) 50 (c)
51 (a) 52 (b) 53 (c) 54 (c) 55 (a) 56 (d) 57 (d) 58 (b) 59 (b) 60 (c)
61 (c) 62 (b) 63 (a) 64 (c) 65 (b) 66 (b) 67 (b) 68 (c) 69 (c) 70 (b)
71 (d) 72 (b) 73 (c) 74 (c) 75 (a) 76 (b) 77 (c) 78 (c) 79 (b) 80 (b)
81 (b) 82 (c) 83 (a) 84 (c) 85 (a) 86 (d) 87 (c) 88 (a) 89 (b) 90 (c)
91 (a) 92 (b) 93 (c) 94 (a) 95 (d) 96 (a) 97 (a) 98 (d) 99 (c) 100 (c)
70 AFCAT ~ Practice Set 8

Hints and Solutions


1. (b) According to the author, the wedding, 16. (a) whether 17. (c) couples 32. (b) Japan successfully launched a Cargo
birthday and Christmas presents may not 18. (d) system 19. (d) sharing ship to the International Space Station.
always be given with any thought of The unpiloted H-II Transport Vehicle-8
20. (a) long
affection. (HTV-8) lifted more than 4 tonnes of
21. (c) The correctly spelt word is ‘Fulfill’ supplies, as well as batteries and a
2. (c) This statement means that the choice
which means ‘to carry out or bring to prototype laser-communications system.
does not involve much care.
realisation’.
3. (c) The ‘unbirthday’ present is the best of It was launched from Tanegashima Space
22. (a) The correctly spelt word is ‘Necessary’ Center in Southern Japan. The ‘Kounotori
all as its choice needs the utmost care.
which means ‘something needed or 8’ which means ‘white stork’ is the
4. (b) A ‘golden rule’ is a rule which is very required for a particular purpose’. world’s biggest transport spaceship
important, which can be inferred from
23. (b) The correctly spelt word is according to Japan Aerospace Exploration
the passage.
‘Guarantee’ which means ‘a promise or Agency (JAXA).
5. (a) It is clearly understood from the assurance, especially one in writing’. 33. (b) The Yangtze river also known as the
passage that you must never give to
24. (c) The correctly spelt word is ‘Example’ Chang Jiang in China, is the longest river
another person anything that you
and it means a thing characteristic of its in Asia and the third-longest in the world.
would not rather keep. So, option (a) is
kind or illustrating a general rule or It rises in the Northern part of the Tibetan
correct.
specimen etc. Plateau and flows 6300 km in a generally
6. (c) Replace ‘total’ by ‘complete’. ‘Total’ is easterly direction to the East China sea.
25. (a) ‘Rancour’ means ‘a feeling of
used to show the sum of individual but
bitterness and anger’. So, option (a) 34. (a) Operational Flood in India, a project of
‘complete’ means ‘thorough’.
‘bitterness’ express the suitable meaning the National Dairy Development Board
7. (b) Replace ‘poet’ by ‘poets’. Phrase ‘one of ‘Rancour’. (NDDB) was the world’s biggest dairy
of the’ agrees with plural noun and development programme. It made India, a
26. (d) ‘Sedate’ means ‘quite and rather dull’
singular verb after it. milk-deficient nation, the largest milk
and ‘staid’ means ‘serious and boring’.
8. (a) Use ‘have’ in place of ‘has’. A producer in the world. This project started
Hence, ‘staid’ is the correct synonym of
number of always takes a plural noun in 1970 and its objective was to create a
‘sedate’.
and a plural verb. nation wide milk grid.
27. (a) ‘Tedium’ means ‘a state of boredom
9. (b) ‘Stumbled’ is usually followed by 35. (d) The Sunderbans National Park is a
or quality of being bored for a long time’.
‘upon’. So, ‘upon’ is the right preposition National Park, Tiger Reserve, UNESCO
Hence, ‘Boredom’ express the correct
to fill the given blank. World Heritage Site and a Biosphere Reserve
meaning of ‘Tedium’.
10. (b) The sentence is in simple present so located in West Bengal, India. It is part of
28. (b) ‘Garrulous’ means ‘excessively the Sunderbans Ganges River Delta.
first form of verb (V1) will be used.
talkative, especially on trivial matters’
Therefore, ‘to prepare’ should be used to It is home to Bengal Tiger, variety of
and ‘Reticent’ means ‘inclined to be
fill the given blank. birds, reptiles and invertebrate species.
silent or unwilling to speak about one’s
11. (d) Comparative adjective ‘Fewer’ should thought’. So, ‘Reticent’ is opposite in 36. (b) Light year is a unit to measure the
be used here as ‘chocolate’ is a distance travelled by light in a vacuum in
meaning to ‘Garrulous’.
countable noun and with countable one Julian year. It was defined by the
nouns adjective ‘fewer’ is used. 29. (c) ‘Forbearance’ means ‘the state or International Astronomical Union (AIU).
quality of being patient and tolerant’ and
12. (c) ‘Deport’ is the process of sending an 37. (c) Otto Hahn, is known for the discovery
‘Intolerance’ means ‘the state or
unwanted person out of the country. of atom bomb. He was a German chemist
quality of being intolerant’. Hence,
So, option (c) is a suitable choice. and Nobel laureate, a pioneer in the fields
‘Intolerance’ is opposite in meaning to
of radioactivity and radiochemistry.
13. (b) The endurance of pain or hardship ‘Forbearance’.
He is regarded as ‘the Father of Nuclear
without display of feelings or complaint 30. (b) ‘Predilection’ means ‘a strong liking Chemistry’.
is called ‘stoicism’. Hence, ‘stoicism’ is or preference’ and ‘Detestation’ means 38. (d) Ideally, the pH of human blood range
the correct one word substitution for the ‘extreme hatred or dislike’. between 7.35-7.45. Respiration and
given statement. So, option (b) ‘Detestation’ is opposite in renal function helps to control out blood
14. (c) ‘Transmigration’ is the process in meaning to ‘Predilection’.
pH. Below 6.8 the death of organisms
which human soul pass to another body 31. (d) The Kailasa or Kailashnatha temple become inevitable.
after death. Hence, option (c) is a is one of the largest Indian rock-cut
suitable choice. 39. (c) Vice President M Venkaiah Naidu
ancient Hindu temples located in the
flagged off a unique service called ‘Tree
15. (d) A photograph or drawing printed on Ellora Caves. The construction of the
Ambulance’ in Chennai to revive and
transparent plastic or glass able to be temple is generally attributed the eighth
provide care to trees on 27th May, 2019.
viewed using a slide projector is called century Rashtrakuta King Krishna I. The ‘Tree Ambulance’ service has been
Transparency. So, option (d) is a suitable The temple architecture shows traces of founded by Dr K Abdul Ghani, also
choice. Pallava & Chalukya styles. known as ‘Green Man of India’.
AFCAT ~ Practice Set 8 71

40. (c) The Reserve Bank of India (RBI) has Assembly in its resolution 49/114, to 55. (a) Let A and C complete the work in
released its vision documents on digital commemorate the signing of the x days.
payment named ‘Payment and Montreal Protocol on substances that 1 ...(i)
(A + B)’s 1 day’s work =
Settlement System in India : Vision deplete the Ozone Layer. 8
2019-21’. It shows the vision of RBI for 49. (b) Bula Choudhary is a former national 1
(B + C)’s 1 day’s work = ...(ii)
growth of Digital Payment in India. women’s swimming champion of India. 12
The move comes as the apex bank She is the first woman to cross seven 1
(C + A)’s 1 day’s work = ...(iii)
expects the number of digital transactions seas. She twice swam the English x
to increase more than four times to 8707 Channel first in 1989 and again in 1999.
crores in December 2021. Then, (A + B + B + C + C + A)’s 1 day’s
She has been awarded Arjuna Award as 1 1 1
41. (b) The first Indian woman to ever win well as Padma Shri Award. work = + +
8 12 x
an Olympic medal was Karnam 50. (c) Bt cotton is a genetically modified 2(A + B + C)’s 1 day’s work
Malleswari who won a bronze medal at variety of cotton producing an insecticide. 3x + 2x + 24
the 2000 Sydney Olympics in It is produced by Monsanto. The =
weightlifting. In 1995, she received the 24x
bacterium Bacillus thuringiensis (Bt) is a
Rajiv Gandhi Khel Ratna Award, India’s 5x + 24
family of over 200 different bacteria that ∴ (A + B + C)’s 1 day’s work =
highest sporting honour. produce chemical. It is harmful to 24 × 2x
42. (b) The Right to Education Act (RTE) selective insects, most notably the larvae According to the question,
which was passed by the Indian of butterflies, etc. 1
Parliament on 4th August, 2009, came (A + B + C)’s 1 day’s work =
51. (a) Let principal = ` P 6
into force on 1st April, 2010. The Act
∴ Amount =
118 1 5x + 24
makes Right to Education a fundamental P ⇒ =
100 6 48x
right of every child between the ages of 6
and 14 yrs. ∵ Principal + Interest = Amount ⇒ 30x + 144 = 48x
118 18 144
43. (d) Right to Vote in India is a ∴ SI = P−P= P ∴ x= = 8 days
constitutional right. Article 326 (Part XV) 100 100 18
of the Constitution gives this right. Article P × R × 3 18 56. (d) Total weight increased
Now, = P
326 of the Constitution provides that the 100 100 = 1 × 25 = 25 kg
elections to the House of the people and
∵ SI = P × R × T and T = 3 yr (given) ∴Weight of new person
to the Legislative Assembly of every state
 100  = Weight of excluded person
shall be on the basis of adult suffrage.
∴ R = 6% + Weight increased
It means that a person should not be less
T = 60 + 25 = 85 kg
Now, amount = P1 +
than 18 years of age. R 

44. (d) Tejas is India’s indegenously built  100 57. (d) Here, a = 11,
light combat aircraft . It is an 3 x = 100,
∴ A = 25000 1 +
6 
single-engine, delta wing, multi-role light  = ` 29775.4 b = 10, y = 110
 100
fighter designed by the Aeronautical
Then, if a articles are bought for ` x and
Development Agency (ADA) and ∴ Compound Interest b articles are sold for ` y, then
Hindustan Aeronautics Limited (HAL) for
= 29775.4 − 25000 = ` 4775.4 ay − bx
the Indian Airforce and Indian Navy. Gain% =  × 100
52. (b) x +
1
=2⇒ x +
1
=2  bx 
45. (a) Trishul is a short range surface to air 1 3
missile developed by India as a part of 1+ 1+ (11 × 110 − 10 × 100)
1 7 = × 100
the Integrated Guided Missile 2+ 10 × 100
3
Development Programme. It has a range 1210 − 1000
7 7 = × 100
of 9 km and is capable of carrying 15 kg ⇒x + =2⇒x =2 −
10 10 1000
warhead.
20 − 7 13 = 21%
46. (c) Headquarters of World Health ∴ x= =
10 10 58. (b) Quantity of lead in 1 kg of first alloy
Organisation (WHO) are situated at
Geneva. Regional offices are situated in 53. (c) 9997 × 10003 =0
Alexandria, Brazzaville, Copenhagen, = (10000 − 3) (10000 + 3) Quantity of lead in 1 kg of second alloy
Manila, New Delhi and Washington. 3 1
= (10000) − (3)
2 2
= = kg
47. (b) Indira Gandhi is the first woman to 12 4
[ ∵(a − b)(a + b) = a2 − b2 ]
get Bharat Ratna Award. She was given Quantity of lead in 2 kg of new alloy
this highest civilian award in the year of = 100000000 − 9 = 99999991 1
1971. = kg
54. (c) A number is divisible by 8, if the 4
48. (b) For the preservation of the Ozone number formed by last 3-digits is
∴Quantity of lead in 1 kg of new alloy
layer, World Ozone Day is celebrated divisible by 8.
1
every year on 16th September. The date From the given options, if * = 3, then = kg
has been designated by the UN General 8
632 is divisible by 8.
72 AFCAT ~ Practice Set 8

59. (b) Let the length of each train = x m 63. (a) ∵ 4x = 5 + 2 67. (b) Given, x + y + z = 1,
1 1 1
+ + =1
x y z
Then, speed of first train 4x = 5 + 1 + 1
and xyz = − 1
=
Distance x
= m/s ⇒ 4x − 1 = 5+1
We know that,
Time 3 and 4x + 1 = 5+ 3 x 3 + y3 + z 3 − 3xyz
x
Similarly, speed of second train = m/s 1 16x 2 − 1 = (x + y + z)(x 2 + y2 + z2 − xy − yz − zx)
4 Then, x − =
16x 16x ⇒ x 3 + y3 + z 3 − 3(−1)
Trains are moving in opposite directions.
(4x − 1) (4x + 1)
x x = = 1[x 2 + y2 + z2 + 2(xy + yz + zx)
∴Relative speed = + 16x
3 4 − 3(xy + yz + zx)]
4x + 3x 7x [∵ a2 − b2 = (a + b) (a − b)]
= = m/s ⇒ x 3 + y3 + z 3 + 3
12 12 ( 5 + 1) ( 5 + 3)   1 1 1 
= = (x + y + z)2 − 3xyz  + +  
Total length = x + x = 2x m 4( 5 + 2) x y z

∴Time taken =
Distance [∵16x = 4 × 4x]
⇒ x + y + z + 3 = [1 − 3(−1)(1)]
3 3 3
Speed 5+ 3 5 + 5 + 3 4 5 + 8
= = ⇒ x 3 + y3 + z 3 + 3 = 1 + 3
=
2x 24
=
3
=3 s 4( 5 + 2) 4( 5 + 2)
7x ⇒ x 3 + y3 + z 3 = 4 − 3 = 1
7 7 4( 5 + 2)
12 = =1 1
4( 5 + 2) 68. (c) Given, P = ` 72, A = ` 81, R = 6 %
60. (c) Suppose the price of cooking gas 4
64. (c) According to the question, Let time be T yr.
= ` 100
1 P× R×T P× R×T
Increased price = ` 115 (B + C)’s 1 day’s work = ...(i) ∴SI = ⇒A − P =
12 100 100
⇒ ` 115, he should reduce = 15 1 1
72 × 6 × T
(A + C)’s 1 day’s work = ...(ii)
15 8 ⇒ 81 − 72 = 4
⇒ ` 1, he should reduce = 100
115 1 ...(iii)
(A + B + C)’s 1 day’s work = 25
15 6 ⇒ 900 = 72 × ×T
⇒ ` 100, he should reduce = × 100 4
115 ∴ C’s 1 day’s work 900 × 4
1 ⇒ T= = 2 yr
= 13 % 1 1 1 2 + 3− 4 1 72 × 25
= + − = =
23 12 8 6 24 24
Alternate Method 69. (c) Neither of the assumptions is implicit.
∴ (A + B)’s 1 day’s work The number of passengers may increase
Reduction percentage in consumption 1 1 4−1 1 after withdrawing the tax. Also nothing
= − = =
 R  6 24 24 8 can be said about safety in trains from the
= × 100 %
(100 + R)  ∴A and B together can complete the statement.
15 1 work = 8 days 70. (b) As ‘King’ is related to ‘Judge’, in the
= × 100 = 13 %
115 23 65. (b) Let the CP of 1 article = ` x same way, ‘Throne’ is related to ‘Bench’.
61. (c) Suppose originally he had x apples. ∴CP of 25 articles = 25x 71. (d) ‘Roar’ is the sound produced by ‘Lion’,
similarly ‘Bray’ is the sound produced by
Then, (100% − 40%) of x = 420 and SP of 20 articles = 25x
‘Ass’.
60 420 × 100 25x 5x
⇒x × = 420 ⇒ x = ∴SP of 1 article = = 72. (b) ‘Frisk’ is the name given to the
100 60 20 4
movement of ‘Lamb’, similarly ‘Scamper’
∴ x = 700 ∴ Gain =
5x
−x=
x
is the name given to the movement of
62. (b) Given, T1 = 6 h, s1 = 5 km/h, T2 = 3 h 4 4 ‘Mouse’.
and s2 = 6 km/h x
× 100 73. (c) All others are precious stones.
Gain × 100 4
∴First distance = Speed × Time ∴ Gain % = = 74. (c) All others are fire arms.
CP x
= 5 × 6 = 30 km = 25% 75. (a) ‘Orange’ is the only citrus fruit.
Similarly, second distance 66. (b) Let the natural number be x. 76. (b) Profit
= 3 × 6 = 18 km According to the question, Bonus Dividend
Total distance = 30 + 18 = 48 km x 2 + x = 90
and total time = 3 + 6 = 9 h ⇒ x 2 + x − 90 = 0 Bonus and Dividend are different from
Total distance each other but both are parts of profit.
∴Average speed = ⇒ x 2 + 10x − 9 x − 90 = 0
Total time 77. (c) Insect
48 16 ⇒ x(x + 10) − 9(x + 10) = 0 Housefly
= = ⇒ (x + 10)(x − 9) = 0 Honey bee
9 3
1 ∴ x = 9 [∵ x = − 10 is not possible] Honey bee is different from housefly but
= 5 km/h
3 Hence, the required number is 9. both come under the class of insect.
AFCAT ~ Practice Set 8 73

78. (c) 83. (a) From problem figure (1) to (2), the
inner most figure becomes outer most
Herbivorous figure and outer most figure becomes the
inner most figure i.e., first and third
Now, it is clear that answer figure (c)
Buffalo Cow figure get interchanged.
Tiger completes the problem figure.
84. (c) From first figure to second figure, the
91. (a) Here, all the three equal parts have
Buffalo and cow are herbivorous animals design rotates through 90° clockwise.
similar design. So, we can obtain the
but both are different. Tiger is different 85. (a) The figure rotates 45° anti-clockwise, answer figure for the missing portion by
from both Buffalo and cow and is not the handle is reversed and the black rotating the problem figure by 90°
herbivorous. portion becomes white and the white clockwise.
79. (b) The direction diagram is as follows portion becomes black.
Starting 86. (d) Arrow rotates 90° clockwise and the
End point point
remaining portion of the figure rotates
E D A
10 km 90° anti-clockwise and then is reversed
at new position. Now, it is clear that answer figure (a)
10 km 10 km
completes the problem figure.
87. (c) Here, all the three equal parts have
similar design. So, we can obtain the 92. (b) It is clear from the given figures that
answer figure for the missing portion by 1, 4, 6 and 3 lie adjacent to 2. So, 5 lies
C B opposite to 2.
5 km rotating the problem figure by 90°
N clockwise. 93. (c) In all other figures, the number of
pins at the top of the figure are equal to
W E
the number of sides of the figure.
S 94. (a) In all other figures, the line is bent
The required distance, AE = AD + ED towards the pins.
Now, it is clear that answer figure (c)
= 5 + 10 [∵ AD = BC] completes the problem figure. 95. (d) In all others, three arcs face outside
and two inside.
= 15 km 88. (a) Here, all the three equal parts have
similar design. So, we can obtain the 96. (a) In figure (a), the vertices of the same
80. (b) As,
J A C O B answer figure for the missing portion by square are shaded, so it is different from
Pairs of opposite other.
letters rotating the problem figure by 90°
Q Z X L Y clockwise. 97. (a) The sequence of the problem figures
Therefore, is
K E N D Y
Pairs of opposite
letters
P V M W B Now, it is clear that answer figure (a)
completes the problem figure.
81. (b) As,
89. (b) Here, all the three equal parts have Problem figure (1) to (2)
N E U R O T I C
similar design. So, we can obtain the Problem figure (3) to (4)
answer figure for the missing portion by
rotating the problem figure by 90° Problem figure (5) to answer figure.
T I C R O N E U clockwise. 98. (d) In each problem figure, one new
design adds and after rotating 90° in
Similarly, anti-clockwise direction, design shifts one
place in anti-clockwise direction.
P S Y C H O T I C
99. (c) Problem figure (1) is same as figure
Now, it is clear that answer figure (b)
(5), hence answer figure will be same as
completes the problem figure.
the figure (2).
T I C C H O P S Y 90. (c) Here, all the three equal parts have
100.(c) The figure rotates 45° and 90°
similar design. So, we can obtain the
anti-clockwise alternatively and the
answer figure for the missing portion by
82. (c) Straight line portion of the figure symbols move one side anti-clockwise
rotating the problem figure by 90°
rotates 90° clockwise, curved line portion each time.
clockwise.
of the figure rotates 90° anti-clockwise.
74 AFCAT ~ Practice Set 9

AFCAT
AIR FORCE COMMON ADMISSION TEST (ONLINE )

Practice Set ~09


Time : 2 Hrs MM : 300

INSTRUCTIONS
1. The set contains a total of 100 questions, Comprising Verbal Ability in English, General Awareness, Numerical Ability and Reasoning and
Military Aptitude Test.
2. Each correct question carry 3 Marks and there will be negative marking of 1 Mark for each incorrect attempt.
3. Total time duration will be 2 hrs (120 minutes).
4. No marks will be deducted for unattempted questions.

Directions (Q. Nos. 1-3) Find out Directions (Q. Nos. 7-10) In the Directions (11-13) Read the following
which part of the sentence has an error. following questions, four alternatives passage carefully and answer the
If there is no error, then choose ‘No are given for the Idiom/Phrase questions that follow.
error’ as your answer. underlined in the sentence. Choose A little man beside me was turning over
1. He was in the temper (a)/and the alternative which best expresses the pages of a magazine quickly and
refused (b)/to discuss the matter the meaning of the given nervously. Opposite me, there was a
again. (c)/No error (d) Idiom/Phrase. young mother who was trying to
7. Ladies fall victim to green eyed restrain her son from making a noise.
2. Despite of the increase in air fares, The boy had obviously grown weary of
(a)/most people still prefer (b)/to monster.
(a) love (b) hatred waiting. He had placed an ashtray on
travel by plane. (c)/No error (d) the floor and was making aeroplane
(c) jealousy (d) flattery
3. His grandfather (a)/had told him noises as he waved a pencil in his hands.
that to smoke (b)/was a bad 8. The teacher was as good as his Near him, an old man was fast asleep,
habit. (c)/No error (d) word. snoring quickly to himself and the boy’s
(a) incapable of action mother was afraid that sooner or later
Directions (Q. Nos. 4-6) Fill in the (b) better than expected her son would wake the gentleman up.
blanks with suitable alternative.
(c) highly pretentious 11. The noise was made by the
4. Don’t blame yourself for the (d) ready to fulfil his promise (a) old man (b) aeroplane
mistake. ……… is perfect. (c) little man (d) boy
9. Police has no clue about his
(a) Nobody
whereabouts and he is at large. 12. The factor common to all the
(b) Anybody
(a) out of country people was that they were all
(c) Everybody
(b) respected by everybody (a) watching a film
(d) Somebody
(c) has no charges against him (b) waiting for something
5. I did ……… well in the test. (d) free (c) looking at the little boy’s playfulness
(a) fully (b) incredibly (d) reading magazine
10. He has made his mark in
(c) lots (d) strongly politics. 13. Among those present, the one who
6. He is weak ……… he does a lot of (a) attained notoriety appeared to be the most bored was
work. (b) ruined his wealth the
(a) because (b) so (c) acquired wealth (a) child (b) little man
(c) and (d) yet (d) distinguished himself (c) old man (d) mother
AFCAT ~ Practice Set 9 75

Directions (Q. Nos. 14-17) In each of 24. To secretly store more than what 36. The Dilwara Temples at Mount
the following questions, choose the is allowed Abu are dedicated to
correctly spelt word. (a) Hoard (b) Store (a) Adinath
14. (a) Semblence (b) Samblance (c) Hide (d) Aboard (b) Bahubali
(c) Semblance (d) Samblence (c) Gautama Buddha
Directions (Q. Nos. 25-27) In the
following questions, out of the four (d) Tukaram
15. (a) Gynecology (b) Gynaecology
alternatives, choose the one which best 37. Which of the following is called
(c) Gynaeccology (d) Gyneccology
expresses the meaning of the given word. ‘The Bible of Tamil Land’?
16. (a) Hypochondria 25. Latent (a) Kural (b) Tolkkappiyam
(b) Hyppochondria (c) Silappatikaram (d) Manimekalai
(a) Apparent (b) Dormant
(c) Hypochondrea
(d) Hyppochondrea
(c) Ample (d) Illegal 38. Which among the following esta
26. Vanity blished the Tattvabodhini Sabha in
17. (a) Puerille (b) Puerrile 1839 at Calcutta (Now Kolkata)?
(a) Humility (b) Pride
(c) Puerile (d) Purrile (a) Keshab Chandra Sen
(c) Ostentation (d) Pity
Directions (Q. Nos. 18-20) In the (b) Debendranath Tagore
27. Laud (c) Rammohan Roy
following passage, some words have (a) Lord (b) Eulogy
been omitted. For each blank, four (d) Sivanath Sastri
(c) Praise (d) condemn
alternatives are provided. Select the 39. Which is the largest fresh water lake
suitable alternative to fill the given Directions (Q. Nos. 28-30) In the in the world?
blanks. following questions, choose the word (a) Great Bear Lake
Recent discoveries ...18... that Indians opposite in meaning to the given word. (b) Great Slave Lake
of early days appear to have been 28. Flimsy (c) Lake Superior
highly civilised in many ways. They (a) Frail (b) Filthy (d) Erie Lake
had massive public buildings and (c) Firm (d) Flippant 40. In which state of India resides the
comfortable dwelling houses ...19... 29. Relinquish maximum number of tribes?
mostly of brick. They had made (a) Madhya Pradesh
(a) Abdicate (b) Renounce
arrangements for good sanitation and (b) Nagaland
(c) Possess (d) Deny
an elaborate drainage system. They (c) West Bengal
knew how to write ...20... their 30. Conciliation
(d) None of the above
language, which has not yet been (a) Dispute (b) Irritation
deciphered was not alphabetic (c) Separation (d) Confrontation 41. Which place is known as ‘Mecca
of Indian Football’?
but syllabic like the Sumerian 31. Biological decay, forest fires and
language. (a) New Delhi (b) Kolkata
volcanic eruptions are example of
(c) Mumbai (d) Chennai
18. (a) shown (b) derived (a) gaseous pollutants
(c) investigated (d) seen (b) water pollutants 42. What can be the maximum strength
(c) human-caused pollution of members in the State Legislature?
19. (a) designed (b) formulated (a) 600 (b) 500 (c) 552 (d) 450
(d) All of the above
(c) built (d) construct
32. Who is the father of 43. Which one among the following is
20. (a) but (b) because ‘Nanotechnology’ in India? the unit raised to protect the
(c) while (d) since (a) CNR Rao naval assets?
(b) Verghese Kurien (a) Sagar Rakshak Bal
Directions (Q. Nos. 21-24) In the (b) Sagar Suraksha Bal
following questions, out of the four (c) Satish Dhawan
(d) Brahma Prakash (c) Sagar Prahari Bal
alternatives, choose the one which can be
(d) Sagar Nigrani Bal
substituted for the given 33. When is the World Day Against
words/sentence. Trafficking in Persons served 44. Which of the following is the
21. A person who kills somebody globally? India’s first lunar probe launched
especially for political reason (a) 28th July (b) 29th July by the Indian Space Research
(c) 30th July (d) 31st July Organisation?
(a) Criminal (b) Murderer
(a) Mangalyaan Programme
(c) Assassin (d) Hangman 34. Which Article was scrapped by (b) Chandrayaan Programme
22. A person who rules without the Parliament? (c) Both (a) and (b)
consulting the opinion of others (a) Article 370 (b) Article 356 (d) Discovery Programme
(a) Democrat (b) Bureaucrat (c) Article 377 (d) Article 246
45. Which of the following constitutional
(c) Autocrat (d) Fanatic 35. Under whose governor-generalship, amendment amended the age of
23. A short stay at a place the Revolt of 1857 occurred? voting from 21 to 18?
(a) Halt (b) Interlude (a) Lord Canning (b) Lord Irwin (a) 51st (b) 48th
(c) Intermission (d) Sojourn (c) Lord Lytton (d) Lord Willington (c) 61st (d) 86th
76 AFCAT ~ Practice Set 9

46. What was the name of operation 56. A shopman bought pens at the rate 64. A cow is tied on the corner of a
conducted by National Security of 7 for ` 10 and sold them at a rectangular field of size 30 m
Guard in 2008 against profit of 40%. How many pens × 20m by a 14 m long rope.
terrorist attack in Taj Hotel, would a customer get for ` 10? The area of the region, that she
Mumbai? (a) 6 (b) 4  22 
can graze, is  use π = 
(a) Operation Black Tornado (c) 5 (d) 3  7
(b) Operation Black Thunder 57. A man purchased a car for (a) 350 m2 (b) 196 m2
(c) Operation Safed Sagar ` 135000 and spent ` 25000 on
(d) Operation Vijay (c) 154 m2 (d) 22 m2
repairs. At what price was the car
47. Which of the following is not one sold, if he suffered 10% loss on it? 65. A sum of money at a certain rate
of the official languages of the (a) ` 176000 (b) ` 144000 per annum of simple interest
UNO? doubles in 5 yr and at a different
(c) ` 121500 (d) ` 150000
rate becomes three times in 12 yr.
(a) French (b) Spanish 58. A man rows a boat 18 km in 4 h The lower rate of interest per
(c) Arabic (d) Italian downstream and returns upstream annum is
48. The first country to commence in 12 h. The speed of the stream (a) 15% (b) 20%
competitive examination in Civil (in km/h) is 3 2
(c)15 % (d)16 %
Services (a) 1 (b) 1.5 4 3
(a) USA (b) Britain (c) 2 (d) 2.5
66. Two statements followed by two
(c) China (d) France 59. A train travels at the average speed conclusions are given. Read the
49. Who among the following 1 conclusions and then decide
of 50 km/h for 2 h and then
designed the logo of Lokpal? 2 which of the conclusions, if any,
(a) Akansha Aggarwal travels at a speed of 70 km/h for logically follows from the two
(b) Prashant Mishra 1 given statements, disregarding the
1 h. The average speed of the
(c) Aaruhi Jain 2 known facts.
(d) Narendra Tomar train in km/h for entire journey is Statements
(a) 59.5 (b) 60 Some rats are dogs.
50. Which isotope of Uranium has Some dogs are cats.
(c) 62.5 (d) 57.5
the capacity to sustain the chain Conclusions
reaction? 60. A and B working together, can do
I. Some rats are not cats.
(a) U-230 (b) U-235 1
a piece of work in 4 h. B and C II. Some cats are not dogs.
(c) U-245 (d) U-225 2
(a) Only Conclusion I follows
51. 5136 working together can do it in 3 h.
+ 459 = ? C and A working together can do (b) Only Conclusion II follows
3
(523 + 333) of 1
it in 2 h. All of them begin the
(c) Neither I nor II follows
4 4 (d) Both Conclusions I and II
(a) 520 (b) 541 work at the same time. Find how follows
(c) 513 (d) 467 long time they will take to finish 67. One statements followed by two
52. What is the least value of K, so the piece of work. assumptions numbered I and II is
that the number 6735K1 is (a) 3 h (b) 2 h given. You have to consider the
divisible by 9? (c) 2.5 h (d) 3.25 h statement and the following
1 1 assumptions and decide which of
(a) 5 (b) 7 61. If a + = 1, b + = 1, then the value
(c) 4 (d) 3 b c the assumptions is implicit in the
1 of abc is statement.
53. A number increased by 137 % (a) 0 (b) − 1 Statement
2
gives 33, then the number is (c) 1 (d) ab “If you want hassle free personal
(a) 16 (b) 24 loans give us a call, we will be at
62. The ratio in which two sugar your door-step.” An advertisement
(c) 20 (d) 28 solutions of the concentration 15% issued by ‘X’ finance company.
54. What is the value of and 40% are to be mixed to get a Assumptions
(443 + 547)2 + (443 − 547)2 solution of concentration of 30% is I. No other financial company
=? (a) 2 : 3 (b) 3 : 2
443 × 443 + 547 × 547 provides service at the
(c) 8 : 9 (d) 9 : 8 door-step of the customer.
(a) 0 (b) 1 (c) 2 (d) 3
63. In a family, the average age of II. People may prefer ‘X’ finance
55. A can do 1/6 of a work in 5 days father and mother is 38 yr. company as they may get
and B can do 2/5 of the work in Whereas, the average age of father, attracted by the
8 days. In how many days can mother and their only daughter is advertisement.
both A and B together do the 28 yr. Then, the age of the (a) Only Assumption I is implicit
work? daughter is (b) Only Assumption II is implicit
(a) 12 days (b) 13 days (a) 5 yr (b) 6 yr (c) Neither I nor II is implicit
(c) 15 days (d) 20 days (c) 8 yr (d) 10 yr (d) Both I and II are implicit
AFCAT ~ Practice Set 9 77

Directions (Q. Nos. 68-72) Find out 79. Which of the following best Directions (Q. Nos. 86-88) In each of
the pair in which the words bear the represents the relation among following questions, find the answer
same relationship to each other as Glassware, Pottery, Crystal? figure in which problem figure is
similar to the words of the given pair embedded.
bear. (a) (b)
86. Problem Figure
68. Run : Race
(a) Enjoy : Journey
(c) (d)
(b) Lecture : Study
(c) Study : Book
(d) Party : Dance Answer Figure
80. Which of the following best
69. Capricious : Reliability represents the relation among
(a) Extemporaneous : Predictability Aircrafts, Jets, Flying objects?
(b) Unreliable : Inhuman
(c) Tenacious : Practicality (a) (b) (a) (b) (c) (d)
(d) Arbitrary : Whimsical
70. Water : Oxygen 87. Problem Figure
(a) Helium : Nitrogen (c) (d)
(b) Salt : Sodium
(c) Tree : Plant
81. Which of the following diagrams
(d) Food : Hunger best depicts the relation among Answer Figure
71. Geeta : Quran Human society youth club,
(a) Orange : Mango Political party and Youths?
(b) Temple : Worship
(c) Good : Man (a) (b)
(d) Army : Defence (a) (b) (c) (d)
72. Sin : Crime 88. Problem Figure
(a) Man : Animal (c) (d)
(b) Home : Court
(c) Morality : Legality
82. Find the figure that best
(d) Jury : Priest represents the group Aeroplane,
Directions (Q. Nos. 73-77) In the pilot and Sky. Answer Figure
following questions, three alternatives
are same in a certain way and so form a (a) (b)
group. Find the odd word that does not
belong to the group. (c) (d)
(a) (b) (c) (d)
73. (a) Red (b) Blue
(c) Yellow (d) Black
83. Some of the letters of the series 89. Which of the following codes will
74. (a) Kanpur (b) Lucknow are missing. Find the missing come in place of question mark?
(c) Lahore (d) Patna letters.
mc_m_a_ca_ca_c_mc
75. (a) Ample (b) Copious
(a) acmmma (b) camcam
(c) Plentiful (d) Abundance (c) aaacmm (d) acmmmc AX DX AZ ?
76. (a) Shiv (b) Ganesh 84. If the word ‘PRINCIPAL’ is (a) DZ (b) AD
(c) Durga (d) Vishnu written as LAPICNIRP, how (c) XD (d) XA
77. (a) Eye (b) Bone ‘ADOLESCENCE’ can be written
in the code? 90. Which of the following codes will
(c) Ear (d) Hand come in place of question mark?
(a) ECNCESELODA
78. Which of the following best (b) ECNECSLEODA
represents the relation among (c) ECNSCEELODA
Donation, Loan and Gifts?
(d) ECNECSELODA
BG CH BI DH DI ?
85. If E = 5 and RED = 27, then
DANCE = ?
(a) (b) (c) (d) (a) 26 (b) 28 (c) 27 (d) 25 (a) CG (b) HC (c) CH (d) DH
78 AFCAT ~ Practice Set 9

Directions (Q. Nos. 91-95) In each Directions (Q.Nos. 96-100) In 98. Problem Figures
of the following questions, four each of the following questions
figures are given. Three are similar in select a figure from the answer
a certain way and so form a figures which will continue the series
group. Find out which one of
as established by the four problem (1) (2) (3) (4)
the figures does not belong to that
group. figures. Answer Figures
91. 96. Problem Figures

(a) (b) (c) (d)


(a) (b) (c) (d)
(1) (2) (3) (4) 99. Problem Figures
92.
Answer Figures

(a) (b) (c) (d) (1) (2) (3) (4)


Answer Figures
93. (a) (b) (c) (d)

97. Problem Figures


(a) (b) (c) (d) (a) (b) (c) (d)

94. 100. Problem Figures

(1) (2) (3) (4)


(a) (b) (c) (d) Answer Figures
(1) (2) (3) (4)
95. Answer Figures

(a) (b) (c) (d)


(a) (b) (c) (d)
(a) (b) (c) (d)

Answers
1 (a) 2 (a) 3 (b) 4 (a) 5 (b) 6 (d) 7 (c) 8 (d) 9 (d) 10 (d)
11 (d) 12 (b) 13 (b) 14 (c) 15 (b) 16 (a) 17 (c) 18 (a) 19 (c) 20 (a)
21 (c) 22 (c) 23 (d) 24 (a) 25 (b) 26 (b) 27 (c) 28 (c) 29 (c) 30 (d)
31 (a) 32 (a) 33 (c) 34 (a) 35 (a) 36 (a) 37 (a) 38 (b) 39 (c) 40 (a)
41 (b) 42 (c) 43 (c) 44 (b) 45 (c) 46 (a) 47 (d) 48 (c) 49 (b) 50 (b)
51 (d) 52 (a) 53 (b) 54 (c) 55 (a) 56 (c) 57 (b) 58 (b) 59 (d) 60 (b)
61 (b) 62 (a) 63 (c) 64 (c) 65 (d) 66 (c) 67 (b) 68 (c) 69 (c) 70 (b)
71 (a) 72 (c) 73 (d) 74 (c) 75 (d) 76 (c) 77 (b) 78 (d) 79 (a) 80 (a)
81 (b) 82 (a) 83 (a) 84 (d) 85 (c) 86 (d) 87 (c) 88 (d) 89 (a) 90 (a)
91 (d) 92 (d) 93 (c) 94 (c) 95 (b) 96 (d) 97 (d) 98 (d) 99 (a) 100 (d)
AFCAT ~ Practice Set 9 79

Hints and Solutions


1. (a) ‘The’ should be removed before ‘temper’ 23. (d) ‘Sojourn’ is a short stay at a place. 33. (c) Every year 30th July has been
as ‘temper’ is an abstract noun and 24. (a) ‘To hoard’ means to secretly store designated by the United Nations as the
abstract noun is not used with articles. more than what is allowed. ‘World Day Against Trafficking in
2. (a) ‘Of’ should be removed as ‘despite’ is Persons’. The day is observed to raise
25. (b) ‘Latent’ means ‘something present
not followed by a preposition. awareness of the situation of victims of
and capable of emerging or developing
human trafficking and for the promotion
3. (b) ‘To smoke’ should be replaced by but not now visible or active’ and ‘Dormant’
and protection of their rights”. The theme
‘smoking’ because here, use of gerund is means ‘not active or growing at the
of ‘World Day Against Trafficking in
more appropriate than infinitive. present time but able to be active later’.
Persons 2019’ was ‘Human Trafficking:
4. (a) ‘Nobody’ is the suitable indefinite Hence, ‘Dormant’ is the suitable Call Your Government to Action’.
pronoun to fill the given blank. synonym for ‘Latent’.
34. (a) Article 370 alongwith Article 35A of
5. (b) ‘Incredibly’ is the right choice to fill in 26. (b) ‘Vanity’ means ‘excessive pride in the Constitution that gave special status
the blank as it tells the degree of doing one’s appearance, qualities or to Jammu and Kashmir was scrapped by
well in the test. achievements’. the Parliament. The Government of India
6. (d) Coordinating conjunction ‘yet’ will be So, option (b) ‘pride’ express the correct revoked the special status of the state
used to show the contrast in the given meaning of ‘vanity’. through a presidential order and the
sentence. 27. (c) ‘Laud’ means ‘to praise someone’. So, passage of a resolution in Parliament.
7. (c) Phrase ‘green eyed monster’ means option (c) ‘praise’ express the correct In addition, the Jammu and Kashmir
‘jealousy’. So, option (c) is a suitable choice. meaning of ‘laud’. Reorganisation Act was passed by the
8. (d) Idiom ‘as good as his word’ means ‘to 28. (c) ‘Flimsy’ means ‘without material strength Parliament, enacting the division of
keep one’s promise’. Hence, option (d) or solidity’ and ‘Firm’ means ‘having a solid, Jammu and Kashmir state into two
hard or rigid structure or surface’. Union Territories i.e. Union Territory of
‘ready to fulfil his promise’ express the
Ladakh and the Union Territory of
correct meaning of given idiom. Hence, ‘firm’ is the word opposite in
Jammu and Kashmir.
9. (d) Phrase ‘at large’ means ‘to be free or meaning to ‘flimsy’.
35. (a) Lord Canning was the British
not yet captured’. Hence, option (d) ‘free’ 29. (c) ‘Relinquish’ means ‘to give up
Governor-General during the Revolt of
is a suitable choice. something such as power or control’ and
1857.
10. (d) Idiom ‘made his mark’ means ‘to ‘possess’ means ‘to have or own
something’. So, ‘possess’ is the opposite Revolt of 1857 or First War of
distinguished himself’. Hence, option (d) Independence was a major, although
is a suitable choice. meaning word to ‘relinquish’.
unsuccessful challenge to British
11. (d) The noise was made by the boy. 30. (d) ‘Conciliation’ means ‘the process of
colonialism. The revolt broke out from
helping two sides in a disagreement’ and
12. (b) The factor common to all the people Meerut and later spread to Delhi, Agra,
‘confrontation’ means ‘a dispute, fight or
was that they were all waiting for Kanpur and Lucknow.
battle between two groups of people’. So,
something. ‘confrontation’ is the opposite meaning 36. (a) The Dilwara Temples are located in
13. (b) The little man was most bored word to ‘conciliation’. Mount Abu, Rajasthan that enshrine
because he was turning the pages of the various Jain ‘Tirthankaras’. Built between
31. (a) Pollutants that leads to air pollution
magazine nervously. 11th and 13th century, the temple
are called gaseous pollutants.
14. (c) The correctly spelt word is ‘semblance’ complex consists of five sections
Biological decay, forest fires, and volcanic dedicated to Lord Mahaveer Swami,
which means ‘having an outward appear eruptions are examples of gaseous Shri Adinathji, Shri Parshavnathji, Shri
ance that resembles something desired’. pollutants. These gaseous pollutants Rishabdaoji and Shri Neminathji.
15. (b) The correctly spelt word is deteriorate natural vegetation and human
health. Ozone and Nitrogen Oxide are the 37. (a) The Tirukkural or Kural is called ‘The
‘Gynaecology’.
most relevant gaseous pollutants in terms Bible of Tamil Nadu’. It is a classic
16. (a) The correct spelt word is Tamil text consisting of 1330 couplets
of adverse health effects.
‘Hypochondria’. or Kurals, dealing with everyday virtues
32. (a) Nano technology is manipulation of
17. (c) The correctly spelt word is ‘Puerile’ of an individual. Authored by
matter on an atomic, molecular and
which means ‘characteristics of or related Thiruvalluvar it is considered as one of
supermolecular scale. Chintamani Nagesa
to child’. the greatest works ever written on ethics
Ramchandra RAO (CNR Rao) is considered
18. (a) shown and morality.
as the ‘Father of Nanotechnology’ in India.
19. (c) built CNR Rao is an Indian Chemist who has 38. (b) On 6th October, 1839, Debendranath
made significant contributions to Tagore established Tattvaranjini
20. (a) but
Nanotechnology. He has authored Sabha which was shortly thereafter
21. (c) ‘Assassin’ is a person who murders renamed the Tattwabodhini
around 160 research papers and 50
someone for political reasons. (Truth-seekers) Sabha. Initially, confined
scientific books. In 2014, he was
22. (c) ‘Autocrat’ is a person who has awarded with Bharat Ratna, the highest to immediate members of the Tagore
absolute power and doesn’t care about civilian award in India. family, in 2 years it mustered over 500
opinions of others. members.
80 AFCAT ~ Practice Set 9

39. (c) North America’s Lake Superior is after the 2008 Mumbai terrorist attacks 54. (c) Given expression,
largest fresh water lake in the world by took place. At least 174 people died, (443 + 547)2 + (443 − 547)2
surface area and the third largest by including 9 attackers, and more than
443 × 443 + 547 × 547
volume. The lake is shared by the 300 were wounded in the attack.
province of Ontario in Canada, and the 47. (d) There are six official languages of the Here, let a = 443, b = 547
states of Minnesota, Wisconsin and (a + b)2 + (a − b)2
UN. These are Arabic, Chinese, English, =
Michigan in the US. French, Russian and Spanish. a× a+ b× b
40. (a) Madhya Pradesh has the maximum The correct interpretation and translation a2 + b2 + 2ab + a2 + b2 − 2ab
=
number of Scheduled Tribes residing in of these six languages, in both spoken a2 + b2
India. There are 46 recognised STs in and written form, is very important to the 2(a2 + b2 )
the state. The tribal population is largely work of the organisation because this = =2
concentrated in and around the forest a2 + b2
enables clear and concise
area of Madhya Pradesh. The population communication on issues of global 1
55. (a) Work done by A in 5 days =
of Scheduled Tribes is 21.1% of the total importance. 6
state population. 1 1
48. (c) First country (state) to start ∴ Work done by A in 1 day = =
41. (b) Kolkata is known as ‘Mecca of Indian competitive exams in civil services was 6 × 5 30
Football’. East Bengal and Mohun Bagan Imperial China. Emperor Wen of Sui (AD 2 1
are the biggest football clubs in the city. Work done by B in 1 day = =
581-604) established the first civil 5 × 8 20
Kolkata’s passion for football has earned service examination in China. The
it the name ‘Mecca of Indian Football’. ∴Work done by (A + B) together in 1 day
imperial exam based on merit was
1 1 2+ 3 5 1
42. (c) The maximum strength of the House designed to select the best administrative = + = = =
(State Legislature) envisaged by Indian officials for the state’s bureaucracy. 30 20 60 60 12
Constitution is 552. Out of these, 530 ∴A and B will complete the work
49. (b) Lokpal Chairman Justice Pinaki
members to represent states, 20 to 1
Chandra Ghose launched the logo of = days
represent Union Territories and not more 1
anti-corruption Ombudsman, Lokpal. The
than 2 members of the Anglo-Indian logo selected for the Lokpal was designed 12
Community to be nominated by the 1
by Prashant Mishra of Uttar Pradesh. The Thus, required time = = 12 days
President, if in his opinion that motto of Lokpal “Ma Gridhah 1
community is not adequately represented Kasyavidhanam” (Do not be greedy for 12
in the house. anyone’s wealth) was also selected. CP × (100 + Profit%)
56. (c) ∵ SP =
43. (c) ‘The Sagar Prahari Bal’ meaning 50. (b) Uranium-235 (235U) is an isotope of 100
‘Ocean Sentinels’, is a unit of the Indian CP × (100 + 40)
uranium making up about 0.72% of ⇒ SP =
Navy formed in 2009, in which is natural uranium. Unlike the predominant 100
entrusted with the responsibility of isotope uranium-238, it is fissile, i.e. it 10 × 140
⇒ SP of 7 pens = = ` 14
patrolling India’s coastal waters and can sustain a fission chain reaction. 100
protect the naval assets. The force 14
consists of 2,000 personnel and is
It is the only fissile isotope that is ∴SP of 1 pen = =`2
primordial and found in relatively 7
equipped with 80 patrol boats.
significant quantities in nature. ∴5 pens were sold for ` 10.
44. (b) Chandrayaan-1 was the first Indian 5136
lunar probe under Chandrayaan 51. (d) + 459 = ? 57. (b) Total CP of car
3
programme. It was launched by the (523 + 333) of = 135000 + 25000 = ` 160000
4
Indian Space Research Organisation in And loss = 10%
5136 5136
2008 and operated until August 2009. ⇒? = + 459 = + 459 (100 − Loss per cent)
3 214 ×3 SP of car = CP ×
The lunar orbiter is best known for 856 × 100
4 (100 − 10)
helping to discover evidence of water = 160000 ×
molecules on the Moon. = 8 + 459 = 467 100
45. (c) The 61st Amendment of the 52. (a) 6 + 7 + 3 + 5 + K + 1 = 22 + K = 1600 × 90 = ` 144000
Constitution of India, officially known as The least number greater than 22 and 18 9
58. (b) Rate of downstream = = km/h
the Constitution (Sixty-first Amendment) divisible by 9 is 27. 4 2
Act, 1988 which lowered the voting age ∴ 27 = 22 + K 18 3
Rate of upstream = = km/h
of elections to the Lok Sabha and to the ⇒ K=5 12 2
Legislative Assemblies of States from 21 53. (b) Let the required number be x. Now, speed of the stream
years to 18 years. This was done by 1
Then, 137 % of x = 33 Rate of downstream− Rate of upstream
amending Article 326 of the =
2 2
Constitution, which concerns elections to
275 1 9 3
the Lok Sabha and the State Assemblies. ⇒ × × x = 33 −
2 100
46. (a) The ‘Operation Black Tornado’ was =2 2
33 × 2 × 100 2
conducted by the National Security ⇒ x= = 24
275 6 3
Guard (NSG) on 29th November, 2008 = = = 15 . km/h
4 2
AFCAT ~ Practice Set 9 81

59. (d) Distance covered by the train in first 64. (c) Area grazed by cow 75. (d) Except ‘Abundance’ all mean
1 sufficient amount but abundance is used
= Speed × Time = 50 × 2 A D
2 for more than sufficient amount.
5
= 50 × = 125 km 76. (c) All others are male Gods.
2
77. (b) All others are external body parts.
Distance covered by the train in last
1 1 78. (d) Gift
= 1 h = 70 × 1 B 14m C Donation
2 2
3
= 70 × = 105 km 90°
= × πr2
2 360° Loan
Total distance covered by the train 1
= × π × 14 × 14
= 125 + 105 = 230 km 4
Now, average speed of the train 1 22
= × × 14 × 14 Donation is a kind of gift but Loan is
Total distance 230 4 7
= = entirely different.
Total time 4 = 154m 2
79. (a)
= 57.5 km/h Glassware
65. (d) The sum gets doubled in 5 yr and
2
60. (b) (A + B)’s 1 h work = …(i) tribled in 12 yr. Clearly, rate of interest Pottery
9 Crystal
for 12 yr will be lower,
1
(B + C)’s 1 h work = …(ii) If principal = ` x, then
3 SI × 100
4 Rate = Crystal is a glassware but pottery is
(C + A)’s 1 h work = …(iii) Principal × Rate different.
9
2x × 100
On adding Eqs. (i), (ii) and (iii), we get = [∵ SI = A − P] 80. (a)
x × 12 Flying objects
2(A + B + C)’s 1 h work
50
2 1 4
= + + =
3 Aircrafts
9 3 9
2 Jets
2 + 3+ 4 9 = 16 %
= = =1
9 9 3
1 66. (c)
(A + B + C)’s 1 h work =
2 All Jets are aircrafts and all aircrafts are
Rats Dogs Cats
∴A, B and C together will complete the flying objects.
work in 2 h. 81. (b) Youth
1 Conclusions I. (×)
61. (b) Given, a + = 1 Human
b II. (×) society Political
youth party
1
⇒ a =1− 67. (b) Only Assumption II is implicit. club
b Company ‘X’ has issued the
1 1 Some youths may be members of
and b + = 1 ⇒ c = advertisement with the assumption that political party and some other youth
c 1− b the people may prefer the company may be members of human society youth
Now, abc = 1 −  × b ×
1 1 as they get attracted by the club.
 b 1− b advertisement. 82. (a)
1 − b
= − 
1 Sky
 × b× = −1 68. (c) As, ‘Race’ is related to ‘Run’, similarly
 b  (1 − b ) a ‘Book’ is related to ‘Study’. Aeroplane
62. (a) By Alligation rule 69. (c) A person who is ‘Capricious’ loses
‘Reliability’. In the same way, if a Pilot
Solution I Solution II
15% 40% person is ‘Tenacious’, he loses
‘Practicality’.
30% 70. (b) ‘Water’ contains ‘Oxygen’ in it and
‘Salt’ contains ‘Sodium’ in it. Pilot sits in aeroplane and he flies
10% 15%
71. (a) ‘Geeta’ and ‘Quran’ belong to the aeroplane in the sky.
∴Required ratio = 10 : 15 = 2 : 3
same class, i.e., religious books and 83. (a) Series pattern :
63. (c) The age of the daughter = (Total age ‘Orange’ and ‘Mango,’ also belong to the
of father, mother and daughter) − (Total mca/mca/mca/mca/mca/mc
same class, i.e., fruits.
age of father and mother) ∴Required answer = acmmma
72. (c) ‘Sin’ is related to ‘Crime’, in the
= (3 × 28) − (2 × 38) 84. (d) The letters have been written in
same way, ‘Morality’ is related to
 Sum of terms  reverse order in the codes.
∵ Average = ‘Legality’.
 Number of terms  As, PRINCIPAL → LAPICNIRP
73. (d) All others are primary colours. Therefore, A D O L E S C E N C E → E C
= (84 − 76) y
= 8 yr 74. (c) All others are Indian cities. NECSELODA
82 AFCAT ~ Practice Set 9

85. (c) According to the question, E = 5 , i.e. 93. (c) Figure (c) is different from other
Position number in the English figures, in the way that this is the only
alphabet. figure in which two semi-circles
R E D are present on the same side of the
89. (a) Here, we see that the first letter of the square.
code represents the shape of each term 94. (c) In all other figures, small semi-circle
and second letter represents the type of and dot inside the large semi-circle are
18+5+4=27
shading in each term. identical. i.e., either shaded or unshaded
Sum of the position numbers of the
Here, letter code A represents triangle, in similar manner.
letters.
letter code X represents slant strips, letter 95. (b) In all figures except (b), the two
Therefore, D A N C E
code D represents circle and letter code arrows are in opposite direction i.e., one
Z represents dots. in clockwise direction and other in
4 + 1+14+3+5 = 27 So, the code for the figure is DZ. Since, anti-clockwise direction.
D represents circle and Z represents 96. (d) Similar figure repeats in every second
86. (d) Clearly, the problem figure is dots. step. Each time a particular figure
embedded in answer figure (d). Hence, option (a) is correct. reappears, it gets rotated through 180°
90. (a) Each element code is represented as and the number of arrowheads increases
by one.
B, C, D 97. (d) The arrow moves 4, 3, 2 and 1
spaces clockwise sequentially and the pin
87. (c) Clearly, the problem figure is and the double line arrangement as moves 3, 4, 5 and 6 spaces
embedded in answer figure (c). anti-clockwise sequentially.
G, H and I 98. (d) A new small line segment is added to
one of the lines in the figure and this
So, code for the figure is CG. addition takes place sequentially in
Hence, option (a) is correct. anti-clockwise direction.
Here, we have selected vertical image 91. (d) Only in figure (d), the number of 99. (a) All the elements move one space
and not horizontal image because we lines (1, 2, 3, 4) are arranged in order clockwise and two spaces clockwise
give preference to the one which depicts but in all other figures, the order of alternately.
the problem figure embedded with least number of lines is not maintained. 100.(d) As we go from left to right, the arrow
change in its orientation.
92. (d) In all other figures, the number of moves 45° and 90° anti-clockwise
88. (d) Clearly, the problem figure is line segments on the boundary of the alternately and the pin moves 45° and
embedded in answer figure (d). figure is two less than the number of 90° clockwise alternately.
sides of the figure.
AFCAT ~ Practice Set 10 83

AFCAT
AIR FORCE COMMON ADMISSION TEST (ONLINE )

Practice Set ~10


Time : 2 Hrs MM : 300

INSTRUCTIONS
1. The set contains a total of 100 questions, Comprising Verbal Ability in English, General Awareness, Numerical Ability and Reasoning and
Military Aptitude Test.
2. Each correct question carry 3 Marks and there will be negative marking of 1 Mark for each incorrect attempt.
3. Total time duration will be 2 hrs (120 minutes).
4. No marks will be deducted for unattempted questions.

Directions (Q. Nos. 1-3) In the 5. Buoyant Directions (Q. Nos. 12-16) Find out
following passage, some words have (a) Joyful (b) Sad the correctly spelt words.
been omitted. For each blank four (c) Angry (d) Clever 12. (a) Consince (b) Conscience
alternatives are suggested. Select the
6. Summit (c) Concinse (d) Consinse
suitable alternative to fill the given
blanks. (a) Base (b) Slope 13. (a) Changable (b) Changeable
(c) Declivity (d) Peak
Squirrels are the most resourceful (c) Changabel (d) Changeabel
animal in the world. They have the Directions (Q. Nos. 7-9) Choose the 14. (a) Jugglar (b) Juglerr
knack of saving up for …… (1) days. correct antonym from the following words.
(c) Jugler (d) Juggler
Autumn can be very entertaining for 7. Wrath
them. That is the time …… (2) they 15. (a) Bankrupsy (b) Bankruptsy
(a) Agitated (b) Gentleness
begin the great harvest …… (3) for (c) Bankruptcy (d) Bankrupay
(c) Anger (d) Worship
their winter store. You can see them 16. (a) Que (b) Queu
8. Brisk
scampering here and there, collecting (c) Qeue (d) Queue
(a) Agile (b) Nimble
nuts of all sorts-Walnuts, beechnuts,
chestnuts, dried berries. (c) Sluggish (d) Quick 17. Fill in the blank with correct
adjective.
1. (a) summer (b) rainy
9. Haphazard
Those are probably the ……
(a) Fortuitous (b) Indifferent curtains in the store.
(c) winter (d) autumn
(c) Deliberate (d) Accidental (a) fancy (b) fancier
2. (a) when (b) that
Directions (Q. Nos. 10 and 11) Find (c) fanciest (d) most fanciest
(c) which (d) then the errors in the following sentences. If
18. Fill in the blank with correct
3. (a) compilation (b) stacking there is no error, then choose ‘No error’
preposition.
(c) collection (d) hoarding as your answer.
Don’t loiter ……… the street.
Directions (Q.Nos. 4-6) Choose the 10. Why did you not (a)/told me that (a) onto (b) about (c) on (d) into
correct synonyms for the following the (b)/meeting was postponed?
words. (c)/No error (d)
19. Choose the appropriate
conjunction to fill the given blank.
4. Assimilate 11. He declared at the(a)/top of his They were ordered to wait ………
(a) Absorb (b) Arrange voice that (b)/it was not possible. the signal was given.
(c) Receive (d) Assemble (c)/No error (d) (a) unless (b) till (c) up to (d) when
84 AFCAT ~ Practice Set 10

Directions (Q. Nos. 20-23) Choose 26. The elephant looked no more 36. Which of the following is India’s
the correct meaning of the idiom bold dangerous than a cow because first indigenously developed
below. (a) it was quietly doing its work nuclear-powered submarine?
20. Parental property has become a (b) unlike lions, it is a vegetarian (a) INS Vikrant
bone of contention between the animal (b) INS Trikhand
siblings. (c) its tusks resemble the cow’s (c) INS Arihant
(a) unifying factor horns (d) INS Sindhurakshak
(b) cause of sadness (d) cows can be very dangerous
37. ‘Duty Unto Death’ is the motto of
(c) cause of happiness sometimes
which of the following Armed
(d) cause of quarrel Directions (Q. Nos. 27-30) In the Forces of India?
21. I am not someone who plays fast questions below, out of the four (a) CRPF (b) ITPB (c) BSF (d) NSG
and loose with other people’s lives. alternatives, choose the one which can
38. What is the name of the India’s
(a) behaves irresponsibly or be substituted for the words/sentence. longest suspension bridge built in
immorally 27. Constant effort to achieve Leh by Indian Army?
(b) behaves politely something (a) Gagan Bridge
(c) acts quickly
(a) Perseverance (b) Assiduous (b) Maitri Bridge
(d) fits
(c) Meticulous (d) Replica (c) Mahatma Bridge
22. Once in a blue moon, we meet (d) Sardar Bridge
each other.
28. Stealing from the writing of
others 39. The provisions of the Citizenship
(a) Frequently
(b) Very seldom indeed (a) Plagiarism (b) Reframing (Amendment) Bill, 2019 are not
(c) Sometimes (c) Copying (d) Reproducing applicable to tribal areas of which
(d) Infrequently 29. Opposed to great or sudden state?
change (a) Meghalaya (b) Jharkhand
23. Why don’t you cut to the chase?
(a) Mercenary (b) Conservative (c) Nagaland (d) Chhattisgarh
(a) stop talking
(b) stop pretending (c) Eccentric (d) Fastidious 40. Which of the following portals was
(c) come to the point 30. Branch of medicine concerned launched by RBI to report Foreign
(d) stop shouting with children and their illness. Liabilities and Assets?
(a) CISBI (b) SACHET
Directions (Q. Nos. 24-26) Read the (a) Pediatrics (b) Osteopathy
passage given below and answer the (c) Morphology (d) Cardiology (c) FLAIR (d) FORGE
questions that follow. 31. Who among the following is the 41. World Consumer Rights Day is
As soon as I saw the elephant I knew author of ‘Harshacharita’? observed every year on
with perfect certainty that I ought not (a) Hiuen Tsang (a) 12th March (b) 15th March
to shoot him. It is a serious matter to (b) Harshavardhana (c) 19th March (d) 25th March
shoot a working elephant.It is (c) Banabhatta 42. The cooking gas (LPG) mainly
comparable to destroying a huge and (d) Bhaskarvarman consists of
costly piece of machinery and (a) Ethane (b) Butane
obviously one ought not to do it if it 32. Which king started the
organisation of Kumbh fair at (c) Propene (d) Ethyne
can possibly be avoided. And at that
distance, peacefully eating, the elephant Prayagraj? 43. Which of the following is the
looked no more dangerous than a cow. (a) Harshavardhana commercial crop in India?
(b) Dhruvasena II (a) Mustard (b) Tobacco
24. The writer was against shooting
(c) Narsimhavarman I (c) Jute (d) All of these
the elephant because
(d) Akbar 44. Which of the following areas in
(a) he suspected it to be a wild one
and was afraid of it 33. In which language Tansen India are known as hotspot of
(b) his heart was full of compassion composed Dhrupad? biodiversity?
for animals (a) Marathi (b) Gujarati (a) Sunderban Deltas
(c) he was certain that the elephant (c) Prakrit (d) Brajabhasha (b) Western Ghats
was innocent (c) Eastern Ghats
(d) it would amount to avoidable 34. Sultan Azlan Shah Cup is
related to which among the (d) Gangetic Plain
waste of useful property.
following sports? 45. Name a metal which is the best
25. The author compares the elephant (a) Badminton (b) Hockey reflector of light.
to a costly machine because (c) Table Tennis (d) Golf (a) Gold (b) Silver
(a) ivory is very expensive
35. The word ‘Agricultural Shot’ is (c) Iron (d) Magnesium
(b) it can do as much work as an
expensive machine used in which among the 46. Which of the following states has
(c) elephants look like big machines following sports? the longest coastline?
(d) elephants and machines have (a) Cricket (b) Hockey (a) Goa (b) Gujarat
similar prices (c) Golf (d) Polo (c) Kerala (d) Andhra Pradesh
AFCAT ~ Practice Set 10 85

47. The first country to prepare a 56. A boat goes a certain distance at 67. There are four towns P, Q, R and
constitution 40 km/h and comes back the same T. Q is the South-West of P, R is
(a) Greece (b) Britain distance at 24 km/h. What is the the East of Q and South-East of P,
(c) France (d) USA average speed for the total journey? and T is to the North of R in line
(a) 32 km/h (b) 28 km/h with QP. In which direction of P is
48. Which Sikh Guru was executed T located?
by Aurangzeb? (c) 34 km/h (d) 30 km/h
(a) Guru Tegh Bahadur 7 . 2 × 7. 2 − 5 . 2 × 5 . 2 (a) South-East (b) North
57. The value of (c) North-East (d) East
(b) Guru Arjan Dev 6 .2 × 2
is
(c) Guru Hargobind
(a) 2 (b) 3 (c) 6.75 (d) 2.56
Directions (Q. Nos. 68-70) Find the
(d) Guru Gobind Singh missing term in the series given below.
58. Two persons can complete a piece
49. When and where was Panchayati 68. Z, U, Q, ?, L
of work in 9 days. How many more
Raj introduced in India? persons are needed to complete (a) I (b) K (c) M (d) N
(a) Mysore, 1955 (b) Nagaur, 1959 double the work in 12 days? 69. AC, FH, KM, PR, ?
(c) Kanpur, 1960 (d) Sonepat, 1958 (a) 3 (b) 2 (c) 4 (d) 1 (a) UX (b) TV (c) UW (D) VW
50. Who among the following is 59. The average cost of 4 items in a 70. OAC, PBD, QCE, RDF, ?
popularly known as ‘Father of shopping list is ` 1250. If one more (a) SGH (b) SHI (c) SEG (d) SIJ
Civil Services in India’? item whose cost is ` 2000 is added
(a) Cartier 71. Which one of the following
to the list, what will be the new
(b) Harry Verelst average? figures represents the relationship
(c) Lord Cornwallis among Shirts, Bedsheets and
(a) ` 1100 (b) ` 1500
(d) Warren Hastings Towels?
(c) ` 1400 (d) ` 1250
51. A sum of money amounts to 1
60. Simplify 21 × 7 + 25 ÷ 5 − 24 × .
` 4840 in 2 yr and to ` 5324 in 8
(a) 150 (b) 147 (c) 148 (d) 149
3 yr at compound interest
compounded annually. The rate of 61. The unit’s digit of the expression (a) (b) (c) (d)
interest per annum is
25 6521
+ 36
528
+ 73 54
is 72. Which one of the following
(a) 10% (b) 9%
(a) 6 (b) 5 (c) 4 (d) 0 figures represents Family, Sons
(c) 11% (d) 8%
62. A book vendor sold a book at a and Daughters?
52. A man can reach a certain place in loss of 20%. Had he sold it for
30 h. If he reduced his speed by ` 108 more, he would have earned
1
th, he goes 10 km less in that a profit of 30%. Find the cost
15 price of the book.
time. Find his speed per hour. (a) (b) (c) (d)
(a) ` 216 (b) ` 648 (c) ` 240(d) ` 432
1
(a) 6 km/h (b) 5 km/h
2 63. A can do a work in 20 days and B Directions (Q. Nos. 73-75) In the
(c) 4 km/h (d) 5 km/h can do the same work in 30 days. following questions, three alternatives
In how many days, can A and B out of four are same in a certain way
53. The length and breadth of a together do the work? and so form a group. Find the odd one
rectangular field are 25 m and (a) 10 (b) 12 (c) 15 (d) 16
15 m, and the two roads each of that does not belong to the group.
width 2 m parallel to length and 64. A person buys 100 cups at ` 10 73. (a) Raft (b) Chariot
breadth respectively exactly in the each. On the way 10 cups are
(c) Sledge (d) Cart
centre of field, cut each other and broken. He sells the remaining
remaining area has grass. The area cups at ` 11 each. His loss per 74. (a) Book (b) Pages
of grass section is cent is (c) Index (d) Chapters
1 1
(a) 295 m 2 (b) 299 m 2 (a) (b) 1 (c)1 (d) 2 75. (a) Huge (b) Tiny
2 2
(c) 300 m 2 (d) 375 m 2 (c) Heavy (d) Small
65. The simple interest on ` 7300
54. If X earns 25% less than Y, what Directions (Q. Nos. 76-79) In each of
per cent more does Y earn than x? from 11 May, 1987 to 10
September, 1987 (both days the following questions, choose the word
(a) 25 (b) 15 that will complete the second pair in the
1 2 included at 5% per annum) is
(c) 33 (d)16 (a) ` 123 (b) ` 103 (c) ` 200 (d) ` 223 same way as first pair.
3 3
66. In a coded language, ‘BRINJAL’ is 76. Knife : Cut : : ? : Guard
55. The HCF of two numbers is 16 (a) Dig (b) Shield
and their LCM is 160. If one of written as LAJNIRB. How will
‘LADYFINGER’ be written in that (c) Oar (d) Bore
the numbers is 32, then the other
number is code? 77. Cobbler : Leather : : Tailor : ?
(a) 48 (b) 80 (a) RNEGIFYDAL (b) RINEGIFYDAL (a) Thread (b) Cloth
(c) 96 (d) 112 (c) REGNIFYDAL (d) RGENIFYDAL (c) Shirt (d) Draper
86 AFCAT ~ Practice Set 10

78. College : Dean : : Museum : ? II. The opening batsmen were Answer Figures
(a) Superviser (b) Custodian spinners.
(c) Warden (d) Curator (a) Only Conclusion I follows
79. Moth : Insect : : Mouse : ? (b) Only Conclusion II follows
(c) Either I or II follows
(a) Cat (b) Mole (a) (b) (c) (d)
(d) Neither I nor II follows
(c) Rodent (d) Rat
Directions (Q. Nos. 84-87) In each of 89. Problem Figure
80. Given that 25th February 2008 is
Monday, what day is 2nd March the following questions, choose the option
of 2008? which contains region(s) common to
(a) Tuesday (b) Saturday circle, square, triangle, similar to that
(c) Sunday (d) Monday marked by the dots in figure (A).
Answer Figures
81. Two statements followed by two 84.
conclusions are given. Read the
conclusion and then decide which
of the conclusions, if any, logically (A)
follows from the two given (a) (b) (c) (d)
statements, disregarding the
known facts. 90. Problem Figure
Statements All lamps are poles.
Some poles are pipes. (a) (b) (c) (d)
Conclusions
I. Some lamps are pipes. 85.
II. Some pipes are poles. Answer Figures
(a) Only Conclusion I follows
(b) Only Conclusion II follows (A)
(c) Neither I nor II follows
(d) Both Conclusions I and II follow
82. Two statements followed by two (a) (b) (c) (d)
conclusions are given. Read the
conclusions and then decide which (a) (b) (c) (d) Directions (Q. Nos. 91-93) In each
of the conclusions, if any, logically of the following questions, find out
follows from the given statement, 86. the answer figure that can replace the
disregarding the known facts. question mark (?) to complete the
Statements problem figure.
Some educated are cigarettes. (A)
Some educated are tables. 91. Problem Figure
Conclusions
I. Some tables are educated.
II. Some educated are not
cigarettes. (a) (b) (c) (d)
(a) Only Conclusion I follows ?
87.
(b) Only Conclusion II follows
(c) Neither I nor II follows Answer Figures
(d) Both Conclusions I and II follow
83. A statement is given followed by (A)
two conclusions is given. You have
to take the given statement to be (a) (b) (c) (d)
true even, if they seem to be at
variance from commonly known (a) (b) (c) (d) 92. Problem Figure
facts. Read the conclusions and
then decide which of the Directions (Q. Nos. 88-90) In the
conclusions, if any, logically follows
following questions, find out that
from the given statement,
disregarding the known facts. answer figure which is embedded in the
Statement In an one day cricket problem figure. ?
match, the total runs made by a 88. Problem Figure
team were 200. Out of these 160 Answer Figures
runs were made by spinners.
Conclusions
I. 80% of the team consists of (a) (b) (c) (d)
spinners.
AFCAT ~ Practice Set 10 87

93. Problem Figure 95. Problem Figures Answer Figures

ii ?
i i
i (1) (2) (3) (4)
(a) (b) (c) (d)
i ? Answer Figures
Directions (Q. Nos. 98-100) In
Answer Figures the following questions, choose the
figure which does not share the
(a) (b) (c) (d) common features with other three
(a) (b) (c) (d) figures.
96. Problem Figures
Directions (Q. Nos. 94-97) In the 98.

+
following questions, select the figure +
?
from the set of answer figures which

+
would come in the place of question (1) (2) (3) (4) (a) (b) (c) (d)
mark (?) ? Answer Figures
94. Problem Figures 99.
+ + + +

?
(a) (b) (c) (d) (a) (b) (c) (d)
(1) (2) (3) (4)
97. Problem Figures 100.
Answer Figures

?
(a) (b) (c) (d)
(1) (2) (3) (4)
(a) (b) (c) (d)

Answers
1 (b) 2 (a) 3 (c) 4 (a) 5 (a) 6 (d) 7 (b) 8 (c) 9 (c) 10 (b)
11 (d) 12 (b) 13 (b) 14 (d) 15 (c) 16 (d) 17 (c) 18 (b) 19 (b) 20 (d)
21 (a) 22 (b) 23 (c) 24 (d) 25 (b) 26 (a) 27 (a) 28 (a) 29 (b) 30 (a)
31 (c) 32 (a) 33 (d) 34 (b) 35 (a) 36 (c) 37 (c) 38 (b) 39 (a) 40 (c)
41 (b) 42 (b) 43 (d) 44 (b) 45 (b) 46 (b) 47 (d) 48 (a) 49 (b) 50 (c)
51 (a) 52 (d) 53 (b) 54 (c) 55 (b) 56 (d) 57 (a) 58 (d) 59 (c) 60 (d)
61 (d) 62 (a) 63 (b) 64 (b) 65 (a) 66 (c) 67 (c) 68 (d) 69 (c) 70 (c)
71 (a) 72 (c) 73 (a) 74 (a) 75 (c) 76 (b) 77 (b) 78 (d) 79 (c) 80 (c)
81 (b) 82 (a) 83 (d) 84 (d) 85 (a) 86 (d) 87 (b) 88 (b) 89 (d) 90 (a)
91 (c) 92 (d) 93 (c) 94 (b) 95 (c) 96 (a) 97 (d) 98 (a) 99 (c) 100 (d)
88 AFCAT ~ Practice Set 10

Hints and Solutions


1. (b) rainy 18. (b) Preposition ‘about’ is used to show 32. (a) King Harshavardhana started the
2. (a) when movement. So, option (b) is the right organisation of Kumbh mela. Kumbh
option. mela is a major pilgrimage and festival in
3. (c) collection
19. (b) ‘till’ is the suitable choice to fill the Hinduism that is celebrated in a term of
4. (a) ‘Assimilate’ is to absorb something. 12 years at four river-bank pilgrimage
given blank. Conjunction ‘till’ is used to
So, option (a) ‘absorb’ is the correct sites i.e. Prayagraj, Haridwar, Nashik and
denote a time period.
synonym of ‘assimilate’. Ujjain.
20. (d) Idiom ‘bone of contention’ means
5. (a) ‘Buoyant’ means ‘cheerful and 33. (d) Tansen’s musical compositions
‘main issue of disagreement’. So, option
optimistic’ and ‘joyful’ means ‘having or covered many themes and employed
(d) ‘cause of quarrel’ is the correct
causing great happiness’. So, option (a) Dhrupad. Most of these were derived from
meaning of the given idiom.
‘joyful’ is a synonym of ‘buoyant’. the Hindu Puranas, composed in
21. (a) Idiom ‘plays fast and loose’ means ‘to
6. (d) ‘Summit’ is the highest point of a Brajabhasha. It was written in praise of
act irresponsibly’. So, option (a) is the
mountain and ‘peak’ also means the Gods and Goddesses such as Ganesha,
correct meaning of the given idiom.
same. Therefore, ‘summit’ and ‘peak’ are Saraswati, Surya, Shiva, Vishnu
synonymous. 22. (b) Idiom ‘Once in a blue moon’ means (Narayana and Krishna Avatar).
‘very rarely’. So, option (b) ‘very seldom
7. (b) ‘Wrath’ means ‘extreme anger’ and 34. (b) The Sultan Azlan Shah Cup is an
indeed’ is the correct meaning of the
‘gentleness’ means ‘calm and kind annual international men’s field hockey
given idiom.
behaviour’. Therefore, option (b) tournament held in Malaysia. It began in
‘gentleness’ is antonym of ‘wrath’. 23. (c) The idiom ‘cut to the chase’ means 1983 as a biennial contest.
‘to come to the point’. So, option (c) is The tournament became an annual event
8. (c) ‘Brisk’ means ‘quick, energetic and
the correct answer. after 1998, following its growth and
active’ and ‘sluggish’ means
‘slow-moving or inactive’. So, option (c) 24. (d) The writer was against shooting the popularity. South Korea won the 2019
‘sluggish is antonym of ‘brisk’. elephant because he felt it would Sultan Azlan Shah Cup after defeating
amount to avoidable waste of useful India in the final.
9. (c) ‘Haphazard’ means ‘happening or
property. 35. (a) The term ‘Agricultural shot’ is
done in a way that is not planned or
organised’ and ‘Deliberate’ means 25. (b) The author compares the elephant associated with ‘cricket’. This shot is
intended or planned. So, option (c) to a costly machine because it can played with the swing of the bat across
‘deliberate’ is the correct antonym of do as much work as an expensive the line of the ball. The shot often causes
‘haphazard’. machine. a chunk of the pitch to be dug up by the
26. (a) The elephant looked no more bat due to which it is called ‘Agricultural
10. (b) ‘told’ should be replaced by ‘tell’
dangerous than a cow because it was shot’.
because the first form of verb is used
with ‘did’. quietly doing its work of eating. 36. (c) INS Arihant is India’s first indigenous
27. (a) ‘Perseverance’ is the continued effort nuclear powered submarine. It was
11. (d) The sentence is grammatically correct.
to do or achieve something despite launched on 26th July, 2009, on the
12. (b) The correctly spelt word is anniversary of Vijay Diwas (Kargil War
difficulties’. Hence, option (a) is a
‘conscience’ which means ‘the inner Victory Day). INS-Arihant is armed with
suitable choice.
sense of what is wrong and right in one’s four K-4 sub marine launched ballistic
conduct or motives’. 28. (a) ‘Plagiarism’ is the practice of stealing
missile with a range of 3500 km.
from the writing of others. So, option (a)
13. (b) The correctly spelt word is ‘changeable’ INS-Arihant is a part of Indian Navy’s
is a suitable choice.
which means ‘able to be changed’. secretive Advanced Technology Vessel (ATV)
29. (b) ‘Conservative’ means ‘averse to great project monitored by the Department of
14. (d) The correctly spelt word is ‘Juggler’.
or sudden change’. Hence, option (b) Atomic Energy.
Juggler is an entertainer who
‘conservative’ is the correct one word
continuously tosses and catches a 37. (c) ‘Duty unto Death’ is the motto of
substitution for the given statement.
number of objects so as to keep at least Border Security Force (BSF). BSF is a
one of them in air. 30. (a) ‘Pediatrics’ is the branch of border defence organisation of India.
medicine dealing with children and their
15. (c) The correctly spelt word is Established on 1st December, 1965, it is
diseases. Hence, option (a) is a suitable
‘Bankruptcy’ which means ‘the state of a Central Armed Police Force charged
choice.
not having enough money or assets to with guarding India’s land border during
pay all debts’. 31. (c) ‘Harshacharita’ is the biography of peace time and preventing transnational
Indian emperor Harsha written by his crime.
16. (d) The correctly spelt word is ‘Queue’
which means ‘a line of people or things court poet Banabhatta.
38. (b) The Indian Army has made a personal
awaiting their turn to be attended to or to The Harshacharita was the first
record after constructing of the longest
proceed’. composition of Banabhatta and is
suspension bridge ‘Maitri Bridge’ over
17. (c) ‘Fanciest’ is the correct option as it is considered to be the beginning of writing
Indus river in Leh within a span of 40
the superlative degree to be used with of historical poetic works in Sanskrit
language. days.
‘the’ in the given sentence.
AFCAT ~ Practice Set 10 89

39. (a) The Citizenship (Amendment) Bill, Sterling silver is used in jewellery, 52. (d) Let the speed of man = x km/h
silverware etc.
30x − 30  x −  = 10
2019 is a bill amending the Citizenship x

Act of 1955 to make illegal migrants who 46. (b) There are total 9 States and 4 Union  15
are Hindus, Sikhs, Buddhists, Jains, Territories that form Indian coastline. Out [∵ distance = time × speed]
Parsis and Christians from Afghanistan, of these, Gujarat state has the longest
30  x − x +
x
Bangladesh and Pakistan eligible for ⇒  = 10
coastline in India. It lies on the Kathiawar  15
Indian Citizenship. The provisions of the region and bounded by the Arabian Sea. x 10
bill are not applicable to the tribal areas ⇒ =
47. (d) USA is the first country to prepare a 15 30
of Assam, Meghalaya, Mizoram and
Constitution. The Constitution was 150
Tripura, included in the Sixth Schedule to ⇒ x= = 5 km/h
adopted by a convention of the states on 30
the Constitution.
17th September, 1787 and was
40. (c) The Reserve Bank of India has 53. (b) ∵ Area of field = 25 × 15
subsequently ratified by several states.
developed a portal, Foreign Liabilities Ratification was completed on 21st June, [∵ area of rectangle = length × breadth]
and Assets Information Reporting (FLAIR) 1788. The US Constitution is made of = 375 m2
system for Indian companies to report the seven articles, a Preamble, and a closing and area of roads
Foreign Liabilities and Assets (FLA). endorsement. In addition, the = 25 × 2 + 15 × 2 − 2 × 2
RBI mandates that all Indian companies Constitution has a Bill of Rights and
= 50 + 30 − 4 = 76 m2
which have received FDI or made FDI several amendments.
abroad i.e. overseas investment in the 48. (a) Guru Tegh Bahadur was the 9th Guru
previous year(s) should file the annual

15 m
of the Sikh religion. In 1675 Guru Tegh
return on Foreign Liabilities and Assets. Bahadur was captured and executed in
41. (b) World Consumer Rights Day is Delhi under the orders of the Mughal
observed on 15th March every year, as a Emperor Aurangzeb. According to Sikh 25 m
means of raising global awareness about history, Guru Tegh Bahadur decided to Area of grass section
consumer rights and needs. The theme confront the forced conversion of religion
∴ = Area of field − Area of roads
for 2019 World Consumer Rights Day is by the Mughal officials.
= 375 − 76 = 299m2
‘Trusted Smart Products’. 49. (b) Panchayati Raj is a system of rural
local Self-government in India. The 54. (c) Required percentage
42. (b) Liquified Petroleum Gas (LPG) or
auto gas mainly consists of propane and Panchayati Raj system was first adopted  R 
by the state of Rajasthan in Nagaur district = × 100
butane. It is produced as a by-product of 100 − R 
natural gas processing and petroleum on 2nd October, 1959. During the 1950s
=  × 100
and 60s, other State Governments also 25
refining. The components of LPG are
adopted this system as laws were passed  100 − 25 
gases at normal temperatures and
to establish panchayats in various states. 25 1
pressures. = × 100 = 33 %
The second state to adopt Panchayati Raj 75 3
LPG is used as a fuel for heating, cooking system was Andhra Pradesh.
and in some vehicles. 55. (b) HCF × LCM = First number
50. (c) Though, Warren Hastings laid the × Second number
43. (d) A commercial crop or cash crop is an foundation of Civil Services but Charles ⇒16 × 160 = 32 × Second Number
agricultural crop which is grown for sale Cornwallis reformed, modernised and 16 × 160
to return a profit. The term is used to rationalised it. Hence, Charles Cornwallis ∴Second number = = 80
32
differentiate from subsistence crops is known as the ‘Father of Indian Civil
which are those fed to the producer’s Services’ in India. He introduced 56. (d) Let the distance covered by boat in
own livestock or grown as food for the Covenanted Civil Services (High Civil each journey be 120 km.
Distance
producer’s family. Some examples of Services) and Uncovenanted Civil ∵ Time =
commercial crops are cotton, tobacco, Services (Lower Civil Services). Speed
mustard, sugarcane, clusterbean, jute, ∴Time taken in ongoing journey
51. (a) Let the rate of interest = r% per annum 120
etc. T = = 3h
Amount = P 1 +
R  40
44. (b) The Western Ghats are one of the ∵ 
 100 and time taken in return journey
world’s biodiversity hotspots with over 2 120
4840 = P 1 +
r  = = 5h
5,000 flowering plants, 139 mammals, ∴  ...(i)
508 birds and 179 amphibian species.  100 24
Total distance
3
∴Average speed =
and 5324 = P 1 +
At least 325 globally threatened r 
 …(ii) Total time
species occur here.  100
120 + 120 240
A biodiversity hotspot is a biographic On dividing Eq. (ii) by Eq. (i), we get = =
region that is both a significant reservoir 5324 r 5324 r 3+ 5 8
of biodiversity and is threatened with =1+ ⇒ −1= = 30 km/h
4840 100 4840 100
destruction. 5324 − 4840 r . )2 − (52
(72 . )2 (72 . − 52 . + 52
. ) (72 .)
⇒ = 57. (a) =
45. (b) Sterling silver is considered as the 4840 100 . ×2
62 62. ×2
best reflector of light. It is an alloy 484 [∵(a2 − b2 ) = (a + b) (a − b)]
⇒ × 100 = r
comprises of about 92.5% silver and 4840 2 × 12.4
= =2
7.5% of another metal, usually copper. r = 10% per annum 62. ×2
90 AFCAT ~ Practice Set 10

58. (d) Given, M1 = 2, D1 = 9, W1 = 1 64. (b) CP of 100 cups = `100 × 10 = ` 1000 72. (c)
Let x more person are needed, then 10 cups are broken. Family
M2 = (2 + x), D2 = 12, W2 = 2 ∴SP of 90 cups = ` (90 × 11) = ` 990
MD M D
By the formula, 1 1 = 2 2 Loss = ` (1000 − 990) = ` 10 Sons
W1 W2 Loss × 100
∴Loss per cent =
2 × 9 (2 + x) × 12 CP Daughters
⇒ = ⇒ 9 = 6 + 3x 10
1 2 = × 100 = 1%
1000
∴ x =1 Sons are different from daughters but they
65. (a) Time from 11 May to 10 September, are members of a family.
59. (c) Total cost of 4 items = 1250 × 4
1987, 73. (a) All except ‘Raft’ are drawn by animals.
= ` 5000 = 21 + 30 + 31 + 31 + 10
Total cost of 5 items = 5000 + 2000 123 74. (a) All other items are contained in a
= 7000 = 123 days = yr book.
365
7000
∴ New average = = ` 1400 P× R×T 75. (c) All other terms are used to denote the
5 ∴ SI = size.
100
1
60. (d) 21 × 7 + 25 ÷ 5 − 24 × 7300 × 123 × 5 76. (b) A ‘Knife’ is used to ‘Cut’, in the same
8 SI = = ` 123
365 × 100 way a ‘Shield’ is used to ‘Guard’.
1
= 147+ 25 × − 3 = 147 + 5 − 3 = 149 77. (b) As ‘Cobbler’ use leather to mend
5 66. (c) The letters have been written in
‘shoes’, similarly ‘Tailor’ uses clothes to
61. (d) Unit’s digit in the expansion of reverse order in the code,
make ‘Dresses’.
56521 = 5 As, B R I N J A L ⇒ L A J N I R B
Similarly, 78. (d) Second is the head of first.
(∵ 5 repeats for every power increased
L A D Y F I N G E R ⇒R E G N I F Y D A L 79. (c) Second denotes the class to which first
36528 = Unit’s digit in 6528 = 6) belongs.
67. (c) The direction diagram is as follows
(∵ 6 repeats for every power increased) 80. (c) Since, 2008 is a leap year. So, there
T
Now, 31 = 3, 32 = 9, 33 = 27, N
N-E are 29 days in the month of February in
N-W
34 = 81, 35 = 243 P W E 2008.
54 Given, 25th February = Monday
Now, remainder of =2 S-W S S-E
4 Q R ∴26th February = Tuesday
∴Unit’s digit = (3) = 9
2 ∴27th February = Wedenesday
Clearly, T is in North-East direction of P.
28th February = Thursday
Required digit = Sum of (9 + 5 + 6) = 20 68. (d) The pattern is as follows ∴29th February = Friday
Hence, required unit’s digit = 0
Z U Q N L ∴ 1st March = Saturday
62. (a) Let the CP of the book be ` x, then
and 2nd March, 2008 = Sunday
–5 –4 –3 –2
According to the question,
∴ ?=N 81. (b)
x × 80 x × 130
+ 108 =
100 100 69. (c) The pattern is as follows
∵SP = CP × 100+profit%  1 3 6 8 11 13 16 18 21 23 Lamps Pipes
 100  A C F H K M P R U W
 
and SP = CP × 100 − Loss%  +2 +3 +2 +3 +2 +3 +2 +3 +2
Poles
 100  ∴ ? = UW
13x 8x 5x
⇒ − = 108 ⇒ = 108 70. (c) The pattern is as follows Conclusions I. (×)
10 10 10 II. (P)
1080 OAC PBD QCE RDF SEG
∴ x= = ` 216 82. (a)
5 +1 +1 +1 +1
+1 +1 +1 +1
63. (b) Since, A can do work in 20 days. +1 +1 +1 +1 Tables Educated Cigarettes
1
∴The work done by A in 1 day =
20 ∴? = SEG
∵ B can do a work in 30 days. 71. (a)
1 Conclusions I. (P)
Now, the work done by B in 1 day = Bedsheets
30 Shirts II. (×)
Now, the work done by A and B together 83. (d) According to the statement, 80% of
1 1 2+ 3 5 1
in one day = + = = = the total runs were made by spinners.
30 20 60 60 12 Towels So, it does not mean that 80% of the
∴A and B will complete the work team consists of spinners. So, I does not
1
= = 12 days Shirts, Bedsheets and Towels are different follow. Nothing about the opening
1 batsmen is given in the statement. So, II
from one another. Therefore, these can
12 be represented by three distinct circles. also does not follow.
AFCAT ~ Practice Set 10 91

84. (d) Common to the 89. (d) Clearly, the answer figure (d) is 95. (c) From figure (1) to (2), the right figure
square and circle embedded in problem figure and has gets inverted and enlarged. The left one
Common to the
circle and rectangle been shown in the below figure. moves to the middle and the middle
Common to the figure gets inverted and reduced in
triangle and rectangle
size and becomes the innermost
85. (a) Common to the element.
square and rectangle
Common to the square, 96. (a) Semi-circular figure rotates 90°
90. (a) Clearly, answer figure (a) is
triangle and rectangle clockwise and all the signs move two
Common to the embedded in problem figure and has
steps in anti-clockwise direction. The
circle and square been shown in the below figure.
second figure moves one step
86. (d) Common to the anti-clockwise and both the circles
square and circle interchange positions.
Common to the circle, 97. (d) From problem figure (1) to (2),
rectangle, square and triangle 91. (c) Answer figure (c) will complete the larger figure rotates 180° in clockwise
Common to the direction and moves one and half step
problem figure.
circle and rectangle
92. (d) Answer figure (d) will complete the in anti-clockwise direction. The
87. (b) Common to the circle,
problem figure. smaller figure moves one step in
square and triangle anti-clockwise direction and becomes
93. (c) Answer figure (c) will complete the
darkened and two new small figures
problem figure.
Common to the circle are added.
and triangle 94. (b) From figure (1) to (2) , the upper left
98. (a) In all other figures, the wheel has
Only rectangle element rotates by 135° anti-clockwise
even number of teeth.
and shifts to lower left. The lower left
88. (b) Clearly, answer figure (b) is rotates by 90° anti-clockwise and shifts 99. (c) All other figures, except figure (c),
embedded in problem figure and has to lower right. The lower right rotates by have even number of leaves.
been shown in the below figure. 135° clockwise and shifts to upper right. 100.(d) Only in the figure (d), arcs on one
The upper right rotates by 90° side of the lines have all the curves in the
anti-clockwise and shifts to the centre. same direction.
92 AFCAT ~ Practice Set 11

AFCAT
AIR FORCE COMMON ADMISSION TEST (ONLINE )

Practice Set ~11


Time : 2 Hrs MM : 300

INSTRUCTIONS
1. The set contains a total of 100 questions, Comprising Verbal Ability in English, General Awareness, Numerical Ability and Reasoning and
Military Aptitude Test.
2. Each correct question carry 3 Marks and there will be negative marking of 1 Mark for each incorrect attempt.
3. Total time duration will be 2 hrs (120 minutes).
4. No marks will be deducted for unattempted questions.

Directions (Q. Nos. 1-4) Read the creating cooperative communities. The 3. What is the biggest obstacle in
passage carefully and then answer the cooperative ideology is indeed too promoting the cooperative system?
questions based on it. Give your precious and realistic to discard. Hard (a) Bureaucratisation
answers on the basis of information introspection and commitment to (b) Government control
given in passage and opinion of the further the cause is needed during the (c) Politicians heading cooperatives
author. centenary. (d) All of the above
There is definite credibility for 1. According to the passage, which 4. Which is/are the essential values
cooperative sector. The process of one of the following statements is that the cooperative sectors
withdrawal of the state from certain not true? need/needs to develop?
service areas should have, in the (a) The Indian cooperative sector is (a) Self help
ordinary course, opened up the doors equipped well to face the (b) Focus on members
for co-operativisation instead of challenges of non-liberal (c) Resources
privatisation. But no one believes that business environment (d) Both ‘a’ and ‘b’
the cooperative sector is competent (b) A definite credibility crisis
enough to accept this challenge. overshadows the cooperative Directions (Q. Nos. 5-7) In the
Unless the sector moves away from the sector following questions, find out which part
clutches of government control and (c) Cooperative ideology is of the sentence has an error. If sentence
worthwhile to be pursued is correct and has no error then mark (d)
comes out with norms for restraining
(d) It would be a welcome as your answer.
politicians from making use of the
development if cooperators 5. It is harmful to sit close (a)/to the
sector as a ladder for their personal become politicians
benefits, the movement is sure to slip television set (b)/as it affects on
down further. It is inevitable that a 2. What are the reasons for the eyes. (c)/No error (d)
good number of our cooperative cooperative movements to 6. She soon lost her heart (a)/to her
enterprises will disappear in the deteriorate? classmate (b)/and decided to
coming years, since they are not strong (a) Government control on become his partner. (c)/
cooperative sector No error (d)
enough to stand the trial of time.
(b) Politician’s ambition to use
However, inculcating the values of cooperative movements for 7. The elderly manager of the bank,
self-help and member centrality can individual gain (a)/together with his staff (b)/have
give rise to cooperatives, which are not (c) Both ‘a’ and ‘b’ resigned. (c)/No error (d)
just enterprises, but instruments of (d) None of the above
AFCAT ~ Practice Set 11 93

Directions (Q. Nos. 8-13) In the 18. The ……… on which you stand, 30. A person, who pretends to be
following passage, some words have been was once isolated. what he is not is called an
omitted. For each blank, four alternatives (a) plateau (b) plataeu (a) imbiber (b) impresario
are provided. Select the suitable (c) plataue (d) plateue (c) imitator (d) imposter
alternative to fill the given blanks.
19. He was paid a handsome amount 31. Who has become the first Indian
An upsurge .....(8) new research ..... (9) to ……… Mr. Gupta. musician to have minor planet
that animals have a much .....(10) level (a) asassinate (b) asasinite named after him?
of brain power ....(11) previously (a) Parveen Sultana
(c) assassinate (d) assasinate
though. If animals do have (b) Pandit Jasraj
intelligence, how do scientists measure 20. Your thoughts are very …. so you
(c) Kumar Gandharva
... (12)? Before defining animals get impressed by anyone easily.
(d) Prabha Atre
intelligence, scientists ....(13) what is (a) malleable (b) maleable
not intelligence. Instinct is not (c) maliable (d) malliable 32. Which of the following mountains
is a volcanic mountain?
intelligence. Directions (Q. Nos. 21-23) In the (a) Himalayan mountains
8. (a) to (b) of following questions, out of the four (b) Aravali mountain
(c) into (d) from alternatives, choose the one which best (c) Mount Fuji
expresses the meaning of the given (d) Rocky mountain
9. (a) suggests (b) concludes
word.
(c) interprets (d) explained 33. Which is the largest cotton
21. Garnish growing state in India?
10. (a) strongest (b) largest
(a) Honour (b) Respect (a) Gujarat (b) Bihar
(c) higher (d) lower (c) Obey (d) Adorn (c) Tamil Nadu (d) Karnataka
11. (a) that (b) than 22. Abandon 34. The resolution of Quit India
(c) those (d) them (a) Excuse (b) Forsake Movement was passed in which of
12. (a) up (b) it (c) Urge (d) Risk the following city?
(c) them (d) those 23. Odious (a) Bombay (b) Calcutta
(c) Madras (d) Lucknow
13. (a) formulated (b) calculated (a) Hateful (b) Rotten
(c) defined (d) utilising (c) Infamous (d) Sick 35. Which among the following is a
folk dance of India?
Directions (Q. Nos. 14-16) Select the Directions (Q. Nos. 24 and 25)
(a) Manipuri
word you consider most appropriate for Choose the word opposite in meaning to
(b) Bihu
the blank space and indicate your the given word.
(c) Kathakali
choice on the answer sheet. 24. Controversial (d) Bharatanatyam
14. When the bus was at full speed, (a) Uncertain (b) Dubious
36. Who was the founder of Gupta
its brakes failed and an accident (c) Undisputed (d) Questionable Empire?
was …… .
25. Alight (a) Vikramaditya
(a) inevitable (b) undeniable
(a) Disembark (b) Embark (b) Chandragupta
(c) fatal (d) miserable
(c) Embalm (d) Align (c) Ram Niwas Gupta
15. To explain his design to his (d) Sri Gupta
visitors, the architect ……… a Directions (Q. Nos. 26-30) Out of the
simple plan on the blackboard. four alternatives, choose the one which 37. Shivaji defeated the Mughals in
can be substituted for the given the battle of
(a) built (b) finalised
word/sentences. (a) Salher (b) Panipat
(c) sketched (d) arranged
(c) Purandar (d) Raigarh
16. Though Bonsai, a well-known art 26. Medicine given to counteract
form, originated in China, it was poison 38. Which among the following is the
……. by the Japanese. (a) Antibiotic (b) Antiseptic first Indian woman went into the
space?
(a) cultivated (b) finished (c) Antidote (d) Antifungal
(a) Venus Williams
(c) perfected (d) enlarged 27. One who hates marriage (b) Sunita Williams
Directions (Q. Nos. 17-20) Four words (a) Misanthrope (b) Misogamist (c) Kalpana Chawla
are given in each question, out of which (c) Misogynist (d) Polygamist (d) Neerja Bhanot
one word is correctly spelt. Find the correctly 28. That which cannot be read
spelt word to fill the given blanks.
39. Who among the following
(a) Negligible (b) Illegible has a record of winning most
17. Don’t give me ……… logics as I (c) Ineligible (d) Incorrigible Oscars?
want to hear clear idea of the 29. One who specialises in the (a) Charlie Chaplain
programme right now. mathematics of insurance (b) Walt Disney
(a) circuitous (b) circuitus (a) A statistian (b) An actuary (c) Tom Cruise
(c) circutous (d) circutious (c) An agent (d) An itinerant (d) Robin Williams
94 AFCAT ~ Practice Set 11

40. Which of the following National 51. The HCF and LCM of two 59. A man travelled a distance of
Park is the only existent habitat numbers are 44 and 264, 61 km in 9 h, partly by walking at
for the nearly extinct Asiatic respectively. If the first number is the speed of 4 km/h and partly on
Lions in India? divided by 2, then quotient is 44. bicycle at the speed of 9 km/h.
(a) Gir National Park The other number is The distance covered by walking is
(b) Mahatma Gandhi Marine (a) 147 (b) 528 (a) 16 km (b) 12 km
National Park (c) 132 (d) 264 (c) 15 km (d) 17 km
(c) Gahirmatha Marine Wildlife
Sanctuary 1 60. Babu and Asha can do a job
52. th part of a tank in full in 40 L together in 7 days. Asha is
(d) National Chambal Sanctuary 10
of oil is poured in that tank. 3
41. Which among the following is the 1 times as efficient as Babu. The
3 4
largest planet in our solar system? Then, th part is full. Find the
5 same job can be done by Asha
(a) Mercury (b) Jupiter
half capacity of tank? alone in
(c) Venus (d) Earth 49 49
(a) 80 L (b) 40 L (a) days (b) days
42. Davis Cup belongs to which sports? (c) 20 L (d) 100 L 4 3
(a) Tennis (b) Hockey 8
53. The cost price of an article is 40% (c) 11 days (d) days
(c) Cricket (d) Chess of the selling price. What per cent 3
43. Which one of following article deals of the cost price is the selling 61. The average of four consecutive
with the appointment of the Prime price? odd numbers is 40. What is the
Minister and other ministers? (a) 140% (b) 200% largest number?
(a) Article 76 (b) Article 74 (c) 220% (d) 250% (a) 42 (b) 45
(c) Article 75 (d) Article 72 54. In an examination, 1100 boys (c) 43 (d) 44
44. Impeachment of the President and 900 girls appeared, 50% of 62. A boat goes 20 km downstream in
can be initiated in the boys and 40% of the girls 1 h and the same distance
(a) Only in Lok Sabha passed the examination. The upstream in 2h. The speed of the
(b) Only in Rajya Sabha percentage of candidates who boat in still water is
(c) In either house of Parliament failed is (a) 15 km/h (b) 10 km/h
(d) Supreme Court (a) 45 (b) 45.5 (c) 5 km/h (d) 7.5 km/h
45. Which was the first nuclear (c) 50 (d) 54.5 63. Which of the following has
reactor in India? 55. If the total cost of 73 articles fractions in ascending order?
(a) Apsara (b) CIRUS having equal cost is ` 5110 2 3 7 9 8
(a) , , , ,
(c) Dhruva (d) Kaiga and the total selling price of 3 5 9 11 9
3 2 9 7 8
46. Who became the first Indian to 89 such articles is ` 5607, (b) , , , ,
then in the transaction, there 5 3 11 9 9
play 150 Ranji matches?
3 2 7 9 8
(a) Sairaj Bahutule will be (c) , , , ,
5 3 9 11 9
(b) Amol Muzumdar (a) a loss of 15% (b) a gain of 10% 8 9 7 2 3
(c) Wasim Jaffer (c) a loss of 10% (d) a gain of 15% (d) , , , ,
9 11 9 3 5
(d) Devendra Bundela
56. Manoj deposited ` 29400 for 64. If a * b = 2 a + 3 b − ab, then the
47. Sanchi Stupa is located near 6 yr at a simple interest. value of (3 * 5 + 5 * 3) is
(a) Bhopal (b) Gaya He got ` 4200 as interest after 6 yr. (a) 10 (b) 6
(c) Bijapur (d) Varanasi The annual rate of interest was (c) 4 (d) 2
48. Who among the following 8 7
(a) 2 % (b) 2 % 65. A, B and C can complete a work in
considered as the ‘Father of 21 20
8 8 2 h. If A does the job alone in 6 h
Artificial Intelligence’ in IT sector? (c) 3 % (d) 4 % and B in 5 h, then how long will it
(a) Charles Babbage 21 21
take for C to finish the job alone?
(b) Lee De Forest
57. Two numbers x and y are 20% 1
(a) 5 h
1
(b) 7 h
(c) John McCarthy
(d) JP Eckert and 50% more than a third 2 2
number respectively , x is how 1
49. Who won 55th Jnanpith Award? (c) 9 h (d) 4 h
much per cent of y? 2
(a) Rajeev Alunkal
(a) 30 (b) 45 66. If blue is called pink, pink is called
(b) Lalithambika Antharjanam
(c) Akkitham (c) 60 (d) 80 green, green is called yellow,
(d) Rafeeq Ahamed 58. Number of composite yellow is called red and red is
numbers lying between 67 and called white. Then, what is the
50. India’s first National Film
101 is colour of turmeric?
Museum was opened in
(a) 27 (b) 24 (a) Red (b) Green
(a) New Delhi (b) Mumbai
(c) 26 (d) 23 (c) Pink (d) yellow
(c) Hyderabad (d) Chennai
AFCAT ~ Practice Set 11 95
Directions (Q. Nos. 67-69) Find the 78. Comets : Meteors Statements
missing term in the series given below. (a) Books : Knowledge All bulbs are birds.
67. DKY, FJW, HIU, JHS, ? (b) Hawk : Crow Some birds are butterflies.
(c) Stars : Fortune Conclusions
(a) LFQ (b) LGQ (c) KGR (d) KFR
(d) Reptiles : Crawl I. All butterflies are bulbs.
68. B E H, K N Q, T W Z, ?
79. Music : Notes II. Some bulbs are butterflies.
(a) C F I (b) D G H
(a) Dance : Music (a) Only Conclusion I follows
(c) P R S (d) F I J
(b) Mathematics : Numbers (b) Only Conclusion II follows
69. C, U, R, E, G, T, W, ? (c) Language : Communication (c) Neither Conclusion I nor II
(a) X (b) E (c) F (d) Z (d) Nations : UN follows
70. Which of the following venn 80. Colour : Eyes (d) Both Conclusions I and II follow
diagram gives the relationship (a) Vision : Spectacles Directions (Q. Nos. 86-90) In each of
between Friend, Guide and (b) Print : Newspaper the following questions, choose the
Philosopher? (c) Medicine : Ailment options which contains region(s)
(d) Fragrance : Nose common to circle, square, triangle,
81. Launcher : Missiles similar to that marked by the dots in
(a) Gun : Revolver figure (A).
(a) (b) (c) (d) (b) Boat : Anchor 86.
71. Which figure represents Rhombus, (c) Catapult : Stone
Quadrilaterals, Polygons? (d) Engine : Train
(A) (a) (b) (c) (d)
82. Sorrow : Misery
(a) Love : Obsession 87.
(b) Amity : Harmony
(a) (b) (c) (d) (c) Happiness : Joy (A) (a) (b) (c) (d)
(d) Enemy : Hatred
72. Which of the following diagrams 88.
represents Smokers, Lawyers, 83. If 8th of April falls on Monday,
Non-smokers? what would be the 30th day of
that month? (A) (a) (b) (c) (d)
(a) Sunday 89.
(b) Monday
(c) Tuesday
(a) (b) (c) (d) (A) (a) (b) (c) (d)
(d) Wednesday
73. Which of the following diagram 84. Two statements followed by two 90.
represents Mammals, Cows and conclusions are given. Read the
Crows? conclusions and then decide (A) (a) (b) (c) (d)
which of the conclusions, if any,
logically follows from the two
Directions (Q. Nos. 91-93) In each of
the following question, Find the answer
given statements, disregarding the
figure in which problem figure is
(a) (b) (c) (d) known facts. embedded.
Statements
Directions (Q. Nos. 74-77) In the Some books are magazines. 91. Problem Answer
following questions, three alternatives Some magazines are novels. Figure Figures
out of four are same in a certain way Conclusions
and so form a group. Find the odd one I. Some books are novels.
that does not belong to the group. II. Some novels are books.
74. (a) Spring (b) Heat (a) Only Conclusion I follows (a) (b) (c) (d)
(c) Winter (d) Autumn (b) Only Conclusions II follows 92. Problem Answer
75. (a) Igloo (b) House (c) Neither Conclusion I nor II Figure Figures
(c) Hut (d) Factory follows
(d) Both Conclusions I and II follow
76. (a) Cholera (b) AIDS
(c) Cancer (d) Health 85. Two statements followed by two
(a) (b) (c) (d)
conclusions are given. Read the
77. (a) Sweater (b) Blouse
conclusions and then decide 93. Problem Answer
(c) Trouser (d) Shirt
which of the conclusions, if any, Figure Figures
Directions (Q. Nos. 78-82) Find logically follows from the two
out the pair in which the words bear the given statements, disregarding the
same relationship to each other as known facts.
similar to the words of the given pair. (a) (b) (c) (d)
96 AFCAT ~ Practice Set 11

Directions (Q. Nos. 94-97) In each of 96. Problem Figure Directions (Q.Nos. 98-100) In each of
the following questions, find out the the following question choose the figure
answer figure that can replace the ‘?’ to which does not share the common
complete the problem figure. features with other three figures.
94. Problem Figure Answer Figures ?
98.
Answer Figures
?
(a) (b) (c) (d)

(a) (b) (c) (d) (a) (b) (c) (d) 99.

95. Problem Figure Answer Figures 97. Problem Figure Answer Figures
(a) (b) (c) (d)
? ?
100.

(a) (b) (c) (d)


(a) (b) (c) (d) (a) (b) (c) (d)

Answers
1 (a) 2 (c) 3 (d) 4 (d) 5 (c) 6 (d) 7 (c) 8 (b) 9 (a) 10 (c)
11 (b) 12 (b) 13 (c) 14 (a) 15 (c) 16 (c) 17 (a) 18 (a) 19 (c) 20 (a)
21 (d) 22 (b) 23 (a) 24 (c) 25 (b) 26 (c) 27 (b) 28 (b) 29 (b) 30 (d)
31 (b) 32 (c) 33 (a) 34 (a) 35 (b) 36 (d) 37 (a) 38 (c) 39 (b) 40 (a)
41 (b) 42 (a) 43 (c) 44 (c) 45 (a) 46 (c) 47 (a) 48 (c) 49 (c) 50 (b)
51 (c) 52 (b) 53 (d) 54 (d) 55 (c) 56 (a) 57 (d) 58 (a) 59 (a) 60 (c)
61 (c) 62 (a) 63 (c) 64 (a) 65 (b) 66 (a) 67 (b) 68 (a) 69 (b) 70 (c)
71 (a) 72 (d) 73 (b) 74 (b) 75 (d) 76 (d) 77 (c) 78 (b) 79 (b) 80 (d)
81 (c) 82 (c) 83 (c) 84 (c) 85 (c) 86 (d) 87 (b) 88 (c) 89 (b) 90 (d)
91 (c) 92 (c) 93 (b) 94 (a) 95 (b) 96 (d) 97 (a) 98 (c) 99 (a) 100 (c)

Hints and Solutions


1. (a) According to the passage, option (a) 5. (c) ‘As it affects the eyes’ should be 16. (c) ‘Perfected’ means ‘make as good as
statement is not true as the Indian used. Use of the preposition ‘on’ is possible’. Therefore, option (c) completes
cooperative sector is not well equipped redundant here. the sentence.
to face the challenges of non-liberal 6. (d) No error. 17. (a) The correctly spelt word is
business environment. ‘circuitous’. ‘Circuitous’ means ‘not
7. (c) The use of plural verb ‘have’ is
2. (c) The control of government on incorrect. It should be replaced by straight or direct’.
cooperative sector and politician’s ambition singular verb ‘has’ as if two subjects are 18. (a) The correctly spelt word is ‘plateau’.
to use the cooperative movements for joined by ‘together with’, then the verb ‘Plateau’ means ‘a high, flat area of land’.
individual’s gain are the reasons for agrees with the subject mentioned first 19. (c) The correctly spelt word is
deterioration of cooperative movements. (Elderly Manager). ‘assassinate’. ‘Assassinate’ means ‘to
3. (d) Bureaucratisation, Government 8. (b) of 9. (a) suggests murder a prominent person either for
control and Politicians heading political or ‘religious reasons’.
10. (c) higher 11. (b) than
cooperatives, are the biggest obstacles in
promoting the cooperative system. 12. (b) it 13. (c) defined 20. (a) The correctly spelt word is ‘Malleable’
Hence, option (d) is its correct answer. which means ‘capable of being changed
14. (a) ‘Inevitable’ means ‘difficult to avoid or
easily’.
4. (d) ‘Self help’ and ‘focus on members’ are prevent’. Therefore, option (a) is
the essential values that the cooperative appropriate in the context of the sentence. 21. (d) ‘Garnish’ means ‘decorate or
sector needs to develop. Resources or embellish’ and ‘adorn’ means ‘to make
15. (c) ‘Sketch’ means ‘to draw a simple
their connection with cooperative sector is more beautiful or attractive’. So, option (d)
outline giving less details’. Therefore,
not discussed in the passage. Hence, ‘adorn’ express the correct meaning of
option (c) suits in the context of the
option (d) is the correct answer. ‘garnish’.
sentence.
AFCAT ~ Practice Set 11 97

22. (b) ‘Abandon and ‘Forsake’ both have It is related to the Bihu festival and an shall be appointed by the President. And
similar meaning i.e. ‘to leave behind’. So, important part of Assamese culture. on the advice of the Prime Minister the
‘Forsake’ is its correct answer. It is performed in groups. other Ministers shall be appointed by the
President.The Ministers shall hold office
23. (a) ‘Odious’ and ‘Hateful’ both have a 36. (d) Sri Gupta was the founder of Gupta
during the pleasure of the President.
similar meaning i.e. horrid, unpleasant, empire. The most notable rulers of the
offensive etc. So, ‘Hateful’ is the correct dynasty were Chandragupta I, 44. (c) The President may also be removed
synonym of ‘odious’. Samudragupta and Chandragupta II. before the expiry of the term through
Gupta age which existed between AD 320 impeachment by the Parliament of India.
24. (c) ‘Controversial’ means ‘disputed’. So, The process may start in either of the two
the antonym for this word is ‘undisputed. to 550, is also known as the Golden Age
of India. houses of the Parliament. A fourteen day
25. (b) ‘Alight’ means ‘to ‘descend from a notice should be given to the President.
37. (a) Shivaji defeated Mughals in the Battle
train, bus or other form of transport’. And 45. (a) India’s as well as Asia’s first nuclear,
of Salher which was fought between the
‘embark’ means ‘to go onboard a ship or reactor was ‘Apsara’ research reactor at
Maratha Empire and the Mughal Empire
aircraft’s. So, ‘embark’ is opposite in Mumbai. It was inaugurated by PM
in February 1672 CE. The battle was
meaning to ‘alight’. Jawaharlal Nehru in 1957. Apsara
fought near the Fort of Salher in the
26. (c) Medicine given to counteract poison Nashik district. The result was a decisive nuclear reactor was designed by the
is called antidote. victory for the Maratha Empire. Bhabha Atomic Research Centre (BARC).
27. (b) ‘Misogamist’ is one who hates 38. (c) Kalpana Chawla was an American 46. (c) Wasim Jaffer became the first Indian
marriage. astronaut, engineer and the first woman cricketer to play 150 Ranji matches.
28. (b) Writing that is impossible to be read of Indian origin to go to space. She was a Jaffer has made his first international
is called Illegible. part of space mission conducted by debut in 2000 against South Africa. In
29. (b) An actuary is a person who deals NASA in 1997. She was amongst the 2018, he also became the first batsman
with the measurement and management six-astronaut crew who travelled in Space to score 11,000 runs in Ranji Trophy.
of insurance risks and premiums. So, Shuttle Columbia flight STS-87. 47. (a) Sanchi stupa is a Buddhist complex
option (b) is the correct answer. In 2003, she died with six others crew at Sanchi Town in Madhya Pradesh. It is
members in Space Shuttle Columbia located 46 km North-East of Bhopal, capital
30. (d) A person, who pretends to be disaster, when it crashed in space. of Madhya Pradesh. The religious
somebody he is not is called an ‘imposter’.
39. (b) Walter Elias Disney was an American establishment at Sanchi was founded by
31. (b) A minor planet, between Mars and Entrepreneur, animator, voice actor and the Mauryan Emperor, Ashoka (272-237
Jupiter, has been named after Indian film producer. As a pioneer of the US BC).
classical vocalist ‘Pandit Jasraj’. animation industry, he introduced several 48. (c) John McCarthy, an American Computer
International Astronomical Union (IAU) developments in the production of Scientist, pioneer and inventor was known
has named minor planet ‘2006 VP32’ as cartoons. He has won 26 Oscars as the ‘Father of Artificial Intelligence’.
‘Pandit Jasraj’. throughout his carrer and holds the McCarthy was one of the founders of the
32. (c) Japan’s Mount Fuji is an active record for winning most Academy Awards discipline of Artificial Intelligence (AI). In
volcano about 100 km South-West of by one person. mid-1950, McCarthy coined the term
Tokyo. It is the country’s tallest peak, 40. (a) Gir National Park and Wildlife Sanctuary ‘Artificial Intelligence’ and defined it as
at 3,376 m and a famous pilgrimage ‘the science and engineering of making
also known as Sasan Gir, is a Wildlife
site. intelligent machines’.
Sanctuary and protected forest area in
33. (a) Gujarat is the largest cotton growing Gujarat, India. It is the only existent habitat 49. (c) Eminent Malayalam poet Akkitham
state in India with a production of 125 has won the 55th Jnanpith Award.
for the nearly extinct Asiatic Lions in India.
lakh bales. Cotton is an important crop Akkitham has authored 55 books out of
41. (b) Jupiter is the largest planet in our which 45 are collections of poems. He
for India as it contributes majorly to the solar system with a diameter of 142,800 has also received several other awards
national agro economy. All credit goes to km. It is fifth planet from the Sun. Its including Padma Shri Award, Sahitya
the perfect conditions in the state like name was derived from the Roman king Akademi Award, Kerala Sahitya Akademi
temperature, soil, water availability, of Gods. Its atmosphere is composed of Award and Vayalar Award.
rainfall, fertilizers and labour availability gases like hydrogen and helium.
that are favourable for the production of 50. (b) India’s first National Film Museum
Jupiter revolves around the Sun once in a was opened in Mumbai. The museum
cotton crop. Maharashtra is the second
period of 11.86 Earth years. showcases the history of Indian Cinema.
largest cotton producing state in India.
42. (a) The Davis Cup is the premier It has ample artefacts, digital elements
34. (a) Mohandas Karamchand Gandhi
international team event in Men’s tennis. including kiosks, interactive digital
launched the ‘Quit India Movement’ at
It is run by the International Tennis screens and screen interfaces, etc.
the All India Congress Committee in
Federation (ITF). It is contested annually Vintage equipments, copies of important
Bombay on 8th August, 1942. Gandhi
between teams from competing countries films, sound tracks, old cinema
made a call to ‘Do or Die’ in his Quit
in a knock-out format. Dwight Filley magazines, trailers are displayed to show
India speech delivered in Bombay at the
Davis, Sr. was the founder of the Davis the history of Indian cinema.
Gowalia Tank Maidan.
Cup International tennis competition. 51. (c) Here, first number = 2 × 44 = 88,
35. (b) The Bihu dance is an indigenous folk 43. (c) According to Article 75 of the
dance from the Indian state of Assam. HCF = 44 and LCM = 264
Constitution of India, the Prime Minister
Now, by the formula,
98 AFCAT ~ Practice Set 11

1st number × 2nd number = HCF × LCM 57. (d) Let third number = 100 62. (a) Given,
⇒ 88 × 2nd number = 44 × 264 Then, x = 100 + 20 Distance = 20 km, Time = 1 h
264 × 44 ∴Speed downstream
⇒ 2nd number = x = 120
88 and y = 100 + 50 = 150 Distance
= = 20 km/h
= 132 ∴Required percentage Time
∴ 2nd number = 132 x Time = 2 h
= × 100%
52. (b) Let half capacity of tank = x L y Distance
Speed upstream =
Then according to the question, 120 Time
= × 100 = 80%
2x 3 20
+ 40 = 2x × 150 = = 10 km/h
10 5 58. (a) There are 27 composite numbers 2
6x 2x
⇒ − = 40 between 67 and 101. Speed of boat in still water
5 10 Speed downstream
68, 69, 70, 72, 74, 75, 76, 77, 78,
12x − 2x
⇒ = 40 80, 81, 82, 84, 85, 86, 87, 88, 90, + Speed upstream
10 =
91, 92, 93, 94, 95, 96, 98, 99 and 2
∴ x = 40 L 100. 10 + 20 30
= =
53. (d) Let the SP of the article = ` 100 59. (a) Distance travelled = 61 km 2 2
40
⇒ 40% of SP = 100 × Time taken = 9 h =15 km/h
100
Let he travelled x km on foot and y km 63. (c) Convert each of the given fraction into
∴ CP = ` 40 on bicycle decimal, we get
∴Required percentage ∴ x + y = 61 …(i) 2 3 7
100 = 066
. , = 06 . , = 077. ,
= × 100 = 250% and
x y
+ =9 3 5 9
40 4 9 9 8
= 081
. and = 088 .
54. (d) Given, number of boys = 1100 9x + 4y = 324 …(ii) 11 9
Number of girls = 900 On solving Eqs. (i) and (ii), we get . < 066
Clearly, 06 . < 077. < 081. < 088 .
Total candidates = 2000 x = 16 and y = 45 3 2 7 9 8
So, < < < <
According to the question, Hence, he covered 16 km by walking. 5 3 9 11 9
Number of failed candidates 60. (c) According to the question, 64. (a) ∵ a * b = 2a + 3b − ab
1100 × 50 900 × 60 Ratio of efficiency of Babu and Asha
= + ∴3 * 5 + 5 * 3 = 2 × 3 + 3 × 5 − 3 × 5
100 100 7
= 1: + 2 × 5+ 3×3− 5× 3
= 550 + 40 = 1090 4 = 6 + 15 − 15 + 10 + 9 − 15
∴Required percentage = 4:7 = 25 − 15 = 10
=
Failed candidates
× 100 ∴Ratio of time taken Babu and Asha
65. (b) Let C alone can finish the job in x h.
Total candidates =7:4
1090 According to the question,
= × 100 = 545. As the time taken is inversely 1
2000 proportional to efficient, therefore if Babu Work done by A, B and C in 1 h =
takes 7x days to complete work, Asha 2
55. (c) Given, CP of 73 articles = ` 5110 1 1 1 1
will take 4x days. + + =
∴CP of 89 articles 6 5 x 2
1 1 1
=
5110
× 89 = ` 6230 ∴ + = 1 1 1 1
73 7x 4x 7 = − −
4+ 7 1 x 2 6 5
Total SP of 89 articles = ` 5607 ⇒ = 15 − 5 − 6
28x 7 =
Loss = ` (6230 − 5607) = ` 623 30
Loss ⇒ 28x = 11 × 7 4
∴Loss percentage = × 100% =
CP 11 × 7 11
⇒ x= = 30
623 28 4
= × 100 = 10% =
2
6230 ∴Asha will complete the work in 15
56. (a) Given, P = ` 29400, T = 6 yr, 11 1
4x = 4 × ∴ x =7 h
SI = ` 4200 4 2
P× R×T = 11 days
SI = 66. (a) The colour of turmeric is yellow. But
100 61. (c) Let the four consecutive odd numbers here yellow called red. So, colour of
29400 × 6 × R be x, x + 2, x + 4, x + 6 turmeric is red.
⇒ 4200 =
100 ∴x + x + 2 + x + 4 + x + 6 = 40 × 4 67. (b) The pattern is as follows
R=
4200  Sum of terms 
294 × 6 ∵ Average = Number of terms  4 6 8 10 12
  D +2 F +2 H +2 J +2 L
50 11 –1 10 –1 9 –1 8 –1 7
= ⇒ 4x + 12 = 160 K J I H G
21 ⇒ 4x = 148
8 25 –2 23 –2 21 –2 19 –2 17
=2 % ⇒ x = 37 Y W U S Q
21
∴Largest odd number = 37 + 6 = 43 ∴? = LGQ
AFCAT ~ Practice Set 11 99

68. (a) The pattern is as follows 73. (b) 87. (b) Common to
Mammals the circle and
B(cd) E(fg) H(ij) K(lm) N(op) Q(rs) T(uv) triangle
W(xy) Z(ab) C(de) F(gh) I (jk) Crows Cows
∴ ? = CFI Common to the
square, circle
69. (b) 3 21 18 5 7 20 23 5 and rectangle
C U R E G T W E
+2 Crows belong to class Aves. Cow is a 88. (c) Only circle
+2 mammal.
+2
+2 74. (b) Except heat, all other terms represent Common to the
the season. circle and triangle
∴ ?=E
75. (d) All others are dwelling places of Common to the
70. (c) human being, while ‘Factory’ is a circle and square
Friend Guide working place.
Common to the
76. (d) Except ‘Health’ all other are different 89. (b) circle and square
kinds of diseases. Common to the circle,
Philosopher 77. (c) Except trouser, all others cover upper triangle and square
part of the body. Common to the
circle and triangle
78. (b) Both the words in the given pair
Some guides may be philosophers and belong to the same category. Common to the
90. (d) circle and square
vice-versa. 79. (b) ‘Music’ contains ‘Notes’ and Common to the
Some friends may be guides and ‘Mathematics’ contains ‘Numbers’. circle and triangle
vice-versa. Common to the
80. (d) ‘Colour’ can be seen with the help of
Some friends may be philosophers and circle, square
‘Eyes’ and ‘Fragrance’ can be smelled and triangle
vice-versa.
with the help of ‘Nose’.
Some philosophers and guides may be 91. (c) The question figure is embedded in
friends. 81. (c) ‘Missiles’ are ejected through answer figure(c).
‘Launcher’. Likewise ‘Stone’ is ejected
71. (a) Polygons 92. (c) The question figure is embedded in
through ‘Catapult’.
answer figure(c).
Quadrilaterals 82. (c) smelled
93. (b) The question figure is embedded in
83. (c) Since, Monday falls on 8th, 15th, answer figure (b).
Rhombus 22nd and 29th.
94. (a) Answer figure (a) will complete the
∴The day on 30th = Tuesday.
question figure.
84. (c)
Books Magazines Novels
95. (b) Answer figure (b) will complete the
question figure.
96. (d) Answer figure (d) will complete the
All rhombus are quadrilaterals. Conclusions I. (×) II. (×)
question figure.
All quadrilaterals are polygons. 85. (c) Birds 97. (a) Answer figure (a) will complete the
72. (d)
question figure.
Non- Bulbs Butterflies 98. (c) In all other figures, both the figures
Smokers Lawyers smokers are inverted image of each other.
99. (a) In all other figures, designs at the
Conclusions I. (×) II. (×) two ends of the line are in opposite
Some smokers may be lawyers and
directions.
vice-versa.Some non-smokers may be 86. (d) Common to the triangle,
lawyers and vice-versa. circle and square 100. (c) All other figures are similar in such a
But smokers are entirely different from Common to the way that pressed and inflated portion of
circle and rectangle each of the figures are identical.
the non-smokers.
100 AFCAT ~ Practice Set 12

AFCAT
AIR FORCE COMMON ADMISSION TEST (ONLINE )

Practice Set ~12


Time : 2 Hrs MM : 300

INSTRUCTIONS
1. The set contains a total of 100 questions, Comprising Verbal Ability in English, General Awareness, Numerical Ability and Reasoning and
Military Aptitude Test.
2. Each correct question carry 3 Marks and there will be negative marking of 1 Mark for each incorrect attempt.
3. Total time duration will be 2 hrs (120 minutes).
4. No marks will be deducted for unattempted questions.

Directions (Q. Nos. 1 and 2) Select 5. My mother has been working 10. (a) Previlege (b) Privilege
the word or group of words that is most hard for the last two weeks and (c) Privilige (d) Prevalage
similar in meaning to the word given in she feels RUN DOWN.
capital letters. 11. (a) Curias (b) Curiaus
(a) Energetic (b) Cold
(c) Emotional (d) Morbid (c) Curious (d) Qurious
1. MOMENTOUS changes are
taking place in the social and 12. (a) Comnnication
Directions (Q.Nos. 6-8) In the
economic life of India. following questions, find out which part (b) Comunication
(a) Notorious (b) Momentary of the sentence has an error. If you think, (c) Communication
(c) Official (d) Enormous sentence is correct and has no error then (d) Communicasion
2. His condition DETERIORATED mark option (d) as your answer. 13. (a) Commette (b) Committee
day by day. 6. He served in this factory (a)/since (c) Comittee (d) Comitte
(a) Went bad (b) Grew worse it started (b)/after a long closure
(c) Went down (d) Grew bad
Directions (Q. Nos. 14-17) In these
in 1960. (c)/No error (d)
questions, four alternatives are given for
Directions (Q. Nos. 3 and 4) Select 7. The hall was already packed the idiom/phrase. Choose the
the word or group of words that is most (a)/and there was less room for alternative which best expresses the
nearly opposite in meaning to the word the group of reporters (b)/who meaning of the idiom/phrase.
given in capital letters. came to cover the event. (c)/No
14. To get away
3. A small ALTERCATION between error (d)
(a) To escape (b) To face
the children started off a riot. 8. The rice from Dehradun is (c) To pass (d) To reach
(a) Quarrel (a)/more superior (b)/to that of
(b) Friendly discussion 15. Cheek by jowl
Saharanpur. (c) No error (d)
(c) Contest (a) Fighting (b) Screaming
(d) Race Directions (Q. Nos. 9-13) Four words (c) Close together (d) Rubbing cheeks
are given in each question, out of which 16. To given the cold shoulder
4. He agreed reluctantly to sign one word is correctly spelt. Find the
the form but looked ILL AT (a) To be unfriendly
correctly spelt word. (b) To abandon someone
EASE.
(a) Embarrassed (b) Comfortable 9. (a) Maintanance (b) Maintanence (c) To abuse someone
(c) Welcome (d) Easy (c) Maintenance (d) Maintenence (d) To curse someone
AFCAT ~ Practice Set 12 101

17. Tough act to follow of his activity seem contrary; this 29. Which of the following temple in
(a) Acting like a tough man external opposition in the two sides India was called as ‘The Black
(b) Difficult to perform like that had much to do with the tragic fate of Pagoda’ by the Europeans?
(c) Difficult to act well the man. But they “stand, (a) Tirumala Tirupati, Andhra Pradesh
(d) Acting skill is tough nevertheless, in the most exact and (b) Kamakhya Temple, Assam
rigidly consistent connection; for just (c) Konark Sun Temple, Odisha
Directions (Q. Nos. 18 and 19) Fill in by depth did Socrates succeed in” (d) Jagannath Temple, Odisha
the blanks with correct alternatives. giving it a construction and fruitful 30. When does a lunar eclipse occur?
18. A trade war …… the world’s turn. (a) When the Sun is between the
major economies would lead to a 23. What can be said after reading Moon and the Earth
significant …… of world trade. the passage? (b) When the Moon is between the
(a) with, increase (a) A student of Socrates is also a Earth and the Sun
(b) in, distance student of Sophism (c) When the Earth is between the
(c) amid, narrow (b) Socrates supported principles of Moon and the Sun
(d) between, contradiction Sophism openly (d) None of the above
19. The student was fairly …… of (c) Socrates philosophy is sophism 31. Which of the following is more
passing, yet previous experiences is its spirit energy efficient?
with this teacher left him …… . (d) Socrates differs from the (a) Incandescent Bulb
(a) uncertain, confused Sophism in method of thought
(b) Tube light
(b) certain, confident 24. Sophism and Socrates (c) CFL
(c) confident, uncertain (a) influenced each other (d) None of the above
(d) confident, certain (b) latter influenced former 32. Monsoon is caused by
(c) former influenced latter
Directions (Q. Nos. 20-22) Read the (a) impact of summer temperature on
(d) not influenced each other the seas
given passage carefully and select the
correct answer for the given blanks out
25. It can be inferred that (b) movement of clouds
(a) Socrates was a revered saint of (c) seasonal reversal of winds
of the four alternatives.
Sophism who helped the cause (d) rise in temperature
Changes in the climatic conditions are of this movement
33. What is it called when two atomic
also having a ……(20) impact on the (b) Socrates was though, in support
nuclei are combined?
water system. It has ……(21) in the of Sophism never came openly
(a) Nuclear fission (b) Nuclear fusion
to support it
……(22) of glaciers and erratic rainfall (c) Nuclear decay (d) Chain reaction
(c) Socrates had conflicting ideas
patterns that in turn are leading to about Sophism that caused him 34. Rechargeable batteries are called as
environmental imbalance. misfortune (a) Primary batteries
20. (a) indifferent (b) positive (d) Socrates never supported (b) Secondary batteries
Sophism in any form (c) Fuel cell
(c) neutral (d) negative
26. Which Mauryan ruler earned the (d) Electrochemical cell
21. (a) affected (b) resulted
title ‘Liberator’? 35. Who discovered cell in 1665?
(c) incorporated (d) decorated (a) Chandragupta Maurya (a) Robert Hooke (b) Robert Crook
22. (a) freezing (b) heating (b) Bindusara (c) David Thomson (d) Marie Francois
(c) melting (d) creating (c) Ashoka
36. Beighton Cup is associated with
(d) Chandragupta II which of the following sports?
Directions (Q. Nos. 23-25) Read the
following passage carefully and answer 27. Which among the following (a) Cricket (b) Hockey
the questions that follows dynasty is also called as Solanki (c) Football (d) Volleyball
Socrates, thus really, resembled the Dynasty? 37. Where were the first
Sophists neither in the spirit of his (a) Chalukyas Dynasty Commonwealth Games held?
philosophy nor in the tendency and (b) Chandella Dynasty (a) Amsterdam, New Zealand
method of thought, nor in the manner (c) Sena Dynasty (b) Hamilton, Canada
of his life. And yet, he was confused as (d) Yaduvansh Dynasty (c) Sydney, Australia
one of them. In this connection, 28. Who of the following was (d) Hyderabad, India
Windelband remarks, “On the one considered by the British 38. Under which among the following
hand, he brought the principle to be ‘The Father of Indian acts, Indian Legislature was made
underlying the sophistic movement to Unrest’? ‘Bi-cameral’?
its clearest and most comprehensive (a) Gopal Krishna Gokhale (a) Government of India Act, 1861
expression; on the other hand, he set (b) Lokmanya Bal Gangadhar Tilak (b) Government of India Act, 1892
himself in the most vigorous manner (c) Lala Lajpat Rai (c) Government of India Act, 1915
against its outcome”, these two sides (d) Madan Mohan Malviya (d) Government of India Act, 1919
102 AFCAT ~ Practice Set 12

39. Which Article is related to ‘Equal 49. Who among the following 58. When a number is divided by 56,
Justice and Free Legal Aid’? presented Union Budget the remainder will be 29. If the
(a) Article 39 (b) Article 39A maximum number of times? same number is divided by 8, then
(c) Article 43 (d) Article 43A (a) P Chidambaram the remainder will be
(b) RK Shanmukham Chetty (a) 6 (b) 7 (c) 5 (d) 3
40. Election Commission of India
(ECI) has recognised Jannayak (c) Pranab Mukherjee 59. A boy has a few coins of denomi
Janta Party (JJP) as state party of (d) Morarji Desai nations 50 paise, 25 paise and
which state? 50. UNESCO has designated which 10 paise in the ratio 1 : 2 : 3. If the
(a) UP (b) Bihar Indian city as ‘Creative City of total amount of the coins is ` 6.50,
(c) Haryana (d) Rajasthan Gastronomy’? the number of 10 paise coins is
(a) Mumbai (b) Chennai (a) 5 (b) 10 (c) 15 (d) 20
41. In which among the following
place, the first Indian National (c) Hyderabad (d) Varanasi 60. If x2 + 2 = 2 x, then the value of
Defence University was set up? 51. The radii of three spherical balls x4 − x3 + x2 + 2 is
(a) Medak, Andhra Pradesh are 3 cm, 4 cm and 5 cm. They (a) 0 (b) 1 (c) − 1 (d) 2
(b) Kanpur, Uttar Pradesh are melted to form a new big
61. A tea merchant professes to sell tea
(c) Gurugram, Haryana sphere, then radius of new
at cost price but uses a false weight
(d) Patiala, Punjab sphere is
of 900 g for 1 kg. The profit per
(a) 6 cm (b) 7 cm
42. Which of the following is not cent in his transaction is
associated with UNO? (c) 8 cm (d) 7.5 cm 1
(a)11 % (b) 10%
(a) ILO (b) WHO 52. The difference between the 9
(c) ASEAN (d) All of these simple interests received from two 1
(c) 9 % (d) 15%
different banks on ` 500 in 2 yr is 11
43. Which of the following is not a ` 2.5. The difference between
member country of Shanghai
their per annum rates of interest
62. Shri X goes to his office by scooter
Cooperation Organisation at a speed of 30 km/h and reaches
is
(SCO)? 6 min earlier. If he goes at a speed
(a) 0.10% (b) 0.25%
(a) China (b) India of 24 km/h, he reaches 5 min late.
(c) 0.50% (d) 1.00% The distance of his office is
(c) Kazakhstan (d) Sri Lanka
53. 8. 7 − [76
. − {6.5 − (5.4 − 4.3 − 2)}] (a) 20 km (b) 21 km
44. Which among the following gas is simplified to
was detected by the (c) 22 km (d) 24 km
(a) 2.5 (b) 3.5
Chandrayaan-2? 63. If A and B together can complete a
(c) 4.5 (d) 5.5
(a) Strontium-40 (b) Argon-40 work in 12 days, B and C together
(c) Nihonium-40 (d) Caesium-40 54. The average of marks obtained by in 15 days and C and A together
100 candidates in a certain in 20 days, then B alone can
45. Name the Arab Nation which examination is 30. If the average
launched its first ‘Underwater complete the work in
marks of passed candidates is 35 (a) 30 days (b) 25 days
Military Museum’?
and that of the failed candidates
(a) Morocco (b) Libya (c) 24 days (d) 20 days
is 10, then what is the number of
(c) Jordan (d) Qatar candidates who passed the 64. Two trains of equal length are
46. World Wide Web was proposed by examination? running of parallel lines in the
(a) Bill Gates (a) 60 (b) 70 same direction at 46 km/h and
(b) Bill Rogers (c) 80 (d) 90 36 km/h. The faster train passes,
(c) Tim Berners Lee the slower train in 36 s. The length
55. The marked price of a ceiling fan of each train is
(d) Vint Cerf is ` 1200 and the shopkeeper (a) 82 m (b) 50 m
47. Where is the headquarters of the allows a discount of 5% on it.
(c) 80 m (d) 72 m
Ordnance Factory Board (OFB) Then, selling price of the fan is
located? (a) ` 1410 (b) ` 1400 65. The rate of working of A and B are
(a) Mumbai (b) Kolkata in the ratio of 2 : 3. The number of
(c) ` 1140 (d) ` 1104
(c) Lucknow (d) Jaipur days taken by them to finish the
56. When 75 is added to 75% of a work is in the ratio
48. What is the full form of number, the answer is the (a) 2 : 3 (b) 4 : 9
‘SAARC’? number. Find 40% of that (c) 3 : 2 (d) 9 : 4
(a) South Australian Association for number.
Regional Cooperation 66. ‘Major’ is related to ‘Lieutenant’ in
(a) 100 (b) 80
(b) South American Association for the same way as ‘Squadron Leader’
(c) 120 (d) 160 is related to
Regional Cooperation
(c) South Asian Association for 57. 3
0.004096 is equal to (a) Group Captain
Regional Cooperation (b) Flying Attendant
(a) 4 (b) 0.4
(d) South African Association for (c) Flying Officer
(c) 0.04 (d) 0.004 (d) Pilot Officer
Regional Cooperation
AFCAT ~ Practice Set 12 103

67. ‘Neck’ is related to ‘Tie’ in the Directions (Q. Nos. 77-79) What 85.
same way as ‘Waist’ is related to comes in place of question mark (?) in
the series given below? BX CZ CY DY DX ?
(a) Watch (b) Belt
(c) Ribbon (d) Shirt 77. AZ, BY, CX, ? (a) DZ (b) CZ
(c) CX (d) BZ
68. ‘Atom’ is related to ‘Molecule’ in (a) EW (b) EU (c) GH (d) DW
the same way as ‘Cell’ is related to 78. AC, FH, K? , PR, UW 86. Two positions of a dice are shown
(a) Matter (b) Nucleus below. What number will be
(a) L (b) J (c) M (d) N
(c) Organism (d) Battery opposite to 3?
79. AMN, BOP, CQR, ? 6
69. ‘Flower’ is related to ‘Petal’ in the 3
(a) BAS (b) DST (c) EQP (d) FRS 1 3
same way as ‘Book’ is related to 4 2
(a) Page 80. Raj travelled from a point X
straight of Y at a distance of (i) (ii)
(b) Content
(c) Author 80 m. He turned right and walked (a) 5 (b) 2
(d) Library 50 m, then again turned right and (c) 3 (d) 1
walked 70 m. Finally, he turned
Directions (Q. Nos. 70-73) In the right and walked 50 m. How far is Directions (Q. Nos. 87-90) In the
following questions, three alternatives he from the starting point? following questions, out of the four
out of four are same in a certain way (a) 10 m (b) 20 m given figures, three are similar in a
and so form a group. Find the odd word (c) 50 m (d) 70 m certain way, one figure is not like the
that does not belong to the group. others. Choose the different figure.
81. Two statements followed by two
70. (a) Cheese (b) Wine conclusions are given. Read the 87.
(c) Milk (d) Curd conclusions and then decide
71. (a) Bronze (b) Silver which of the conclusions, if any
(c) Cadmium (d) Platinum logically follows from the two (a) (b) (c) (d)
given statements, disregarding the
72. (a) Herd (b) Flight
known facts. 88. × ×
(c) Hound (d) Swarm ×
Statements
73. (a) Frequency polygon All birds are peacocks. ×
(b) Rectangle (a) (b) (c) (d)
Rohit is a peacock.
(c) Bar
Conclusions 89.
(d) Pi
I. Rohit is a bird.
74. Which of the following diagrams II. All peacocks are birds.
illustrate the relationship among (a) (b) (c) (d)
(a) Only Conclusion I follows
Science, Maths and Biology?
(b) Only Conclusion II follows 90.
(c) Neither I nor II follows
(d) Both Conclusions I and II follow
82. If ‘BOMBAY’ is coded as (a) (b) (c) (d)
(a) (b) (c) (d) FSQFEC, which word could be
coded as QCWSVI? Directions (Q. Nos. 91-93) In each of
75. Which one of the following (a) MANDYA (b) MANDAL the following questions, find out the
figures represents relationship (c) MYSORE (d) MYSOER figure that can replace the problem
among furniture, chairs, tables? mark (?) to complete the problem figure.
83. In a certain code, PEN is coded as
123, PENCIL as 123456 and 91. Problem Figure
CABLE as 48962, then
6283123456 means what?
?
(a) (b) (c) (d) (a) LAENPENCIL (b) LEANPNCLI
(c) LANPENCIL (d) LEANPENCIL
76. Which one of the following Directions (Q. Nos. 84 and 85)
represents men, rodents and living Choose the correct code for the uncoded
beings? term. Answer Figures
84.

BX EY BZ OZ CW ?
(a) (b) (c) (d)
(a) (b) (c) (d) (a) DY (b) EX (c) EZ (d) BW
104 AFCAT ~ Practice Set 12

92. Problem Figure 94. Problem Figures 97. Problem Figure

Answer Figures
(A)
Answer Figures Answer Figures

(a) (b) (c) (d)

(a) (b) (c) (d) 95. Problem Figure (a) (b) (c) (d)
93. Problem Figure
98. Problem Figure
?
Answer Figures

(A)

Answer Figures Answer Figures


(a) (b) (c) (d)

Directions (Q. Nos. 97-100) In the


(a) (b) (c) (d) following questions, choose the option (a) (b) (c) (d)
which contains region (s) common to
Directions (Q.Nos. 94-96) In each of circle, square, triangle similar to that 99. Problem Figure
the following questions, find out that marked by the dot in figure (A).
answer figure which is embedded in the
96. Problem Figure
problem figure.
Problem Figure
(A)
Answer Figures
(A)
Answer Figures
Answer Figures

(a) (b) (c) (d)

(a) (b) (c) (d)


(a) (b) (c) (d)

Answers
1 (d) 2 (b) 3 (b) 4 (b) 5 (a) 6 (c) 7 (c) 8 (b) 9 (c) 10 (b)
11 (c) 12 (c) 13 (b) 14 (a) 15 (c) 16 (a) 17 (b) 18 (d) 19 (c) 20 (d)
21 (b) 22 (c) 23 (d) 24 (b) 25 (c) 26 (a) 27 (a) 28 (b) 29 (c) 30 (c)
31 (c) 32 (a) 33 (b) 34 (b) 35 (a) 36 (b) 37 (b) 38 (d) 39 (b) 40 (c)
41 (c) 42 (c) 43 (d) 44 (b) 45 (c) 46 (c) 47 (b) 48 (c) 49 (d) 50 (c)
51 (a) 52 (b) 53 (c) 54 (c) 55 (c) 56 (c) 57 (b) 58 (c) 59 (c) 60 (a)
61 (a) 62 (c) 63 (d) 64 (b) 65 (c) 66 (c) 67 (b) 68 (c) 69 (a) 70 (b)
71 (a) 72 (c) 73 (b) 74 (a) 75 (c) 76 (c) 77 (d) 78 (c) 79 (b) 80 (a)
81 (c) 82 (c) 83 (d) 84 (b) 85 (d) 86 (a) 87 (c) 88 (c) 89 (c) 90 (d)
91 (d) 92 (b) 93 (b) 94 (d) 95 (b) 96 (a) 97 (a) 98 (a) 99 (a) 100 (b)
AFCAT~Practice Set 12 105

Hints and Solutions


1. (d) ‘Momentous’ means ‘very big or 20. (d) negative 21. (b) resulted with high atmospheric pressure) toward
great’. So ‘Enormous’ is its correct similar 22. (c) melting heat sources (warm region characterised
meaning word as it also means ‘very by low atmospheric pressure).
23. (d) After reading the passage, it can be Consequently, monsoon winds typically
large in size, quantity or extent’.
clearly inferred that Socrates differs from travel from sea to land in summer and
2. (b) ‘Deteriorated’ means ‘became worse’. the Sophism in method of thought.
So, option (b) ‘Grew worse’ is its correct vice-versa in winter.
24. (b) In Sophism and Socrates, latter 33. (b) Nuclear fusion is the process of
answer.
influenced former. combining two atomic nuclei to form a
3. (b) ‘Altercation’ means ‘a noisy argument
or disagreement’. So, option (b) ‘Friendly 25. (c) It can be inferred that Socrates had single heavier nucleus. In this process, a
discussion’ is the obvious antonym of it. conflicting ideas about Sophism that large amount of energy is released. In
caused him misfortune. contrast to Nuclear fusion, the process of
4. (b) ‘Ill at Ease’ means ‘to be worried and
26. (a) Chandragupta Maurya was the nuclear fission involves splitting of heavy
uncomfortable’. Its antonym would be
founder of Mauryan empire. He built one unstable nucleus into two lighter nuclei.
‘comfortable.’
of the largest empires on the Indian Fusion is the process by which the Sun
5. (a) ‘Run down’ means ‘very tired’. So its and other stars generate light and heat.
Subcontinent. He also earned the title of
correct antonym would be ‘energetic’ as
Liberator. 34. (b) Rechargeable battery, also called a
it means ‘full of energy’.
27. (a) The Chalukya dynasty ruled parts of storage battery or a secondary cell is a
6. (c) The given part (c) should be written type of battery that can be recharged over
North-Western Gujarat and Rajasthan.
as ‘long after its closure in 1960’. and over again with electrical energy.
Their capital was located at Anahilavada
7. (c) ‘had come’ should be used in place of (modern Patan). The Medieval legends Like other batteries, rechargeable
‘came’ because the sentence is in Past also called them ‘Agnivanshi Rajputs’ batteries can be recycled.
Perfect tense. and they are also known as the ‘Solanki Some of rechargeable batteries are:
8. (b) ‘Superior’ itself shows better in Dynasty’ in the vernacular literature. Nickel Cadmium battery, Lead acid
comparison, so use of ‘more’ is redundant. 28. (b) Lokmanya Bal Gangadhar Tilak was battery, Lithium Ion (Li-ion), battery,
9. (c) The correctly spelt word is ‘Maintenance’. called ‘The Father of Indian Unrest’ by Nickel-Metal Hydride battery and Lithium
‘Maintenance’ means ‘the act of maintaining’. the British officer Valentine Chirol. Tilak Ion Polymer (Li-ion polymer).
10. (b) The correctly spelt word is ‘privilege’. was born as Keshav Gangadhar Tilak on 35. (a) The cell was first discovered and
‘Privilege’ means ‘an advantage given to 23rd July, 1856 in Ratnagiri, named by Robert Hooke in 1665. Robert
a person or group’. Maharashtra. He was the first leader of Hooke was 17th century’s natural
the Indian Independence Movement. philosopher, physicist and a scientist.
11. (c) The correctly spelt word is ‘curious’.
‘Curious’ means eager to know or learn He was one third of the ‘Lal Bal Pal’ Hooke is best known for identification of
something. triumvirate. the cellular structure of plants.
12. (c) The correctly spelt word is 29. (c) Konark Sun Temple is a 13th century 36. (b) Beighton Cup is one of the oldest field
‘Communication’. ‘Communication’ means temple of Odisha, built by hockey tournaments instituted in 1895.
‘act of transferring information from one Narasimhadeva of the Eastern Ganga It is organised by Bengal Hockey
Dynasty. The temple is a UNESCO World Association. It is usually held on the
place, person or group to another’.
Heritage Site and also a major pilgrimage ground of Mohun Bagan Stadium in
13. (b) The correctly spelt word is site for Hindus. Kolkata.
‘Committee’. ‘Committee’ means ‘an The monument was also called ‘The 37. (b) The first Commonwealth Games were
association of people for a particular Black Pagoda’ by European sailors. held in 1930 in Hamilton, Canada. 400
purpose’.
30. (c) A lunar eclipse occurs when the Earth Athletes from11 countries participated in
14. (a) Idiom ‘To get away’ means ‘to escape’. comes between the Sun and the Moon six sports and 59 events in this game.
15. (c) Phrase ‘Cheek by jowl’ means ‘close and covers all or part of the lunar surface 38. (d) The Government of India Act, 1919
together’. with its shadow. The phenomenon of also known as Montague-Chelmsford
16. (a) Idiom ‘to give the cold shoulder’ lunar eclipse can occur only on the night Reforms was instituted in the British
of full moon. India to introduce the Bicameral
means ‘to ignore someone or treat
someone in a unfriendly way’. So, option 31. (c) CFLs are more energy efficient than Legislature, i.e., executive councilors and
(a) is a suitable choice. traditional incandescent bulb and popular ministers. The Act came into
tubelight. Though, CFLs initially cost force in 1921 and introduced,
17. (b) Idiom ‘Tough act to follow’ means ‘too
more, but they are less expensive in long bicameralism and direct elections in the
difficult to perform like that’ .
run. CFLs use 70% less energy than country. This bicameral legislature
18. (d) The pair of words given in option (d) incandescent bulb and tubelight. consisted of an Upper House (Council of
i.e. ‘between, contradict’ fits in the
32. (a) Monsoon is caused by impact of State) and a Lower House (Legislative
blanks perfectly. summer temperature on the seas. Assembly).
19. (c) The usage of ‘yet’ express the contrast Seasonal changes in temperature are 39. (b) Article 39A of the Constitution
in the sentence. So, pair of opposite large over land but small over ocean provides equal justice and free legal aid.
words i.e. confident, uncertain is suitable waters and monsoons blow from In reference to the Article, the State
to fill the given blanks. atmospheric heat sinks (i.e. cold region shall secure that the operation of the
106 AFCAT~Practice Set 12

legal system promotes justice and known as the web, is an information 53. (c) 8. 7 − [76
. − {6.5 − (54
. − 43
. − 2)}]
provide free legal aid by suitable system where documents and web [using VBODMAS rule]
legislation. resources are identified by URL (Uniform = 8. 7 − [76
. − {6.5 − (54
. − 2.3)}]
Resources Locators). The resources of
40. (c) The Election Commission of India = 8.7 − [76
. − {6.5 − 31. }]
(ECI) has recently recognised the the WWW may be accessed by users by
= 8. 7 − [76
. − 34
. }] = 8.7 − 42
. = 45
.
Jannayak Janta Party (JJP) as state a software application called web
browser. 54. (c) Let the number of passed students be x.
party of Haryana and alloted the symbol
∴Number of failed students be 100 – x.
of “key” for the party. The decision in 47. (b) Ordnance Factory Board (OFB) is an
this regard was taken on the basis of the industrial organisation functioning under Then, total marks obtained by 100
students = 100 × 30 = 3000
poll performance of the party in the the Ministry of Defence, Government of
 Sum of terms 
recent 2019 Assembly elections in India with its headquarter at Kolkata. It ∵ Average = 
Haryana. is engaged in research, development,  Number of terms 
41. (c) Indian National Defence University production, testing, marketing in the According to the question,
(INDU) is a mega project of Ministry of areas of air, land and sea systems. 35x + (100 − x)10 = 3000
Defence, Govt of India. It is situated at 48. (c) The South Asian Association for ⇒ 35x + 1000 − 10x = 3000
Binola village in Gurugram, Haryana. Regional Cooperation (SAARC). is the ⇒ 25x = 2000 ⇒ x = 80
It was first proposed in 1967. regional intergovernmental organisation 55. (c) ∵ Marked price = ` 1200
and geopolitical Union of States in South
The objectives of this university is to Discount = 5%
promote research-oriented national Asia. It was established on 8th
∴Selling price
defence policy, to develop the leadership December, 1985 in Kathmandu, Nepal.
100 − Discount per cent
qualities in military officers. Its member states comprise Afghanistan, = MP ×
Bangladesh, Bhutan, India, the 100
42. (c) The Association of South-East Asian 1200 × (100 − 5)
Maldives, Nepal, Pakistan and Sri =
Nations (ASEAN) is not associated with 100
Lanka.
UNO. It was established on 8th August, 1200
49. (d) With eight annual and two interim = × 95
1967 in Bangkok by the five original 100
member countries : Indonesia, Malaysia, budgets, Morarji Desai is the Finance
= ` 1140
Philippines, Singapore and Thailand. Minister who has presented the maximum
number of Union Budgets. 56. (c) Let the number be x.
The ASEAN is an intergovernmental
organisation aimed primarily at 50. (c) Hyderabad has been designated as 75% of x + 75 = x
75
promoting economic growth and regional ‘Creative City of Gastronomy’ by UNESCO. ⇒ x + 75 = x
stability among its member countries. The organisation has also included 100
75
43. (d) The Shanghai Cooperation or Mumbai as ‘Creative City of Films’ in its ⇒ 75 = x − x
network of ‘Creative Cities’. 100
Shanghai Pact (SCO) is a Eurasian 25
political, economic and security UNESCO Creative Cities Network ⇒ 75 = x
(UCCN) is a project of UNESCO to 100
alliance.The organisation was founded in
promote cooperation with and among ⇒ x = 75 × 4 = 300
2001 with China, Kazakhastan, Kyrgyz 40
Republic, Russia, Tajikistan and cities that have identified creativity as a Now, 40% of x = × 300
major factor in urban development. 100
Uzbekistan as its founding members.
51. (a) ∵ Volume of big sphere = 120
Later India and Pakistan also joined SCO
as a full members on 9th June, 2017. 4 4
= π(33 + 43 + 53) = π × 216 57. (b) Expression = 3
0004096
.
44. (b) Chandrayaan-2 orbiter has detected 3 3 In this type of question the decimal digit
the Argon-40 (Ar) in the lunar exosphere and let radius of big sphere is r, then is converted into decimal fraction.
4 3 4
from an altitude of ~ 100 km on the πr = π × 216
3 3 4096 212 24
moon surface. Argon-40, is one of the = 3 = 3
6
=
isotopes of the noble gas Argon. It is an ⇒ r 3 = 216 ⇒ r = 3 216 = 6 cm 1000000 10 102
important constituent of the lunar 52. (b) Let the two rates be R1% and R2 % 22 4
exosphere. It originates from the = = = 04
.
per annum. 10 10
radioactive disintegration of Potassium
-40 ( 40k), which has a half-life of ~ Then, R1 > R2 58. (c) Let the number be y, then
. × 109 years.
12 Difference in interest = 2.5 y = 56x + 29 = 8 × 7x + 8 × 3 + 5
P × R1 × T P × R2 × T
= − = 2.5 = 8(7x + 3) + 5
45. (c) Jordan is the first Arab nation, who 100 100 ∴If the same number is divided by 8,
unveiled its first underwater military ∵ SI = P × R × T  then the remainder will be 5.
 
museum off the coast of Aqaba. The dive  100 
site includes several tanks, an 59. (c) Ratio of the value of the coins
500 × 2 × R1 500 × 2 × R2
ambulance, a military crane, a troop ⇒ − = 2.5 1 2 3
= : : = 5: 5: 3
100 100 2 4 10
carrier, a combat helicopter, etc.
⇒ 10(R1 − R2 ) = 2.5 Let the value of 50 paise coins = ` 5x,
46. (c) English scientist Tim Berners-Lee 2.5
⇒ R1 − R2 = the value of 25 paise coins = ` 5x and
invented the World Wide Web (WWW) in 10 the value of 10 paise coins = ` 3x
1989. The World Wide Web, commonly = 0. 25% per annum
AFCAT~Practice Set 12 107

Now, Thus, B alone can do the work in 76. (c)


5x + 5x + 3x = 6.50 (total amount) 20 days. Living Beings
⇒ 13x = 6.50, x = 050
. 64. (b) Let the length of each train be x m.
Hence, the value of 10 paise coins Covered distance = x + x = 2x m Rod-
Men
ents
= 3 × 0 .50 = ` 1.5 Relative speed = 46 − 36 = 10 km/h
and number of 10 paise coins 10 × 5 25
= m/s = m/s
15. 18 9
= × 100 = 15
10 Covered distance
Time taken = Both men and rodents are living beings,
60. (a) Given, x 2 + 2 = 2x …(i) Relative speed of train
but men are different from rodents.
2x 36 × 25
On squaring both sides of Eq. (i), we get ∴ = 36 ⇒ 2x = 77. (d) The pattern is as follows
25 9
x 4 + 4 + 4x 2 = 4x 2 –1 –1 –1
9
⇒ x4 + 4 = 0 36 × 25
⇒ x= 1 26 2 25 3 24 4 23
⇒ x4 = − 4 …(ii) 18 A Z B Y C X D W
Again, on multiplying by x in Eq. (i) ∴ x = 50 m
+1 +1 +1
x 3 + 2x = 2x 2 65. (c) We know that,
1 ∴ ? = DW
⇒ x − x = x − 2x
3 2 2
…(iii) Efficiency ∝
Time taken 78. (c) The pattern is as follows
On subtracting Eq. (iii) from Eq. (ii), we
∴Required ratio = 3 : 2 1 3 6 8 11 13 16 18 21 23
get A C F H K M P R U W
x 4 − x 3 + x 2 = − 4 − x 2 + 2x 66. (c) In army and air force ‘Major’ and
‘Squadron Leader’ are equivalent ranks +2 +3 +2 +3 +2 +3 +2 +3 +2
⇒ x 4 − x 3 + x 2 + 2 = − 2 − x 2 + 2x
and so are ‘Lieutenant’ and ‘Flying ∴ ?=M
⇒ x 4 − x 3 + x 2 + 2 = − (2 + x 2 ) + 2x Officer’.
79. (b) The pattern is as follows
= − 2x + 2x [from Eq. (i)] 67. (b) ‘Tie’ is worn in the ‘Neck’ and ‘Belt’ is 1 2 3 4
=0 worn on the ‘Waist’. A
+1
B
+1
C
+1
D
61. (a) Tea is sold at cost price. 68. (c) First constitutes the second. 13 15 17 19
+2 +2 +2
Now, instead of using 1000 g weight the M O Q S
69. (a) ‘Petals’ constitute a ‘Flower’, likewise
tea merchant uses 900 g weight. 14 16 18 20
‘Pages’ constitute a ‘Book’. +2 +2 +2
∴Gain percentage of tea merchant N P R T
1000 − 900 70. (b) ‘Wine’ is alcohol while others got by
=   × 100% animal.
∴ ? = DST
 900 
80. (a) The direction diagram is as follows
100 100 1 71. (a) ‘Bronze’ is an alloy while
= × 100% = % = 11 % silver, cadmium and platinum are the
End
900 9 9 point
elements. C 80 m Y
62. (c) Let the distance of office be x km. (Starting X
72. (c) Except ‘Hound’, all represent group. point)
According to the question,
50 m 50 m
x x 11 73. (b) Frequency polygon, Bar and Pi are
− =
24 30 60 different type of graphs while ‘rectangle’
5x − 4x 11 x 11 is a geometrical figure. B 70 m A
⇒ = ⇒ =
120 60 120 60 74. (a)
N
11 Science
∴ x= × 120 = 22 km
60 W E
63. (d) Given, Biology Maths
S
1
(A + B)’s 1 day’s work = …(i) ∴ Required distance,
12
1 XC = XY − CY
(B + C)’s 1 day’s work = …(ii) = 80 − 70
15 Biology comes under the class science.
1 But, Mathematics is different from the = 10 m [∵CY = BA]
(C + A)’s day’s work = …(iii)
20 both. 81. (c)
On adding, 2 (A + B + C)’s 1 day’s work 75. (c)
1 1 1 Furniture
= + +
12 15 20
5+ 4+ 3 1 Birds Rohit
= =
60 5 Chair Table
1
∴(A + B + C)’s 1 day’s work = Peacocks
10
1 1
B’s 1 day’s work = −
10 20 Conclusions I. (×) II. (×)
2 −1 1 Chair is different from table, but both
= = Hence, neither I nor II follows.
20 20 come under the class furniture.
108 AFCAT~Practice Set 12

82. (c) As, From the above codes, we see that 94. (d) Answer figure (d) is embedded in
code for the last figure is EX, where E problem figure.
B O M B A Y F S Q F E C specifies circle and X specifies position
+4 of arrow.
+4
+4 Hence, option (b) is correct.
+4
+4
85. (d) Here, the coded series can be
+4 decoded as
Circle having four divisions = B, Circle 95. (b) Answer figure (b) is embedded in
Similarly,
having two divisions = C, Circle having problem figure.
Q C W S V I M Y S O R E three divisions = D and circle having
–4 two shaded divisions = X, circle having
–4
–4
one shaded division = Y and circle
–4 having no shaded division = Z.
–4 From the above codes, we see that
–4
code for the last figure is BZ, where B 96. (a) Answer figure (a) is embedded in
83. (d) As, specifies number of divisions and Z problem figure.
specifies type of shade.
P E N
Hence, option (d) is correct.
86. (a) From the given positions, numbers
1 2 3 4, 1, 6 and 2 lie adjacent to 3. So, the
remaining number i.e. 5. will lie
P E N C I L opposite to 3. 97. (a) Common to square,
triangle and circle
87. (c) Only in figure (c), the cross inside
Common to triangle
the circle is directly attached with the
1 2 3 4 5 6 and square
main line segment.
98. (a) Only triangle
88. (c) Except (c), all have two plus and
C A B L E Common to the
one minus but in figure (c) it is circle and square
vice-versa.
4 8 9 6 2 89. (c) In all other figures, inner figure has
less number of sides than the outer
Therefore, figure.
6 2 8 3 1 2 3 4 5 6 99. (a) Common to the triangle,
90. (d) In all the figures, except figure (d), circle and square
the line is placed adjacent to the Common to the
L E A N P E N C I L
unshaded circle. triangle and circle
91. (d) Answer figure (d) will complete the
84. (b) Here, the coded series can be decoded problem figure. 100. (b)
as
92. (b) Answer figure (b) will complete the Common to the
B, problem figure. circle, square
X, Y, Z, and and triangle
C/W 93. (b) Answer figure (b) will complete the
E, O and W/C problem figure.
AFCAT ~ Practice Set 13 109

AFCAT
AIR FORCE COMMON ADMISSION TEST (ONLINE )

Practice Set ~13


Time : 2 Hrs MM : 300

INSTRUCTIONS
1. The set contains a total of 100 questions, Comprising Verbal Ability in English, General Awareness, Numerical Ability and Reasoning and
Military Aptitude Test.
2. Each correct question carry 3 Marks and there will be negative marking of 1 Mark for each incorrect attempt.
3. Total time duration will be 2 hrs (120 minutes).
4. No marks will be deducted for unattempted questions.

Directions (Q. Nos. 1-3) Read the 1. It can be inferred that 5. Now that my children are
following passage carefully and answer (a) male dominance has ended and all grown up and gone out
the questions that follow. women dominance has taken over into the world (a)/I would like
The first question is whether men have (b) both men and women are at par to spend (b)/my old age in
always been this way or whether this in the family standing the village I was born. (c)/
behaviour has been precipitated by the (c) women have become sole No error (d)
profound changes in the roles of men provider for family
and women which have occurred in the (d) role of men and women has Directions (Q. Nos. 6-8) In the
20th century. Another issue is that changed but not completely following passage, some of the words
these changes in the relationships 2. What could have been the cause have been left out. Read the passage
between men and women, which of ‘mid-life-crises’ as discussed in carefully and select the correct answer
started around the time of the Second the passage? for the given blank out of the four
World War when women first moved (a) Male dominance alternatives.
into the workplace in large numbers, (b) Increasing age The recently launched ‘Swachh Bharat
has undergone another profound (c) Change in role of women Mission’ focusing on …(6)… sanitation
change in the 1970s and 1980s as the (c) None of the above facilities in schools and every rural
roles of men and women in the 3. Most probably the passage is written household by 2019, can prove to be
workplace have been equalised. No (a) during Second World War very …(7)… in bringing down the
longer is the man the sole provider for (b) in 1970-1980 …(8)… of girls dropping out of
his family or the provider who is (c) in 20th century school.
assisted by his wife, but now they are (d) cannot be determined
joint providers and many American 6. (a) depriving (b) improving
families are now experiencing the Directions (Q. Nos. 4 and 5) In each (c) residing (d) resisting
situation where the women is the of the following questions, find out
which part of the sentence has an error. 7. (a) significant
dominant provider. The men and women (b) meaningless
who come from the generation of this If the sentence has no error then mark
option (d) as your answer. (c) dominant
last change are just entering their mid- (d) buoyant
forties now, so there really has been no 4. It is nothing else than (a)/his
stupidity which stands (b)/in the 8. (a) cost (b) rate
experience with the passage of these
couples through their ‘mid-life-crises’. way of his progress. (c)/No error (d) (c) profit (d) price
110 AFCAT ~ Practice Set 13

Directions (Q. Nos. 9-11) In the Directions (Q Nos. 21-23) In the 31. Who was the founder of Haryanka
following questions, the sentence given following questions, out of the given four Dynasty?
with blank to be filled in with an alternatives, select the one which best (a) Bindusara
appropriate word. expresses the meaning of the given word. (b) Bimbisara
(c) Mahapadma Nand
9. We must insist that free oratory 21. Verbatim
(d) Kalashoka
is only the beginning of free (a) Exactly (b) Virtually
speech, it is not the end, but a (c) Carelessly (d) Loosely 32. Who was the first Muslim invader
.....… to an end. in India?
22. Temerity (a) Muhammed Ghori
(a) medium (b) middle
(c) trail (d) means (a) Boldness (b) Cowardice (b) Mahmud of Ghaznavi
(c) Distrust (d) Uncertainty (c) Muhammed Bin Kasim
10. There is absolutely no point in (d) Babur
complaining that over the years, 23. Ruck
there has been ……… for (a) Straighten (b) Mass 33. UNFCCC stands for
increased productivity and higher (c) Unbend (d) Leave (a) United Nations Framework
earnings for workers in industry. Convention on Climate Change
Directions (Q. Nos. 24-26) In the (b) United Nations Federation
(a) urge (b) force Convention on Climate Change
following questions, out of the given
(c) pressure (d) limit (c) United Nations Framework
four alternatives, select the one which is
11. Pidgins are languages that are not opposite in meaning of the given word.
Council on Climate Change
acquired as mother tongue and (d) United Nations Federation Council
that are used for a ……… set of 24. Rueful on Climate Change
communicative functions. (a) Contrite (b) Regretful 34. What is the SI unit of electric power?
(a) restricted (b) spacious (c) Defiant (d) Dolent (a) Coulomb (b) Watt
(c) still (d) many 25. Penury (c) Volt (d) Meter

Directions (Q. Nos. 12-15) In the (a) Dearth (b) Need 35. Commercially, sodium bicarbonate
following questions, out of the given four (c) Wealth (d) Adversity is known as
alternatives, select the alternative, which (a) Washing soda
26. Overweening
is the best substitute of the given phrase. (b) Baking soda
(a) Arrogant (b) Haughty (c) Bleaching power
12. Fear of riding in a car (c) Brash (d) Modest (d) Soda ash
(a) Astrophobia (b) Amaxophobia Directions (Q Nos. 27-30) In the 36. Which one of the following kinds
(c) Cacophobia (d) Genophobia following questions, out of the given of organisms causes malaria?
13. Loss of speech due to medical four alternatives, select the alternative (a) Bacterium (b) Fungus
problem which best expresses the meaning of the (c) Protozoa (d) Virus
(a) Anorexia (b) Anosmia given Idiom/Phrase.
37. Who is the first Lokpal of India?
(c) Aphasia (d) Hypogeusia 27. Play one’s ace (a) AR Lakshmanan
14. Place where cows are sheltered (a) To fight vehemently (b) GS Singhvi
(a) Pyre (b) Dyke (b) To use one’s best weapon or (c) Pinaki Chandra Ghose
(c) Byre (d) Convey resource (d) Madan Lokur
15. A low level tract of uncultivated (c) To win something 38. Who became the first woman of
grassland (d) To depend on chance Indian Navy to fly Dornier
(a) Island (b) Peninsula 28. Aid and abet Surveillance aircraft?
(c) Meadow (d) Oasis (a) To relieve someone of their duties (a) Punita Arora
(b) Shivangi Swaroop
Directions (Q. Nos. 16-20) In the (b) To help someone to commit a
(c) Minty Aggarwal
following questions, four words have been crime
(d) Shaliza Dhami
given out of which one word is incorrectly (c) To be charitable
spelt. Select the incorrectly spelt word. (d) To waste money 39. Which of the following Article
talks about the establishment of
16. (a) Camarderie (b) Comouflage 29. All over hell’s half acre the Supreme Court?
(c) Capricious (d) Carnivorous (a) Degrading oneself (a) Article 176 (b) Article 153
(b) Everywhere (c) Article 124 (d) Article 324
17. (a) Desicration (b) Disparaging
(c) To own a large piece of land
(c) Dilapidated (d) Diminution 40. What is the period of appointment
(d) To be sinful
18. (a) Exasperated (b) Excruchisating of the Comptroller and Auditor
30. An arrow in the quiver General of India?
(c) Exaggeration (d) Exultation
(a) Strategies that can be followed (a) 6 years
19. (a) Cease (b) Seize (b) Being vengeful (b) Up to 65 years of age
(c) Beseige (d) Beseech (c) Extremely old fashioned (c) Up to 64 years of age
20. (a) Carrier (b) Carreer (d) Types of weapons in one’s (d) 6 years or 65 years of age
(c) Courier (d) Barrier possession whichever is earlier
AFCAT ~ Practice Set 13 111

41. Who was the first Indian to receive 53. A train with a uniform speed 61. A, B and C individually can do a
the Ramon Magsaysay Award? passes a platform, 122 m long, in piece of work in 10 days, 12 days
(a) Indira Gandhi 17s and a bridge, 210 m long, in and 15 days, respectively. If they start
(b) TN Sheshan 25s. The speed of the train is working together, then the number
(c) Arvind Kejriwal (a) 46.5 km/h (b) 37.5 km/h of days required to finish the work is
(d) Acharya Vinoba Bhave (c) 37.6 km/h (d) 39.6 km/h (a) 16 days (b) 8 days
(c) 4 days (d) 2 days
42. ‘Dinar’ is the currency of which of 54. A person started his journey in
the following countries? the morning. At 11 am he covered 62. The value of
(a) Bahrain (b) Brazil 3    8  
of the journey and on the same 3 ÷ (8 − 5) ÷ (4 − 2) + 2 +  
(c) Bhutan (d) Georgia 8    13  
5
43. Name of the first country to issue day at 4 : 30 pm he covered of is
paper currency? 6 15 13 15 60
the journey. He started his (a) (b) (c) (d)
(a) China (b) UK 17 17 19 13
journey at
(c) Portugal (d) Greece 63. The difference between
(a) 6 : 00 am (b) 3 : 30 am
44. Where was the first computer (c) 7 : 00 am (d) 6 : 30 am compound interest (compounded
installed in India? annually) and simple interest on a
55. ` 180 contained in a box consists certain sum of money at 10% per
(a) New Delhi (b) Bangalore of ` 1, 50 paise and 25 paise coins
(c) Mumbai (d) Kolkata annum for 2 yr is ` 40. The sum is
in the ratio 2 : 3 : 4. What is the (a) ` 4000 (b) ` 3600
45. Which of the following river is number of 50 paise coins?
(c) ` 4200 (d) ` 3200
known as Dakshin Ganga? (a) 60 (b) 120 (c) 150 (d) 180 1 1
(a) Krishna (b) Mahanadi 64. If x = p + and y = p − , then
56. In a quarterly examination a p p
(c) Godavari (d) Cauvery student secured 30% marks and the value of x4 − 2 x2 y2 + y4 is
46. Who among the following British failed by 12 marks. In the same (a) 24 (b) 4 (c) 16 (d) 8
Governor General abolished the examination another student 65. A person deposited ` 400 for 2 yr,
slavery in India? secured 40% marks and got 28 ` 550 for 4 yr and ` 1200 for 6 yr.
(a) Lord Ellenborough marks more than minimum marks He received the total simple
(b) Sir Charles (Lord) Metcalfe to pass. The maximum marks in interest of ` 1020. The rate of
(c) Lord Auckland the examination is interest per annum is
(d) Lord Amherst (a) 300 (b) 500 (c) 700 (d) 400 (a) 10% (b) 5% (c) 15% (d) 20%
47. Who is the writer of the book 57. A dishonest fruit vendor sells his Directions (Q. Nos. 66-70) In the
‘Train to Pakistan’? goods at cost price but he uses a following questions, select the pair that
(a) Amrita Pritam weight of 900 g for the 1 kg has the same relationship as the
(b) Khushwant Singh weight. His gain per cent is original pair of words.
(c) Amrita Shergill 1
(a) 12% (b)11 % 66. Minute : Hour
(d) Mahadevi Verma 9
1 (a) Drop : Ocean (b) People : Crowd
48. Which is the most abundant gas (c)10 % (d) 10% (c) Cup : Tea set (d) Paisa : Rupee
9
in Earth’s atmosphere? 67. Statute : Law
(a) Oxygen (b) Nitrogen 58. A sells an article to B at a profit of
10%, B sells the article back to A at (a) Proviso : Clause
(c) Sulphur (d) Methane (b) Chapter : Exercise
a loss of 10%. In this transaction
49. On which of the following day is (a) A neither loses nor gains
(c) University : School
the Navy Day celebrated in India? (d) Section : Illustration
(b) A makes a profit of 11%
(a) 15th January (b) 8th October (c) A makes a profit of 20% 68. Round : Earth
(c) 4th December (d) 1st February (d) B loses 20% (a) Thin : Paper
50. Aluminium metal is extracted 59. A can do a piece of work in 20 (b) Height : Mountain
mainly from its ore days and B in 40 days. If they (c) Transparent : Glass
work on it together for 5 days, (d) Cube : Dice
(a) Magnetite (b) Bauxite
(c) Dolomite (d) Lime Pigment then the fraction of the work that 69. Plaintiff : Defendant
left is (a) Judge : Jury
51. The product of two 2-digit 5 8 7 1
(a) (b) (c) (d) (b) Court : Law
numbers is 2160 and their HCF is (c) Attorney : Lawyer
8 15 15 10
12. The numbers are (d) Injured : Accused
(a) (12, 60) (b) (72, 30) 60. The mean of 100 items was 46.
Later on, it was discovered that an 70. Buddhists : Pagoda
(c) (36, 60) (d) (44, 12)
item 16 was misread as 61 and (a) Parsis : Temple
52. If (6767 + 67) is divided by 68. another item 43 was misread as 34. (b) Christians : Cross
Then, the remainder is Then what is the correct mean? (c) Jains : Sun Temple
(a) 1 (b) 67 (c) 63 (d) 66 (a) 50 (b) 45.64 (c) 52 (d) 52.74 (d) Jews : Synagogue
112 AFCAT ~ Practice Set 13

Directions (Q. Nos. 71-75) In the Statement Directions (Q. Nos. 83 and 84) In
following questions, three alternatives “If you are first class graduate, each of the following question, find the
are same in a certain way out of four our organisation is the best place answer figure in which problem figure is
and so form a group. Find out the odd for you to work”. embedded.
word that does not belong to the group. Assumptions
I. No other organisation may 83. Problem Figure
71. (a) Cello (b) Guitar
require first class graduates as
(c) Flute (d) Violin
they may not get adequate
72. (a) Sweetness (b) Elegant number of applications.
(c) Bright (d) Beautiful II. First class graduates may get
attracted and apply to this Answer Figures
73. (a) Mile (b) Centimetre
organisation.
(c) Litre (d) Yard (a) Only Assumption I is implicit
74. (a) Eyes (b) Ears (b) Only Assumption II is implicit
(c) Throat (d) Skin (c) Neither Assumption I nor II is (a) (b) (c) (d)
implicit
75. (a) 14th November (b) 15th August
(d) Both Assumptions I and II are 84. Problem Figure
(c) 26th January (d) 2nd October implicit
76. Which one of the following figure Directions (Q. Nos. 80-82) Each
represents the relationship among question given below has a set of three or
Males, Fathers, Advocates ? four statements. Each set of statement
is further divided into three segments. Answer Figures
(a) (b) Choose the alternative, where the third
segment in the statement can be logically
deduced using both the preceding two,
(c) (d) but not just from one of them
80. A : X is an actor. Some actors are (a) (b) (c) (d)
77. Choose the correct alternative pretty. X is pretty.
that contains the missing letters B : Some men are cops. All cops are Directions (Q. Nos. 85-88) In each of
of the following series. brave. Some brave men are cops. the following question, choose the figure
a_cdaab_cc_daa_bbb_ccddd that can replace the question mark (?) to
C : All actors are brave. Some men
(a) bdbda (b) bddca are actors. Some men are brave. complete the problem figure.
(c) abdcb (d) bbdac 85. Problem Figure
D : All actors are pretty. X is not
78. A statement is given followed by an actor. X is not pretty.
two conclusions. You have to take (a) Only A (b) B and C
the given statement to be true even, (c) Only C (d) Only D
?
if they seem to be at variance
from commonly known facts.
81. A : All beautiful things are sad.
She is beautiful. She is sad.
Read the conclusions and then
decide which of the conclusions, B : All nice things are flat. TVs
if any, logically follows from the are flat. TVs are nice things. Answer Figures
two given statements, C : Potatoes are stems. All stems
disregarding the known facts. are fruits. Potatoes are fruits,
Statement The old order changed (a) Only A (b) A and B
yielding place to new. (c) Only C (d) A and C (a) (b) (c) (d)
Conclusions 82. A : All mammals are viviparous.
I. Change is the law of nature. Some fish are viviparous. 86. Problem Figure
II. Discard old ideas because they Some fish are mammals.
are old. B : All birds are oviparous. Some
(a) Only Conclusion I follows fish are not oviparous. Some
(b) Only Conclusion II follows fish are birds.
(c) Either Conclusion I or II follows C : No mammal is oviparous.
(d) Both Conclusions I and II follow
?
Some creatures are oviparous.
79. One statement is given followed Some creatures are not
by two assumptions numbered I mammals. Answer Figures
and II.You have to consider the D : Some creatures are mammals.
statement and the following Some creatures are viviparous.
assumptions and decide which of Some mammals are viviparous.
the assumptions is implicit in the (a) Only A (b) Only B
(a) (b) (c) (d)
statement. (c) Only C (d) Only D
AFCAT ~ Practice Set 13 113

87. Problem Figure Answer Figures Answer Figures

(a) (b) (c) (d) (a) (b) (c) (d)


? 91. Problem Figures 95. Problem Figures

Answer Figures
?

1 2 3 4 1 2 3 4 5
Answer Figures Answer Figures

(a) (b) (c) (d)

88. Problem Figure


(a) (b) (c) (d)
(a) (b) (c) (d)
96. Problem Figures
? Directions (Q.Nos. 92-96) In each
of the following questions, select
a figure from the answer figures
which will continue the series 1 2 3 4 5
as established by the five problem Answer Figures
Answer Figures figures.
92. Problem Figures

(a) (b) (c) (d)


(a) (b) (c) (d)
1 2 3 4 5 Directions (Q. Nos. 97-100) In each
Directions (Q. Nos. 89-91) In the of the following question, out of the four
Answer Figures given figures, three are similar in a
following questions, select that figure
from the set of answer figures, certain way, one figure is not like the
which would come in place of question others. Choose the different figure.
C S S S 97.
mark? 1 4
(a) (b) (c) (d) 4 3 3 2
89. Problem Figures 2 4 2 5 3 2
5 1 5 1 3 4
93. Problem Figures 5 1
(a) (b) (c) (d)
?
98.
1 2 3 4
Answer Figures 1 2 3 4 5
Answer Figures
(a) (b) (c) (d)
99.

(a) (b) (c) (d) (a) (b) (c) (d)


(a) (b) (c) (d)
90. Problem Figures 94. Problem Figures
100.
?
? I I I I I
+
1 2 3 4 1 2 3 4 5 (a) (b) (c) (d)
114 AFCAT ~ Practice Set 13

Answers
1 (d) 2 (a) 3 (c) 4 (a) 5 (a) 6 (b) 7 (a) 8 (b) 9 (d) 10 (c)
11 (a) 12 (b) 13 (c) 14 (c) 15 (c) 16 (a) 17 (a) 18 (b) 19 (c) 20 (b)
21 (a) 22 (a) 23 (b) 24 (c) 25 (c) 26 (d) 27 (b) 28 (b) 29 (b) 30 (a)
31 (b) 32 (c) 33 (a) 34 (b) 35 (b) 36 (c) 37 (c) 38 (b) 39 (c) 40 (d)
41 (d) 42 (a) 43 (a) 44 (d) 45 (c) 46 (a) 47 (b) 48 (b) 49 (c) 50 (b)
51 (c) 52 (d) 53 (d) 54 (d) 55 (b) 56 (d) 57 (b) 58 (b) 59 (a) 60 (b)
61 (c) 62 (d) 63 (a) 64 (c) 65 (a) 66 (c) 67 (a) 68 (d) 69 (d) 70 (d)
71 (c) 72 (a) 73 (c) 74 (c) 75 (a) 76 (a) 77 (d) 78 (a) 79 (b) 80 (b)
81 (d) 82 (c) 83 (d) 84 (a) 85 (d) 86 (d) 87 (a) 88 (b) 89 (d) 90 (a)
91 (d) 92 (d) 93 (c) 94 (d) 95 (d) 96 (a) 97 (d) 98 (b) 99 (d) 100 (d)

Hints and Solutions


1. (d) It can be inferred that role of men 16. (a) Among the given options, 27. (b) The Idiom ‘Play one’s ace’ means ‘to
and women has changed but not ‘Camarderie’ is incorrectly spelled. Its use one’s best weapon or ‘resource’. So,
completely. correct spelling is ‘Camaraderie’. option (b) best expresses the meaning of
2. (a) Mid-file-crises as discussed in the 17. (a) Among the give options, ‘Desicration’ given idiom.
passage are the result of male is incorrectly spelled. Its correct spelling 28. (b) Idiom ‘Aid and abet’ means ‘to help
dominance in the family. is ‘Desecration’. someone to do something illegal or
3. (c) The passage is written in 20th 18. (b) Among the given options, option (b) wrong’. So, option (b) ‘To help someone
century. Excruchisating is incorrectly spelled here. to commit a crime’ best expresses its
Its correct spelling is ‘Excruciating’. correct meaning.
4. (a) ‘Nothing else’ is followed by ‘but’, not
‘than’. 19. (c) The wrongly spelt word is ‘Beseige’. 29. (b) ‘All over hell‘s half acre’ is an idiom
Its correct spelling is ‘Besiege’. which means ‘all over the place or spread
5. (a) It should be ‘are grown up’ (delete
out across a great distance or area’. So,
‘all’) and ‘have gone out’ (add ‘have’) to 20. (b) The wrongly spelt word is ‘Carreer’.
option (b) i.e. ‘Everywhere’ best expresses
make the sentence grammatically correct. Its correct spelling is ‘career’.
its meaning.
6. (b) improving 21. (a) The word ‘Verbatim’ means ‘using the
30. (a) The Idiom ‘An arrow in the quiver’
7. (a) Significant exact words as were originally used’, So,
means ‘one of a number of resources’ or
out of the given options ‘Exactly’ best
8. (b) rate strategies that can be used to achieve a
expresses the meaning of the given word.
9. (d) In the context of the sentence, option goal’. So, option (a) i.e. ‘Strategies that can
22. (a) ‘Temerity’ means ‘excessive be followed’ best expresses its meaning.
(d) ‘means’ is a suitable choice.
confidence or boldness’. So, ‘Boldness’
‘Means’ is a method or system by which 31. (b) Bimbisara was the founder of
from the given options best expresses the
a result is achieved. Haryanka dynasty. Initially, the capital of
meaning of the given word.
10. (c) ‘Pressure’ is the most appropriate the dynasty was Rajagriha which was
23. (b) ‘Ruck’ means ‘a large number of later sifted to Pataliputra.
word to fill the blank in the given
quantity; mass, especially of ordinary or
sentence. The Haryanka dynasty was the second
undistinguished people or things’. So,
11. (a) ‘Restricted’ means ‘limited in extent, ruling dynasty of Magadha which was later
option (b) i.e. ‘Mass’ best expresses the
scope or action’. So, option (a) ‘restricted’ succeeded by the Shishunaga dynasty.
meaning of ‘Ruck’.
is a suitable choice to fill the given blank. 32. (c) Muhammed Bin Kasim (672 CE) was
24. (c) ‘Rueful’ is feeling sorry and ‘Defiant’
12. (b) ‘Amaxophobia’ is the fear of riding in the first Muslim general to invade the
means ‘not feeling sorry’. So, option (c)
a vechicle. So, option (b) is correct here. Indian subcontinent in the year 715 AD.
is the antonym of ‘rueful’.
Kasim defeated the king Dahir of Sindh in
13. (c) Inability or impaired ability to 25. (c) ‘Penury’ means ‘the state of being 716 AD.
understand or produce speech, as a extremely poor’. Out of the given options,
result of brain damage is called ‘Aphasia. 33. (a) UNFCCC stands for United Nations
‘Wealth’ is its obvious opposite which
So, option (c) is correct here. Framework Convention on Climate
means ‘having lot of money’.
Change. The UNFCCC is an
14. (c) ‘Byre’ is a place where cows are 26. (d) ‘Overweening’ means ‘being too intergovernmental treaty developed to
sheltered. So, option (c) is correct here. proud or confident in ‘oneself’. So, option address the problem of climate change.
15. (c) A ‘Meadow’ is a low level tract of (d) ‘Modest’ is its opposite, which means The UNFCCC entered into force on 21st
uncultivated land. So, option (c) is the ‘unassuming in estimation of one’s March, 1994 and has been ratified by
best substitute of the phrase. abilities or achievements.’ 197 countries.
AFCAT ~ Practice Set 13 115

34. (b) Electric power is the rate, per unit various categories including Government 49. (c) ‘Navy Day’ in India is observed on 4th
time, at which electrical energy is and Public Service, Literature, December every year to celebrate the
transferred by an electric circuit. Journalism, etc. This award globally magnificence, achievements and role of
The SI Unit of electric power is watt, one holds the rank equivalent to Nobel the Navel Force to the country.
jule per second. Awards in Asia. Indian Journalist Ravish 50. (b) Aluminium ore is called Bauxite. The
J Kumar is the winner of 2019 Ramon Bauxite is purified to produce aluminium
Watt (W) W = , W = V × A oxide. It is a white powder from which
S Magsaysay Award.
42. (a) The Bahraini Dinar is the currency of aluminium can be extracted. The extraction
35. (b) Sodium bicarbonate is commercially is done by electrolysis. The ions in the
known as baking soda. It is a chemical Bahrain. The currency code for Dinars is
aluminium oxide must be free to move so
compound with the formula NaHCO 3. It is BHD and currency symbol is BD. It is issued
that electricity can pass through it.
a salt composed of sodium cation (Na+) by the Central Bank of Bahrain (CBB).
51. (c) Let the 2-digit numbers be12x and12 y.
and biocarbonate axion (HCO −3). The Bahraini Dinar was introduced in
Sodium bicarbonate is a white solid that 1965. Then, 12x . 12 y = 2160
is crystalline, but appears as a fine 43. (a) The first country to print paper ⇒ 144 x . y = 2160 ⇒ x. y = 15
powder. currency was China. It started with the Two cases are possible
36. (c) Malaria is caused by protozoa of the Tang Dynasty (618 AD-907AD). Paper (i) 3, 5 (ii) 1,15
genus Plasmodium. Four species cause currency was a natural progression of (i) 3, 5
disease in humans : P falciparum, P things to come, because of the Chinese Two numbers are 12 × 3
vivax, P ovale and P malariae. Malaria is block-printing. The paper currency was and 12 × 5 = 36 and 60 respectively
spread to humans by the bite of female known as flying money. (ii) 1, 15
mosquitoes of the genus Anopheles. 44. (d) The first digital computer in India was Two numbers are 12 × 1
37. (c) Retired Supreme Court Judge Pinaki installed in Indian Statistical Institute, and 12 × 15 = 12 and 180, respectively
Chandra ghose is appointed as the first Kolkata in 1955. This computer was Here, 180 is a 3-digits number.
Lokpal (ombudsman) of India. The first named HEC-2M built by a British named Hence, Two number are 36 and 60.
Lokpal was proposed by Advocate Shanti AD Booth. The computer cost was ` 10 52. (d) 6767 + 67 = 6767 + 1 − 1 + 67
Bhusan in 1986. A Lokpal is an lakh and huge in size.
= (6767 + 167 ) + 66
anti-corruption authority of Ombudsman, 45. (c) The Godavari River is known as
who represents the public interest in the ‘Dakshin Ganga’ i.e. ‘River Ganga of We know that x n + an , is divisible by
Republic of India. It is commonly known x + a, if n is only odd numbers.
South India’ because it is the largest river
as The Lokpal and Lokayuktas Act, of the South India. It flows East for ∴6767 + 167 is divisible by (67 + 1 = 68).
2013. 1,465 km (910 mi) draining the states of ∴Required remainder = 66
38. (b) Sub-lieutenant Shivangi Swaroop Maharashtra, Madhya Pradesh, Odisha 53. (d) Given, length of platform = 122 m
became the first woman pilot of Indian and Andhra Pradesh. Ultimately it and length of bridge = 210 m
Navy. Shivangi was commissioned into empties into the Bay of Bengal through
Let the length of the train = xm
the Indian Navy in 2018 after her initial its network of tributaries.
According to the question,
training. 46. (a) British Governor General of India, x + 122 x + 210
Lord Ellenborough abolished Slavery in =
She will be flying the Dornier Surveillance 17 25
aircraft of the Indian Navy. India in year 1843. Indian Slavery Act,
[speed of train is equal in both case]
1843, also known as Act V of 1843, was
39. (c) Article 124 of the Constitution of ⇒ 25 x + 3050 = 17x + 3570
an act passed in British India, under East
India deals with the establishment of ⇒ 25 x − 17x = 3570 − 3050
Indian company rule which outlawed
Supreme Court in India. The Article
many economical transactions associated ⇒ 8 x = 520
states that ‘There shall be a Supreme 520
with slavery. ∴ x= = 65 m
Court of India, consisting of a Chief 8
Justice of India’ and seven other judges. 47. (b) The author of the book ‘Train to
∴Speed of the train
Pakistan’ is Khushwant Singh. It is a
The Supreme Court of India was 65 + 122
historical novel published in 1956. = [putting the value of x]
established on 28th January, 1950. 17
It recounts the Partition of India in
It succeeded the Federal Court and 187
August, 1947. Khushwant Singh, was = = 11 m/s
established under the Government of 17
one of the most famous and foremost
India Act, 1935. 11 × 18 1 m / s = 18 km / h
writers and the editor of ‘The Hindustan = km/h
40. (d) Comptroller and Auditor General of Times’, a daily English newspaper. 5  5 
India (CAG) holds office for 6 years or till
He served as the member of the Rajya = 39. 6 km/h
65 years of age, whichever is earlier. The
Sabha from 1980 to 1986. 54. (d) Difference of time
CAG is the head of Indian Audit and
48. (b) The most abundant gas in the Earth’s 1 11
Accounts Department. = 4 : 30 pm − 11 : 00 am = 5 h = h
atmosphere is Nitrogen which accounts 2 2
41. (d) Acharya Vinoba Bhave was the first 11
for about 78% of the mass of dry air, Distance covered in this period of time
Indian to receive the prestigious Ramon
followed by oxygen (20.95%). 2
Magsaysay Award. Bhave received this 5 3 20 − 9 11
In dry air, the third most abundant gas is = − = = part
award in 1958 for ‘Community
Argon (0.93%) a monoatomic noble gas. 6 8 24 24
Leadership’. This award is conferred in
116 AFCAT ~ Practice Set 13

Since,
11
part of the journey is covered = 4600 + 16 − 61 + 43 − 34 66. (c) Former is one of the subsets of the
24 = 4564 latter.
11
in h. ∴Correct mean of 100 items 67. (a) ‘Statute’ is a legislative body that
2 4564
= = 45. 64 passes the ‘Law’. Similarly, ‘Proviso’ has a
3
∴ part of the journey is covered in 100 ‘Clause’.
8
11 24 3 9 1 61. (c) Given, A, B and C individually can do 68. (d) Shape of ‘Earth’ is ‘Round’. Similarly
= × × = h= 4 h shape of ‘Dice’ is ‘Cube’.
2 11 8 2 2 a piece of work in 10 days, 12 days and
Clearly, the person started at 6 : 30 am. 15 days, respectively. 69. (d) ‘Injured’ is the ‘Plaintiff’ and similarly,
1 ‘Accused’ is the ‘Defendant’.
55. (b) Ratio of the values of the coins Hence, work done by A in one day =
3 4 3 10 70. (d) As, ‘Buddhists’ gather in pagoda for
= 2 : : = 2 : :1 1
2 4 2 Work done by B in one day = worship. Similarly, jews gather in
12 synagogue.
= 4 : 3 :2 1
Sum of the ratio = 4 + 3 + 2 = 9 Work done by C in one day = 71. (c) All except flute have string to play the
15
∴Value of 50 paise coins music but flute does not have.
3 Now, work done by A, B and C in 1 day
= × 180 = ` 60 72. (a) All except sweetness are related with
1 1 1 6 + 5 + 4 15 1
9 = + + = = = beauty but sweetness is related with taste.
Numbers of 50 paise coins =120 10 12 15 60 60 4
73. (c) Except ‘Litre’ all others are units to
56. (d) Let maximum marks of examination ∴a, b and c finish the work together measure distance.
= 4 days
= x Then, according to the question, 74. (c) Except ‘Throat’, all words having four
 8 

30
+ 12 = x ×
40
− 28 62. (d) 3 ÷ (8 − 5) ÷ (4 − 2) +  2 +   letters.
100 100    13  
75. (a) Except 14th November, all others are
40 x × 30  34
⇒x × − = 12 + 28 = 3 ÷ 3 ÷  2 +  national holidays.
100 100   13 
x 76. (a)
= 3 ÷ 3 ÷  = 3 ÷ 3 ×
⇒ = 40 60 13  Males
 13   60
10
⇒ x = 40 × 10 = 400 39 60 60 Fathers Advocates
⇒ x = 400 = 3÷ = 3× =
60 39 13
57. (b) We know, 1 kg = 1000 g 2
63. (a) Difference = P 
r 
Gain percent 
 100 All fathers must be males. Some males
True weight − False weight
= × 100 2 2
⇒ 40 = P 
10  1 may be advocates and vice-versa. Some
False weight  ⇒ 40 = P  
1000 − 900  100  10 fathers may be advocates and vice-versa.
= × 100 77. (d) Series pattern
900 ⇒ P = 40 × (10) 2 ⇒ P = 40 × 100
abcd/aabbccdd/aaabbbcccddd
=
100 1
= 11 % ∴ = ` 4000
9 9 Clearly, each letter of first sequence is
1 1
64. (c) Given, x = p + and y = p − repeated two times in the second
58. (b) Let the CP of the article for A = 100 p p sequence and three times in the third
CP for B = 110 x 4 − 2x 2 y2 + y4 = [(x 2 − y2 )]2 sequence.
Again, CP for A = 110 ×
90 [(x + y)(x − y)]2 ∴Required answer = bbdac
100 2 78. (a) Clearly, Conclusion I directly follows
 1 1  1 1
A = 11 × 9 = 99 =  p + + p −   p + − p +   from the given statement. Also, it is
Gain of A = 110 − 99 = ` 11  p p  p p
mentioned that old ideas are replaced by
11 [putting the values of x and y]
Gain percent = × 100 = 11% new ones, as thinking changes with the
100 2
 2 4 progressing time. So, Conclusion II does
= (2p)   = 4p2 × 2 = 4 × 4 = 16
59. (a) Time taken by A = 20 days  p not follow.
 p
and time taken by B = 40 days 79. (b) Assumption I is not implicit as nothing
65. (a) Let the rate of interest be R% can be assumed about other organisation
Then, (A + B )’s 5 days work
 2 + 1 = 15 = 3 per annum. from the given statement. Assumption II is
= 5 
1 1
+  = 5  400 × 2 × R 550 × 4 × R
 20 40  40  40 8 Then, +
implicit because it states the motive
3 5 100 100 behind the statement.
∴Remaining work = 1 − =
8 8 1200 × 6 × R 80. (b) From (A),
+ = 1020
Hence, required fraction is 5/8. 100
⇒ 8R + 22R + 72R = 1020 X
60. (b) Given, average of 100 items = 46
Pretty
Sum of 100 items = 46 × 100 = 4600 ⇒ 102 R = 1020 Actor
According to the question, 1020
∴ R= = 10%
Correct sum of 100 items 102 Here, we cannot say that X is pretty.
AFCAT ~ Practice Set 13 117

From (B), From (B), 90. (a) From problem figure (1) to (2) , the
whole figure gets inverted vertically. Now,
Men Cops Birds the upper part gets half-shaded while the
Fish lower one is reduced to half.
Brave Oviparous 91. (d) Both the upper and lower pair of figures
Clearly, some brave men are cops. interchange positions.
From (C), Here, we cannot say, that some fish are 1
92. (d) Circle moves and 1 steps alternatively
birds. 2
Actor Men From (C). in anticlock wise direction in each
subsequent block and a new figure is added
Brave Mammals Oviparous before it. Each time new figure is preceded
×
Clearly, some men are brave. by another new figure in the subsequent
From (D), Creatures figures.
Actors 93. (c) In each successive problem figure, there
X Clearly, some creatures are not mammals. is an addition of 3 and 2 dots, respectively
pretty From (D) and 2 and 3 lines, respectively.
94. (d) The series of letters present horizontally is
Here, it is wrong to say that X is pretty. moving one position downwards in each
Creatures Mammals
81. (d) From (A), subsequent figure along with addition of a
new element in the vertical series in place of
Viviparous an old element. Also, from problem figures
Beautiful (1) to (2), the two letters present on left side
Here, we cannot say, that some of the vertical line interchange their positions
She
mammals are viviparous. with each other. Same is the case for right
Sad side letters. From problem figures (2) to (3),
83. (d) The problem figure is embedded in
the letters present on left side interchange
Clearly, she is sad. answer figure (d).
their position with the letters present on the
From (b), right side. Now, the same process repeats.
Nice 95. (d) Triangle becomes reversed and one dot is
increasing one-by-one.
Flat TV 96. (a) The sequence of the problem figures ‘↔’ is
84. (a) The problem figure is embedded in
Here, we cannot say, that TVs are nice answer figure (a). S RS S RS S
things.
From (C),
S = Same design
RS = Reverse same design
Potatoes 85. (d) Answer figure (d) will complete the Problem figure (1) to (2)
problem figure. Problem figure (3) to (4)
Stems Problem figure (5) to answer figure.
86. (d) Answer figure (d) will complete the
Fruits problem figure. 97. (d) In all other figures, all the numbers from
1 to 5 are in a serial order.
Clearly, potatoes are fruits. 87. (a) Answer figure (a) will complete the
problem figure. 98. (b) In figure (b), the arrow of the design is
82. (c) From (A), missing which is present in all other figures.
88. (b) Answer figure (b) will complete the
problem figure. 99. (d) In all other figures, number of line segments
Mammals in upper and lower figures are the same.
89. (d) From problem figure (1) to (2) , the
Fish whole figure gets laterally inverted while 100.(d) In all other figures, the lines
Viviparous the L shaped figure rotates by 180°. Also, perpendicular to each other do not have any
one line is added to the other figure. sign in between them.
Here, we cannot say, that some fish
are mammals.
118 AFCAT ~ Practice Set 14

AFCAT
AIR FORCE COMMON ADMISSION TEST (ONLINE )

Practice Set ~14


Time : 2 Hrs MM : 300

INSTRUCTIONS
1. The set contains a total of 100 questions, Comprising Verbal Ability in English, General Awareness, Numerical Ability and Reasoning and
Military Aptitude Test.
2. Each correct question carry 3 Marks and there will be negative marking of 1 Mark for each incorrect attempt.
3. Total time duration will be 2 hrs (120 minutes).
4. No marks will be deducted for unattempted questions.

Directions (Q. Nos. 1-5) Read the provide constant feedback. Students 3. The writer is dissatisfied with the
passage carefully and choose the best enjoy practising at home what they are examiners because they test
answer to each questions out of the four taught in class. As they realise their students
alternatives. progress by constant reinforcement they (a) memory
This is the thorny side of the prevailing welcome and enjoy examination in (b) originality
examination system. Most examiners class. Under the watchful guidance of (c) aptitude
have perfected their skill in making it committed teachers, students grow and (d) creativity
a veritable nightmare for majority of blossom out as well-trained artistes. 4. The writer’s intention to compare
the students. Quite unwittingly we This is possible and feasible because the the topic of discussion with that
have increased the enrolment in schools teacher-pupil ratio is ideal and the of dance and music is to
alarmingly. Most of the students have attitude of the learner is based on (a) show how students of dance
neither the requisite aptitude to learn devotion and dedication. and music enjoy not
nor any clear cut goal in life. The 1. “Ultimately all these are gone with only learning, but also
destiny of students would be decided the wind.” The above sentence examinations
in the final examination of a written shows that the writer (b) popularise dance and music
nature to test bookish, rote memory. (a) enjoys the prevailing situation among all children
All laudable objectives of kindling (b) regrets our ignoring the aims of (c) congratulate teachers who take
originality and problem solving ability true education good care of their students
are trumpeted only in educational (c) is quite satisfactory about the (d) prove that dance and music
seminars and workshops. Ultimately syllabus alone can bring peace to us
all these are gone with the wind. No (d) makes fun of teachers and their
5. According to the passage,
wonder examination hangs like a students
the objectives of education
‘Damocles’ sword. 2. The passage emphasises the need for should be
Compare this with a related discipline (a) making dance and music (a) to teach dance, music and drama
such as music and dance. None would compulsory in schools to students in schools and
venture to seek entry into such centres (b) making examinations an colleges
of excellence unless one has proven enjoyable experience (b) to reduce teacher pupil ratio
aptitude to profit from training. Here (c) seeking easy questions in the (c) not to test bookish, rote
the students have excellent rapport examinations memory
with their teachers who evaluate their (d) warning examiners who harass (d) to encourage originality and
performance on a day-to-day basis and students in the examinations problem solving ability
AFCAT ~ Practice Set 14 119

Directions (Q. Nos. 6 and 7) Find out 15. The hunter killed two ……… for 25. A person who is interested in
which part of the sentence has an error. dinner. antiques
If there is no error in the sentence, mark (a) flour (b) fowl (c) foul (d) fool (a) Traditionalist (b) Antiquarian
option (d) ‘No error’ as your answer. (c) Acquarium (d) Epicure
Directions (Q. Nos. 16 and 17) Each
6. After his illness, (a)/the patient of the following questions consists of a 26. Which of these ancient forms
was (b)/sick with life. (c)/ No word in capital letters followed by four of writing has never been
error (d) answer choices. You have to mark one translated?
7. I told him (a)/that I availed of choice as your answer which is most (a) Egyptian hieroglyphs
(b)/the opportunity.(c)/ similar to meaning of the word in the (b) Indus valley script
No error (d) question. (c) Viking writing
16. CATAPULT (d) Ancient Indian language of
Directions (Q. Nos. 8-12) In the Sanskrit
following passage, some of the words (a) Throw in air (b) Swim in water
have been left. Try to understand (c) Drive in race (d) Flow of air 27. Who was the most famous ruler
what it is about, then fill in the of Tuluva dynasty?
17. APLOMB
blanks with the help of the alternatives (a) Sri Krishna Dev Raya
(a) Confidence (b) Enjoyment
given. (c) A superior (d) Supreme being (b) Hari Hara I
(c) Tirumala Raya
‘Quit India’ came not from the lips, Directions (Q. Nos. 18-20) Each of (d) Sadasiva Raya
but the aching hearts of millions. In the following questions consists of a
this open rebellion, the Indian …(8)… word in capital and bold letters 28. The founder of Arya Samaj was
reached its climax. The British were followed by four choices. You have to (a) Swami Dayanand Saraswati
not only …(9)… by it, but also were choose one choice which is most (b) Annie Besant
obliged to quit unilaterally. The dissimilar to meaning of the word in (c) Swami Vivekananda
importance of Quit India can be the question. (d) Debendranth Tagore
…(10)… from Lord Linlithgow’s 29. Which heritage site is famous for
18. REITERATE
statement, ‘I am engaged here in its Chalukya style of architecture?
(a) To say again and again
meeting by far the most …(11)… (a) Buland Darwaja
(b) No communication
rebellion since, that of 1857. The (c) Elegance (b) Rani Ki Vav
gravity and extent of which we have so (d) No feedback (c) Pattadakal
far …(12)… from the world for reasons (d) Konark Sun Temple
of military security’.
19. DISPARAGE
(a) Evolve (b) Expose 30. Ice as compare to water is
8. (a) freedom (b) patriotism (c) Appreciate (d) Converge (a) less dense (b) more dense
(c) liberation (d) revolution
20. HINDRANCE (c) alike (d) constant
9. (a) enfeebled (b) inspired (a) Aid (b) Persuasion 31. Who was the father of ‘Green
(c) attacked (d) impressed (c) Cooperation (d) Agreement Revolution’ in India?
10. (a) diffused (b) gauged Directions (Q. Nos. 21-25) In each of (a) C Rangarajan
(c) established (d) determined the following questions, choose the word (b) KV Kamath
11. (a) trivial (b) magnificent which can be substituted for the given (c) MS Swaminathan
phrases/sentences. (d) Rakesh Mohan
(c) serious (d) auspicious
12. (a) excluded (b) elicited 21. A fault that may be forgiven 32. Which of the following mammals
(a) Mercenary (b) Venial is the most endangered species on
(c) prevented (d) concealed
(c) Unforgivable (d) Inexcusable the planet?
Directions (Q. Nos. 13-15) In the (a) Hippopotamus
22. A person in his seventies
following questions, select the word (b) Asian elephant
(a) Nonagenarian (c) Javan rhinoceros
you consider most appropriate for
(b) Centenarian (d) African elephant
the blank space and indicate your
(c) Octogenarian
answer. 33. Which of the following is the
(d) Septuagenarian
13. The ……… of cotton is very fastest process of heat transfer?
23. A person guilty of maliciously
heavy. (a) Conduction (b) Convection
setting to cause fire of property
(a) bald (b) bail (c) Radiation (d) Insolation
(a) Plagiarist (b) Anarchist
(c) bale (d) band 34. Which type of radioactive decay
(c) Arsonist (d) Incendiarist
14. The principal has given his ……… doesn’t change the atomic number?
24. The act of violating the sanctity of
to the match. (a) Alpha
a church
(a) except (b) ascent (b) Beta
(a) Heresy (b) Desecration
(c) refuse (d) assent (c) Gamma
(c) Sacrilege (d) Blasphemy
(d) All forms of radioactive decay
120 AFCAT ~ Practice Set 14

35. Who is known as the ‘Father of 46. Which of the following banks was 54. If `12000 is divided into two parts
Genetics’? inaugurated by Mahatma Gandhi such that the simple interest on
(a) Augustinian Friar in 1919? the first part for 3 yr at 12% per
(b) Gregor Mendel (a) Bank of Maharashtra annum is equal to the simple
(c) Norman Borlaug (b) Bank of Baroda interest on the second part for
(d) MS Swaminathan (c) Union Bank of India 1
4 yr at 16% per annum, the
36. What is the currency of Tunisia? (d) State Bank of India 2
47. Which of the following report greater part is
(a) Tunisian rial
(b) Tunisian pound is not released by the World (a) ` 8000 (b) ` 6000
(c) Tunisian dinar Bank? (c) ` 7000 (d) ` 7500
(d) Tunisian shekel (a) World Development Report 55. If a3 + b3 = 341 and ab = 30, then
(b) Ease of Doing Business what is value of a + b ?
37. When is the World Development
Information Day observed? (c) Global Economic Prospects (a) 1 (b) 9 (c) 7 (d) 11
(a) 21st October (b) 24th October (d) Global Gender Gap Report 56. A man goes from A to B at a
(c) 12th May (d) 14th May 48. Which country has introduced uniform speed of 12 km/h and
mandatory face scans for mobile returns with a uniform speed of
38. Who discovered the Polio
users? 4 km/h. His average speed
Vaccine?
(a) China (b) India (in km/h) for whole journey is
(a) Jonas Salk
(c) USA (d) Spain (a) 8 (b) 7.5 (c) 6 (d) 4.5
(b) Albert Bruce Sabin
(c) Alexander Fleming 49. Which Indian State produces the 57. The ratio of the ages of two
(d) Both (a) and (b) largest quantity of pulses? persons is 4 : 7 and age of one of
(a) Maharashtra them is greater than that of the
39. Which among the following is the other by 30 yr. The sum of their
capital of Mangolia? (b) Uttar Pradesh
(c) Madhya Pradesh ages (in years) is
(a) Kabul (b) Zhengzhou (a) 110 (b) 100 (c) 70 (d) 40
(d) Rajasthan
(c) Ulaanbaatar (d) Warsaw
50. There is a 4% increase in volume 2 3 1 7
40. CK Nayudu Trophy belongs to 58. 9 − 1 of 3 ÷ 5 of is equal to
when a liquid freezes to its solid 9 11 7 9
which game?
state. The percentage decrease 32 3
(a) Chess (b) Billiards (a) 8 (b) 9 (c) 8 (d)
when solid melts to liquid again, is 81 4
(c) Table-Tennis (d) Cricket 3
(a) 3 % (b) 4% 59. The average temperature of
41. Jaspal Singh Rana is a 13 Monday, Tuesday, Wednesday
distinguished athlete of which 1
(c) 4 %
11
(d) 3 % and Thursday is 60°. The average
game? 13 13 for Tuesday, Wednesday,
(a) Shooting (b) Wrestling 51. A bag contains ` 90 in coin of Thursday and Friday is 63°. If the
(c) Football (d) Basketball denominations of 50 paise, ratio of temperature for Monday
42. Union Public Service Commission 25 paise and 10 paise. If coins of and Friday is 21 : 25, then what is
was founded on 50 paise, 25 paise and 10 paise the temperature of Friday?
(a) 1st August, 1925 are in the ratio 2 : 3 : 5, then the (a) 70° (b) 73° (c) 75° (d) 78°
(b) 1st October, 1926 number of 25 paise coins in the 60. With a 5% discount on the cost of
(c) 1st November, 1927 bag is sugar, a buyer could purchase 2 kg
(d) 1st June, 1928 (a) 80 (b) 120 more sugar for ` 608. Selling price
(c) 100 (d) 135 of sugar is
43. Which Article of the Indian
Constitution provides for an 52. A is twice as good a workman as (a) ` 15.50 (b) ` 15
B and B is twice as good a (c) ` 16.50 (d) ` 16
Election Commission in India?
(a) Article 324 (b) Article 128
workman as C. If A and B can 61. To travel 720 km, a express train
(c) Article 256 (d) Article 378
together finish a piece of work in takes 6 h more than Duronto. If
4 days, then C can do it by however, the speed of the express
44. Indian Military Training Team himself in train is doubled, it takes 2 h less
(IMTRAT)imparts training to (a) 6 days (b) 8 days than Duronto. The speed of
army officers of (c) 24 days (d) 12 days Duronto is
(a) Nepal (b) Bhutan
53. A coconut merchant finds that (a) 60 km/h (b) 72 km/h
(c) Bangladesh (d) Maldives
the cost price of 2750 coconuts is (c) 66 km/h (d) 78 km/h
45. At which of the following places is the same as the selling price of 62. The sum of two numbers is 528
the College of Defence 2500 coconuts. His loss or gain and their HCF is 33. How many
Management located? will be pairs of such numbers can be?
(a) Dehradun (b) Pune (a) 5% loss (b) 10% gain (a) 1 (b) 2
(c) Secunderabad (d) Chennai (c) 15% loss (d) 20% gain (c) 3 (d) 4
AFCAT ~ Practice Set 14 121

63. 3 yr ago, the average age of a family 72. XWA, VTC, SPF, OKJ, ? 84.
of 5 members was 17 yr. After (a) JDN (b) JEO (c) LPN (d) JDP
born a baby, the average age of
73. Which diagram correctly represents
the family is not change. The (a) (b) (c) (d)
the relationship between politicians,
present age of the baby is
poets and women?
1 85.
(a) 1 yr (b)1 yr (c) 2 yr (d) 3 yr
2
64. Find the unit’s digit in the product
(4387)245 × (621)72 (a) (b) (c) (d)
(a) (b) (c) (d)
(a) 1 (b) 2 (c) 5 (d) 7
86.
65. Ram earns ` 125 in 8 days and Directions (Q. Nos. 74-78) In each of
Shyam earns ` 140 in 10 days, the the following question, three out of four
ratio of their earning is alternatives are same in a certain way,
(a) 125 : 110 (b) 112 : 125 and so form group. Find that word that (a) (b) (c) (d)
(c) 125 : 112 (d) 100 : 112 does not belong to the group.
66. [(7−1 − 8−1)−1 − (3 −1 − 4 −1)−1] is 74. (a) Bhutan (b) Bangladesh Directions (Q.Nos. 87-91) In each
equal to (c) China (d) Pakistan of the following question, choose the
(a) 56 (b) 44 (c) 50 (d) 48 correct answer figure, which will
75. (a) John F Kennedy
complete the series given in problem
67. In a particular way of coding, the (b) Abraham Lincoln figures.
word ‘CENTRAL’ is coded as (c) George Washington
ABCDEFG and PLANETARIUM (d) Gerald Ford 87. Problem Figures
as HGFCBDFEIJK. With the 76. (a) Gun (b) Pistol
same coding how can we express
(c) Dagger (d) Atom bomb
the word ‘LANTERN’?
(a) GFCDFEG (b) GFCDBEC 77. (a) Indira Gandhi 1 2 3 4
(c) GFCDEFG (d) GFCDBEB (b) Lal Bahadur Shastri
Answer Figures
68. Two statements are given (c) Jawahar Lal Nehru
followed by two conclusions. Read (d) Dr Rajendra Prasad
the conclusions and then decide 78. (a) Head (b) Heed
which of the conclusions, if any, (c) Sledge (d) Led
logically follows from the two (a) (b) (c) (d)
given statements, disregarding the Directions (Q.Nos. 79-82) In the
known facts. following questions, choose the missing 88. Problem Figures
Statements term that will complete the second pair
Some tanks are ponds. in the same way as first pair.
Some ponds are buckets. 79. Cougar : South America : : Okapi : ?
Conclusions (a) India (b) Central Africa (1) (2) (3) (4)
I. Some buckets are ponds. (c) North America (d) Pakistan
II. Some tanks are buckets. Answer Figures
80. ABCDE : FGHIJ : : PQRST : ?
(a) Only Conclusion I follows
(b) Only Conclusion II follows (a) TSRQP (b) UVWXY
(c) Neither Conclusion I nor II follows (c) KLMNO (d) EDCBA
(d) Both Conclusions I and II follow 81. XMAE : 16 : : VTNG : ? (a) (b) (c) (d)
69. If the 1st of November falls on (a) 21 (b) 17 (c) 35 (d) 18
Monday, what day will the 25th 82. 85 : 40 : : 77 : ? 89. Problem Figures
of November be? (a) 14 (b) 48 (c) 49 (d) 50
(a) Tuesday (b) Thursday
(c) Wednesday (d) Friday Directions (Q. Nos. 83-86) Out of the
four given figures, three are similar in a
Directions (Q. Nos. 70-72) What (1) (2) (3) (4)
comes in place of question mark (?) in certain way, one figure is not like the
the series given below? others. Choose the different figure. Answer Figures
70. NZ, OY, PX, QW, RV, ? 83.
(a) FS (b) SU (c) UF (d) TU
71. AMV, BNW, COX, ?, EQZ (a) (b) (c) (d) (a) (b) (c) (d)
(a) DMO (b) DPY (c) DYP (d) DLZ
122 AFCAT ~ Practice Set 14

90. Problem Figures Answer Figures 97. Problem Figure

(A)
(1) (2) (3) (4) (a) (b) (c) (d)
Answer Figures
Answer Figures 94. Problem Figure

?
(a) (b) (c) (d)
(a) (b) (c) (d)
98. Problem Figure
91. Problem Figures Answer Figures

(A)
(1) (2) (3) (4) (a) (b) (c) (d) Answer Figures
Answer Figures 95. Problem Figure

(a) (b) (c) (d)


(a) (b) (c) (d) ?
99. Problem Figure
Directions (Q. Nos. 92-95) In each of Answer Figures
the following question, choose the
figure that can replace the ‘?’ to
complete the problem figure. (A)
92. Problem Figure (a) (b) (c) (d) Answer Figures
Directions (Q.Nos. 96-100) In each
of the following question, choose the
correct option which contains region(s)
? common to circle, square, triangle (a) (b) (c) (d)
similar to that marked by the dots in
Answer Figures figure (A). 100. Problem Figure
96. Problem Figure

(a) (b) (c) (d)


(A)
(A)
93. Problem Figure Answer Figures
Answer Figures

(a) (b) (c) (d)


?
(a) (b) (c) (d)

Answers
1 (b) 2 (b) 3 (a) 4 (a) 5 (d) 6 (c) 7 (b) 8 (d) 9 (a) 10 (b)
11 (b) 12 (d) 13 (c) 14 (d) 15 (b) 16 (a) 17 (a) 18 (d) 19 (c) 20 (a)
21 (b) 22 (d) 23 (d) 24 (c) 25 (b) 26 (b) 27 (a) 28 (a) 29 (c) 30 (a)
31 (c) 32 (c) 33 (c) 34 (c) 35 (b) 36 (c) 37 (b) 38 (d) 39 (c) 40 (d)
41 (a) 42 (b) 43 (a) 44 (b) 45 (c) 46 (c) 47 (d) 48 (a) 49 (c) 50 (d)
51 (b) 52 (c) 53 (b) 54 (a) 55 (d) 56 (c) 57 (a) 58 (a) 59 (c) 60 (d)
61 (b) 62 (d) 63 (c) 64 (d) 65 (c) 66 (b) 67 (b) 68 (a) 69 (b) 70 (b)
71 (b) 72 (b) 73 (d) 74 (c) 75 (d) 76 (c) 77 (d) 78 (d) 79 (b) 80 (b)
81 (d) 82 (c) 83 (b) 84 (c) 85 (d) 86 (a) 87 (d) 88 (b) 89 (c) 90 (d)
91 (d) 92 (a) 93 (b) 94 (c) 95 (b) 96 (c) 97 (a) 98 (c) 99 (a) 100 (d)
AFCAT ~ Practice Set 14 123

Hints and Solutions


1. (b) The given sentence shows that the 20. (a) ‘Hindrance’ means ‘an obstacle or 30. (a) Ice is less dense as compare to water.
writer regrets our ignoring the aims of something that makes it difficult to It is about 9% less dense than liquid water.
true education. achieve something’ and ‘Aid’ means ‘help Ice takes up about 9% more space than
2. (b) The passage emphasises the need for or support in the achievement of water, so a litre of ice weighs less than a
making examinations an enjoyable something’. litre of water.
experience. So, ‘Aid’ is the correct antonym of In other words, density of ice is 0.9 g/cm
3
‘Hindrance’. , whereas the density of water is 1 g/cm
3. (a) The writer is dissatisfied with the 3
. So, the relative density of ice as
examiners because they test students’ 21. (b) ‘Venial’ is a fault that may be
compared to water will be 0.9.
memory. forgiven.
31. (c) MS Swaminathan is an Indian
4. (a) The writer’s intention to compare the 22. (d) A person in his seventies is called
geneticist and administrator.
topic of discussion with that of dance ‘Septuagenarian’.
Swaminathan is known as the ‘Father of
and music is to show how students of 23. (d) ‘Incendiarist’ is one who Green Revolution’. He is better known for
dance and music enjoy not only learning deliberately and maliciously sets fire his role in India’s ‘Green Revolution’, a
but also examinations. to property. programme under which
5. (d) According to the passage, the 24. (c) The act of violating the sanctity of a High-Yield-Varieties (HYVs) of wheat and
objectives of education should be to church is called ‘Sacrilege’. rice were planted. Later, he promoted
encourage originality and problem solving sustainable development, which he
25. (b) ‘Antiquarian’ is a person who is
ability. called, the ‘Evergreen Revolution’.
interested in antiques.
6. (c) Use ‘of’ in place of ‘with’ as ‘sick of’ 32. (c) Javan Rhinoceros is the most
26. (b) The Indus valley script writing
means ‘bored with something or annoyed endangered mammal species on Earth.
system could not be translated because
with something’. Javan Rhinoceros belongs to the same
the texts are too short. The Indus script
7. (b) Use ‘had availed myself’ in place of (also known as the Harappan script) is genus as the Indian Rhinoceros. The
‘availed’ as after ‘availed’ reflexive a corpus of symbols produced by the species is critically endangered with the
pronoun (myself) is used. Indus valley civilisation. This script total population of only 60 in Ujung
8. (d) revolution shows no significant changes over time. Kulon National Park in Java, Indonesia.
9. (a) enfeebled One of the major causes is the absence 33. (c) Radiation in the fastest mode of
of information on their context and transfer of heat because radiation travels
10. (b) gauged at the speed of light. The slowest mode
usage.
11. (b) magnificent of transfer of heat is conduction because
27. (a) Sri Krishna Deva Raya was the most
12. (d) concealed it takes place from particle to particle.
famous king of Tuluva dynasty of
13. (c) For a collection of cotton, we use Vijayanagara Empire, who reigned from 34. (c) Gamma rays are usually emitted from
term ‘bale’. So, here, ‘bale of cotton’ 1509-1529. He is the third ruler of the the nuclei along with alpha or beta
should be used. Tuluva dynasty. He is regarded as an particles during radioactive decay. They
icon by many historians. have no mass and no electrical charge
14. (d) ‘Assent’ is used when allowance is to which means no change in the atomic
be shown because it means approval. Tuluva dynasty was at its zenith during number or mass number when gamma
So, option (d) is a suitable choice. the rule of Krishna Devaraya, the second rays are emitted.
15. (b) Use of ‘fowl’ is proper here because it son of Tuluva Narasa Nayaka. He was
35. (b) Gregor Mendel known as ‘Father of
means ‘chicken kept for poultry or meat undefeated during his rule. He built the
Genetics’ was. Born in 1822 in Austria,
purpose’. famous Vithalswami and Hazara temple
Mendel discovered the basic principles of
in Hampi.
16. (a) ‘Catapult’ is a device that can throw heredity through experiments in his
28. (a) Arya Samaj was founded by monastery’s garden.
objects at a high speed in air. So, ‘throw
in air’ is the most similar meaning of Swami Dayananda Saraswati in 1875. 36. (c) The Tunisian Dinar is the official
‘catapult’. Arya Samaj is a vigorous reform movement currency in Tunisia, subdivided into
of modern Hinduism in India. Arya Samaj 1,000 milim or millimes. The dinar was
17. (a) ‘Aplomb’ means ‘self-confidence or promotes the moral values and practices set out as the new currency in Tunisia in
assurance, especially when in a based on the belief in the infallible 1958. Before 1958, the official currency
demanding situation’. So, option (a)
authority of the Vedas. of Tunisia was Franc. The Tunisian Dinar
‘confidence’ is a suitable choice.
29. (c) Pattadakal is a UNESCO World is commonly abbreviated as DT.
18. (d) ‘Reiterate’ means ‘to say something 37. (b) World Development Information as
Heritage Site located in Karnataka,
again and again typically for emphasis or Day is observed annually on 24th
famous for its Chalukya style of
clarity’. So, option (d) ‘No feedback’ is October. The day focuses on improving
architecture. It was the capital of
opposite in meaning to ‘Reiterate’. the dissemination of information and
Chalukya dynasty between 6th and 8th
19. (c) ‘Disparage’ means ‘not to give so century AD. There are 10 major temples mobilising public opinion, among young
much value as one deserves’. Its opposite people. The primary goal for this day is to
in Pattadakal, all dedicated to Lord
is to ‘praise or appreciate’. So, option (c) create awareness for problems of
Shiva.
is a suitable choice. development.
124 AFCAT ~ Practice Set 14

38. (d) The polio vaccine was discovered by imparting management training to 52. (c) According to the question, A takes
two American physicians Jonas Salk and officers of defence services. Located in x days to complete the work, B will take
Albert Bruce Sabin in 1955. Polio Secunderabad the institute is 2x days and C will take 4x days.
vaccines are used to prevent considered as the only exclusive college 1
Now, (A + B)′ s 1 day’s work =
poliomyelitie (polio). Poliomyelitis is an in Asia for Defence Management 4
acute paralytic disease caused by three training to the Armed Forces. It is run According to the question,
poliovirus (PV) serotypes. under the management of the Indian 1 1 1
Armed Forces. + =
39. (c) Ulaanbaatar, is the capital and x 2x 4
largest city of Mongolia. It is situated on 46. (c) Union Bank of India (UBI) was 2 +1 1
⇒ =
the Tuul River on a windswept plateau at registered on 11th November, 1919 as a 2x 4
an elevation of 4,430 feet (1,350 m). limited company in Mumbai. UBI is one ⇒ 2x = 12
Ulaanbaatar is the Mongolia’s cultural, of the largest state owned banks in the ⇒ x =6
industrial and financial heart. This city is country and inaugurated by Mahatma ∴ C will complete the work in 24 days.
also connected by road network and by Gandhi in 1919. The Government of
53. (b) Given, x = 2750 and y = 2500
rail to both the Trans-Siberian Railway India owns 90% of its share capital. The
x − y 
in Russia and the Chinese Railway bank was nationalised in 1969. Gain % =  × 100 %
system. 47. (d) Global Gender Gap Report is not  y 
(2750 − 2500)
40. (d) CK Nayudu Trophy belongs to released by the World Bank. Global = × 100
cricket. It is dedicated to the first captain Gender Gap Report benchmarks 149 2500
250
of the Indian cricket team in test countries on their progress towards = × 100
matches. In his remembrance the CK gender parity across four thematic 2500
Nayudu Trophy cricket tournament is dimensions. = 10%
organised. This report was first published in 2006 54. (a) Let the larger part of the sum = ` x
41. (a) Jaspal Rana is an Indian shooter. He by the World Economic Forum. It was ∴Smaller part = ` (12000 − x)
contested mainly in the 25 m Centre Fire designed to measure gender equality. According to the question,
Pistol category. He was a gold medallist x × 3 × 12 (12000 − x) × 9 × 16
48. (a) China has introduced mandatory face =
at the 1994 Asian Games, 1998 scans for mobile users. People in China 100 2 × 100
Commonwealth Games, 2002 are now required to have their faces ⇒ 36x = (12000 − x) × 72
Commonwealth Games and 2006 Asian scanned when registering new mobile ⇒ x = (12000 − x) × 2
Games in Men’s Center Fire Pistol Pairs. phone services. ⇒ x + 2x = 24000
42. (b) Union Public Service Commission The new regulation for telecom operators ⇒ 3x = 24000
(UPSC) was founded on 1st October, was framed to ensure that the 24000
1926. The UPSC is India’s premiere ⇒ x= = ` 8000
government can identify all mobile 3
central recruiting agency. It is phone users.
responsible for appointments to and 55. (d) Given, a3 + b3 = 341
49. (c) Madhya Pradesh is the largest
examinations for All India Services and and ab = 30
producing state of pulse in india, which 30
Group A & B of Central Services. ⇒ a= …(i)
accounts for 23% of total pulse
Article 315 to 323 of part XIV of b
production in the country. Madhya 3
Constitution, titled as services, provide ∴  30 + b3 = 341
Pradesh is followed by UP (18%),  
for a Public Service Commission for Mahrashtra (14%), Rajasthan (11%)  b
Union and for each state. and Andhra Pradesh (9%). 27000 + b6
⇒ = 341
43. (a) Article 324 envisages the provision of 50. (d) Let the volume in liquid state be b3
the Election Commission of India. Article 100 unit 3. ⇒ 27000 + b6 − 341b3 = 0
324 empowers the Election commission 104
∴Volume in solid state = × 100 Let b3 = x
of India to issue rules and guidelines for 100
conducting elections of Parliament, ∴ x 2 − 341x + 27000 = 0
= 104 unit 3
President, Vice-President and ⇒ x 2 − 216x − 125 x + 27000 = 0
State Legislature. Assemblies in the Required percentage decrease
104 − 100 11 ⇒ (x − 216)(x − 125) = 0
country. = × 100 = 3 % ∴ x = 216, 125
44. (b) Indian Military Training Team 104 13
Now, b3 = x
(IMTRAT) imparts training to army 51. (b) Ratio of values of 50 paise, 25 paise
officers of Bhutan. It is responsible for and 10 paise coins b= 3
x
2 3 5 3 1 b = 216, 3 125
3
the training of Royal Bhutan Army (RBA) = : : = 1: : = 4: 3:2
and Royal Body Guards (RBG). All 2 4 10 4 2 b = 3 216, 3 125 = 6, 5
RBA and RBG officers are trained at Sum of the ratios = 4 + 3 + 2 = 9 30
the Indian Army’s NDA and IMA, 3 Then, a = =5
Value of 25 paise coins = × 90 = ` 30 6
Dehradun. 9 30
Number of 25 paise coins or =6
45. (c) The College of Defence Management 5
is a premier tri-service institution = 30 × 4 = 120
In both cases, a + b = 11
AFCAT ~ Practice Set 14 125

56. (c) Here, x = 12 km/h, y = 4 km/h 61. (b) Let speed of express train = x km/h and
∴Average speed =
2xy and speed of Duronto train = y km/h P L A N E T A R I U M
x+ y Now, according to the question,
2 × 12 × 4 720 720 H G F C B D F E I J K
= = 6 km/h − =6 …(i)
12 + 4 x y
Therefore,
720 720
57. (a) Let the ages of two persons be 4x and and − = 2 …(ii) L A N T E R N
7x, respectively. y 2x
According to the question, On solving Eqs. (i) and (ii), we get
G F C D B E C
7x − 4x = 30 ⇒ 3x = 30 x = 45 and y = 72 km/h
∴ x = 10 ∴Speed of Duronto train = 72 km/h 68. (a)
Now, age of first person = 4 × 10 = 40 yr Tanks Ponds Buckets
62. (d) Here, the HCF = 33
and age of second person Then, let the two numbers be 33a and
= 7 × 10 = 70 yr Conclusions I. (✓) II. (✕)
33b, respectively, where a and b are
Thus, sum of their ages coprime. 69. (b) Since, Monday falls on 1st, 8th, 15th
= 40 + 70 = 110 yr Then, 33a + 33b = 528 ⇒ a + b = 16 and 22nd of November
2 3 1 7 Hence, values of (a, b) which are coprime ∴ 23 rd = Tuesday
58. (a) 9 − 1 of 3 ÷ 5 of
9 11 7 9 to each other can be ∴ 24th = Wednesday
11 36 36 7 (1, 15), (3, 13), (5, 11) and (7, 9). and 25th = Thursday
= 9− of ÷ of
9 11 7 9 Hence, required number of pairs is 4. 70. (b) Here, the pattern is as follows
= 9 −  ×
11 36   36 7 
÷
  ×  63. (c) Present age of family of 5 members –1 –1 –1 –1 –1
 9 11   7 9 = 17 × 5 + 5 × 3 = 85 + 15 = 100 yr
[using VBODMAS rule] Total age of family of 6 members with NZ OY PX QW RV SU
36 9 baby = 6 × 17 = 102 yr
= 9− × = 9−1= 8
+1 +1 +1 +1 +1
9 36 ∴Present age of baby = 102 − 100 = 2 yr
59. (c) Average temperature of Monday, 64. (d) Given, (4387) 245
× (621) 72 71. (b) The pattern is as follows
Tuesday, Wednesday and Thursday +1 +1
To find the unit’s digit, we have to divide +1 +1
= 60° the index of given numbers by 4.
+1 +1
∴Sum of temperature of these days In, 245 ÷ 4, remainder = 1 and in 72 ÷ 4, AMV BNW COX DPY EQZ
= 60 × 4 = 240° …(i) remainder = 0. +1 +1
Similarly, sum of temperature of [∵ when remainder comes 0, +1 +1
Tuesday, Wednesday, Thursday and the index would be 4] +1 +1
Friday = 63 × 4 = 252° …(ii) Hence, (4387)1 × (621)4 ∴= DPY
From Eqs. (i) and (ii), we get
= 71 × 14 = 7 × 1 = 7 72. (b) The pattern is as follows
difference in temperature of Friday
and Monday = 252 − 240 = 12° 65. (c) Since, earnings of Ram and Shyam 3rd Letter
Let temperature of Monday be 21x 125 140 24 22 19 15 10
be ` and ` per day. –2 –3 –4 –5
8 10 X V S O J
and temperature of Friday be 25x.
125 140 23 20 16 11 5
[∵ the ratio of temperature for Then, required ratio = : W
–3
T
–4
P
–5
K
–6
E
Monday and Friday is 21 : 25] 8 10
1 3 6 10 15
Now, 25x − 21x = 12° ⇒ 4x = 12 = 125 × 10 : 140 × 8 A
+2
C
+3
F
+4
J
+5
O
∴ x = 3° = 125 : 112
∴ ? = JEO
Hence, temperature of Friday 66. (b) [(7−1 − 8−1)−1 − (3−1 − 4−1)−1]
73. (d)
= 25 × 3 = 75°
Politicians

  1 1  −1  1 1  −1 
60. (d) Let the original SP of sugar =  −  −  −   Poets Women
  7 8  3 4 

= ` x per kg
95x  8 − 7  − 1
 4 − 3 
−1
SP after discount = ` =   −   Some politicians may be poets and
100   56   12  
 vice-versa.
19x
=` per kg   1  −1  1  −1  Some politicians may be women and
20 =   −   
  56   12   vice-versa.

608 608
− =2 
19x No poet can be women as women poet
x = 56 − 12 = 44 is called poetess.
20 67. (b) As, 74. (c) All except China are democratic
608  20 1 
−  =2 ⇒
608
=2 C E N T R A L
 19x x  countries.
19x
608 75. (d) Among these four persons,
⇒ x= = ` 16 A B C D E F G Gerald Ford has never been the President
19 × 2
of America.
126 AFCAT ~ Practice Set 14

76. (c) Among all these weapons, ‘Dagger’ is 82. (c) As, 85 40 92. (a) Answer figure (a) will complete the
an ancient sword, whereas all the other problem figure.
weapons are modern. 8×5
93. (b) Answer figure (b) will complete the
77. (d) ‘Dr Rajendra Prasad’ was the President Similarly, 77 49 problem figure.
of India, whereas others were the Prime 94. (c) Answer figure (c) will complete the
7×7
Ministers. problem figure.
78. (d) All except led have two vowels. 83. (b) In all the other figures, one
95. (b) Answer figure (b) will complete the
shaded semicircle attached to the line
79. (b) As, ‘Cougar’ (a type of lion) is found problem figure.
and the small line segment attached
in ‘South America’, similarly, Okapi is Common to the
to the other line, face each other. triangle and square
found in Central Africa.
84. (c) In all the other figures, both arrow
80. (b) As, A BCDE : F GH I J Common to the
+5
and pin are perpendicular to each triangle and circle
+5 other.
+5
+5 85. (d) Only in figure (d), the arrow is Only square
+5
originating from one of the sides of 97. (a) Only circle
Similarly, P Q R S T : U V WX Y the triangle and not from any of the
+5
+5
vertices. And it is only in (d), that all Common to the triangle,
+5 vertices all away from the point of circle and square
+5
+5 intersection of the two chords. Common to the circle
86. (a) In option (a), dot and crrow are and square
81. (d) As, X M A E
opposite to each other. 98. (c) Common to the
87. (d) In each step, one of the circles triangle and circle

24 13 1 5 turns black and moves to a corner of Common to the triangle,


the square boundary. circle and rectangle
Common to the square,
88. (b) The shading moves one, two,
circle and triangle
(2+4) (1+3) (0+1) (0+5) three, four, …… spaces
anti-clockwise sequentially. 99. (a) Common to the triangle
and square
89. (c) The number of sides of the figure
Common to the triangle
6 + 4 + 1 + 5 =16 reduces by one in each step. and circle
V T N G 90. (d) The circle moves sequentially Common to the triangle
Similarly,
and rectangle
one, two, three, four, …spaces (each
space is equal to half-a-side of the 100. (d) Common to the circle
22 20 14 7 square boundary) in anti-clockwise and rectangle
direction. Common to the circle,
square and rectangle
91. (d) Similar figure repeats in every
Common to the circle,
(2+2) (2+0) (1+4) (0+7) second step and each time a square and triangle
particular figure reappears, it gets
rotated through 180°.
4 + 2 + 5 + 7=18
∴ ? = 18
AFCAT ~ Practice Set 15 127

AFCAT
AIR FORCE COMMON ADMISSION TEST (ONLINE )

Practice Set ~15


Time : 2 Hrs MM : 300

INSTRUCTIONS
1. The set contains a total of 100 questions, Comprising Verbal Ability in English, General Awareness, Numerical Ability and Reasoning and
Military Aptitude Test.
2. Each correct question carry 3 Marks and there will be negative marking of 1 Mark for each incorrect attempt.
3. Total time duration will be 2 hrs (120 minutes).
4. No marks will be deducted for unattempted questions.

Directions (Q. Nos. 1-3) In the Directions (Q. Nos. 7-10) Out of the 12. The ……… of the ‘Chief’ was
following questions, select the word that four alternatives, choose the one which ……… upon him.
is most similar in meaning to the given can be substituted for the given (a) name; dismissed
word. words/sentences. (b) title; imposed
1. EXONERATE 7. Study of the interaction of people (c) title; bestowed
(a) To free (b) To imprison with their environment. (d) appointment; thrown
(c) To harass (d) To push (a) Ecology (b) Psychology 13. The little boy ran ……… fast that
2. EMACIATED (c) Philosophy (d) Geography he was ……… for breath.
(a) Strong (b) Weak 8. A list of explanation of rare, (a) so; gasping (b) very; struggling
(c) Obese (d) Thin technical or obsolete words. (c) so; inhailing (d) too; fighting
(a) Dictionary (b) Glossary Directions (Q. Nos. 14-16) Read the
3. APROPOS
(c) Lexicon (d) Catalogue following passage carefully and choose
(a) To the point
(b) Out of context 9. Underground place for storing the correct answer to each question out
(c) On papers wine or other provisions of the four alternatives and fill in the
(d) An argument (a) Garage (b) Cellar blanks.
(c) Attic (d) Hall A team of Indian engineers …(14)…
Directions (Q. Nos. 4-6) In the
following questions, select the word that 10. Free somebody from blame or guilt. shown that digital photographs of the
is opposite in meaning to the given (a) Excuse (b) Reprimand hand may …(15)… cholestrol levels
word. (c) Exonerate (d) Plunder in the body and serve as an early
Directions (Q. Nos. 11-13) In the warning signal …(16)… poor
4. CALLOUS lipid health without a standard
(a) Careful (b) Wrinkled following questions, sentences are given
along with the blanks to be filled in blood test.
(c) Bloated (d) Emancipated
with an appropriate word(s). 14. (a) has been (b) has
5. GERMANE
11. He was so … at his job that he was (c) are (d) have
(a) Considerate (b) Genuine
asked to give a … to the visitors. 15. (a) conduct (b) conjecture
(c) Irrelevant (d) Upstart
(a) agile; demolition
(c) reveal (d) conceal
6. LAGGARD (b) adept; demonstration
(a) Novice (b) Neophyte (c) able; disclosure 16. (a) at (b) by
(c) Expert (d) Pioneer (d) accurate; display (c) of (d) before
128 AFCAT ~ Practice Set 15

Directions (Q. Nos. 17-19) In the 26. He lays out 50% of his income 31. In which among the following places
following questions, some parts of the on bonds and shares. Second Buddhist Council was held?
sentences have errors and some are (a) allots (b) distributes (a) Sattapani (b) Vaishali
correct. Find out which part of a sentence (e) donates (d) spends (c) Pataliputra (d) Kundalvana
has an error. If a sentence is free from
error, choose ‘No error’ option. 27. I will do the work if I am allowed 32. Who is known as the ‘Father of
a free hand in the choice of Indian Army’?
17. Our beloved Prime Minister is materials. (a) Thomas Walker Elmhirst
known to the prominent figures (a) complete liberty (b) Robert Lockhart
all over the world. (b) an expense account (c) Stringer Lawrence
(a) the prominent figures all over (c) to employ men to work (d) Sam Manekshaw
the world
(d) unlimited funds 33. What is the motto of Indian Army?
(b) Our beloved Prime Minister
(c) is known to Directions (Q. Nos. 28-30) Read the (a) Duty Unto Death
(d) No error following passage carefully and answer (b) May the Lord of water be
the questions that follow. Auspicious Unto US
18. Everyone were happy to hear (c) Nabhah Sparsham Diptam
about his success. There is more than a modicum of
truth in the assertion that “a working (d) Service Before Self
(a) to hear
(b) Everyone were happy knowledge of ancient history is 34. In which year first census was
(c) about his success necessary to the intelligent conducted in India?
(d) No error interpretation of current events”. But (a) 1884 (b) 1872 (c) 1881 (d) 1856
the sage who uttered these words of 35. The Indian Botanical Garden is
19. The lions kill the animals and eat wisdom might well have added
his meat. located at
something on the benefits of studying, (a) Delhi (b) Allahabad
(a) kill the animals particularly, the famous battles of
(b) and eat his meat (c) Kolkata (d) Lucknow
history for the lessons they contain for
(c) The lions those of us who lead or aspire for 36. World Malaria Day is observed on
(d) No error leadership. Such a study will reveal (a) 24th April (b) 25th April
certain qualities and attributes which (c) 26th April (d) 27th April
Directions (Q. Nos. 20-23) There are
four words in each question, out of enabled the winners to win and certain 37. Who among the following is
which only one word is correctly spelt. deficiencies which caused the losers to known as ‘Payyoli Express’?
Find the correctly spelt word. lose. And the student will see that the (a) PT Usha (b) Sania Mirza
20. (a) Ansestral (b) Encestral same pattern recurs consistently, again (c) Saina Nehwal (d) Jyoti Kumari
and again, throughout the centuries. 38. ‘Libero’ is related to which sports?
(c) Ansastral (d) Ancestral
28. The expression ‘more than a (a) Hockey (b) Volleyball
21. (a) Symetry (b) Cemetary
modicum of truth’ means (c) Football (d) Cricket
(c) Cemetery (d) Cemetry (a) nothing but truth
39. Which is one of the most extreme
22. (a) Entecedent (b) Antecedent (b) some truth
volcanoes of the world?
(c) Intecedent (d) Antecedant (c) much truth
(a) Cotopaxi (b) Mount Etna
(d) more than a small amount of
23. (a) Naseating (b) Aberrance (c) Popocatepetl (d) Krakatoa
truth
(d) Delibarate (d) Maditation 40. Which among the following is the
29. In this context, ‘intelligent
Directions (Q. Nos. 24-27) Choose oldest refinery of India?
interpretation of current events’
the alternatives which best expresses means (a) Chennai (b) Digboi
the meaning of the idiom/phrase given (c) Jamnagar (d) Panipat
(a) skilful interpretation of events
in bold words. (b) intellectual outlook on events 41. Montreal Protocol is related to the
24. If you want to be happy, cut (c) appropriate understanding of (a) Global warming
your coat according to your events (b) Ozone layer depletion
cloth. (d) rational explanation of events (c) Sustainable development
(a) be honest in your dealings 30. A person who aspires to lead (d) Food security
(b) work according to your capacity could learn from the history of 42. Joule is a unit of
(c) live within your means battles (a) Energy (work) (b) Power
(d) don’t be too ambitious (a) the qualities and deficiencies of (c) Momentum (d) None of these
25. She broke down in the middle of commanders of these battles
43. Who demonstrated that mass of an
her speech. (b) what led the previous leaders
atom is located in the nucleus?
(a) could not proceed win a battle
(c) what made them lose a battle (a) Niels Bohr
(b) fell down (b) Ernest Rutherford
(c) became angry (d) the strategies they evolved in
course of these battles (c) Alfred Nobel
(d) cried (d) Isaac Newton
AFCAT ~ Practice Set 15 129

44. The headquarter of International 54. A, B and C are employed to do a 62. A shopkeeper makes a profit of
Atomic Energy Agency (IAEA) is piece of work for ` 575. A and C 20% even after giving a discount
located is 19 of 10% on the marked price of an
are supposed to finish of the
(a) Geneva (b) Washington 23 article. If marked price ` 500,
(c) Vienna (d) London work together. Amount shall be then the cost price of the article
is
45. FIFA World Cup 2022 will be paid to B is
(a) ` 350 (b) ` 375
organised in which country? (a) ` 210 (b) ` 100 (c) ` 200(d) ` 475
(c) ` 425 (d) ` 475
(a) India (b) Bhutan 55. A motorist travels to a place
(c) Qatar (d) Spain 150 km away at an average speed 63. The simplification of
46. Which among the following is a of 50 km/h and returns at 30 1 − 1  1 − 1  1 − 1  ... 1 − 1 
       
Union Territory, which was made km/h. His average speed for the  2  3  4  100 
by bifurcating the Jammu and whole journey (in km/h) is results in
Kashmir state? (a) 37.5 (b) 37 (c) 35 (d) 40 (a) 0.01 (b) 0.001
(a) Shimla (b) Srinagar 56. Priya got 9 marks more in History (c) 1 (d) 0.1
(c) Ladakh (d) Jammu than what she got in Geography. 64. A godown is in form of a cuboid,
47. Which city has become the first Her History marks are 56% of the whose measures are 8m × 6 m × 3 m .
sky-cycling-park? sum of her History and If a bag of wheat occupies 0.65 m 3
(a) Kullu (b) Mount Abu Geography marks. What are her space, then the number of
(c) Nainital (d) Shimla Geography marks? bags that can be stored in
(a) 42 (b) 65 (c) 53 (d) 33 godown, is
48. Which historical monument has
57. 2 men and 3 women can do a (a) 220 (b) 218
got India’s first ever architectural
LED illumination? piece of work in 10 days, while 3 (c) 221 (d) 222
(a) Lal Kila (b) Taj Mahal men and 2 women can do the 65. In a division problem, the
(c) Qutb Minar (d) Hawa Mahal same work in 8 days. Then, 2 men divisor is 4 times the quotient
and 1 woman can do the same and 3 times the remainder.
49. Name the particles which make work in If remainder is 4, then the
up matter. 1
(a) 12 days (b)12 days dividend is
(a) Non-metals (b) Metals 2 (a) 36 (b) 40
(c) Atoms (d) Metalloids 1
(c) 13 days (d)13 days (c) 12 (d) 30
50. Which among the following 2
66. In a code language,
countries has undertaken the pilot 58. If the profit per cent got on ‘TEMPORARY’ is written as
project ‘Samudrayaan’? selling an article is numerically ‘EPRSAYOYM’ and ‘EXCUSE’ as
(a) Brazil (b) Afghanistan equal to its cost price in rupees ‘PGNVXP’. How is ‘ASSURE’
(c) India (d) South Korea and the selling price is ` 39, then written in that code?
51. A train starts from a places A at cost price (in `) will be (a) OPPVYP (b) OXXVYP
6 am and arrives at another place (a) 20 (b) 22 (c) OPPVXP (d) OXXYVP
B at 4 : 30 pm on the same day. If (c) 28 (d) 30
67. Two statements are given
the speed of the train is 40 km/h, 59. The simplified value of followed by two conclusions. Read
find the distance travelled by the
112
×
576
×
256 the conclusions and then decide
train? is
196 12 8 which of the conclusions, if any,
(a) 420 km (b) 230 km
(a) 12 (b) 8 logically follows from the two
(c) 320 km (d) 400 km given statements, disregarding the
(c) 6 (d) 32
52. In what time ` 8000 will amount known facts.
to ` 9261 at 10% per annum 60. The average marks obtained by a Statements
compound interest, when the class of 60 students is 65. The Some shirts are socks.
interest in compounded average marks of half of the No socks is red.
half-yearly? students is found to be 85. The Conclusions
1 1 average marks of the remaining I. Some socks are shirts.
(a) 3 yr (b)1 yr students is II. No shirt is red.
2 2
1 (a) 35 (b) 45 (a) Only Conclusion I follows
(c) 2 yr (d) 2 yr (c) 55 (d) 65
2 (b) Only Conclusion II follows
61. If a number is increased by 20% (c) Neither Conclusion I nor II
53. What number must be subtracted follows
from both the numerator and the and the resulting number is
27 again increased by 20%, then (d) Both Conclusions I and II follow
denominator of the fraction , what per cent is the total
35 68. One statement is given followed
2 increase? by two assumptions numbered I
so that it becomes ? (a) 48 (b) 44 and II. You have to consider the
3
(a) 6 (b) 8 (c) 9 (d) 11
(c) 41 (d) 40 statement and the following
130 AFCAT ~ Practice Set 15

assumptions, and decide which of Directions (Q.Nos. 72 and 73) In each 82. Select from the alternatives, the box
the assumption is implicit in the of the following question, there are that cannot be formed by folding
statement. three elements related in same way. the sheet shown in the figure (X)
Statement Find the diagram in which these three
The government has instructed all elements fit. 5
the premier institutes offering
professional courses to reduce the (a) (b) 4 3
fees by 50 per cent and increase
the number of students. 1 2
Assumptions
I. These institutes may be able (c) (d) 6
to continue providing quality
education with less fees and 5 6 4 5
more students. 72. Author, Lawyer, Singer 3 3 2 2
4 5 1 1
II. The institutes may continue 73. Class, Blackboard, School
charging more fees to provide (a) (b) (c) (d)
Directions (Q. Nos. 74-76) In each of
quality education. the following question, three alternatives
(a) Only Assumption I is implicit Directions (Q. Nos. 83-86) Which of
are same in a certain way out of four the following codes will come in place of
(b) Only Assumption II is implicit and so form a group. Choose the word question mark ?
(c) Neither Conclusion I nor II is that does not belong to the group.
implicit 83.
74. (a) Debit (b) Deposit
(d) Both Assumptions I and II are
implicit (c) Deduction (d) Withdrawal
75. (a) Jupiter (b) Uranus PX WY WX ?
69. A statement is given followed by
two conclusions. You have to take (c) Mercury (d) Earth (a) XP (b) XY
the given statement to be true (c) WP (d) PY
76. (a) Rain (b) Shower
even, if they seem to be at 84.
(c) Sleet (d) Raisin
variance from commonly known
facts. Read the conclusions and Directions (Q. Nos. 77-81) Find out AX CX CY AY FX ?
then decide which of the the pair in which the words bear the same
(a) FX (b) AX
conclusion, if any, logically relationship to each other as similar to
the words of the given pair bear. (c) FY (d) FA
follows from the two given
statements, disregarding the 77. Book : Author 85.
known facts.
(a) Rain : Flood (b) Light : Switch
Statements Vegetable prices are (c) Symphony : Composer AV CX EY CY EV ?
soaring in the market.
(d) Song : Music (a) CV (b) AY
Conclusions
78. Identity : Anonymity (c) AX (d) EX
I. Vegetables are becoming a
rare commodity. (a) Flaw : Perfection 86.
II. People cannot eat vegetables. (b) Careless : Mistake
(a) Only Conclusion I follows (c) Truth : Lie VL QD TL VS TD ?
(b) Only Conclusion II follows (d) Fear : Joy
(a) QD
(c) Either Conclusion I or II follows 79. Suggestion : Order (b) VS
(d) Neither Conclusion I nor II follows (a) Advise : Suggest (c) VD
70. A man walks 30 m towards South. (b) Smile : Laugh (d) QS
Then, turning to his right, he (c) Plan : Implement
Directions (Q.Nos. 87-89) Out of the
walks 30 m. Then, turning to his (d) Anger : Shout
four given figure, three are similar in a
left, he walks 20m. Again, he turns 80. Interview : Service certain way, one figure is not like the
to his left and walks 30 m. How
(a) Travel : Bus others. choose the different figure.
far is he from his initial position?
(b) Examination : Degree 87.
(a) 20 m (b) 30 m
(c) Ticket : Travel
(c) 80 m (d) 50 m
(d) Light : Darkness
71. In the following series, find the (a) (b) (c) (d)
81. Bihu : Asom
missing letters.
aa_aaa_aaaa_aaaa_b (a) Garba : Bengal 88.
(b) Gidd : Gujarat
(a) baaa (b) bbaa
(c) Yakshgan : Bihar
(c) bbbb (d) bbba (a) (b) (c) (d)
(d) Bhangra : Punjab
AFCAT ~ Practice Set 15 131

89. 93. Problem Figure Answer Figures

(a) (b) (c) (d)


?
Directions (Q. Nos. 90-91) In each of (a) (b) (c) (d)
the following question choose the figure
which will complete the second pair in 97. Problem Figure
the same way as first pair. Answer Figures
90. Problem Figures
+ + +
?
+
+

+ + Answer Figures
+ + (a) (b) (c) (d)
1 2 3 4
Answer Figures 94. Problem Figure

+ + + (a) (b) (c) (d)


+++ + + + +
+

+ +
+ + +
98. Problem Figure
(a) (b) (c) (d)
?
91. Problem Figures
Answer Figures
? Answer Figures

1 2 3 4
Answer Figures (a) (b) (c) (d)
(a) (b) (c) (d)
95. Problem Figure
99. Problem Figure
(a) (b) (c) (d)

Directions (Q.Nos. 92-95) In each of


the following question, choose the figure ?
that can replace the question mark to Answer Figures
complete the problem figure.
Answer Figures
92. Problem Figure

(a) (b) (c) (d)


(a) (b) (c) (d) 100. Problem Figure
? Directions (Q. Nos. 96-100) In each of
the following question, Find the answer
figure in which the problem figure is
Answer Figure embedded.
Answer Figures
96. Problem Figure
(a) (b) (c) (d)

(a) (b) (c) (d)


132 AFCAT ~ Practice Set 15

Answers
1 (a) 2 (b) 3 (a) 4 (a) 5 (c) 6 (d) 7 (a) 8 (b) 9 (b) 10 (c)
11 (b) 12 (c) 13 (a) 14 (b) 15 (c) 16 (c) 17 (a) 18 (b) 19 (b) 20 (d)
21 (c) 22 (b) 23 (b) 24 (c) 25 (d) 26 (d) 27 (a) 28 (d) 29 (d) 30 (a)
31 (b) 32 (c) 33 (d) 34 (b) 35 (c) 36 (b) 37 (a) 38 (b) 39 (a) 40 (b)
41 (b) 42 (a) 43 (b) 44 (c) 45 (c) 46 (c) 47 (a) 48 (c) 49 (c) 50 (c)
51 (a) 52 (b) 53 (d) 54 (b) 55 (a) 56 (d) 57 (b) 58 (d) 59 (d) 60 (b)
61 (b) 62 (b) 63 (a) 64 (c) 65 (b) 66 (b) 67 (a) 68 (a) 69 (d) 70 (d)
71 (d) 72 (b) 73 (a) 74 (b) 75 (c) 76 (d) 77 (c) 78 (a) 79 (b) 80 (b)
81 (b) 82 (d) 83 (d) 84 (c) 85 (c) 86 (d) 87 (b) 88 (a) 89 (d) 90 (b)
91 (d) 92 (a) 93 (c) 94 (d) 95 (b) 96 (c) 97 (c) 98 (b) 99 (a) 100 (c)

Hints and Solutions


1. (a) ‘Exonerate’ means ‘to free from Hence, option (c) ‘chief and bestowed’ 27. (a) Idiom ‘A free hand’ means ‘the freedom
charge’. So, option (a) ‘to free’ is a are the correct alternatives. to do things and make decisions without
suitable choice. 13. (a) ‘So’ is the adverb for ‘fast’ and due being controlled by another.’ So, option
2. (b) ‘Emaciated’ means ‘abnormally thin to running ‘gasping’ is the obvious (a) ‘complete liberty’ is a correct answer.
or weak especially’ because of illness or result. Hence, option (a) is its correct 28. (d) In context of the passage, the
a lack of food. So, ‘weak’ is its correct answer. expression ‘more than a modicum of truth’
similar meaning word. means more than a small amount of truth.
14. (b) has 15. (c) reveal 16. (c) of
3. (a) ‘Apropos’ means ‘with regard to or 29. (d) As per the given context, intelligent
17. (a) Preposition ‘from’ is missing before
something concerning. Hence, ‘To the interpretation of current events’ means
‘all over the world’.
point’ is its correct synonym. rational explanation of events.
18. (b) ‘Everyone’ is singular so helping verb
4. (a) ‘Callous’ means ‘uncaring, heartless’. 30. (a) A person who aspires to lead could
should also be singular. So, ‘was’ will be
So, ‘careful’ is its correct antonym. learn the qualities and deficiencies of
used in place of ‘were’.
commanders of these battles, from the
5. (c) ‘Germane’ means ‘relevant to a 19. (b) Plural pronoun ‘their’ will be used in history of battles.
subject under consideration’ So, place of ‘his’ as it is used for plural noun
‘Irrelevant’ is its correct antonym. 31. (b) The Second Buddhist Council was
animals. held at Vaishali. It is an ancient city, now
6. (d) ‘Laggard’ is a person who makes 20. (d) ‘Ancestral’ is the correctly spelt word in modern Bihar in Northern India. This
slow progress and falls behind others which means ‘belonging to or inherited Second Council took place in near about
and ‘Pioneer’ is a person who is one of from an ancestor’. 383 BCE, seventy years after the
the first people to do something. So, Buddha’s Parinirvana.
21. (c) ‘Cemetery’ is the correctly spelt word
‘Pioneer’ is the correct antonym of The Second Buddhist Council was convened
which means ‘graveyard’.
‘Laggard’. by King Kalasoka, following conflicts
22. (b) ‘Antecedent’ is the correctly spelt
7. (a) ‘Ecology’ is the study of the between the conservative and liberal
word which means ‘a thing that existed
interaction of people with their elements of Sangha. The council ended
before, or logically precedes another’.
environment. with the rejection of the Mahasamghikas.
23. (b) ‘Aberrance’ is the correctly spelt word 32. (c) Major General Stringer Lawrence was
8. (b) ‘Glossary’ is a list of explanations of
which means ‘unusual or abnormal’. an English soldiers, the first
rare, technical or obsolete words.
24. (c) ‘Cut your coat according to your cloth’ Commander-in-Chief, India. He is known
9. (b) Basement or underground place for
means ‘to do only what you have enough as ‘The Father of Indian Army’. He was a
storing wine or other provisions is called
money to do and no more’. So, option British Army officer, who commanded the
cellar.
(c) ‘living within your means’ best East India Company Army.
10. (c) ‘Exonerate means ‘to free someone expreses the meaning of given idiom. 33. (d) The motto of Indian Army is ‘Service
from blame or guilt.’
25. (d) Phrase ‘Break down’ means ‘to lose Before Self’. The Indian Army is the land
11. (b) In the given blanks, ‘adept and based branch and the largest Component
control of your feeling and start crying’.
demonstration’ are the correct of the Indian Armed Forces. The President
So, option (d) ‘cried’ best expresses the
alternative. One has to be adept in the of India is the Supreme Commander of
meaning of given idiom.
task, to demonstrate. the Indian Army and is commanded by
26. (d) Phrase ’Lays out’ means ‘to spend’. the Chief of Army Staff. It was founded on
12. (c) ‘Chief’ is a title of responsibilities and
So, option (d) ‘spends’ best express the 1st April, 1895. The Indian Army is the
it cannot be forced or imposed, so
meaning of given idiom. 2nd largest army in the world.
‘bestowed’ is the correct word for it.
AFCAT ~ Practice Set 15 133

34. (b) The first census was conducted in 42. (a) Joule is a derived unit of energy or 49. (c) The particles which make up matter
1872 in India. But the first regular work in the International System of Units. are atoms. Atoms are then divided into
census was taken under British Rule on It is equal to the work done by a force of electrons, protons and neutrons, which
17th February, 1881 by WC Plowden, one newton acting through one metre. are made up of quark. Protons and
Census Commissioner of India. Since It was named in honour of the English neutrons make up the center of the atom
then, censuses have been undertaken Physicist James Prescott Joule. It equals called the nucleus.
uninterruptedly once every ten years. 107 ergs or approximately 0.7377 foot 50. (c) The ‘Samudrayaan’ is a part of the
35. (c) Indian Botanical Garden, now pounds. Ministry of Earth Sciences’ pilot project
Acharya Jagdish Chandra Bose Indian m2 for deep ocean mining for rare minerals.
Botanical Garden is located in Howrah, 1 Joule = 1 kg
s2 The project proposed to send a
Kolkata in West Bengal. It is famous for submersible vehicle with three persons to
its enormous collections of orchids, 43. (b) Ernest Rutherford demonstrated that
a depth of 6000 m to carry out deep
bamboos, palms and plants Screwpine the mass of an atom is located in the
under nature studies.This project is
genus (Pandanus). It is operated by the nucleus. Most of the mass of the atom is
undertaken by the National Institute of
Botanical Survey of India. concentrated in nucleus, which has a
Ocean Technology (NIOT) of India.
36. (b) World Malaria Day is observed on positive charge. He discovered that all
the positive charge of an atom was 51. (a) Time = 6 : 00 am to 4 : 30 pm
25th April every year. This day gives
located in a tiny dense object at the 1 21
people the chance to promote about the = 10 h = h
efforts made to prevent and reduce center of the atom. 2 2
Malaria around the world. 44. (c) The headquarters of International ∴Speed = 40 km/
The theme of ‘World Malaria Day 2019’ Atomic Energy Agency is in Vienna, ∴Distance = Speed × Time
21
was ‘‘Zero malaria Starts with Me’’. Austria. It was established on 29th July, = 40 × = 420 km
1957. IAEA is an international 2
37. (a) PT Usha is a retired Indian track and
field athlete and has been associated organisation that seeks to promote the 52. (b) Interest is compounded half-yearly.
with Indian athletics since 1979 and is peaceful use of nuclear energy and to ∴Rate of interest = 5%
better known as ‘Payyoli Express’. Payyoli inhibit its use for any military purpose, n
time = yr then n = half-years
is a famous town on the Malabar Coast including nuclear weapons. 2
t
A = P 1 +
of Kozhikode district in Kerala state and 45. (c) FIFA World Cup, 2022 is the 22nd r 

the birth place of PT Usha. edition of the quadrennial international  100
38. (b) The term ‘Libero’ is widely used in the men’s association football championship n
9261 = 8000 1 +
5 
Volleyball game. A libero is a defensive contested by the national teams of the ⇒ 
 100
specialist position in indoor Volleyball. member associations of FIFA. It will be
n 3 n
9261  21
=   ⇒   =  
The Libero position was added in 1999 in organised in Arabian State Qatar in 21 21

volleyball game along with a set of special 2022. Qatar would be the first Arab 8000  20  20  20
rules in order to foster more digs and Nation to host the FIFA World Cup. ⇒ n = 3 half-years
rallies. 3 1
46. (c) Ladakh region in Jammu and ∴ Time = yr = 1 yr
39. (a) Cotopaxi is one of South America’s Kashmir was reorganised as a Union 2 2
and world’s most active volcano. With its Territory (UT) by implementation of the 53. (d) Let the number be x.
5897 m (19,347 ft) height, it also ranks Jammu and Kashmir Reorganisation 27 − x 2
among the world’s most dangerous and According to the question, =
Act, 2019. Two UT, Jammu and 35 − x 3
active volcano. It is situated about 50 km Kashmir as well as Ladakh were made
(30 miles) from the capital city Quito of 3(27 − x) = 2 (35 − x)
by bifurcating the Jammu and Kashmir
Ecuador. state. It means the special status enjoyed 81 − 3x = 70 − 2x
40. (b) Digboi Refinery of Indian Oil by Jammu and Kashmir under Article 3x − 2x = 81 − 70
Corporation is the oldest refinery in India. 370 came to an end. x = 11
It is infact the world’s oldest operating Hence, the required number is 11.
47. (a) The first sky-cycling-park was opened
refinery. It was commissioned in the year
in Gulaba area in Kullu, near Manali. It is 54. (b) Share of work done by A and C
1901. It is now operated by Assam Oil 19
first eco-friendly park, which is 350 together =
Division (AOD).
metre long, in the country. The sky 23
41. (b) The Montreal Protocol is a global cycling path is at a height of 9,000 feet ∴Amount of work done by B alone
agreement to protect the stratospheric above sea level. This park will give a fillip 19 4
ozone layer which took place on 16th =1− =
to tourism in Manali and visitors will get 23 23
September, 1987 in Montreal, Canada. 4
an enriching experience of adventure ∴Share of B = × 575
This protocol also regulates the
sports. 23
production and consumption of nearly
100 man-made chemicals referred to as 48. (c) The historic Qutb Minar got the India’s = 4 × 25 = ` 100
Ozone-Depleting-Substances (ODSs). first ever architectural LED illumination. 55. (a) Here, x = 50, y = 30
Ozone layer is the Earth’s protective With the illumination, the architectural 2xy 2 × 50 × 30
beauty of the 12th century monument ∴Average speed = =
shield that protects humans and the x+ y 50 + 30
environment from harmful levels of Ultra will display its historic majesty after sunset.
Violet (UV) radiation from the Sun.
134 AFCAT ~ Practice Set 15

2 × 50 × 30 Total marks of 30 students


= 66. (b) As, T E M P O R A R Y
80 = 30 × 85 = 2550
= 37.5 km/h ∴Average marks of the remaining students
56. (d) Let Priya got x marks in Geography, 3900 − 2550 E P R S A Y O Y M
=
∴According to the question, 30 and E X C U S E
Marks obtained in History = x + 9 =
1350
= 45
According to the question, 30
x + 9=
56
(x + x + 9 ) 61. (b) Let the original number = 100 P G N V X P
100 The number increased by 20%
⇒100 x + 900 = 112x + 504 Therefore, A S S U R E
= 100 + 100 × 20%
⇒ 12x = 396 ⇒ x = 33 100 × 20
= 100 + = 120
57. (b) According to the question, 100 O X X V Y P
Number of days. Again the number increased by 20%
(M1 + W1) = Number of days. 120 × 20 67. (a)
= 120 + = 120 + 24 = 144 Shirts Socks Red
(M2 + W2 ) 100
10 (2 men + 3 women) Now, change in number Conclusions I. ✓ II. ✗
= 8 (3 men + 2 women) = 144 − 100 = 44
68. (a) The institutes may be able to continue
⇒ 20 men + 30 women In percentage
44 providing quality education with less fees
= 24 men and 16 women = × 100 = 44% [increase] and more student that’s why the
⇒ 4 men = 14 women 100
government has instructed the institutes
⇒ 2 men = 7 women 62. (b) If the cost price of article be ` x. to reduce the fees and increase the
∴2 men + 3 women = 10 women Then, according to the question, number of students. So, Assumption I is
2 men + 1 women = 8 women MP × (100 – Discount%) implicit. Assumption II is not implicit
=
By the formula, M1D1 = M2 D2 100 because if the government give
⇒ 10 × 10 = 8 × D2 CP × (100 + Profit%) instructions, then the institutes may not
=
25 1 100 continue charging more fees.
⇒ D2 = = 12 days 500 × 90 x × 120
2 2 ⇒ = 69. (d) Nothing is given in the statement
100 100 about the availability of vegetables. So,
58. (d) Let the CP of the article be ` x. 6x
⇒ 450 = conclusion I does not follow. Also,
Gain % = x% 5 conclusion II is not directly related to the
According to the question, ⇒ 6x = 5 × 450 statement and so, it also does not follow.
SP − CP 5 × 450
× 100 = Gain % ⇒ x= = ` 375 70. (d) The direction diagram is as follow
CP 6 A (starting point)
39 − x
× 100 = x 63. (a) Given,
x 1 − 1  1 − 1  1 − 1  ... 1 − 1  30 m
N
⇒ 3900 − 100 x = x 2      
 2  3  4  100 30 m W E
⇒ x 2 + 100x − 3900 = 0 1 2 3
= × × × ... ×
99 C B
2 3 4 100 20 m
⇒ x 2 + 130x − 30x − 3900 = 0 S
1
⇒ x(x + 130) − 30 (x + 130) = 0 = = 001
. D E (end point)
100 30 m
⇒ (x − 30) (x + 130) = 0 ∴Required distance, AE = AB + BE
64. (c) ∵ Volume of godown
∴x = 30 as x cannot be negative. = 30 + 20 = 50m
= 8 × 6 × 3 = 144 m3
112 576 256 71. (d) Series pattern :
59. (d) × × and volume of one bag = 0.65 m3
196 12 8 aab/aaab/aaaab/aaaaab
Volume of godown
112 ∴Number of bags = Clearly, the number of ‘a’ is increasing by
= Volume of one bag
2×2×7×7 one in the successive sequence.
144
= = 221 ∴Required answer = bbba
2 ×2 ×2 ×2 ×2 ×2 × 3× 3 065
.
× 72. (b)
12 65. (b) Given, remainder = 4
2×2×2×2×2×2×2×2 Author Lawyer
× According to the question,
8 Divisor = 3 × Remainder = 3 × 4 = 12
112 2×2×2× 3 2×2×2×2
= × × Again, divisor = 4 × Quotient
2×7 12 8 ⇒ 4 × Quotient = 12
112 24 16 12 Singer
= × × = 8 × 2 × 2 = 32 ⇒ Quotient = =3
14 12 8 4 Some Authors can be lawyer, Some
60. (b) Total marks of 60 students Now, Dividend = Quotient × Divisor Lawyers can be Singers, also some
= 65 × 60 = 3900 + Remainder Singers can be Authors and some Authors
= 3 × 12 + 4 = 40 can be both lawyers and Singers.
AFCAT ~ Practice Set 15 135

73. (a) School 83. (d) In the given question, P stands for 90. (b) From first figure to second figure,
Class square, W stands for triangle, X stands both designs inside the circle move in
Blackboard for cross lines shading and Y stands for clockwise direction and one design
doted circle shading. appears in the middle of the two designs.
So, code for the figure is PY. 91. (d) The figure rotates through 90°
Blackboard is in the class and class in Hence, option (d) is correct. clockwise. The inner figure is inverted
the school.
84. (c) In the given question, A stands for and comes out side and the out side
74. (b) In this question, the words Debit, rhombus, C stands for square and figure goes inside.
Deduction and Withdrawal are very F stands for circle and XY, respectively 92. (a) Answer figure (a) will complete the
much similar in the meaning, stand for their internal patterns. problem figure.
whereas Deposit is antonym of So, code for the figure is FY.
these words. 93. (c) Answer figure (c) will complete the
Hence, option (c) is correct. problem figure.
75. (c) Each of Jupiter, Uranus and Earth
85. (c) In the given question, each element 94. (d) Answer figure (d) will complete the
has its satellite while Mercury doesn’t
code is represented as problem figure.
have its satellite. Thus, Mercury is
different. A, C, E 95. (b) Answer figure (b) will complete the
76. (d) Here, all options are related to rain or problem figure.
water except Raisin. The meaning of and the external shapes as 96. (c) The problem figure is embedded in
‘Raisin’ is a partially dried grapes which answer figure (c).
is different from all others. V, X, Y
77. (c) Book is written by Author, in the
same way Symphony is composed by So, code for the figure is AX.
Composer. Hence, option (c) is correct.
78. (a) Second is the state of lack of first. 86. (d) In the given question, each element 97. (c) The problem figure is embedded in
code is represented as answer figure (c).
79. (b) ‘Suggestion’ is a light form of ‘Order’.
In the same way, ‘Smile’ is a light form of Straight circle i.e. V
‘Laugh’.
80. (b) ‘Interview’ is conducted to provide
‘Service’ and ‘Examination’ is conducted Slant circle i.e. Q
to provide ‘Degree’. 98. (b) The problem figure is embedded in
81. (d) ‘Bihu’ is a dance form of ‘Asom’. In and T answer figure (b).
the same way, ‘Bhangra’ is a dance form
of ‘Punjab’. Now, for shaded arrangement
82. (d)
5 L D S

4 3 Opposite So, code for the figure is QS. 99. (a) The problem figure is embedded in
Hence, option (d) is correct. answer figure (a).
Opposite 1 2 87. (b) All other figure, contain different
figures with one, two, three and four
6
sides. Figure (b) contains two figures of
Opposite two sides each.
∴‘5’ is opposite to ‘2’, ‘4’ is opposite to 88. (a) In all other figures, one dot is outside 100.(c) The problem figure is embedded in
‘6’ and ‘1’ is opposite to ‘3’. the figure and one is inside. answer figure (c).
In figure (d), ‘5’ and ‘2’ appear 89. (d) In all other figures, arrows on the
adjacent to each other. So, the circumference of both the circles are in
cube given in the figure (d) cannot be opposite directions.
formed.

You might also like